0% found this document useful (0 votes)
130 views287 pages

Taking Your Examination Offline: Eastern Time On March 30, 2007

ASPS

Uploaded by

omar waleed
Copyright
© © All Rights Reserved
We take content rights seriously. If you suspect this is your content, claim it here.
Available Formats
Download as PDF, TXT or read online on Scribd
0% found this document useful (0 votes)
130 views287 pages

Taking Your Examination Offline: Eastern Time On March 30, 2007

ASPS

Uploaded by

omar waleed
Copyright
© © All Rights Reserved
We take content rights seriously. If you suspect this is your content, claim it here.
Available Formats
Download as PDF, TXT or read online on Scribd
You are on page 1/ 287

American Society of Plastic Surgeons and the Plastic Surgery Educational Foundation

In-Service Examination

Taking Your Examination Offline


You may take the In-Service Examination offline, but you must submit your answers online. Select the
“Download In-Service Exam (PDF)” link to open and print the pdf version of the exam. There will be
notations next to the color pictorials if you do not have a color printer. When you are ready to enter your
answers online, initiate your exam by selecting the “Start Exam” link and read and acknowledge the honor
code statement. Pause the exam, and then select the “Enter answer sheet” link provided on the website;
more specific details are provided below. You may save and print the answer sheet at any time, but do
not select “Submit” until all responses have been entered.

Once you have completed your examination offline, follow these steps to enter and submit your answers
for scoring.
• Log in to the website.
• Select the “Start Exam” link and read and acknowledge the honor code statement.
• Pause the exam.
• Select the “Enter answer sheet” link.
• Enter your answers in the provided form. If the form already contains answers, these are the
answers you previously entered in your online examination or using the answer sheet.
• Once you have finished entering your answers, be sure to save them by clicking “Save.” If you
close the answer sheet page without clicking one of these links, your answers will not be saved.
• You may return to the answer sheet to enter or review answers as many times as you like
during the testing window as long as you do not submit your answers or select “End Exam” in the
online exam.

When you are ready to submit your final answers for scoring, click the “Submit” link. A pop-up box will
appear, asking you to confirm your decision to submit your responses at this time. Click “OK” to
continue with submission or “Cancel” to return to the answer sheet. Once you click “Submit” and
respond “OK” to the pop-up box, you will receive a message indicating your responses have been
submitted. If you have selected the option for an email confirmation, an email will be sent to you
indicating that the examination has been completed. Once you submit your final answers for scoring,
you will not be able to review or modify your answers. All final answers must be submitted by noon
Eastern Time on March 30, 2007.

When your answers have been submitted successfully, you will be directed to an online survey about
your examination experience. Your participation is critical to future planning so please answer all the
questions.

This examination contains test materials that are owned and copyrighted by the American Society of
Plastic Surgeons and the Plastic Surgery Educational Foundation. Any reproduction of these materials or
any part of them, through any means, including but not limitied to, copying or printing electronic files,
reconstruction through memorization or dictation, and/or dissemination of these materials or any part of
them is strictly prohibited. Keep printed materials in a secure location when you are not reviewing them
and discard them in a secure manner, such as shredding, when you have completed the examination.

Page 1 of 287
American Society of Plastic Surgeons and the Plastic Surgery Educational Foundation
In-Service Examination

American Society of Plastic Surgeons and


the Plastic Surgery Educational Foundation
In-Service Examination

Copyright© 2007 by the American Society of Plastic Surgeons Inc. and the Plastic Surgery Educational
Foundation. All rights reserved. Copyright© 2007 by Internet Testing Systems (ITS) as to the computer
presentation. All rights reserved.

This examination contains test materials that are owned and copyrighted by the American Society of
Plastic Surgeons Inc. and the Plastic Surgery Educational Foundation. Any reproduction of these
materials or any part of them, through any means, including but not limited to, copying or printing
electronic files, reconstruction through memorization or dictation, and/or dissemination of these materials
or any part of them is strictly prohibited.

Prepared in cooperation with the National Board of Medical Examiners®.

The mission of the American Society of Plastic Surgeons® is to support its members in their efforts to
provide the highest quality patient care and maintain professional and ethical standards through
education, research and advocacy of socioeconomic and other professional activities.
The Plastic Surgery Educational Foundation® is the educational arm of the American Society of Plastic
Surgeons. The Foundation is proud to display the ASPS Symbol of Excellence signifying the professional
expertise of plastic surgeons certified by The American Board of Plastic Surgery® or The Royal College of
Physicians and Surgeons of Canada®.
The mission of the Plastic Surgery Educational Foundation is to develop and support the domestic and
international education, research and public service activities of plastic surgeons.
This In-Service Examination has been designed to enhance the participant’s clinical knowledge base of
plastic surgery and serve as a self-assessment tool. Each participant will become updated on the core
curriculum of plastic surgery. Upon completion of questions and thoughtful review of individual scores,
participants can establish a benchmark for areas of focus needing further concentrated educational
efforts. It is expected that completion of this educational activity will assist participants in obtaining better
outcomes in practice.
Material released March 2007.

This examination contains test materials that are owned and copyrighted by the American Society of
Plastic Surgeons and the Plastic Surgery Educational Foundation. Any reproduction of these materials or
any part of them, through any means, including but not limitied to, copying or printing electronic files,
reconstruction through memorization or dictation, and/or dissemination of these materials or any part of
them is strictly prohibited. Keep printed materials in a secure location when you are not reviewing them
and discard them in a secure manner, such as shredding, when you have completed the examination.

Page 2 of 287
American Society of Plastic Surgeons and the Plastic Surgery Educational Foundation
In-Service Examination

2007 IN-SERVICE EXAMINATION Educational Assessment Committee


Lawrence C. Kurtzman, MD, Overall Chair
Comprehensive Cosmetic Craniomaxillofacial Hand
Philip D. Wey, MD, Sheri Slezak, MD, Chair Stephen A. Chidyllo, MD, DDS, Mark F. Hendrickson, MD,
Chair Chair Chair
Kaveh Alizadeh, MD Pierre M. Chevray, MD, Lisa R. David, MD, FACS Jerome Chao, MD
Royal Gerow, MD PhD Joseph L. Daw, Jr., MD, DDS Minas Chrysopoulo, MD
Karol A. Gutowski, Claudio DeLorenzi, MD Frederic W.-B. Deleyiannis, William W. Dzwierzynski,
MD Daniel N. Driscoll, MD MD, MPhil, MPH MD
Charles Scott Parham A. Ganchi, Joseph J. Disa, MD Lawrence Glassman, MD
Hultman, MD PhD, MD John A. Girotto, MD, FAAP Steven Moran, MD
Jeffrey Janis, MD David Kahn, MD Larry Hollier, MD, FACS Zubin J. Panthaki, MD
Yvonne Karanas, MD Anthony N. LaBruna, Joseph E. Losee, MD Bradon Wilhelmi, MD
John T. Lindsey, MD MD Delora Mount, MD Michael S. Wong, MD
Stephen Milner, MD Charles W. Perry, MD Davinder Singh, MD
Michele A. Shermak, Elsa Raskin, MD Helen E. Tadjalli, MD, FACS
MD Ali Sajjadian, MD Peter Taub, MD
Michael Suzman, MD Michel Samson, MD
Mia Talmor, MD Nicole Sommer, MD
Jason Wendel, MD

The In-Service Examination is comprised of a question and answer examination,


individual score sheet, norm table and syllabus which includes questions, answers and
references. The materials have been compiled by the PSEF In-Service Examination
Committee along with the National Board of Medical Examiners professional staff. All
questions were reviewed and found to be objective and supported by appropriate review
of the literature and align with evidence based research. Content in this educational
activity has been reviewed by peers and professionals to determine that the content is
fair, balanced and unbiased.

This examination contains test materials that are owned and copyrighted by the American Society of
Plastic Surgeons and the Plastic Surgery Educational Foundation. Any reproduction of these materials or
any part of them, through any means, including but not limitied to, copying or printing electronic files,
reconstruction through memorization or dictation, and/or dissemination of these materials or any part of
them is strictly prohibited. Keep printed materials in a secure location when you are not reviewing them
and discard them in a secure manner, such as shredding, when you have completed the examination.

Page 3 of 287
American Society of Plastic Surgeons and the Plastic Surgery Educational Foundation
In-Service Examination

CME and Designation Statement


The American Society of Plastic Surgeons is accredited by the Accreditation Council for
Continuing Medical Education (ACCME) to provide continuing medical education for
physicians.

The American Society of Plastic Surgeons designates this educational activity for a
maximum 30 AMA PRA Category I Credit™.

Of the 30 credits, 2 credits have been identified as programming applicable towards the
ASPS Patient Safety requirement. Physicians should only claim credit commensurate
with the extent of their participation in the activity.

The management, discussion and recommended answers for this test represent the
combined opinions of the authors and reviewers. They are not endorsed by nor do they
constitute a standard of practice of the American Society of Plastic Surgeons, Inc. or the
Plastic Surgery Educational Foundation.

© 2007 American Society of Plastic Surgeons. All rights reserved.

Prepared in cooperation with the National Board of Medical Examiners.

This examination contains test materials that are owned and copyrighted by the American Society of
Plastic Surgeons and the Plastic Surgery Educational Foundation. Any reproduction of these materials or
any part of them, through any means, including but not limitied to, copying or printing electronic files,
reconstruction through memorization or dictation, and/or dissemination of these materials or any part of
them is strictly prohibited. Keep printed materials in a secure location when you are not reviewing them
and discard them in a secure manner, such as shredding, when you have completed the examination.

Page 4 of 287
American Society of Plastic Surgeons and the Plastic Surgery Educational Foundation
In-Service Examination

The American Society of Plastic Surgeons, Inc.


and the Plastic Surgery Educational Foundation
HONOR CODE STATEMENT

for the

In-Service Examination in Plastic Surgery

I understand that successful completion of this examination is intended to attest to my


current knowledge of the specialty of plastic surgery. I understand also that this is a
proctored examination for resident participation and is a non-proctored examination for
surgeons in practice. I am bound on my honor to take it alone – without the aid of, or
consultation with, any other individual. Thus, I may not contact, directly or indirectly,
other participants in this In-Service Examination.

Although the images and questions may be downloaded and printed for use by
participants during the examination period, they remain the property of the American
Society of Plastic Surgeons and Plastic Surgery Educational Foundation and may not
be shared with anyone else or used in any other context. Electronic files of the
examination must be deleted.

By taking this examination, I acknowledge that I have read this honor code statement
and agree to abide by the terms stated therein.

This examination contains test materials that are owned and copyrighted by the American Society of
Plastic Surgeons and the Plastic Surgery Educational Foundation. Any reproduction of these materials or
any part of them, through any means, including but not limitied to, copying or printing electronic files,
reconstruction through memorization or dictation, and/or dissemination of these materials or any part of
them is strictly prohibited. Keep printed materials in a secure location when you are not reviewing them
and discard them in a secure manner, such as shredding, when you have completed the examination.

Page 5 of 287
American Society of Plastic Surgeons and the Plastic Surgery Educational Foundation
In-Service Examination

Section 1: Comprehensive
1. A 60-year-old woman with Fitzpatrick type 1 skin and coarse panfacial rhytides
comes to the office for rejuvenation. Which of the following is the most effective
treatment?
A) Dermabrasion
B) Glycolic acid peel
C) Jessner solution peel
D) Phenol peel
E) Trichloroacetic acid (TCA) peel

The correct response is Option D.

Trichloroacetic acid (TCA) chemical peels are excellent for treating rhytides of the infraorbital
skin and the full face. TCA concentration can be varied, from 10% to 25% for a light peel, 30%
to 35% for an intermediate peel, and 50% to 60% for a deep peel. TCA is more effective than
other treatments for neck, chest, and hands because the concentration can be lowered to avoid
hypertrophic scarring, which may occur in these areas. Although TCA peels result in less facial
bleaching than phenol peels, TCA cannot provide as profound an effect as phenol. Therefore,
phenol is better for coarse wrinkles, severe sun damage, and significant pigment problems.

Although TCA peels can provide varying levels of treatment, the response to phenol peels is
"all-or-none." Histologic studies show that phenol provides double the penetration and double
the amount of neocollagen formation compared with a deep TCA peel. Phenol results are long
lasting. Problems associated with phenol include the bleaching effect (making it less useful for
darker complected individuals), prolonged recovery period, and potential for cardiac toxicity.

Performed with a motor-driven diamond fraise or steel brush, dermabrasion is a technique of


skin abrasion that can sand down scars or sculpt the surrounding tissue to a level where the
defect becomes less noticeable. Dermabrasion removes the entire epidermis and the upper and
mid-dermis, including the upper part of skin adnexa. Depth of treatment is controllable and may
be varied to meet the needs of the patient; deeper penetration can improve deeper scars.
Dermabrasion is thus better than peels for perioral rhytides and acne scarring. Re-epithelization
and repigmentation take place from the residual portions of the hair follicles, sebaceous glands,
and sweat ducts. Dermabrasion has less of a bleaching effect than phenol, so it is better for
patients with darker complexions who need improvement in rhytides of the upper and lower lip

This examination contains test materials that are owned and copyrighted by the American Society of
Plastic Surgeons and the Plastic Surgery Educational Foundation. Any reproduction of these materials or
any part of them, through any means, including but not limitied to, copying or printing electronic files,
reconstruction through memorization or dictation, and/or dissemination of these materials or any part of
them is strictly prohibited. Keep printed materials in a secure location when you are not reviewing them
and discard them in a secure manner, such as shredding, when you have completed the examination.

Page 6 of 287
American Society of Plastic Surgeons and the Plastic Surgery Educational Foundation
In-Service Examination

and chin. Dermabrasion cannot be safely performed around the eye and is difficult to perform for
full face treatment.

Glycolic acid peels and Jessner solution peels only provide treatment for superficial wrinkles,
acne scars, and uneven pigmentation.

References

1. Greenbaum SS. Chemical peeling, injectable collagen implants, and dermabrasion.


In: Aston SJ, Beasley RW, Thorne CH, eds. Grabb and Smith's Plastic Surgery. 5th
ed. Philadelphia: Lippincott-Raven; 1997:597-608.
2. Stuzin JM, Baker TJ, Baker TM. Treatment of photoaging: facial chemical peeling.
In: Achauer BM, Eriksson E, Guyuron B, eds. Plastic Surgery: Indications,
Operations and Outcomes. St. Louis: Mosby; 2000:2435-2456.

This examination contains test materials that are owned and copyrighted by the American Society of
Plastic Surgeons and the Plastic Surgery Educational Foundation. Any reproduction of these materials or
any part of them, through any means, including but not limitied to, copying or printing electronic files,
reconstruction through memorization or dictation, and/or dissemination of these materials or any part of
them is strictly prohibited. Keep printed materials in a secure location when you are not reviewing them
and discard them in a secure manner, such as shredding, when you have completed the examination.

Page 7 of 287
American Society of Plastic Surgeons and the Plastic Surgery Educational Foundation
In-Service Examination

2. A 3-month-old girl is referred to the office by her pediatrician for consultation


regarding a hemangioma on the forehead that obstructs the vision of her right eye.
Physical examination shows a nonulcerated, raised, 1.5-cm mass on the upper
eyelid. Which of the following is the most appropriate management?
A) Injection of a corticosteroid
B) Intense pulsed-light therapy
C) Interferon therapy
D) Surgical excision
E) Observation

The correct response is Option D.

Hemangiomas exhibit a characteristic evolution, with early rapid growth (proliferation) followed
by slow involution. The earliest sign of a hemangioma is blanching of the involved skin. This
may be followed by fine telangiectasias and then a red or crimson macule. Rarely, a shallow
ulceration may be the first sign of an incipient hemangioma.

Rapid growth during the neonatal period (birth to four weeks) is the historical hallmark of
hemangiomas. This rate is characteristically beyond the growth rate of the infant, thereby
differentiating this neoplasm from vascular malformations that grow commensurate with the
infant. As proliferation ensues, the hemangioma becomes elevated and may be dome-shaped,
lobulated, plaque-like, tumoral, or any combination of these morphologies. The proliferation
phase occurs during the first year, with the most growth occurring during the first six months of
life. Proliferation slows between the middle and end of the first year of life. During this time, the
hemangioma may remain quiescent or may begin to involute.

The involutional phase may be rapid or prolonged. No specific characteristics appear to influence
the rate or completeness of involution. The exception is a separate type of hemangioma referred
to as a rapidly involuting congenital hemangioma (proliferates in utero and is fully developed at
birth), which tends to complete involution during the second year of life. A total of 50% of
infantile hemangiomas complete involution by age 5 years and 70% by age 7 years; the
remainder may take an additional three to five years to complete the process. Of lesions that have
involuted by age 6 years, 38% will have residual evidence with scar formation, telangiectasia, or
redundant or anetodermic skin. Hemangiomas that take longer to involute have a higher
incidence of permanent cutaneous residua. A total of 80% of lesions that complete involution
after age 6 years may exhibit significant cosmetic deformities.

The benefits to early surgical excision include saving a life or vision and decreasing the negative

This examination contains test materials that are owned and copyrighted by the American Society of
Plastic Surgeons and the Plastic Surgery Educational Foundation. Any reproduction of these materials or
any part of them, through any means, including but not limitied to, copying or printing electronic files,
reconstruction through memorization or dictation, and/or dissemination of these materials or any part of
them is strictly prohibited. Keep printed materials in a secure location when you are not reviewing them
and discard them in a secure manner, such as shredding, when you have completed the examination.

Page 8 of 287
American Society of Plastic Surgeons and the Plastic Surgery Educational Foundation
In-Service Examination

psychosocial effects associated with a cosmetically disfiguring lesion during early childhood.
Other benefits of early excision include the use of naturally expanded skin to aid in primary
closure and the ability to employ a relatively avascular tissue plane surrounding actively growing
hemangiomas. New advancements in surgical instruments that cauterize while cutting lessen the
risk of hemorrhage.

Expectant observation would not be wise in a child with blocked vision due to possibility of
amblyopia. Laser surgery is beneficial in treating both proliferating and residual vessels from
hemangiomas. The flash lamp–pumped pulsed dye laser has become the most widely used laser
for selective ablation of vascular tissue in childhood. Intense pulsed light is not indicated in
treatment of hemangiomas.

Oral and intralesional corticosteroids are effective at slowing the growth and decreasing the size
of proliferating hemangiomas. The mechanism of action has not been elucidated completely;
however, corticosteroids seem to act by potentiating vasoconstrictive effects of epinephrine and
norepinephrine on vascular smooth muscle. Evidence indicates that corticosteroids block
estradiol receptors in hemangiomas in vitro. Wide variation in response rates exists, from less
than 40% to greater than 90%, depending on dose, duration of treatment, and age at which
corticosteroid therapy is initiated. Corticosteroid should be administered during the proliferative
phase because they have a negligible effect on involuting otherwise stable lesions; also, because
they have prolonged and variable response times, they are not to be used in life- or sight-
threatening scenarios.

Interferon alfa-2a can be used in lesions that are unresponsive to corticosteroids. In fact, unlike
corticosteroids, interferon alfa-2a does not require administration during the proliferation phase
to be effective. The onset of action is slower than that of corticosteroids, usually requiring
several weeks; this makes it less attractive for use in acute life- or sight-threatening situations.
Interferon alfa-2a should be used only if corticosteroid therapy fails. The most significant
adverse effect limiting its use in hemangiomas is potentially irreversible spastic diplegia, now
reported in several infants; although most infants displayed significant recovery of spasticity of
lower extremities, it appeared permanent in other infants.

References

1. Achauer BM, Chang CJ, Vander Kam VM. Management of hemangioma of infancy:
review of 245 patients. Plast Reconstr Surg. 1997;99:1301-1308.
2. Boon LM, MacDonald DM, Mulliken JB. Complications of systemic corticosteroid
therapy for problematic hemangioma. Plast Reconstr Surg. 1999;104:1616-1623.
3. Frieden IJ, Eichenfield LF, Esterly NB, et al. Guidelines of care for hemangiomas of
infancy: American Academy of Dermatology Guidelines/Outcomes Committee. J Am
Acad Dermatol. 1997;37:6.

This examination contains test materials that are owned and copyrighted by the American Society of
Plastic Surgeons and the Plastic Surgery Educational Foundation. Any reproduction of these materials or
any part of them, through any means, including but not limitied to, copying or printing electronic files,
reconstruction through memorization or dictation, and/or dissemination of these materials or any part of
them is strictly prohibited. Keep printed materials in a secure location when you are not reviewing them
and discard them in a secure manner, such as shredding, when you have completed the examination.

Page 9 of 287
American Society of Plastic Surgeons and the Plastic Surgery Educational Foundation
In-Service Examination

4. Poetke M, Philipp C, Berlien HP: Flashlamp-pumped pulsed dye laser for


hemangiomas in infancy: treatment of superficial vs mixed hemangiomas. Arch
Dermatol. 2000;136:628-637.
5. Tamayo L, Ortiz DM, Orozco-Covarrubias L, et al. Therapeutic efficacy of interferon
alfa-2b in infants with life-threatening giant hemangiomas. Arch Dermatol.
1997;133:1567-1571.

This examination contains test materials that are owned and copyrighted by the American Society of
Plastic Surgeons and the Plastic Surgery Educational Foundation. Any reproduction of these materials or
any part of them, through any means, including but not limitied to, copying or printing electronic files,
reconstruction through memorization or dictation, and/or dissemination of these materials or any part of
them is strictly prohibited. Keep printed materials in a secure location when you are not reviewing them
and discard them in a secure manner, such as shredding, when you have completed the examination.

Page 10 of 287
American Society of Plastic Surgeons and the Plastic Surgery Educational Foundation
In-Service Examination

3. A 2-year-old boy is brought to the emergency department because he has had


lethargy, fever, and a rash over the extremities for the past 10 hours. Temperature is
39.9°C (103.8°F). Physical examination shows petechiae over the trunk and arms.
Over the next three hours, the rash coalesces to hemorrhagic bullae, and the
diagnosis of purpura fulminans is confirmed. Each of the following management
interventions is appropriate EXCEPT
A) administration of activated protein C
B) broad-spectrum antibiotic therapy
C) early wound debridement and amputation of ischemic digits
D) fasciotomy of extremities
E) fluid resuscitation with inotropic support

The correct response is Option C.

Purpura fulminans is a frequently fatal, rapidly evolving syndrome of septic shock and
hemorrhagic bullae, which can result in massive desquamation. Management includes prompt
recognition of the infection (which is usually caused by Neisseria meningitidis), initiation of
broad-spectrum antibiotic therapy, mechanical ventilation, and aggressive fluid resuscitation
with inotropic support. Disseminated intravascular coagulopathy (DIC) develops, and patients
seem to benefit from replacement of activated protein C. A recently published multicenter,
retrospective review of 70 patients documented an amputation rate of 90% and suggested the
need for early fasciotomy to improve limb salvage. It is difficult to determine tissue viability
during the resuscitation period; therefore, debridement, coverage, and amputation are delayed
until demarcation has occurred.

References

1. Warner PM, Kagan RJ, Yakuboff KP, et al. Current management of purpura
fulminans: a multicenter study. J Burn Care Rehabil. 2003;24:119-126.
2. Sheridan RL, Briggs SE, Remensnyder JP, et al. Management strategy in purpura
fulminans with multiple organ failure in children. Burns. 1996;22:53-56.
3. Dhainaut JF, Yan SB, Claessens YE. Protein C/activated protein C pathway:
overview of clinical trial results in severe sepsis. Crit Care Med. 2004;32(5
Suppl):S194-S201.

This examination contains test materials that are owned and copyrighted by the American Society of
Plastic Surgeons and the Plastic Surgery Educational Foundation. Any reproduction of these materials or
any part of them, through any means, including but not limitied to, copying or printing electronic files,
reconstruction through memorization or dictation, and/or dissemination of these materials or any part of
them is strictly prohibited. Keep printed materials in a secure location when you are not reviewing them
and discard them in a secure manner, such as shredding, when you have completed the examination.

Page 11 of 287
American Society of Plastic Surgeons and the Plastic Surgery Educational Foundation
In-Service Examination

4. Under average conditions, which of the following percentages represents the


expected six-month resorption rate of fat injections?
A) 10%
B) 30%
C) 50%
D) 70%
E) 90%

Please note: Question 4 has been deleted from the examination.


The correct response is Option B.
The key points of the theory of fat survival are as follows: 1) fat is a dynamic tissue; 2) cells that
suffer trauma lose more volume; and 3) the recipient site makes an important difference in graft
survival based on the vascularity of the bed. Careful handling of the graft is critical. Therefore,
given the ideal conditions of minimal traumatic technique, meticulous fat graft size, and recipient
bed selection, we should expect approximately 60% to 80% long-term graft survival based on
several studies.
Moore and co-workers reported on the effects of mechanical damage to fat cells harvested with
syringe suction lipectomy versus excision with local anesthetics. The mechanical damage to
tissue aspirated versus tissue that was excised was unchanged. Lidocaine inhibited the glucose
metabolism and lipolysis of adipocytes in culture. The effect was maintained only for as long as
the lidocaine was present.
Also, as Smahel reported, fat grafts, like other grafts, must be small enough to be revascularized
but large enough to maintain structural integrity and some native blood vessels. When part of the
graft is not revascularized, the fat becomes necrotic and is not simply resorbed but is broken up
and removed by the cellular elements.

Karaoglu and co-workers have studied the recipient site of fat grafts and seen a statistically
significant increase in fat graft survival in supramuscular layer (81.95 ± 4.40%) than in
subcutaneous (41.62 ± 3.29%) and submuscular layer (37.31 ± 5.77%) (P < 0.05). This study
demonstrates that selection of an appropriate recipient site should enhance ultimate survival of
the fat graft.

References

1. Karacaoglu E, Kizilkaya E, Cermik H, et al The role of recipient sites in fat-graft


survival: experimental study. Ann Plast Surg. 2005;55:63-68.
2. Coleman SR. Facial recontouring with lipostructure. Clin Plast Surg.
1997;24:347-367.

This examination contains test materials that are owned and copyrighted by the American Society of
Plastic Surgeons and the Plastic Surgery Educational Foundation. Any reproduction of these materials or
any part of them, through any means, including but not limitied to, copying or printing electronic files,
reconstruction through memorization or dictation, and/or dissemination of these materials or any part of
them is strictly prohibited. Keep printed materials in a secure location when you are not reviewing them
and discard them in a secure manner, such as shredding, when you have completed the examination.

Page 12 of 287
American Society of Plastic Surgeons and the Plastic Surgery Educational Foundation
In-Service Examination

5. In healthy human subjects, individual hairs develop from cells at which of the
following locations?
A) Base of the follicle
B) Cuticle
C) Infundibulum
D) Outer root sheath
E) Sebaceous gland

The correct response is Option A.

Each hair is produced through the proliferation of matrix cells at the base of the hair follicles.
The progeny of these cells become displaced from below, become mature, and produce keratin.

The outermost layer of the hair is called the hair cuticle, composed of hard keratin, and is
responsible for anchoring the hair in its follicle by a system of interlocking scales on its inner
surface.

The infundibulum is the upper portion of the hair follicle above the sebaceous duct. It is lined by
surface epithelium.

The outer root sheath covers the inner root sheath and extends upward from the matrix cells at
the lower end of the hair bulb to the entrance of the sebaceous gland duct. The basal layer of the
outer root sheath contains inactive pigmented amelanotic melanocytes, which can produce
melanin after injury such as chemical peels or dermabrasion and migrate toward the epidermis.

Sebaceous glands produce sebum and open into the hair follicle.

References

1. Teumer J, Cooley J. Follicular cell implantation: an emerging cell therapy for hair
loss. Hair Restoration: State of the Art Seminars in Plastic Surgery.
2005;19:193-200.
2. Orentreich N, Durr NP. Biology of scalp hair growth. Clin Plast Surg. 1982;9:197.

This examination contains test materials that are owned and copyrighted by the American Society of
Plastic Surgeons and the Plastic Surgery Educational Foundation. Any reproduction of these materials or
any part of them, through any means, including but not limitied to, copying or printing electronic files,
reconstruction through memorization or dictation, and/or dissemination of these materials or any part of
them is strictly prohibited. Keep printed materials in a secure location when you are not reviewing them
and discard them in a secure manner, such as shredding, when you have completed the examination.

Page 13 of 287
American Society of Plastic Surgeons and the Plastic Surgery Educational Foundation
In-Service Examination

6. After harvesting of a split-thickness skin graft from the lateral aspect of the thigh,
application of which of the following types of wound care agents will yield the most
rapid epithelialization at the donor site?
A) Normal saline wet-to-dry gauze
B) Occlusive dressing
C) Petrolatum-impregnated gauze
D) Semi-occlusive dressing
E) Silver sulfadiazine cream

The correct response is Option D.

The ideal donor site dressing is one that promotes rapid re-epithelialization, causes little pain,
requires little care, is inexpensive, and has a low rate of infection. Options include occlusive
dressings (Duoderm), semi-occlusive dressings (OpSite, Tegaderm), semi-open dressings
(petrolatum-impregnated gauze, Xeroform or scarlet red), and no dressing.

Although semi-open techniques using a heat lamp to dry the donor site covered with Xeroform
or scarlet red are historically popular, these dressings do not meet current concepts of promoting
wound healing by providing a moist environment.

In multiple studies, the superior dressings have been shown to be semi-occlusive. These products
have been shown to have the fastest healing rates (average nine days to re-epithelialization),
lowest subjective pain scores, lowest infection rates (~3%), and are among the lowest in cost.
They have the advantage of being transparent, which allows ongoing inspection of the site while
maintaining sterility. Some fluid collects under these materials, which promotes moist wound
healing and probably accounts for the more rapid healing rates and decreased subjective pain
scores.

Donor sites for split-thickness skin grafts heal spontaneously from epithelial cells remaining in
epithelial appendages within the dermis and at the wound edges. Healing begins within 24 hours
of harvesting, and the rate of healing is directly proportional to the number of epithelial
appendages remaining and inversely proportional to the thickness of graft harvested. When the
epidermis has regenerated it may be reharvested; however, each harvesting removes a portion of
dermis that is not regenerated. The initial epithelium that is regenerated is very delicate and easy
to disrupt with tape or dressing changes. This is another reason to use the semi-occlusive
dressing technique that does not need to be removed until healing is complete.

This examination contains test materials that are owned and copyrighted by the American Society of
Plastic Surgeons and the Plastic Surgery Educational Foundation. Any reproduction of these materials or
any part of them, through any means, including but not limitied to, copying or printing electronic files,
reconstruction through memorization or dictation, and/or dissemination of these materials or any part of
them is strictly prohibited. Keep printed materials in a secure location when you are not reviewing them
and discard them in a secure manner, such as shredding, when you have completed the examination.

Page 14 of 287
American Society of Plastic Surgeons and the Plastic Surgery Educational Foundation
In-Service Examination

References

1. Disa J, Alizadeh K, Smith J, et al. Evaluation of a combined calcium sodium alginate


(Kaltostat) and bio-occlusive membrane dressing (Tegaderm) in the management of
split thickness skin graft donor sites. Ann Plast Surg. 2001;46:405-408.
2. Feldman DL. Which dressing for split-thickness skin graft donor sites? Ann Plast
Surg. 1991;27:288-291.
3. Gallico GG 3d. Biologic skin substitutes. Clin Plast Surg. 1990;17:519-526.

This examination contains test materials that are owned and copyrighted by the American Society of
Plastic Surgeons and the Plastic Surgery Educational Foundation. Any reproduction of these materials or
any part of them, through any means, including but not limitied to, copying or printing electronic files,
reconstruction through memorization or dictation, and/or dissemination of these materials or any part of
them is strictly prohibited. Keep printed materials in a secure location when you are not reviewing them
and discard them in a secure manner, such as shredding, when you have completed the examination.

Page 15 of 287
American Society of Plastic Surgeons and the Plastic Surgery Educational Foundation
In-Service Examination

7. A 42-year-old woman comes to the office for consultation regarding improvement of


the appearance of her facial skin. Resurfacing with a carbon dioxide laser is
contraindicated in this patient if she has been treated recently with which of the
following drugs?
A) Acyclovir
B) Glycolic acid
C) Hydroquinone
D) Isotretinoin
E) Tretinoin

The correct response is Option D.

Unlike the other pretreatments, isotretinoin (Accutane) is contraindicated in any skin resurfacing
procedures. Although its use for the treatment of acne is regulated, because of its significant risk
for causing birth defects, it may also interfere with healing after resurfacing procedures. This
complication may occur even if the medication has not been used for one year. For this reason,
all patients should be asked about isotretinoin use before any resurfacing procedure is performed.

The remaining treatments are not a contraindication to facial skin resurfacing. Acyclovir is
frequently administered before and after resurfacing procedures, particularly in patients with a
history of herpes virus outbreaks. A glycolic acid peel may also be administered before
resurfacing; however, little data show that it will prevent pigment changes or decrease
inflammation after resurfacing. Hydroquinone may also be administered, but again, there is lack
of good data that it provides significant improvement of pigment changes. Although similar to
isotretinoin, tretinoin (Renova, Retin A) does not cause an increased risk of healing
complications after laser resurfacing. It is frequently used as a pretreatment in such cases.

References

1. Alster TS, Lupton JR. Prevention and treatment of side effects and complications of
cutaneous laser resurfacing. Plast Reconstr Surg. 2002;109:308-318.
2. Alster TS. Cutaneous resurfacing with CO2 and erbium:YAG lasers: preoperative,
intraoperative, and postoperative considerations. Plast Reconstr Surg.
1999;103:619-632.

This examination contains test materials that are owned and copyrighted by the American Society of
Plastic Surgeons and the Plastic Surgery Educational Foundation. Any reproduction of these materials or
any part of them, through any means, including but not limitied to, copying or printing electronic files,
reconstruction through memorization or dictation, and/or dissemination of these materials or any part of
them is strictly prohibited. Keep printed materials in a secure location when you are not reviewing them
and discard them in a secure manner, such as shredding, when you have completed the examination.

Page 16 of 287
American Society of Plastic Surgeons and the Plastic Surgery Educational Foundation
In-Service Examination

8. A 30-year-old man undergoes reconstruction of the right lower leg after sustaining an
open fracture of the tibia. A skin graft is harvested and placed over a free muscle
flap. Two days postoperatively, which of the following findings on microscopic
examination of the skin graft is most likely?
A) Early ischemic injury
B) Increased collagen cross-linking
C) Neovascular circulation
D) Significant edema
E) Venous congestion

The correct response is Option D.

Within the first 24 hours after placement, the graft survives by serum imbibition, which is
absorption of nutrients from the serum leaked from the donor site (muscle in this case). At 24
hours, the healing graft will have increased in mass from edema by up to 30%. Leukocytes can
be seen invading the graft, which may help stimulate endothelial migration and revascularization.
By 24 hours, donor site vessels have begun to invade the graft vascular channels in a process
called inosculation. The graft vessels degenerate and become replaced by the growing donor site
vessels. Circulation is reestablished by day 4 to 5 at the earliest. Whereas early graft ischemia
results in a lowering of the pH and a decrease in metabolism, ischemic injury and necrosis are
not seen. Graft maturation and collagen turnover occur over weeks to months. Early recognition
and correction of the venous thrombosis are not likely to affect early graft nourishment and
healing.

References

1. Paletta CE, Pokorny JJ, Rumbolo P. Skin grafts. In: Mathes S, ed. Plastic Surgery.
2nd ed. Philadelphia: WB Saunders; 2006:293-316.
2. Converse JM, Uhlschmid GK, Ballantyne DL. "Plasmatic circulation" in skin grafts:
the phase of serum imbibition. Plast Reconstr Surg. 1969;43:495-499.

This examination contains test materials that are owned and copyrighted by the American Society of
Plastic Surgeons and the Plastic Surgery Educational Foundation. Any reproduction of these materials or
any part of them, through any means, including but not limitied to, copying or printing electronic files,
reconstruction through memorization or dictation, and/or dissemination of these materials or any part of
them is strictly prohibited. Keep printed materials in a secure location when you are not reviewing them
and discard them in a secure manner, such as shredding, when you have completed the examination.

Page 17 of 287
American Society of Plastic Surgeons and the Plastic Surgery Educational Foundation
In-Service Examination

9. A 20-year-old man undergoes reduction and external fixation of a grade IIIB fracture
of the right lower extremity. One day postoperatively, the plastic surgery team is
called for wound care management. Which of the following is the most appropriate
initial step?
A) Application of wet-to-dry saline dressings
B) Enzymatic debridement
C) Negative-pressure wound therapy
D) Operative debridement
E) Observation

The correct response is Option D.

The tibia is a long bone, the anterior third of which is placed subcutaneously in the leg
throughout most of its length. It is covered only by a thin layer of skin and subcutaneous tissue.
This anatomic location may be explained by the need to protect the blood vessels and nerves of
the leg, which are located deep laterally, medially, and posteriorly. This position makes the tibia
easily vulnerable to trauma, often resulting in exposed fractures.

The extensive Orthopaedic Trauma Association classification of long bone fractures divides
them according to the line of fracture as follows: linear (transverse, oblique, spiral), comminuted
(<50%, >50%, butterfly <50%, butterfly >50%), segmental (2 level, 3 level, longitudinal split,
comminuted), and bone loss (<50%, >50%, complete).

The clinical classification considers the involvement of soft tissues in open fractures. In 1984,
Gustilo and colleagues described five grades in the Association of Osteosynthesis classification,
as follows:
Grade 1: Skin lesion smaller than 1 cm; clean; simple bone fracture with minimal
comminution
Grade 2: Skin lesion larger than 1 cm; no extensive soft tissue damage; minimal crushing;
moderate comminution and contamination
Grade 3: Extensive skin damage with muscle and neurovascular involvement; high-speed
injuries; comminution of the fracture; instability
Grade 3a: Extensive laceration of soft tissues with bone fragments covered; usually
high-speed traumas with severe comminution or segmental fractures
Grade 3b: Extensive lesion of soft tissues with periosteal stripping, contamination, and
severe comminution due to high-speed traumas; usually, exposed bone must be
replaced with a local or free flap as a cover

This examination contains test materials that are owned and copyrighted by the American Society of
Plastic Surgeons and the Plastic Surgery Educational Foundation. Any reproduction of these materials or
any part of them, through any means, including but not limitied to, copying or printing electronic files,
reconstruction through memorization or dictation, and/or dissemination of these materials or any part of
them is strictly prohibited. Keep printed materials in a secure location when you are not reviewing them
and discard them in a secure manner, such as shredding, when you have completed the examination.

Page 18 of 287
American Society of Plastic Surgeons and the Plastic Surgery Educational Foundation
In-Service Examination

Grade 3c: Exposed fracture with arterial damage that requires repair

Because of the exposure of devascularized bone in a contaminated bed, the spontaneous healing
of these wounds is usually aggravated by complications such as delayed union, nonunion,
infection, and osteomyelitis. It is of utmost importance to irrigate the wounds with jet lavage and
debride all segments of devascularized bone in addition to skin and muscle to help prevent these
complications. Application of a vacuum-assisted closure device is helpful only as an adjunct to a
comprehensive irrigation and debridement of the wound.

References

1. Lee KS, Park JW. Free vascularized osteocutaneous fibular graft to the tibia.
Microsurgery. 1999;19:141-147.
2. Byrd HS, Spicer TE, Cierney G 3rd. Management of open tibial fractures. Plast
Reconstr Surg. 1985;76:7.
3. Gustilo RB. Fracture Classification Manual. St. Louis: Mosby-Year Book;
1991:19-30.

This examination contains test materials that are owned and copyrighted by the American Society of
Plastic Surgeons and the Plastic Surgery Educational Foundation. Any reproduction of these materials or
any part of them, through any means, including but not limitied to, copying or printing electronic files,
reconstruction through memorization or dictation, and/or dissemination of these materials or any part of
them is strictly prohibited. Keep printed materials in a secure location when you are not reviewing them
and discard them in a secure manner, such as shredding, when you have completed the examination.

Page 19 of 287
American Society of Plastic Surgeons and the Plastic Surgery Educational Foundation
In-Service Examination

10. A 14-year-old boy is referred to the office by his pediatrician for consultation
regarding a purple mass on the right hand. The lesion has been present since birth
but has been enlarging over the past year. The patient says that the mass
increases in size when he exercises. On physical examination, the lesion is warm
to touch and has a palpable thrill. Which of the following is the most likely
diagnosis?
A) Arteriovenous malformation
B) Capillary malformation
C) Hemangioma
D) Lymphatic malformation
E) Venous malformation

The correct response is Option A.

Arteriovenous malformations are present at birth but may be mistaken at that time for
hemangiomas. However, they do not regress and can increase in size with puberty. The fast-flow
nature of these lesions results in the cutaneous warmth and palpable thrill. Hemangiomas usually
grow during the first year of life and usually regress spontaneously by age 5 to 10 years.
Vascular malformations are present at birth but will not spontaneously involute. They are
characterized by channel type (venous and capillary) and by flow rate. Arteriovenous
malformations are characterized by a red-purple color with warmth and palpable thrill.

References

1. Mulliken JB. Vascular anomalies. In: Aston SJ, Beasley RW, Thorne CHM, eds.
Grabb and Smith's Plastic Surgery. 5th ed. Philadelphia: Lippincott-Raven;
1997:191-203.
2. Spring MA, Bentz ML. Cutaneous vascular lesions. Clin Plast Surg.
2005;32:171-186.

This examination contains test materials that are owned and copyrighted by the American Society of
Plastic Surgeons and the Plastic Surgery Educational Foundation. Any reproduction of these materials or
any part of them, through any means, including but not limitied to, copying or printing electronic files,
reconstruction through memorization or dictation, and/or dissemination of these materials or any part of
them is strictly prohibited. Keep printed materials in a secure location when you are not reviewing them
and discard them in a secure manner, such as shredding, when you have completed the examination.

Page 20 of 287
American Society of Plastic Surgeons and the Plastic Surgery Educational Foundation
In-Service Examination

11. A 7-year-old girl who sustained burns to the head three years ago is scheduled to
undergo reconstruction with tissue expansion. Preoperative physical examination
shows a 7 × 14-cm area of alopecia on the scalp and scarring on the forehead.
Which of the following is the most appropriate placement of tissue expanders in this
patient?
A) Deep to the galea and deep to the frontalis muscle
B) Deep to the galea and superficial to the frontalis muscle
C) Deep to the pericranium and deep to the frontalis muscle
D) Deep to the pericranium and superficial to the frontalis muscle
E) Superficial to the galea and deep to the frontalis muscle

The correct response is Option A.

To provide the maximal amount of safe expansion, tissue expanders are placed between the galea
and periosteum in the scalp and between the frontalis muscle and periosteum in the forehead.
Placement of the expander deep to the periosteum would result in difficult, painful, and
unpredictable expansion. Placement of the expander superficial to the galea might result in
premature extrusion of the device and an unreliable flap after its removal. Expansion of the galea
and frontalis is critical in optimizing blood supply to the expanded forehead skin and scalp.

References

1. Rivera R, LoGiudice J, Gosain AK. Tissue expansion in pediatric patients. Clin Plast
Surg. 2005;32:35-44.
2. Argenta LC. Pediatric tissue expansion. In: Bentz ML, ed. Pediatric Plastic Surgery.
Stamford, CT: Appleton & Lange; 1998:635-675.

This examination contains test materials that are owned and copyrighted by the American Society of
Plastic Surgeons and the Plastic Surgery Educational Foundation. Any reproduction of these materials or
any part of them, through any means, including but not limitied to, copying or printing electronic files,
reconstruction through memorization or dictation, and/or dissemination of these materials or any part of
them is strictly prohibited. Keep printed materials in a secure location when you are not reviewing them
and discard them in a secure manner, such as shredding, when you have completed the examination.

Page 21 of 287
American Society of Plastic Surgeons and the Plastic Surgery Educational Foundation
In-Service Examination

(Please note that this pictorial appears in color in the online examination)

12. A 3-year-old girl is brought to the office by her parents two months after sustaining
an injury to the right ankle for evaluation of the scar shown. Which of the following
is the most appropriate management?
A) Excision
B) Oral administration of a corticosteroid
C) Radiation therapy
D) Topical administration of vitamin E
E) Observation

This examination contains test materials that are owned and copyrighted by the American Society of
Plastic Surgeons and the Plastic Surgery Educational Foundation. Any reproduction of these materials or
any part of them, through any means, including but not limitied to, copying or printing electronic files,
reconstruction through memorization or dictation, and/or dissemination of these materials or any part of
them is strictly prohibited. Keep printed materials in a secure location when you are not reviewing them
and discard them in a secure manner, such as shredding, when you have completed the examination.

Page 22 of 287
American Society of Plastic Surgeons and the Plastic Surgery Educational Foundation
In-Service Examination

The correct response is Option E.

This patient has a hypertrophic scar. Correct diagnosis of the abnormal scar will directly
influence treatment options for this patient.

Hypertrophic scars and keloid scars are clinically distinct entities with different treatment
approaches.

Hypertrophic scars develop soon after the injury (within six to eight weeks). They can worsen up
to six months but subside with time. The extent of scarring relates to the initial depth of injury.
Hypertrophic scars can produce contractures, especially over joints. The boundaries of the
original scar are maintained. Hypertrophic scars have a predilection to occur over the flexor
surface of joints.

Keloid scars may develop months after the injury. They seldom regress and are not associated
with contractures. The boundaries of the original wound are overgrown, and the extent of the
scar can far exceed the original tissue injury. Keloid scars are commonly found on the deltoid,
upper back, chest, and earlobes.

Both hypertrophic and keloid scars are raised, erythematous, and often pruritic.

References

1. Niessen FB, Spauwen PH, Schalkwizk J, et al. On the nature of hypertrophic scars
and keloids: a review. Plast Reconstr Surg. 1999;104:1435-1458.
2. Shahrad RR, Garner WL. Fibroproliferative scars. Clin Plast Surg. 2003;30:77-89.
3. Shaffer JJ, Taylor SC, Cook-Bolden F. Keloidal scars: a review with a critical look at
therapeutic options. J Am Acad Dermatol. 2002;46:S63-S97.
4. Su CY, Alizadeh K, Boddie A, et al. The problem scar. Clin Plast Surg.
1998;25:451-465.

This examination contains test materials that are owned and copyrighted by the American Society of
Plastic Surgeons and the Plastic Surgery Educational Foundation. Any reproduction of these materials or
any part of them, through any means, including but not limitied to, copying or printing electronic files,
reconstruction through memorization or dictation, and/or dissemination of these materials or any part of
them is strictly prohibited. Keep printed materials in a secure location when you are not reviewing them
and discard them in a secure manner, such as shredding, when you have completed the examination.

Page 23 of 287
American Society of Plastic Surgeons and the Plastic Surgery Educational Foundation
In-Service Examination

(Please note that this pictorial appears in color in the online examination)

13. A 12-year-old girl is brought to the office by her parents for consultation regarding a
lesion on the posterior aspect of the scalp (shown). The lesion has been present
since birth and has grown proportionately with the child. Which of the following is
the most likely diagnosis?
A) Compound melanocytic nevus
B) Keratoacanthoma
C) Nevus of Ito
D) Nevus of Ota
E) Nevus sebaceus of Jadassohn

This examination contains test materials that are owned and copyrighted by the American Society of
Plastic Surgeons and the Plastic Surgery Educational Foundation. Any reproduction of these materials or
any part of them, through any means, including but not limitied to, copying or printing electronic files,
reconstruction through memorization or dictation, and/or dissemination of these materials or any part of
them is strictly prohibited. Keep printed materials in a secure location when you are not reviewing them
and discard them in a secure manner, such as shredding, when you have completed the examination.

Page 24 of 287
American Society of Plastic Surgeons and the Plastic Surgery Educational Foundation
In-Service Examination

The correct response is Option A.

The lesion shown represents a compound melanocytic nevus, which is a benign neoplasm
composed mostly of melanocytes. Congenital melanocytic nevi, such as this one, are believed to
represent an anomaly in embryogenesis and, as such, could be considered a malformation or a
hamartoma. Melanocytic nevi are common in light- or fair-skinned patients and are relatively
uncommon lesions in dark-skinned individuals. There is no sexual predilection, and melanocytic
nevi are most common in children and young adults. Excisional biopsy is the treatment of
choice.

Keratoacanthomas are round, firm, pinkish red or flesh-colored papules that rapidly progress to
dome-shaped nodules with a central crateriform ulceration or a horn-like keratin plug. It is a
relatively common low-grade malignancy that originates in the pilosebaceous glands and closely
and pathologically resembles squamous cell carcinoma.

A nevus of Ota is a blue-gray lesion occurring on the face in the distribution of the trigeminal
nerve (particularly the ophthalmic and maxillary branches—V1 and V2).

A nevus of Ito is similar to the nevus of Ota but follows the distribution of the lateral brachial
cutaneous and supraclavicular nerves.

A nevus sebaceus of Jadassohn is a yellow-orange, slightly elevated plaque that usually occurs
on the face and scalp. It has the potential for malignant transformation to a basal cell carcinoma.

References

1. Marghoob AA. Congenital melanocytic nevi: evaluation and management. Dermatol


Clin. 2002;20:607.
2. Zarem HA, Lowe NJ. Benign growths and generalized skin disorders. In: Aston SJ,
Beasley RW, Thorne CH, eds. Grabb and Smith’s Plastic Surgery. 5th ed.
Philadelphia: Lippincott-Raven; 1997:141-159.

This examination contains test materials that are owned and copyrighted by the American Society of
Plastic Surgeons and the Plastic Surgery Educational Foundation. Any reproduction of these materials or
any part of them, through any means, including but not limitied to, copying or printing electronic files,
reconstruction through memorization or dictation, and/or dissemination of these materials or any part of
them is strictly prohibited. Keep printed materials in a secure location when you are not reviewing them
and discard them in a secure manner, such as shredding, when you have completed the examination.

Page 25 of 287
American Society of Plastic Surgeons and the Plastic Surgery Educational Foundation
In-Service Examination

14. Which of the following is an advantage of using bupivacaine over other local
anesthetic agents?
A) Increased vasoconstriction
B) Less cardiotoxicity
C) Less motor blockade
D) Rapid metabolism
E) Rapid onset of action

The correct response is Option C.

One of the advantages of using bupivacaine (Marcaine) is its ability to achieve sensory blockade
at low concentrations with much less effect on motor blockade. This is also true of ropivacaine,
which is the single S-enantiomer of racemic bupivacaine.

Unfortunately, bupivacaine has a much higher risk for cardiotoxicity, especially compared with
lidocaine. Unlike lidocaine, which is a fast-in and fast-out sodium channel blocker, bupivacaine
enters the sodium channel quickly but departs slowly and, therefore, may cause cardiac
arrhythmias.

Bupivacaine does not cause significant vasoconstriction. However, used topically, cocaine
produces excellent local anesthesia and vasoconstriction and, therefore, is used as a topical
anesthetic for mucous membranes of the nose and mouth.

Bupivacaine has a somewhat longer time to onset of action compared with lidocaine and also has
a long duration of action, lasting four to six hours. Therefore, it is indicated for longer
procedures or procedures in which postoperative pain control is desired.

Unlike lidocaine, which can be added to tumescent fluid, bupivacaine is not recommended in
such situations because of the high risk for cardiotoxicity.

References

1. Hijjawi JB, Mustoe TA. Anesthesia. In: McCarthy JG, Galiano RD, Boutros SG, eds.
Current Therapy in Plastic Surgery. Philadelphia: Elsevier-Saunders; 2006:28-33.
2. White PF, Waddle JP. Anesthesia for plastic surgery. In: Mathes SJ, Hentz VR, eds.
Plastic Surgery. 2nd ed. Philadelphia: Elsevier-Saunders; 2006:167-207.

This examination contains test materials that are owned and copyrighted by the American Society of
Plastic Surgeons and the Plastic Surgery Educational Foundation. Any reproduction of these materials or
any part of them, through any means, including but not limitied to, copying or printing electronic files,
reconstruction through memorization or dictation, and/or dissemination of these materials or any part of
them is strictly prohibited. Keep printed materials in a secure location when you are not reviewing them
and discard them in a secure manner, such as shredding, when you have completed the examination.

Page 26 of 287
American Society of Plastic Surgeons and the Plastic Surgery Educational Foundation
In-Service Examination

15. A 19-year-old man who sustained burns to 40% of the total body surface area at 4
years of age comes to the burn clinic for consultation regarding correction of the
scalp defect shown. Physical examination shows patchy alopecia of the temporal
scalp and hypertrophic scarring. Which of the following is the most appropriate
surgical procedure for reconstruction of this patient's scalp?
A) Hair transplantation with micrografts and minigrafts
B) Orticochea flap
C) Serial excision and closure
D) Temporoparietooccipital (Juri) flap
E) Tissue expansion

The correct response is Option E.

The patient shown has a large (>25 cm2) parietal scalp defect that resulted from a flame burn. It
is clear that hair transplantation has already been attempted with minimal take and coverage.
Hair transplantation with micrografts and minigrafts is a technique that has been described to
treat large areas of burn alopecia. However, it is usually reserved for smaller areas, or as a

This examination contains test materials that are owned and copyrighted by the American Society of
Plastic Surgeons and the Plastic Surgery Educational Foundation. Any reproduction of these materials or
any part of them, through any means, including but not limitied to, copying or printing electronic files,
reconstruction through memorization or dictation, and/or dissemination of these materials or any part of
them is strictly prohibited. Keep printed materials in a secure location when you are not reviewing them
and discard them in a secure manner, such as shredding, when you have completed the examination.

Page 27 of 287
American Society of Plastic Surgeons and the Plastic Surgery Educational Foundation
In-Service Examination

revisional procedure to camouflage incisions that result from rotational flaps.

Tissue expansion is the preferred method for secondary reconstruction of a large parietal scalp
defect. Approximately 50% of scalp can be reconstructed with expanded scalp tissue, although
this may require multiple stages. Expander complication rates may be as high as 25% and may
include infection, exposure, extrusion, and device failure. Scalp expansion in children is difficult.
There is a higher incidence of infection and decreased tolerance of pain. Moreover, there is a risk
of skull deformation after a prolonged period of expansion. Therefore, deferral until the patient
reaches young adulthood is reasonable.

Direct closure in a staged manner with serial excision or as a single-stage procedure with rotation
advancement flaps is appropriate for small to medium-sized defects of the anterior and parietal
scalp. The size of this defect would preclude this therapy in this case.

Temporoparietooccipital flaps as described by Juri are optimally suited for reconstruction of


large defects of the anterior scalp. Orticochea flaps are classically described for reconstruction of
large defects of the occipital scalp. They are not useful for parietal defects because tissue
advancement from the contralateral parietal scalp up over the vertex is often inadequate for
defect closure.

References

1. Leedy JE, Janis JE, Rohrich RJ. Reconstruction of acquired scalp defects: an
algorithmic approach. Plast Reconstr Surg. 2005;116:54e-72e.
2. Barrera A. The use of micrografts and minigrafts for the treatment of burn alopecia.
Plast Reconstr Surg. 1999;103:581.
3. Barrera A. The use of micrografts and minigrafts in the aesthetic reconstruction of the
face and scalp. Plast Reconstr Surg. 2005;112:883.

This examination contains test materials that are owned and copyrighted by the American Society of
Plastic Surgeons and the Plastic Surgery Educational Foundation. Any reproduction of these materials or
any part of them, through any means, including but not limitied to, copying or printing electronic files,
reconstruction through memorization or dictation, and/or dissemination of these materials or any part of
them is strictly prohibited. Keep printed materials in a secure location when you are not reviewing them
and discard them in a secure manner, such as shredding, when you have completed the examination.

Page 28 of 287
American Society of Plastic Surgeons and the Plastic Surgery Educational Foundation
In-Service Examination

(Please note that this pictorial appears in color in the online examination)

16. A 60-year-old woman undergoes surgical excision of a 9-mm basal cell carcinoma
from the dorsum of the nose. The resulting defect is 1.5 cm in diameter (shown).
Which of the following modalities is most appropriate for aesthetic reconstruction of
this defect?
A) Banner flap
B) Bilobed flap
C) Full-thickness skin graft
D) Healing by secondary intention
E) Nasolabial flap

This examination contains test materials that are owned and copyrighted by the American Society of
Plastic Surgeons and the Plastic Surgery Educational Foundation. Any reproduction of these materials or
any part of them, through any means, including but not limitied to, copying or printing electronic files,
reconstruction through memorization or dictation, and/or dissemination of these materials or any part of
them is strictly prohibited. Keep printed materials in a secure location when you are not reviewing them
and discard them in a secure manner, such as shredding, when you have completed the examination.

Page 29 of 287
American Society of Plastic Surgeons and the Plastic Surgery Educational Foundation
In-Service Examination

The correct response is Option B.

The bilobed flap is the most appropriate choice for 0.5- to 1.5-cm defects of the nasal tip and ala
(see photograph). Excess nasal skin high in the middle of the nose or high on the lateral aspect of
the nose is the donor tissue. Generally, a laterally based design is used for defects of the tip and a
medially based design is used for defects of the alar lobule. Undermining must be wide and just
above the level of periosteum and perichondrium to preserve blood supply. The diameter of the
first lobe is equal to the defect, and the second lobe is reduced in width to ease donor site
closure. The bilobed flap donor incisions heal well in the skin of the nose. The bilobed flap is a
single stage reconstruction.

Single-lobed flaps such as the banner flap are preferred in thin-skinned areas of the nose and may
give a better aesthetic result than full-thickness skin grafting. In thicker skinned areas of the nose
where the skin is less flexible, transposition of single-lobed flaps may distort adjacent areas.

Full-thickness skin grafting is best suited to the upper two-thirds of the nose with defects up to
2.5 cm wide. Skin grafting works well in areas of thin nasal skin and does not replace the thick

This examination contains test materials that are owned and copyrighted by the American Society of
Plastic Surgeons and the Plastic Surgery Educational Foundation. Any reproduction of these materials or
any part of them, through any means, including but not limitied to, copying or printing electronic files,
reconstruction through memorization or dictation, and/or dissemination of these materials or any part of
them is strictly prohibited. Keep printed materials in a secure location when you are not reviewing them
and discard them in a secure manner, such as shredding, when you have completed the examination.

Page 30 of 287
American Society of Plastic Surgeons and the Plastic Surgery Educational Foundation
In-Service Examination

skin of the lower nose in a cosmetically acceptable way. The skins grafts may appear depressed,
shiny, and off-colored. Skin grafting requires a moist, vascularized wound bed and quilting
sutures or bolster dressings for immobilization of the graft for the healing period.

Healing by secondary intention in the nasal tip will lead to distortion of the tip and is often not a
viable option because of exposed cartilage in the tip after skin cancer excision.

The nasolabial flap is pedicled superiorly or inferiorly based on branches of the facial and
angular arteries, and the donor site is closed in the contour of the nasolabial crease. The flap may
be taken as a pedicled flap requiring pedicle division or as an island flap that may be performed
in a single stage. Problems associated with use of the nasolabial flap include texture and color
differences between the skin of the cheek and nose and possible loss of the cheek-nose concavity
when reconstructing the nasal ala.

References

1. Burget GC, Menick FJ. Aesthetic Reconstruction of the Nose. St. Louis: Mosby-Year
Book; 1994.
2. Jackson IT. Local Flaps in Head and Neck Reconstruction. St. Louis: CV Mosby;
1985.

This examination contains test materials that are owned and copyrighted by the American Society of
Plastic Surgeons and the Plastic Surgery Educational Foundation. Any reproduction of these materials or
any part of them, through any means, including but not limitied to, copying or printing electronic files,
reconstruction through memorization or dictation, and/or dissemination of these materials or any part of
them is strictly prohibited. Keep printed materials in a secure location when you are not reviewing them
and discard them in a secure manner, such as shredding, when you have completed the examination.

Page 31 of 287
American Society of Plastic Surgeons and the Plastic Surgery Educational Foundation
In-Service Examination

17. A 53-year-old man has a Stage III pressure sore on the ischium. Negative-pressure
wound therapy is planned. Which of the following wound characteristics is most
likely to impair the efficacy of this treatment modality?
A) Depth
B) Diameter
C) Edema
D) Fibrosis
E) Undermining

The correct response is Option D.

Chronic, fibrotic pressure sores are characterized by impaired wound healing and, therefore,
respond poorly to negative-pressure wound therapy (NPWT). Debridement of fibrotic tissue and
devitalized tissue in these pressure sores improves the response to NPWT.

NPWT is most appropriate for management of full-thickness pressure sores that are large enough
for adequate contact between the foam dressing and the wound bed and for safe removal of the
foam. NPWT can be used successfully for management of large pressure sores, shallow or deep
pressure sores, and those with undermining or tunneling.

If the patient is a surgical candidate, NPWT can be used to temporize the wound until the patient
has been readied for surgery (nutrition, antibiotics, stabilization of coagulopathy, etc.). NPWT
may allow wound improvement such that a lesser surgical procedure may be done. If the
pressure sore deteriorates or fails to progress in two to four weeks, NPWT may need to be
discontinued.

References

1. Argenta LC, Morykwas MJ. Vacuum-assisted closure: a new method for wound
control and treatment: clinical experience. Ann Plast Surg.1997;38:563-576.
2. Gupta S, Baharestani M, Baranoski S, et al. Guidelines for managing pressure ulcers
with negative pressure wound therapy. Adv Skin Wound Care. 2004;17 Suppl 2:1-16.
3. Mendez-Eastman S. Guidelines for using negative pressure wound therapy. Adv Skin
Wound Care. 2001;14:314-323.

This examination contains test materials that are owned and copyrighted by the American Society of
Plastic Surgeons and the Plastic Surgery Educational Foundation. Any reproduction of these materials or
any part of them, through any means, including but not limitied to, copying or printing electronic files,
reconstruction through memorization or dictation, and/or dissemination of these materials or any part of
them is strictly prohibited. Keep printed materials in a secure location when you are not reviewing them
and discard them in a secure manner, such as shredding, when you have completed the examination.

Page 32 of 287
American Society of Plastic Surgeons and the Plastic Surgery Educational Foundation
In-Service Examination

18. Which of the following percentages best represents the risk of transformation of a
congenital giant nevus to malignant melanoma?
A) 10%
B) 20%
C) 30%
D) 40%
E) 50%

The correct response is Option A.

Although it is generally accepted that giant congenital nevi may undergo transformation to
malignant melanoma, the exact incidence is difficult to determine, due in part to limitations in
methodology of the available studies. Recent reports cite an incidence ranging from 2.9% to
12.2%.

Giant congenital nevi at greatest risk for transformation to melanoma are those lesions that have
a predicted largest diameter of 20 cm in adulthood. Current recommendations regarding the
timing of excision are controversial, but these lesions should be removed as early as possible in
childhood.

In addition to congenital melanocytic nevi, other precursor lesions that can degenerate into
malignant melanoma include common acquired melanocytic nevi, dysplastic nevi, and melanoma
in situ.

References

1. Zaal LH, Mooi WJ, Klip H, et al. Risk of malignant transformation of congenital
melanocytic nevi: a retrospective nationwide study from the Netherlands. Plast
Reconstr Surg. 2005;116:1902-1909.
2. Bett BJ. Large or multiple congenital melanocytic nevi: occurrence of cutaneous
melanoma in 1008 persons. J Am Acad Dermatol. 2005;52:793-797.

This examination contains test materials that are owned and copyrighted by the American Society of
Plastic Surgeons and the Plastic Surgery Educational Foundation. Any reproduction of these materials or
any part of them, through any means, including but not limitied to, copying or printing electronic files,
reconstruction through memorization or dictation, and/or dissemination of these materials or any part of
them is strictly prohibited. Keep printed materials in a secure location when you are not reviewing them
and discard them in a secure manner, such as shredding, when you have completed the examination.

Page 33 of 287
American Society of Plastic Surgeons and the Plastic Surgery Educational Foundation
In-Service Examination

19. A 37-year-old man is brought to the emergency department because he has pain,
paresthesia, and difficulty extending the fingers of the right hand one hour after he
sustained a high-voltage electrical injury. Physical examination shows burns over
20% of the total body surface area with an entrance wound in the right forearm and
an exit wound in the left lower abdomen. Which of the following is the most
appropriate management?
A) Decompressive fasciotomy of the right forearm
B) Elevation and splinting of the right upper extremity
C) Infusion of calcium gluconate
D) Serial physical examinations of the right upper extremity over eight hours
E) Tangential excision of burned tissue and coverage with allografts

The correct response is Option A.


Compartment pressures greater than 30 mmHg, worsening paresthesia, pain on passive range of
motion, or pulselessness in an extremity mandate immediate decompression as part of acute
resuscitation after electrical injury. Fasciotomies should also include decompression of the
median and ulnar nerves at the level of the wrist, as well as decompression of the pronator
quadratus, which often sustains significant collateral injury, given its proximity to bone, which
has higher resistance than the soft tissue and generates greater heat than more superficial
structures. Decompression of the ulnar nerve at the level of the cubital tunnel should also be
considered.
Elevation and splinting are helpful adjuncts after burn injury to an extremity, but these measures
should not delay the definitive management of compartment syndrome, which is fasciotomy.
Infusion of calcium gluconate is indicated after hydrofluoric acid chemical burns.
Although serial physical examination is a guiding diagnostic principle of surgery, this approach
is contraindicated in a patient who meets criteria for compartment syndrome and requires
fasciotomy.
Although wide excision of burned tissue is desirable, it is often difficult to appreciate the full
magnitude of tissue loss, arguing against immediate excision and definitive coverage. Staged
excision, combined with temporary coverage, is acceptable after distal perfusion has been
restored.
References

1. Luce E. Electrical burns. Clin Plast Surg. 2000;27:133-143.


2. Graves TA, Cioffi WG, Vaughan GM, et al. The renal effects of low-dose dopamine
in thermally injured patients. J Trauma. 1993;35:97-102.

This examination contains test materials that are owned and copyrighted by the American Society of
Plastic Surgeons and the Plastic Surgery Educational Foundation. Any reproduction of these materials or
any part of them, through any means, including but not limitied to, copying or printing electronic files,
reconstruction through memorization or dictation, and/or dissemination of these materials or any part of
them is strictly prohibited. Keep printed materials in a secure location when you are not reviewing them
and discard them in a secure manner, such as shredding, when you have completed the examination.

Page 34 of 287
American Society of Plastic Surgeons and the Plastic Surgery Educational Foundation
In-Service Examination

20. A 59-year-old man comes to the office for follow-up examination three weeks after
undergoing left total knee replacement. Physical examination shows a draining
sinus on the distal aspect of the knee. Surgical removal of the hardware and flap
reconstruction are planned. Which of the following is the dominant pedicle to the
muscle flap that is most appropriate for reconstruction?
A) Anterior tibial artery
B) Ascending branch of the medial circumflex femoral artery
C) Medial sural artery
D) Perforating branch of the distal superficial femoral artery
E) Proximal branches of the posterior tibial artery

The correct response is Option C.

The muscle flap best suited to reconstruct this defect is a medial head of gastrocnemius muscle
rotation flap. Its dominant pedicle is the medial sural artery, which is a branch of the popliteal
artery.

The ascending branch of the medial circumflex artery is the dominant pedicle to the gracilis flap,
whereas the minor pedicles are perforating branches of the distal superficial femoral artery. A
proximally based gracilis flap would not have the arc of rotation to adequately cover the knee. A
distally based flap would require a delay procedure and is not the best choice for this defect.

The lateral sural artery is the dominant blood supply to the lateral head of the gastrocnemius,
which is smaller, and, therefore, not as well suited for coverage of large patellar defects such as
this one.

The soleus is based off the proximal two branches of the posterior tibial artery, branches of the
proximal popliteal, and branches of the peroneal artery. It is used for reconstruction of defects of
the middle third of the lower extremity and does not have an arc of rotation sufficient enough to
cover this defect.

References

1. Mathes SJ, Nahai F, eds. Reconstructive Surgery: Principles, Anatomy, and


Technique. New York: Churchill Livingstone; 1997.
2. Nahabedian MY, Mont MA, Orlando JC, et al. Operative management and outcome
of complex wounds following total knee arthroplasty. Plast Reconstr Surg.
1999;104:1688.

This examination contains test materials that are owned and copyrighted by the American Society of
Plastic Surgeons and the Plastic Surgery Educational Foundation. Any reproduction of these materials or
any part of them, through any means, including but not limitied to, copying or printing electronic files,
reconstruction through memorization or dictation, and/or dissemination of these materials or any part of
them is strictly prohibited. Keep printed materials in a secure location when you are not reviewing them
and discard them in a secure manner, such as shredding, when you have completed the examination.

Page 35 of 287
American Society of Plastic Surgeons and the Plastic Surgery Educational Foundation
In-Service Examination

21. A 49-year-old woman with a 20-year history of paraplegia comes to the office for
consultation regarding a chronic ulcer of the right ischium. She has been treating
the wound twice daily with saline-soaked gauze dressings. Wound drainage has
increased and become foul-smelling during the past week. The patient has smoked
one pack of cigarettes daily for the past 15 years. Height is 5 ft 1 in and weight is
92 lb. The patient breathes comfortably on room air, is afebrile, and is not in acute
distress. Physical examination of the ischial region shows a 5-cm-deep wound that
is necrotic and extends to the ischial bone. Hip flexion contractures are noted.
Culture of the wound shows gram-positive cocci and gram-negative rods.
Laboratory studies show the following:

Hemoglobin 9.1 g/dL


Hematocrit 27%
Leukocyte count 12.5/mm3
Serum albumin 2.4 g/dL
Urine:
Leukocytes Many
Leukoesterase Positive

Which of the following is the most appropriate next step in management?


A) Bone scan
B) Debridement and closure of the wound with a muscle flap
C) Excision of the ulcer
D) In-office debridement and application of collagenase dressings
E) Intravenous antibiotic therapy

The correct response is Option C.

The patient described has an ischial decubitus ulcer resulting from prolonged pressure in the
sitting position. The wound necrosis, elevated white blood cell count, and Gram stain merit
immediate surgical intervention. The best immediate surgical option is complete ulcer excision
including the underlying bursa and exposed ischium. Bone should be resected until firm,
bleeding cortex is encountered and debrided bone should be sent for culture. The extension of the
ulcer to bone indicates clinical osteomyelitis, and a bone scan will not change management in
this patient.

This examination contains test materials that are owned and copyrighted by the American Society of
Plastic Surgeons and the Plastic Surgery Educational Foundation. Any reproduction of these materials or
any part of them, through any means, including but not limitied to, copying or printing electronic files,
reconstruction through memorization or dictation, and/or dissemination of these materials or any part of
them is strictly prohibited. Keep printed materials in a secure location when you are not reviewing them
and discard them in a secure manner, such as shredding, when you have completed the examination.

Page 36 of 287
American Society of Plastic Surgeons and the Plastic Surgery Educational Foundation
In-Service Examination

Office debridement and chemical debridement dressings are not sufficient treatments for a
wound of this severity. Although culture-specific antibiotics are an essential part of treatment,
this patient's clinical status will not improve without debridement of all nonviable tissues. This
patient has multiple risk factors for healing problems and ulcer recurrence, including
malnutrition, systemic infection, smoking, and contractures that will inhibit rehabilitation and
positioning. A single-stage debridement and flap reconstruction can be performed in optimal
patients. Considering this patient's high risk of recurrence, debridement should be followed by
wound care, antibiotics, nutritional optimization, smoking cessation, and physical therapy to
improve her hip contractures. After her overall health status improves, flap reconstruction will
have a higher chance of success. Recurrence of ulceration is extremely common over the long
term, exceeding 50%. Having a patient willing to make the lifestyle changes and sacrifices
necessary to avoid excess pressure and maintain optimal health cannot be overemphasized as the
key to long-term success.

References

1. Foster R. Pressure sores. In: Mathes S, ed. Plastic Surgery. 2nd ed. Philadelphia:
W.B. Saunders; 2006:1317-1353.
2. Riou JP, Cohen JR, Johnson H. Factors influencing wound dehiscence. Am J Surg.
1992;163:324-330.
3. Conway H, Griffith BH. Plastic Surgery for closure of decubitus ulcers in patients
with paraplegia: based on experience with 1,000 cases. Am J Surg. 1956;91:946-975.
4. Lewis VL, Bailey MH, Pulawski G, et al. The diagnosis of osteomyelitis in patients
with pressure sores. Plast Reconstr Surg. 1988;81:229-232.

This examination contains test materials that are owned and copyrighted by the American Society of
Plastic Surgeons and the Plastic Surgery Educational Foundation. Any reproduction of these materials or
any part of them, through any means, including but not limitied to, copying or printing electronic files,
reconstruction through memorization or dictation, and/or dissemination of these materials or any part of
them is strictly prohibited. Keep printed materials in a secure location when you are not reviewing them
and discard them in a secure manner, such as shredding, when you have completed the examination.

Page 37 of 287
American Society of Plastic Surgeons and the Plastic Surgery Educational Foundation
In-Service Examination

22. A 37-year-old African American woman comes to the office for consultation
regarding a 5-cm nodule on the posterior aspect of the left earlobe that has been
enlarging over the past three months. There is no history of trauma. Which of the
following are the most likely biologic and morphologic characteristics of this
patient's lesion?

Fibroblast Myofibroblast Blood Vessel


Proliferation Status Density
A) Increased Absent Decreased
B) Increased Present Increased
C) Normal Absent Increased
D) Normal Present Decreased
E) Normal Present Increased

The correct response is Option A.

The patient described has an earlobe keloid. The distinction between hypertrophic and keloid
scars is often difficult to make based on clinical features. Keloids are more frequently associated
with more darkly pigmented skin and are commonly seen on earlobes or in the deltoid or
presternal region.

Keloids and hypertrophic scars can also be difficult to differentiate histopathologically and
biologically. Recent advances in cellular biology have identified several key differences between
the two.

The predominant cell of scar tissue is the fibroblast, which is responsible for producing
collagens. Various in vitro studies have shown that fibroblasts cultured from keloids have
increased proliferation rates and decreased apoptosis when compared with fibroblasts cultured
from both hypertrophic scars as well as from normal skin.

While morphologic and immunohistochemical changes occur within a scar as it matures,


immature hypertrophic scars express an increased density of blood vessels, whereas keloid scars
show a decreased density of blood vessels. Myofibroblasts are present in hypertrophic scars but
absent in keloids, which in general, are far less cellular.

Both transforming growth factor (TGF)-β1 and TGF-β2 have been shown to be expressed in
greater levels in fibroblasts from both keloids and hypertrophic scars when compared with those
from normal skin.

This examination contains test materials that are owned and copyrighted by the American Society of
Plastic Surgeons and the Plastic Surgery Educational Foundation. Any reproduction of these materials or
any part of them, through any means, including but not limitied to, copying or printing electronic files,
reconstruction through memorization or dictation, and/or dissemination of these materials or any part of
them is strictly prohibited. Keep printed materials in a secure location when you are not reviewing them
and discard them in a secure manner, such as shredding, when you have completed the examination.

Page 38 of 287
American Society of Plastic Surgeons and the Plastic Surgery Educational Foundation
In-Service Examination

References

1. Burd A, Huang L. Hypertrophic response and keloid diathesis: two very different
forms of scar. Plast Reconstr Surg. 2005;116:150e-157e.
2. Luo S, Benathan M, Raffoul W, et al. Abnormal balance between proliferation and
apoptotic cell death in fibroblasts derived from keloid lesions. Plast Reconstr Surg.
2001;107:87-97.

This examination contains test materials that are owned and copyrighted by the American Society of
Plastic Surgeons and the Plastic Surgery Educational Foundation. Any reproduction of these materials or
any part of them, through any means, including but not limitied to, copying or printing electronic files,
reconstruction through memorization or dictation, and/or dissemination of these materials or any part of
them is strictly prohibited. Keep printed materials in a secure location when you are not reviewing them
and discard them in a secure manner, such as shredding, when you have completed the examination.

Page 39 of 287
American Society of Plastic Surgeons and the Plastic Surgery Educational Foundation
In-Service Examination

(Please note that this pictorial appears in color in the online examination)

23. A 27-year-old woman with a history of intravenous drug use has a persistent
abscess and cellulitis of the dorsal aspect of the left hand. A photograph of the
hand is shown. Three days ago, incision and drainage were performed using local
anesthesia in the emergency department, and the patient was then admitted to the
hospital for intravenous administration of piperacillin with tazobactam. Results of
culture of the wound tissue are pending. Current physical examination shows that
the cellulitis has not resolved satisfactorily. Which of the following is the most likely
causative organism of this infection?
A) Bacteroides fragilis
B) Escherichia coli
C) Group B streptococcus
D) Pseudomonas aeruginosa
E) Staphylococcus aureus

The correct response is Option E.

The infection described is most likely caused by a subset of Staphylococcus aureus, which is
methicillin-resistant. Although methicillin-resistant S. aureus (MRSA) has traditionally been

This examination contains test materials that are owned and copyrighted by the American Society of
Plastic Surgeons and the Plastic Surgery Educational Foundation. Any reproduction of these materials or
any part of them, through any means, including but not limitied to, copying or printing electronic files,
reconstruction through memorization or dictation, and/or dissemination of these materials or any part of
them is strictly prohibited. Keep printed materials in a secure location when you are not reviewing them
and discard them in a secure manner, such as shredding, when you have completed the examination.

Page 40 of 287
American Society of Plastic Surgeons and the Plastic Surgery Educational Foundation
In-Service Examination

iatrogenic in its etiology, community-acquired MRSA is becoming more prevalent. In the


Emergency Department population, the proportion of soft tissue infections presenting with
MRSA is increasing. MRSA is not sensitive to penicillin-related antibiotics such as
piperacillin/tazobactam and typically requires vancomycin or linezolid for effective treatment.
Older antibiotics such as clindamycin and trimethoprim-sulfamethoxazole may also be effective
in treating MRSA. MRSA takes longer than the other bacteria listed to grow out in culture.

Bacteroides fragilis, Escherichia coli, group B streptococcus, and Pseudomonas aeruginosa are
all common bacteria found in abscesses of the hand; however, they are all sensitive to
penicillin-related antibiotics such as piperacillin/tazobactam.

References

1. Digenis A, Beasley RW. Infections of the upper limb. In: Aston SJ, Beasley RW,
Thorne CH, eds. Grabb and Smith's Plastic Surgery. 5th ed. Philadelphia:
Lippincott-Raven; 1997:901-910.
2. Ellis MW, Lewis JS 2nd. Treatment approaches for community-acquired
methicillin-resistant staphylococcus aureus infections. Curr Opin Infect Dis.
2005;18:496-501.
3. Iyer S, Jones DH. Community-acquired methicillin-resistant Staphylococcus aureus
skin infection: a retrospective analysis of clinical presentation and treatment of a local
outbreak. J Am Acad Dermatol. 2004;50:854-858.

This examination contains test materials that are owned and copyrighted by the American Society of
Plastic Surgeons and the Plastic Surgery Educational Foundation. Any reproduction of these materials or
any part of them, through any means, including but not limitied to, copying or printing electronic files,
reconstruction through memorization or dictation, and/or dissemination of these materials or any part of
them is strictly prohibited. Keep printed materials in a secure location when you are not reviewing them
and discard them in a secure manner, such as shredding, when you have completed the examination.

Page 41 of 287
American Society of Plastic Surgeons and the Plastic Surgery Educational Foundation
In-Service Examination

24. A 56-year-old woman has a 1.6-cm full-thickness defect of the alar rim after
undergoing Mohs micrographic surgery of the nose because of skin cancer. Which
of the following flaps is most appropriate for reconstruction of this defect?
A) Axial frontonasal (Rieger) flap
B) Helical root free flap
C) Nasolabial flap
D) Paramedian forehead flap
E) Slide-swing lateral nasal wall flap

The correct response is Option C.

Although all of the approaches listed have been described for nasal reconstruction, a two-stage
nasolabial flap provides the best reconstruction, in terms of form and function, of this 1.6-cm alar
rim defect. An axial frontonasal flap, first described by Rieger and later modified by Marchac
and Toth, is an excellent option for midline dorsal defects less than 2.0 cm, but this flap will not
easily reach the most lateral portion of the alar rim. Although a single-stage nasolabial flap is
technically possible, the base remains bulky and the cheek-sidewall junction is flattened. Finally,
both the paramedian forehead flap and helical root free flap can be used for this defect but are
considerably more complex procedures than the nasolabial flap and are not justified. The slide-
swing flap would not provide adequate tissue for closure of the defect.

References

1. Marchac D, Toth B. The axial frontonasal flap revisited. Plast Reconstr Surg.
1985;76:686-694.
2. Pribaz JJ, Falco N. Nasal reconstruction with auricular microvascular transplant. Ann
Plast Surg. 1993;31:289-297.
3. Burget GC, Menick FJ. Aesthetic reconstruction of the nose. St. Louis: Mosby;
1994:57-116.

This examination contains test materials that are owned and copyrighted by the American Society of
Plastic Surgeons and the Plastic Surgery Educational Foundation. Any reproduction of these materials or
any part of them, through any means, including but not limitied to, copying or printing electronic files,
reconstruction through memorization or dictation, and/or dissemination of these materials or any part of
them is strictly prohibited. Keep printed materials in a secure location when you are not reviewing them
and discard them in a secure manner, such as shredding, when you have completed the examination.

Page 42 of 287
American Society of Plastic Surgeons and the Plastic Surgery Educational Foundation
In-Service Examination

25. A 37-year-old woman who underwent full-face resurfacing with a carbon dioxide
laser seven days ago comes to the office because she has chills, malaise, and
painful lesions of the face. Temperature is 38.7°C (101.7°F). Physical examination
shows ulcerated, diffusely distributed lesions over the face. A four-day regimen of
acyclovir 400 mg three times daily has not alleviated the patient’s symptoms.
Results of Tzanck smear and viral culture are pending. Which of the following
therapies is the most appropriate next step in management?
A) Intravenous acyclovir and ciprofloxacin
B) Oral acyclovir at a higher dose
C) Oral acyclovir at a more frequent dose
D) Oral valacyclovir
E) Oral valacyclovir and ciprofloxacin

The correct response is Option A.


The patient described has a presumptive diagnosis of cutaneous herpes simplex. Herpes simplex
commonly is activated by laser resurfacing, spreads readily, and may scar. To avoid this
complication, antiviral prophylaxis with one of the established drugs (acyclovir [Zovirax],
famciclovir [Famvir], or valacyclovir [Valtrex]) should be started 24 to 48 hours before a laser
resurfacing procedure and continued for 10 days until re-epithelialization is complete.
Herpes simplex is difficult to recognize on resurfaced skin. Moreover, primary or secondary
bacterial infection cannot be overlooked. Therefore, antibiotic coverage is prudent, particularly
until culture results are available.
When a patient experiences an outbreak of herpes simplex despite the use of antiviral
prophylaxis, the physician should perform a Tzanck smear and send viral, fungal, and bacterial
cultures. The anti-herpetic dosage should be increased to zoster levels as well (acyclovir 800 mg
five times a day, valacyclovir 500 mg three times a day).
This patient presented with signs and symptoms of systemic herpetic infection, which may
include disseminated cutaneous lesions, shortness of breath, high fever and chills, malaise,
headache, and neurologic changes. When these are present, hospital-based treatment with
intravenous antiviral and antibiotic therapy is warranted.
References

1. Gregory RD, Alster TS, Mayl N, et al. Complications in laser resurfacing. Aesth Surg
J. 2000;20:232-237.
2. Weinstein C, Roberts TL. Aesthetic skin resurfacing with the high energy ultrapulsed
CO2 laser. Clin Plast Surg. 1997;24:379-405.

This examination contains test materials that are owned and copyrighted by the American Society of
Plastic Surgeons and the Plastic Surgery Educational Foundation. Any reproduction of these materials or
any part of them, through any means, including but not limitied to, copying or printing electronic files,
reconstruction through memorization or dictation, and/or dissemination of these materials or any part of
them is strictly prohibited. Keep printed materials in a secure location when you are not reviewing them
and discard them in a secure manner, such as shredding, when you have completed the examination.

Page 43 of 287
American Society of Plastic Surgeons and the Plastic Surgery Educational Foundation
In-Service Examination

26. A healthy 7-year-old boy with anotia undergoes reconstruction of the right ear via
implantation of a high-density porous polyethylene prosthesis. Compared with use
of autologous cartilage, which of the following is the most likely result of this
procedure?
A) Higher incidence of contour deformities
B) Higher incidence of extrusion
C) Increased likelihood of malposition
D) Lower incidence of infection
E) More resorption of the implant over time

The correct response is Option B.

High-density porous polyethylene (Medpor) is an alloplastic implant material that is


nonresorbable and highly biocompatible. It has an intramaterial porosity with a pore size
between 125 and 250 μm, which permits extensive fibrovascular ingrowth throughout the
implant. Although limited bony ingrowth may occur in select clinical circumstances, the material
should not be considered truly osteoconductive. This bony and fibrovascular ingrowth can make
this type of material somewhat difficult to remove. Studies have shown less underlying bone
resorption occurs with high-density polyethylene than with other implant materials. As an
alloplastic material, it should not be placed in contaminated wound beds, because it can get
easily infected, and should always be positioned in areas of adequate soft tissue cover to prevent
extrusion. The incidence of extrusion is higher in microtia reconstruction and nasal dorsum
augmentation for this reason.

References

1. Rubin JP, Yaremchuk MJ. Complications and toxicities of implantable biomaterials


used in facial reconstructive and aesthetic surgery: a comprehensive review of the
literature. Plast Reconstr Surg. 1997;100:1336.
2. Eppley BL. Alloplastic implantation. Plast Reconstr Surg. 1999;104:1761.
3. Weinzweig J, Pantaloni M, Spangenberger A, et al. Osteochondral reconstruction of a
non-weight-bearing joint using a high-density porous polyethylene implant. Plast
Reconstr Surg. 2000;106:1547.
4. Bikhazi HB, van Antwerp R. The use of Medpor in cosmetic and reconstructive
surgery: experimental and clinical evidence. In: Stucker F, ed. Plastic and
Reconstructive Surgery of the Head and Neck. St. Louis: Mosby; 1990:271.

This examination contains test materials that are owned and copyrighted by the American Society of
Plastic Surgeons and the Plastic Surgery Educational Foundation. Any reproduction of these materials or
any part of them, through any means, including but not limitied to, copying or printing electronic files,
reconstruction through memorization or dictation, and/or dissemination of these materials or any part of
them is strictly prohibited. Keep printed materials in a secure location when you are not reviewing them
and discard them in a secure manner, such as shredding, when you have completed the examination.

Page 44 of 287
American Society of Plastic Surgeons and the Plastic Surgery Educational Foundation
In-Service Examination

(Please note that this pictorial appears in color in the online examination)

27. A 38-year-old man undergoes extensive debridement of skin, subcutaneous tissue,


and fascia on the right chest wall (shown) for progressive Type II monomicrobial
necrotizing fasciitis. Results of culture are pending. In combination with penicillin,
which of the following antibiotics is the most appropriate initial therapy?
A) Ciprofloxacin
B) Clindamycin
C) Gentamicin
D) Metronidazole
E) Vancomycin

The correct response is Option B.

Necrotizing fasciitis is a severe form of subcutaneous infection that tracks along fascial planes
often extending beyond the superficial signs of infections. Early diagnosis and treatment are
critical in reducing mortality. Early surgical intervention is often required for diagnosis and
treatment.

Initial antibiotic therapy is directed at the most likely pathogens. Two clinical subtypes of

This examination contains test materials that are owned and copyrighted by the American Society of
Plastic Surgeons and the Plastic Surgery Educational Foundation. Any reproduction of these materials or
any part of them, through any means, including but not limitied to, copying or printing electronic files,
reconstruction through memorization or dictation, and/or dissemination of these materials or any part of
them is strictly prohibited. Keep printed materials in a secure location when you are not reviewing them
and discard them in a secure manner, such as shredding, when you have completed the examination.

Page 45 of 287
American Society of Plastic Surgeons and the Plastic Surgery Educational Foundation
In-Service Examination

necrotizing fasciitis have been identified. Type I is a polymicrobial infection caused by aerobic
and anaerobic bacteria occurring primarily in patients who are immunocompromised or have
certain chronic diseases like diabetes.

Type II is a monomicrobial infection that can occur in healthy individuals in any age group.
Often, the individual has a history of blunt trauma, penetrating injuries or lacerations (often
minor), surgical procedures, childbirth, or burns.

In Type II fasciitis, the most common causative organisms are group A streptococci.
Clindamycin and penicillin in combination are recommended. Clindamycin has been shown to
suppress toxin production, whereas metronidazole has not. Clindamycin also facilitates
phagocytosis of Streptococcus pyogenes. Penicillin is added because of increasing resistance of
group A streptococci to clindamycin.

Infection with Staphylococcus aureus is a less common cause of Type II necrotizing fasciitis.
Oxacillin could be used to treat susceptible S. aureus infection, whereas vancomycin is
appropriate therapy for methicillin-resistant S. aureus (MRSA).

Type I necrotizing fasciitis requires a multiple antibiotic regimen to cover aerobic and anaerobic
bacteria. Therefore, broad-spectrum coverage with ampicillin-sulbactam, clindamycin,
ciprofloxacin, or gentamicin is recommended.

References

1. Stevens DL, Bisno AL, Chambers HF, et al. Practice guidelines for the diagnosis and
management of skin and soft-tissue infections. Clin Infect Dis. 2005;41:1373-1406.
2. Wong CH, Wang YS. The diagnosis of necrotizing fasciitis. Curr Opin Infect Dis.
2005;18:101-106.

This examination contains test materials that are owned and copyrighted by the American Society of
Plastic Surgeons and the Plastic Surgery Educational Foundation. Any reproduction of these materials or
any part of them, through any means, including but not limitied to, copying or printing electronic files,
reconstruction through memorization or dictation, and/or dissemination of these materials or any part of
them is strictly prohibited. Keep printed materials in a secure location when you are not reviewing them
and discard them in a secure manner, such as shredding, when you have completed the examination.

Page 46 of 287
American Society of Plastic Surgeons and the Plastic Surgery Educational Foundation
In-Service Examination

28. A 45-year-old woman comes to the office for consultation regarding multiple actinic
keratoses on the face. Physical examination shows fair-skinned complexion and
small scaly patches on the face and ears. Which of the following interventions will
result in the most desirable long-term aesthetic appearance in this patient?
A) Cryosurgery
B) Electrodesiccation and curettage
C) Excision of the lesions with 2-mm margins
D) Microdermabrasion
E) Topical application of 0.5% fluorouracil cream

The correct response is Option E.

Several formulations and concentrations of topical fluorouracil have received U.S. Food & Drug
Administration (FDA) approval for the treatment of keratotic lesions. Fluorouracil 5% and 0.5%
creams have demonstrated, in clinical trials, a marked ability to eradicate keratotic lesions. The
cosmetic result is often far better than surgical excision. Irritation at the application site,
erythema, and burning are common side effects of both formulations, but comparative data
suggest that the fluorouracil 0.5% cream is more cost-effective and may be safer, more tolerable,
and as effective as fluorouracil 5% cream. Actinic keratosis is a common sun-induced skin
disease. Recent molecular studies indicate an association between actinic keratosis and squamous
cell carcinoma. Although 60% of squamous cell carcinoma cases begin as actinic keratosis, the
risk of progression to squamous cell carcinoma is minimal. Other methods of treatment such as
cryosurgery, topical trichloracetic acid, and curettage should be used with caution because
hypopigmentation with scarring may result. Microdermabrasion is not an acceptable treatment
for actinic keratosis.

References

1. Jorizzo JL, Carney PS, Ko WT, et al. Treatment options in the management of actinic
keratosis. Cutis. 2004;74:9-17.
2. Spencer JM, Hazan C, Hsiung SH, et al. Therapeutic decision making in the therapy
of actinic keratoses. J Drugs Dermatol. 2005;4:296-301.

This examination contains test materials that are owned and copyrighted by the American Society of
Plastic Surgeons and the Plastic Surgery Educational Foundation. Any reproduction of these materials or
any part of them, through any means, including but not limitied to, copying or printing electronic files,
reconstruction through memorization or dictation, and/or dissemination of these materials or any part of
them is strictly prohibited. Keep printed materials in a secure location when you are not reviewing them
and discard them in a secure manner, such as shredding, when you have completed the examination.

Page 47 of 287
American Society of Plastic Surgeons and the Plastic Surgery Educational Foundation
In-Service Examination

29. Which of the following injectable fillers has the highest concentration of hyaluronic
acid per volume of injectate?
A) CosmoDerm
B) CosmoPlast
C) Radiesse
D) Restylane
E) Sculptra

The correct response is Option D.

Restylane is a U.S. Food & Drug Administration (FDA)-approved non-animal-stabilized


hyaluronic acid derivative used for soft-tissue augmentation. Unlike Hylaform gel, it is derived
from streptococcal bacterial fermentation and does not require an animal source. At 20 mg/ml,
Restylane has a higher concentration of hyaluronic acid than Hylaform gel. It is used to treat
rhytides and scars and in lip augmentation. Restylane correction was noted to be 82% at three
months and 33% at one year in a study involving 285 wrinkles treated in 113 patients.

Hylaform gel, previously known as Hylan B gel, is a form of cross-linked hyaluronic acid (5.5
mg/ml) derived from the rooster combs of domestic fowl. It is reported to be less immunogenic
and longer lasting than bovine collagen. One study showed 86% histologic persistence of
Hylaform gel compared with 25% of bovine collagen in guinea pigs at 26 weeks. Besides being
longer lasting than collagen, proponents say it has less risk of clumping and goes in more
smoothly. Overcorrection is not needed with this product.

CosmoDerm and CosmoPlast are FDA-approved fillers consisting of natural human collagen
grown under controlled laboratory conditions. Pretreatment skin tests are not necessary, and the
materials are screened for purity. Both products contain 35 mg/ml of human-derived collagen in
phosphate-buffered physiologic saline containing 0.3% lidocaine. CosmoDerm is not
cross-linked and is used to treat superficial lines and wrinkles, whereas CosmoPlast is
cross-linked with glutaraldehyde and can be used for deeper wrinkles.

Radiesse is composed of microspheres of calcium hydroxyl apatite suspended in an aqueous gel


carrier. These biodegradable microspheres serve as a lattice upon which the body forms a
scaffold for tissue infiltration. The spheres degrade slowly over years for a longer-lasting,
semipermanent effect.

Sculptra is a poly-L-lactic acid that is FDA approved for management of HIV facial lipoatrophy.
It serves as a volume enhancer and is used for indications similar to those for autologous fat
transfer. Results are not immediate; treatment is performed as a series of three to five treatments

This examination contains test materials that are owned and copyrighted by the American Society of
Plastic Surgeons and the Plastic Surgery Educational Foundation. Any reproduction of these materials or
any part of them, through any means, including but not limitied to, copying or printing electronic files,
reconstruction through memorization or dictation, and/or dissemination of these materials or any part of
them is strictly prohibited. Keep printed materials in a secure location when you are not reviewing them
and discard them in a secure manner, such as shredding, when you have completed the examination.

Page 48 of 287
American Society of Plastic Surgeons and the Plastic Surgery Educational Foundation
In-Service Examination

approximately one month apart.

References

1. Klein AW, Elson ML. The history of substances for soft tissue augmentation.
Dermatol Surg. 2000;26:1096-1105.
2. Millikan LE. The evolution of dermal implants. Cosmet Dermatol. 2001;14:27-30.
3. Narins RS, Brandt F, Leyden J, et al. A randomized, double-blind, multicenter
comparison of the efficacy and tolerability of Restylane versus Zyplast for the
correction of nasolabial folds. Dermatol Surg. 2003;29:588-595.
4. Krauss M, Klein AW. Soft tissue augmentation. In: Kaminer MS, Dover JS, Arndt
KA, eds. Atlas of Cosmetic Surgery. Philadelphia: WB Saunders; 2002:264-290.

This examination contains test materials that are owned and copyrighted by the American Society of
Plastic Surgeons and the Plastic Surgery Educational Foundation. Any reproduction of these materials or
any part of them, through any means, including but not limitied to, copying or printing electronic files,
reconstruction through memorization or dictation, and/or dissemination of these materials or any part of
them is strictly prohibited. Keep printed materials in a secure location when you are not reviewing them
and discard them in a secure manner, such as shredding, when you have completed the examination.

Page 49 of 287
American Society of Plastic Surgeons and the Plastic Surgery Educational Foundation
In-Service Examination

30. A 17-year-old girl comes to the office because she has had lesions on the upper
arms and in the axillary folds for the past eight years. Physical examination shows
clusters of white, vesicular lesions ranging from 2 to 5 mm in diameter. The lesions
are obliterated with gentle pressure but refill when pressure is removed. Diffuse
swelling is palpable in the underlying subcutaneous tissue. Which of the following is
the most likely diagnosis?
A) Blue rubber bleb nevus syndrome
B) Lymphangioma circumscriptum
C) Maffucci syndrome
D) Osler-Weber-Rendu syndrome
E) Sebaceous nevus

The correct response is Option B.

Lymphangioma circumscriptum consists of clusters of small to moderate-sized, clear to


whitish-appearing vesicular lesions. These lesions can be obliterated with gentle pressure, but
they refill with the lymph when the pressure is removed due to the presence of communications
with larger underlying cisterns of lymphatic malformation. Clear vesicles may be present on
apparently normal skin or may top small papules. Although the classic lesion can occur
anywhere on the body, it is particularly common over proximal parts of limbs and adjacent parts
of the limb girdle including the upper arm, axillary, pectoral, and scapular regions. Adequate
treatment of lymphangioma circumscriptum usually involves excision of the involved skin as
well as the deeper lymphatic components.

Blue rubber bleb nevi usually present as raised blue to purple rubbery cutaneous lesions that are
easily compressible. This is a rare morphogenetic disorder consisting of malformed vascular
channels within the skin and bowel. When blood is expressed from the vesicles, the deflated
blister becomes a wrinkled sac. When pressure is released, the deflated blisters refill with blood.
The lesions sometime present at birth, but more frequently appear throughout adolescence.
Subtotal excisions of tender or painful skin lesions can be of symptomatic relief.

Maffucci syndrome classically presents as blood-like, spongy, papular, or pedunculated vascular


malformations in association with enchondromas. Visceral vascular malformations may also be
present, and the involved bones are usually shortened and deformed. Approximately 20% of the
enchondromas can degenerate into chondrosarcoma, which is usually manifest by increasing size
and pain of a particular lesion.

Osler-Weber-Rendu syndrome presents as malformed ectatic vessels in the skin, mucous

This examination contains test materials that are owned and copyrighted by the American Society of
Plastic Surgeons and the Plastic Surgery Educational Foundation. Any reproduction of these materials or
any part of them, through any means, including but not limitied to, copying or printing electronic files,
reconstruction through memorization or dictation, and/or dissemination of these materials or any part of
them is strictly prohibited. Keep printed materials in a secure location when you are not reviewing them
and discard them in a secure manner, such as shredding, when you have completed the examination.

Page 50 of 287
American Society of Plastic Surgeons and the Plastic Surgery Educational Foundation
In-Service Examination

membranes, and viscera appearing after puberty and multiplying with advancing age.
Hemorrhage from the lesions presents as epistaxis, hematemesis, hematuria, or melena.

Sebaceous nevi usually develop as firm, plaque-like, waxy-appearing yellowish lesions


developing on the scalp. These are potentially premalignant lesions. Generally, these lesions
should be excised, because there is an approximately 5% incidence of malignant degeneration,
particularly during adolescence. Hair growth in these lesions is sparse to absent.

References

1. Bauer BS, Kernahan DA, Hugo NE. Lymphangioma circumscriptum: a


clinicopathological review. Ann Plast Surg. 1981;7:318-326.
2. Peachey RDG, Lim CC, Whimster IW. Lymphangioma of the skin: a review of 65
cases. Br J Dermatol. 1970;83:519.
3. Williams HB. Vascular neoplasms. Clin Plast Surg. 1980;7:397.
4. Goldman MP, Bennett RG. Treatment of telangiectasia: a review. J Am Acad
Dermatol. 1987;17:167.
5. Stal S, Friedman J. Common pediatric plastic and reconstructive diagnoses. Clin Plast
Surg. 1998;25:509.

This examination contains test materials that are owned and copyrighted by the American Society of
Plastic Surgeons and the Plastic Surgery Educational Foundation. Any reproduction of these materials or
any part of them, through any means, including but not limitied to, copying or printing electronic files,
reconstruction through memorization or dictation, and/or dissemination of these materials or any part of
them is strictly prohibited. Keep printed materials in a secure location when you are not reviewing them
and discard them in a secure manner, such as shredding, when you have completed the examination.

Page 51 of 287
American Society of Plastic Surgeons and the Plastic Surgery Educational Foundation
In-Service Examination

31. A 27-year-old woman is scheduled to undergo rhinoplasty using homograft rib


cartilage for reconstruction of the dorsum of the nose. Which of the following
interventions during this procedure is most effective to reduce long-term warping of
the graft?
A) Access to a peripheral segment
B) Insertion of the graft at least 30 minutes after carving
C) Scoring of the graft
D) Suture fixation
E) Use of nonirradiated material

The correct response is Option B.

To minimize the long-term clinical effects of cartilage warping, it is recommended to wait at


least 30 minutes after carving the graft to allow initial warping to occur. The observed warping
can then be accounted for in the final graft placement. Further warping may continue for some
time, but the majority will occur within the first 30 to 60 minutes.

Use of nonirradiated graft material may predispose more warping than irradiated material;
however, this may depend on the dose of radiation. Doses of 3 to 4 million rads are less likely to
result in graft warping compared with no radiation, but at doses of 1.5 to 2.5 million rads, the
warping may be similar to that of nonirradiated grafts.

Central cuts of cartilage grafts are less likely to warp than are peripheral cuts. Scoring of the
graft will result in warping of the graft. Suture fixation will not prevent graft warping.

References

1. Strauch B, Wallach SG. Reconstruction with irradiated homograft costal cartilage.


Plast Reconstr Surg. 2003;111: 2405-2412.
2. Adams WP, Rohrich RJ, Gunter JP, et al. The rate of warping in irradiated and
nonirradiated homograft rib cartilage: A controlled comparison and clinical
implications. Plast Reconstr Surg. 1999;103:265-270.

This examination contains test materials that are owned and copyrighted by the American Society of
Plastic Surgeons and the Plastic Surgery Educational Foundation. Any reproduction of these materials or
any part of them, through any means, including but not limitied to, copying or printing electronic files,
reconstruction through memorization or dictation, and/or dissemination of these materials or any part of
them is strictly prohibited. Keep printed materials in a secure location when you are not reviewing them
and discard them in a secure manner, such as shredding, when you have completed the examination.

Page 52 of 287
American Society of Plastic Surgeons and the Plastic Surgery Educational Foundation
In-Service Examination

32. A 23-year-old woman with neurofibromatosis comes to the office for consultation
regarding lesions on the face and trunk that have appeared during the past 10
years. She is dissatisfied with the cosmetic appearance of a lesion on her nose and
is concerned about a lesion on the left hip that has been enlarging and is painful on
application of pressure. Physical examination shows multiple small, flesh-colored
masses on the face and trunk ranging from 5 to 35 mm in diameter. Punch biopsy
shows neurofibromas. Which of the following is the most appropriate management
of the lesions on the patient's nose and hip?
A) Injection of a corticosteroid
B) Nd:YAG laser resurfacing
C) Radiation therapy
D) Surgical excision
E) Observation only

The correct response is Option D.

Neurofibromatosis is an autosomal-dominant disorder that can affect the skin, soft tissues, bone,
and central nervous system. The treatment of isolated soft tissue and skin nodules is primarily
surgical. Lesions are composed of Schwann cells, fibroblasts, and mast cells. Malignant
transformation may occur in 3% to 15% of patients over their lifetime. Any lesions that grow
rapidly or cause pain should be excised or biopsied to exclude neurofibrosarcoma. Observation
is, therefore, not appropriate in this case. Laser resurfacing is not useful for lesions that originate
in the subcutaneous tissues. Injection of corticosteroids and radiation have no role in treatment.
In healthy patients, lesions may also be removed for cosmetic reasons.

This condition is known as von Recklinghausen's neurofibromatosis. The disease can range from
mild, as in this case, to severe and devastating deformity. Café-au-lait macules may develop in
early childhood. Other physical signs may include hamartomas of the iris (Lisch nodules) and
axillary freckling. Central nervous system lesions, such as acoustic neuromas and meningiomas,
may occur. Many patients have learning disabilities, and up to 10% may have mental retardation.
Pheochromocytomas are also more common in this patient population.

Multiple variants of the disease exist with neurofibromatosis type 1 (NF-1) primarily affecting
the peripheral nervous system and neurofibromatosis type 2 (NF-2) affecting the central nervous
system. The incidence is approximately 1 in 3000 live births for NF-1 and 1 in 50,000-120,000
live births for NF-2. It occurs with equal frequency across all races and among men and women.
Genetic mutations have been identified in the production of a protein, neurofibromin, which acts
as a tumor suppressor.

This examination contains test materials that are owned and copyrighted by the American Society of
Plastic Surgeons and the Plastic Surgery Educational Foundation. Any reproduction of these materials or
any part of them, through any means, including but not limitied to, copying or printing electronic files,
reconstruction through memorization or dictation, and/or dissemination of these materials or any part of
them is strictly prohibited. Keep printed materials in a secure location when you are not reviewing them
and discard them in a secure manner, such as shredding, when you have completed the examination.

Page 53 of 287
American Society of Plastic Surgeons and the Plastic Surgery Educational Foundation
In-Service Examination

References

1. Zarem H, Lowe N. Benign growths and generalized skin disorders. In: Aston SJ,
Beasley RW, Thorne CH, eds. Grabb and Smith's Plastic Surgery. 5th ed.
Philadelphia: Lippincott-Raven; 1997:150-151.
2. Fitzpatrick TB. Neurofibromatosis. In: Fitzpatrick TB, Johnson RA, Polano MK, eds.
Color Atlas and Synopsis of Clinical Dermatology: Common and Serious Diseases.
3rd ed. New York: McGraw-Hill; 1997:458-462.

This examination contains test materials that are owned and copyrighted by the American Society of
Plastic Surgeons and the Plastic Surgery Educational Foundation. Any reproduction of these materials or
any part of them, through any means, including but not limitied to, copying or printing electronic files,
reconstruction through memorization or dictation, and/or dissemination of these materials or any part of
them is strictly prohibited. Keep printed materials in a secure location when you are not reviewing them
and discard them in a secure manner, such as shredding, when you have completed the examination.

Page 54 of 287
American Society of Plastic Surgeons and the Plastic Surgery Educational Foundation
In-Service Examination

33. Which of the following laboratory results of fluid analysis is increased in chronic
wounds relative to acute wounds?
A) Growth factor level
B) Matrix deposition
C) Metalloproteinase level
D) Protease inhibitor level
E) Tissue oxygen tension

The correct response is Option C.

Chronic wounds have an interruption in the natural sequence of wound healing involving a
highly regulated cascade of events among many cell types, soluble factors, and matrix
components. The chronic wound microenvironment is characterized by an imbalance between
matrix-degrading enzymes and their inhibitors. Metalloproteinase levels are elevated relative to
acute wounds, resulting in extracellular matrix degradation.

In chronic wounds, the healing process is disrupted by a prolonged inflammatory phase.


Proinflammatory cytokines are elevated, which leads to an increase in protease activity and a
decrease in protease inhibitor and growth factor levels. This results in decreased matrix
deposition, which prevents epithelization and healing.

Tissue oxygen tension is abnormally low in the central aspect of chronic wounds.

References

1. Burns JL, Mancoll JS, Phillips LG. Impairments to wound healing. Clin Plast Surg.
2003;30:44-56.
2. Nwomeh BC, Yager DR, Cohen IK. Physiology of the chronic wound. Clin Plast
Surg. 1998;25:341-356.
3. Robson MC. Cytokine manipulation of the wound. Clin Plast Surg. 2003;30:57-65.

This examination contains test materials that are owned and copyrighted by the American Society of
Plastic Surgeons and the Plastic Surgery Educational Foundation. Any reproduction of these materials or
any part of them, through any means, including but not limitied to, copying or printing electronic files,
reconstruction through memorization or dictation, and/or dissemination of these materials or any part of
them is strictly prohibited. Keep printed materials in a secure location when you are not reviewing them
and discard them in a secure manner, such as shredding, when you have completed the examination.

Page 55 of 287
American Society of Plastic Surgeons and the Plastic Surgery Educational Foundation
In-Service Examination

34. A 14-year-old boy is brought to the emergency department by his parents one hour
after he had sudden onset of swelling, tenderness, and tension of a lymphatic
malformation on the right leg. The boy’s parents say that the lesion has been
present since 3 years of age and has been enlarging since that time. Which of the
following is the most appropriate initial management?
A) Administration of an antibiotic
B) Compression of the entire leg
C) Lymphovenous shunting
D) Pulsed-dye laser therapy
E) Surgical decompression

The correct response is Option A.

In a patient with a large lymphatic malformation who presents with sudden expansion of a lesion
along with tenderness and tenseness, the most likely diagnosis is infection. Although
prophylactic antibiotics are generally not indicated for patients with lymphatic malformations, it
is wise to prescribe a broad-spectrum antibiotic with instructions to administer it and see a
physician at the first signs of infection.

Although elevation of the extremity may be reasonable, there is no role for compression if
infection is ongoing. Laser therapy may be used to treat lymphatic vesicles on a mucosal surface
but will not treat the deeper malformations. In the case of acute infection, there is no role for
surgical decompression, and lymphatic malformations do not respond to shunting of the
lymphatic fluid.

References

1. Marler JJ, Mulliken JB. Vascular anomalies. In: Mathes SJ, Hentz VR, eds. Plastic
Surgery. Philadelphia: Elsevier-Saunders; 2006:19-68.
2. Marler JJ, Mulliken JB. Current management of hemangiomas and vascular
malformations. Clin Plast Surg. 2005;32:99-116.

This examination contains test materials that are owned and copyrighted by the American Society of
Plastic Surgeons and the Plastic Surgery Educational Foundation. Any reproduction of these materials or
any part of them, through any means, including but not limitied to, copying or printing electronic files,
reconstruction through memorization or dictation, and/or dissemination of these materials or any part of
them is strictly prohibited. Keep printed materials in a secure location when you are not reviewing them
and discard them in a secure manner, such as shredding, when you have completed the examination.

Page 56 of 287
American Society of Plastic Surgeons and the Plastic Surgery Educational Foundation
In-Service Examination

35. Alopecia results when which of the following changes in the hair growth cycle
occur?

Shortened Prolonged
A) Anagen phase Catagen phase
B) Anagen phase Telogen phase
C) Catagen phase Telogen phase
D) Telogen phase Anagen phase
E) Telogen phase Catagen phase

The correct response is Option B.

Balding occurs when the anagen phase is shortened and the telogen phase is prolonged.

Human hair undergoes a normal cycle of growth and rest characterized by three stages: anagen,
catagen, and telogen. The anagen (active) phase lasts 1000 days in men and two to five years
longer in women. At any one time, 85% to 90% of hairs are in the anagen phase. The catagen
(degradation) phase follows anagen and lasts several weeks. During this phase, the follicular
bulb atrophies and degrades. After this, the telogen (resting) phase begins and lasts two to four
months. At any given time, approximately 10% of hairs are in the telogen phase. On average, 50
to 100 telogen hairs fall out every day and are replaced with new growing hairs. Balding occurs
when the anagen (active) phase is shortened and the telogen (resting) phase is prolonged.

References

1. Orentreich N. Scalp hair replacement in man. In: Advances in Biology of Skin. Vol. 9.
New York: Pergamon Press; 1969.
2. Orentreich N, Durr NP. Biology of scalp hair growth. Clin Plast Surg. 1982;9:197.

This examination contains test materials that are owned and copyrighted by the American Society of
Plastic Surgeons and the Plastic Surgery Educational Foundation. Any reproduction of these materials or
any part of them, through any means, including but not limitied to, copying or printing electronic files,
reconstruction through memorization or dictation, and/or dissemination of these materials or any part of
them is strictly prohibited. Keep printed materials in a secure location when you are not reviewing them
and discard them in a secure manner, such as shredding, when you have completed the examination.

Page 57 of 287
American Society of Plastic Surgeons and the Plastic Surgery Educational Foundation
In-Service Examination

36. Which of the following is most appropriate in preparation of deep partial-thickness


burn injuries to the hand before skin grafting?
A) Application of topical enzymatic debriding agents
B) Delay of surgical intervention until natural separation of eschar occurs
C) Excision of the skin to the viable subcutaneous fat
D) Tangential excision until pinpoint bleeding occurs
E) Whirlpool hydrotherapy until indeterminate areas are declared

The correct response is Option D.

The best treatment of eschar in deep partial thickness or indeterminate burn injuries is early
tangential excision until pinpoint bleeding occurs.

Proper diagnosis, evaluation, stabilization, and resuscitation are required in every case. In
patients with multiple traumatic injuries, other injuries must be assessed and treated before
excision of the primary burn. After it has been established that no contraindications to immediate
primary excision exist, the patient is taken to the operating room, where all burn excisions,
tangential or full thickness, are performed with the patient under general anesthesia administered
by personnel experienced in burn injuries. Areas of obvious full-thickness burn are excised to the
level of viable fat or fascia using electrocautery.

Delay of surgical intervention until natural eschar separation has been shown to significantly
increase mortality, increase hospitalization, increase incidence of burn wound sepsis, and also
contribute to burn scar hypertrophy and contracture.

Early surgical excision into viable subcutaneous fat is not appropriate for partial-thickness or
indeterminate burns. It is, however, appropriate management of obvious full-thickness burns.

Deep partial-thickness eschars, and also areas of indeterminate burn, are treated by tangential
excision using a Goulian knife. Tangential excision continues in these areas until pinpoint
bleeding is seen. The points of bleeding are close together in the upper layers of the dermis,
known as the papillary dermis. Burn wounds at this level are partial-thickness injuries that heal
adequately with a biologic barrier material. If tangential excision proceeds to the reticular
dermis, which is in the deeper dermal layers, bleeding points become more widely separated. At
these deeper levels of tangential excision, the yellow hue of subcutaneous fat may become
visible. Tangential excision to this deeper layer is best treated with a biologic barrier material
followed by skin grafting.

Application of topical enzymatic debriding agents and whirlpool hydrotherapy are inappropriate

This examination contains test materials that are owned and copyrighted by the American Society of
Plastic Surgeons and the Plastic Surgery Educational Foundation. Any reproduction of these materials or
any part of them, through any means, including but not limitied to, copying or printing electronic files,
reconstruction through memorization or dictation, and/or dissemination of these materials or any part of
them is strictly prohibited. Keep printed materials in a secure location when you are not reviewing them
and discard them in a secure manner, such as shredding, when you have completed the examination.

Page 58 of 287
American Society of Plastic Surgeons and the Plastic Surgery Educational Foundation
In-Service Examination

for management of burn eschar.

References

1. Chamania S, Patidar GP, Dembani B, et al. A retrospective analysis of early excision


and skin grafting from 1993 to 1995. Burns. 1998;24:177-180.
2. Cope O, Langohr J, Moore F, et al. Expeditious care of full thickness burn wounds by
surgical excision and grafting. Ann Surg. 1947;125:1-22.
3. Engrav L, Heinbach D, Reus J, et al. Early excision and grafting versus non-operative
treatment of burns of indeterminate depth: a randomized prospective study. J Trauma.
1983;23:1001-1004.
4. Herndon D, Barrow R, Rutan R, et al. A comparison of conservative versus early
excision: therapies in severely burned patients. Ann Surg. 1989;209:547-552.
5. Muller M, Nicolai M, Wiggins R, et al. Modern treatment of burn wounds. In:
Herndon DN, ed. Total Burn Care. Philadelphia: WB Saunders; 1996:136.
6. Still JM, Law EJ. Primary excision of the burn wound. Clin Plast Surg.
2000;27:21-47.

This examination contains test materials that are owned and copyrighted by the American Society of
Plastic Surgeons and the Plastic Surgery Educational Foundation. Any reproduction of these materials or
any part of them, through any means, including but not limitied to, copying or printing electronic files,
reconstruction through memorization or dictation, and/or dissemination of these materials or any part of
them is strictly prohibited. Keep printed materials in a secure location when you are not reviewing them
and discard them in a secure manner, such as shredding, when you have completed the examination.

Page 59 of 287
American Society of Plastic Surgeons and the Plastic Surgery Educational Foundation
In-Service Examination

37. Compared with cortical bone, which of the following best characterizes autologous
cancellous bone grafts?
A) Effective in bridging defects larger than 6 cm
B) Greater structural strength
C) Less osteoconductive
D) Less osteoinductive
E) More readily revascularized and remodeled

The correct response is Option E.

Relative to cortical bone, cancellous bone grafts are more osteoconductive (the property of the
scaffold-like matrix to accommodate the ingrowth of new bone) and more osteoinductive (the
capacity to induce mesenchymal cells from the recipient bed to produce active osteoblasts).
Cancellous bone is more quickly revascularized, which usually occurs within two weeks of
grafting, whereas cortical bone can take up to two months to revascularize. Cancellous grafts are
also more easily remodeled. Cancellous bone grafts are ideal for bridging bone gaps of less than
5 to 6 cm, but suffer from a lack of structural rigidity until 6 to 12 months after grafting, when
they are generally as strong as cortical bone grafts.

References

1. Finkmeier CG. Bone grafting and bone-graft substitutes. J Bone Joint Surg.
2002;84A:454.
2. Burchardt H. Biology of bone transplantation. Orthop Clin North Am. 1987;18:187.

This examination contains test materials that are owned and copyrighted by the American Society of
Plastic Surgeons and the Plastic Surgery Educational Foundation. Any reproduction of these materials or
any part of them, through any means, including but not limitied to, copying or printing electronic files,
reconstruction through memorization or dictation, and/or dissemination of these materials or any part of
them is strictly prohibited. Keep printed materials in a secure location when you are not reviewing them
and discard them in a secure manner, such as shredding, when you have completed the examination.

Page 60 of 287
American Society of Plastic Surgeons and the Plastic Surgery Educational Foundation
In-Service Examination

38. The dominant pedicle of the gracilis muscle flap is located between the adductor
longus and which of the following muscles?
A) Adductor brevis
B) Adductor magnus
C) Pectineus
D) Sartorius
E) Semitendinosus

The correct response is Option A.

The dominant pedicle of the gracilis muscle flap is located between the adductor longus and
adductor brevis muscles. The pectineus, adductor magnus, sartorius, and semitendinosus muscles
are anatomically incorrect relative to the location of this pedicle.

Although the origin of the dominant vascular pedicle to the gracilis muscle flap may originate
from either the profundus femoris or medial circumflex femoral vessels, the course of this
vascular pedicle is consistent and runs between the adductor longus and adductor brevis muscles.
This vascular pedicle enters the gracilis muscle approximately 8 to 10 cm below the pubic
tubercle. The superficial position of the gracilis muscle, its thin, ribbon-like character, and also
its consistent vascular anatomy allow this muscle flap to remain one of the most useful muscles
in the field of microvascular surgery. Although the dominant vascular pedicle easily nourishes
the entire muscle, the distal third of the skin overlying the gracilis muscle does not receive
sufficient blood supply for reliable transfer.

References

1. Strauch V, Han-Ling Y. Atlas of Microvascular Surgery. New York: Thieme Medical


Publishers; 1993:166-173.
2. Manktelow RT, Zuker RM. Microvascular free gracilis muscle and musculocutaneous
flap. In: Strauch B, ed. Grabb's Encyclopedia of Flaps. Boston: Little Brown;
1990:1783.

This examination contains test materials that are owned and copyrighted by the American Society of
Plastic Surgeons and the Plastic Surgery Educational Foundation. Any reproduction of these materials or
any part of them, through any means, including but not limitied to, copying or printing electronic files,
reconstruction through memorization or dictation, and/or dissemination of these materials or any part of
them is strictly prohibited. Keep printed materials in a secure location when you are not reviewing them
and discard them in a secure manner, such as shredding, when you have completed the examination.

Page 61 of 287
American Society of Plastic Surgeons and the Plastic Surgery Educational Foundation
In-Service Examination

39. A 22-year-old man comes to the emergency department because he has a lump on
the distal volar forearm three weeks after he sustained a stab wound just proximal
to the wrist. He did not seek treatment at the time of the injury. Current physical
examination shows a warm 2 × 2-cm mass with surrounding prominent veins. A
thrill is palpated and a bruit is heard over the mass. A positive Branham sign is
noted. Which of the following is the most appropriate next step?
A) Comparative plethysmography with the contralateral limb
B) Ligation of feeding vessels without resection
C) Magnetic resonance angiogram of the affected wrist
D) Percutaneous biopsy of the mass
E) Observation
The correct response is Option C.
The patient described has an acquired arteriovenous fistula after penetrating trauma, which
represents an abnormal connection between an artery and vein. The affected area is usually
enlarged, warm, and has distended and often pulsatile superficial veins. A thrill can be palpated
over the fistula, and a bruit may be present. In severe cases, altered hemodynamics may cause
heart failure if a significant portion of the cardiac output is diverted through the fistula. The
Branham sign indicates slowing of the heart rate upon compression proximal to the arteriovenous
malformation.
The next appropriate step is a magnetic resonance angiogram of the affected wrist to delineate
the nature and extent of the fistula. It also helps rule out traumatic pseudoaneurysm and helps
plan for the subsequent surgical procedure.
Comparative plethysmography with the contralateral limb is typically used to quantify flow in an
entire limb, but assessing blood flow through localized areas is difficult.
Ligation of feeding vessels without surgical resection will result in rapid recruitment of collateral
vessels and is contraindicated.
Percutaneous biopsy is contraindicated in pulsatile lesions.
Observation in a case of an enlarging pulsatile lesion is not appropriate.

References

1. Turowski GA, Amjadi N, Sterling A, et al. Aneurysm of the radial artery following
blunt trauma to the wrist. Ann Plast Surg. 1997;38:527.
2. Jones NF, Brown EE, Khiabani KT. Iatrogenic arteriovenous fistula after
revascularization of the hand. J Reconstr Microsurg. 2005;21:283-286.

This examination contains test materials that are owned and copyrighted by the American Society of
Plastic Surgeons and the Plastic Surgery Educational Foundation. Any reproduction of these materials or
any part of them, through any means, including but not limitied to, copying or printing electronic files,
reconstruction through memorization or dictation, and/or dissemination of these materials or any part of
them is strictly prohibited. Keep printed materials in a secure location when you are not reviewing them
and discard them in a secure manner, such as shredding, when you have completed the examination.

Page 62 of 287
American Society of Plastic Surgeons and the Plastic Surgery Educational Foundation
In-Service Examination

40. A 57-year-old man comes to the office for follow-up examination eight days after he
underwent composite grafting from the right auricle for repair of a 1.4-cm alar
defect. He does not smoke cigarettes. Physical examination shows a 5-mm region
of yellow eschar in the center of the graft, which is cyanotic. Which of the following
is the most appropriate next step in management?
A) Application of ice compresses
B) Debridement of eschar and coverage with a local flap
C) Leech therapy
D) Continued observation
The correct response is Option D.
Auricular composite grafts are frequently used to reconstruct small defects of the alar rim.
Unlike skin grafts, composite grafts only interface with the recipient bed along their perimeter;
therefore, graft size should be no larger than 1.5 cm in diameter, so the center of the graft is
never more than 5–8 mm from a blood supply.
The clinical appearance of a healing composite graft follows a reproducible pattern. The graft
can look very white for the first six to eight hours. A slight pink tinge thereafter signals the
earliest stage of revascularization. Within one to two days, the graft should appear cyanotic
secondary to venous congestion, and this can persist for approximately one week. The graft will
ultimately become pink and then more red as revascularization intensifies. The redness will fade,
and the tissue acquires its final tone within six months to one year.
A lingering whitish area in the center of a cyanotic graft most likely represents impending
necrosis. However, even if eschar develops, it should not be debrided. It may only be superficial,
with the underlying dermis and cartilage still viable. Removing the eschar may dislodge the
surviving portion of the graft and also places it at increased risk for infection. The eschar will
likely slough within three to four weeks. Therefore, continued observation is the most prudent
initial step in management.
Postoperative cooling is recommended to decrease the metabolic rate of the grafted tissue until
secondary revascularization has taken place. This therapy would likely be ineffective one week
after surgery and may compromise the neovascularization through vasoconstriction.
References

1. Adams C, Ratner D. Composite and free cartilage grafting. Dermatol Clin. 2005;23:129-140.
2. Renner G, McClane SD, Early E et al. Enhancement of auricular composite graft survival
with hyperbaric oxygen therapy. Arch Facial Plast Surg. 2002;4:102-104.
3. Rohrich RJ, Barton FE, Hollier L. Nasal reconstruction. In: Aston SJ, Beasley RW, Thorne
CHM, eds. Grabb and Smith’s Plastic Surgery. Boston: Little Brown;1997:44.

This examination contains test materials that are owned and copyrighted by the American Society of
Plastic Surgeons and the Plastic Surgery Educational Foundation. Any reproduction of these materials or
any part of them, through any means, including but not limitied to, copying or printing electronic files,
reconstruction through memorization or dictation, and/or dissemination of these materials or any part of
them is strictly prohibited. Keep printed materials in a secure location when you are not reviewing them
and discard them in a secure manner, such as shredding, when you have completed the examination.

Page 63 of 287
American Society of Plastic Surgeons and the Plastic Surgery Educational Foundation
In-Service Examination

41. During dissection for a microsurgical free parascapular flap, the circumflex scapular
artery is located in the triangular space bordered by the teres minor, the long head
of the triceps, and which of the following muscles?
A) Infraspinatus
B) Latissimus dorsi
C) Subscapularis
D) Teres major
E) Trapezius

The correct response is Option D.

The scapular area supplied by the circumflex scapular artery is a commonly used donor site in
reconstructive microsurgery because of the consistent and reliable vascular anatomy, easily
accessible pedicle, and good vessel diameter and length. The circumflex scapular artery arises
from scapular artery approximately 3 to 4 cm from its origin at the axillary artery. It then passes
through the triangular space, which is bordered above by the teres minor, the teres major below,
and the long head of triceps laterally. Flap size can reach 10 × 25 cm with generally closure of
the defect directly with acceptable scarring. Parascapular osteocutaneous flaps can also be
harvested using the lateral border of the scapula, which is useful for reconstruction of the hand
and face.

References

1. Arnez ZM, Scamp T, Planinsek F, et al. Lateral extension of the free scapular flap. Br
J Plast Surg. 1994;47:268-271.
2. Vaienti L, Soresina M, Menozzi A. Parascapular free flap and fat grafts: combined
surgical methods in morphological restoration of hemifacial progressive atrophy.
Plast Reconstr Surg. 2005;116:699-711.

This examination contains test materials that are owned and copyrighted by the American Society of
Plastic Surgeons and the Plastic Surgery Educational Foundation. Any reproduction of these materials or
any part of them, through any means, including but not limitied to, copying or printing electronic files,
reconstruction through memorization or dictation, and/or dissemination of these materials or any part of
them is strictly prohibited. Keep printed materials in a secure location when you are not reviewing them
and discard them in a secure manner, such as shredding, when you have completed the examination.

Page 64 of 287
American Society of Plastic Surgeons and the Plastic Surgery Educational Foundation
In-Service Examination

42. Which of the following skin lesions meets criteria for sentinel lymph node biopsy
(SLNBx)?
A) Basal cell carcinoma (8 cm wide) on the anterior chest
B) Melanoma (1.6 mm thick) of the breast with bulky axillary adenopathy
C) Melanoma-in-situ on the shoulder
D) Squamous cell carcinoma (1.8 cm wide) on the dorsum of the hand
E) Squamous cell carcinoma (2 cm wide) in a 26-year-old burn scar of the foot

The correct response is Option E.

Sentinel lymph node biopsy (SLNBx) is a well-established staging procedure for melanoma and
breast cancer. A patient with melanoma in situ, by definition, does not have invasion and,
therefore, would not benefit from SLNBx. Conversely, the patient with bulky adenopathy most
likely has regional metastatic disease and requires formal lymphadenectomy. Indications for
SLNBx in non-melanoma skin cancers are evolving and currently include squamous cell
carcinoma greater than 2 cm in diameter, Merkel cell carcinoma, and Marjolin ulcer (burn scar
carcinoma). Basal cell carcinomas almost never demonstrate lymphatic spread; therefore,
SLNBx would not add any diagnostic information.

References

1. Nahabedian MY, Tufaro AP, Manson PN. Sentinel lymph node biopsy for the T1
(thin) melanoma: is it necessary? Ann Plast Surg. 2003;50:601-606.
2. Stitzenberg KB, Groben PA, Stern SL, et al. Indications for lymphatic mapping and
sentinel lymphadenectomy in patients with thin melanoma (Breslow thickness < or =
1.0 mm). Ann Surg Oncol. 2004;11:900-906.
3. Eastman AL, Erdman WA, Lindberg GM, et al. Sentinel lymph node biopsy identifies
occult nodal metastases in patients with Marjolin’s ulcer. J Burn Care Rehabil.
2004;25:241-245.
4. Wagner JD, Evdokimow DZ, Weisberger E, et al. Sentinel node biopsy for high-risk
nonmelanoma cutaneous malignancy. Arch Dermatol. 2004;140:75-79.

This examination contains test materials that are owned and copyrighted by the American Society of
Plastic Surgeons and the Plastic Surgery Educational Foundation. Any reproduction of these materials or
any part of them, through any means, including but not limitied to, copying or printing electronic files,
reconstruction through memorization or dictation, and/or dissemination of these materials or any part of
them is strictly prohibited. Keep printed materials in a secure location when you are not reviewing them
and discard them in a secure manner, such as shredding, when you have completed the examination.

Page 65 of 287
American Society of Plastic Surgeons and the Plastic Surgery Educational Foundation
In-Service Examination

43. Which of the following is the mechanism of action of pressure garments in


management of fibroproliferative scars?
A) Alteration in cell shape
B) Hypoxia of local tissue
C) Increase in synthesis of tissue proteinases
D) Increase in temperature of the scar
E) Induction of matrix-specific autoantibodies

The correct response is Option B.

The mechanisms of pressure garments in management of fibroproliferative scars include local


tissue hypoxia, reduced fibroblast proliferation, and reduced collagen synthesis.

Pressure therapy is a conservative treatment modality that has been used for many years,
particularly in the treatment of hypertrophic scars after burn injury. Numerous studies have
documented that pressure therapy reduces the size as well as softens hypertrophic scars. The
mechanism of action behind pressure garments is believed to be secondary to tissue ischemia.
The pressure leads to local hypoxia, which, in turn, decreases tissue metabolism and increases
collagenase activity. It also reduces fibroblast proliferation and collagen synthesis. The exerted
pressure is effective between 24 and 30 mmHg. Studies have shown a response to pressures as
low as 5 to 15 mmHg. At these pressures, the inherent capillary flow, but not the peripheral
circulation, is overcome, and there is occlusion of the small vessels within the scar. Pressure
therapy should begin as soon as re-epithelialization occurs.

Alteration in cell shape has not been shown to occur with use of pressure garments in
management of fibroproliferative scars.

Increased synthesis of tissue proteinases is a mechanism of corticosteroids. When used as a


single therapy in fibroproliferative scars, corticosteroid injections have a variable response rate
that ranges from 50% to 100%. In addition to softening the scars, corticosteroid injections often
provide symptomatic relief of itching and pain.

Increased temperature of the scar has not been proven as a possible effect of topical silicone gel
sheeting. Other possible mechanisms of action of silicone are changes in skin hydration and
downregulation of wound healing by the negative charge of the silicone.

Induction of matrix-specific autoantibodies is a molecular mechanism that blocks the effect of


transforming growth factor (TGF)-β. The inhibition of TGF-β may serve as a new approach to
scar therapy. Autoantibodies and binding proteins that function against TGF-β are still

This examination contains test materials that are owned and copyrighted by the American Society of
Plastic Surgeons and the Plastic Surgery Educational Foundation. Any reproduction of these materials or
any part of them, through any means, including but not limitied to, copying or printing electronic files,
reconstruction through memorization or dictation, and/or dissemination of these materials or any part of
them is strictly prohibited. Keep printed materials in a secure location when you are not reviewing them
and discard them in a secure manner, such as shredding, when you have completed the examination.

Page 66 of 287
American Society of Plastic Surgeons and the Plastic Surgery Educational Foundation
In-Service Examination

experimental and are not available for routine clinical use.

References

1. Rockwell WB, Cohen IK, Ehrlich HP. Keloids and hypertrophic scars: a
comprehensive review. Plast Reconstr Surg. 1989;84:827-837.
2. Kazeen AA. The immunological aspects of keloid tumor formation. J Surg Oncol.
1988;38:16-18.
3. Kischer CW, Shetlar MR, Shetlar CL. Alterations in hypertrophic scars induced by
mechanical pressure. Arch Dermatol. 1975;111:60-64.
4. Rahban SR, Garner WL. Fibroproliferative scars. Clin Plast Surg. 2003;30:77-89.

This examination contains test materials that are owned and copyrighted by the American Society of
Plastic Surgeons and the Plastic Surgery Educational Foundation. Any reproduction of these materials or
any part of them, through any means, including but not limitied to, copying or printing electronic files,
reconstruction through memorization or dictation, and/or dissemination of these materials or any part of
them is strictly prohibited. Keep printed materials in a secure location when you are not reviewing them
and discard them in a secure manner, such as shredding, when you have completed the examination.

Page 67 of 287
American Society of Plastic Surgeons and the Plastic Surgery Educational Foundation
In-Service Examination

44. A 42-year-old woman is scheduled to undergo augmentation of the malar region


with an alloplastic prosthesis. Which of the following tissue planes is most
appropriate for placement of the prosthesis to decrease risk of infection and
extrusion?
A) Subcutaneous
B) Submalar
C) Submuscular
D) Subperiosteal
E) Sub-SMAS

The correct response is Option D.

The single most important concept for promoting healing over alloplastic material is providing
ample soft-tissue coverage. By placing a malar or genial implant directly on bone, a
well-perfused musculocutaneous coverage layer is ensured and the desired aesthetic
augmentation of the overlying soft tissues is accomplished. Dissection in the subperiosteal plane
also allows for good visualization of the infraorbital nerve.

Dissection in the immediate submuscular plane offers no advantages over subperiosteal


placement and risks injury or division of the overlying muscles or the motor nerve supply. Sub-
SMAS and subcutaneous placements are also inferior options in regard to soft-tissue coverage.
Submalar placement refers to the position of the implant in relation to the malar bone. Submalar
augmentation helps to correct the malar and nasolabial atrophy associated with aging. Direct
malar placement augments a deficiency in the projection of the malar bone. Submalar versus
malar placement is an aesthetic judgment and does not influence extrusion or infection rates.

Other important considerations for alloplastic augmentation include antibiotic prophylaxis,


sterile implant handling, correct pocket size dissection, and avoidance of skin incisions directly
over implants.

References

1. Terino E, Sofino M. Facial contouring with alloplastic implants. In: Aston SJ,
Beasley RW, Thorne CH, eds. Grabb and Smith's Plastic Surgery. 5th ed.
Philadelphia: Lippincott-Raven; 1997:702.
2. Atagi T, Young VL. Alloplastic materials. In: Mathes S, ed. Plastic Surgery. 2nd ed.
Philadelphia: W.B. Saunders; 2006:745-768.

This examination contains test materials that are owned and copyrighted by the American Society of
Plastic Surgeons and the Plastic Surgery Educational Foundation. Any reproduction of these materials or
any part of them, through any means, including but not limitied to, copying or printing electronic files,
reconstruction through memorization or dictation, and/or dissemination of these materials or any part of
them is strictly prohibited. Keep printed materials in a secure location when you are not reviewing them
and discard them in a secure manner, such as shredding, when you have completed the examination.

Page 68 of 287
American Society of Plastic Surgeons and the Plastic Surgery Educational Foundation
In-Service Examination

45. A 45-year-old woman is referred to the office because she has had chronic
infection, drainage, and pain of the left armpit for the past 10 years. The
contralateral axilla is similarly affected. Medical history includes multiple courses of
antibiotic therapy and intermittent drainage procedures. Physical examination
shows a 10 x 15-cm area of involvement. Which of the following is the most
effective management?
A) Oral administration of an antibiotic
B) Intravenous administration of an antibiotic
C) Injection of botulinum toxin A
D) Incision and drainage of the region
E) Complete excision of the scarred area
The correct response is Option E.
Hidradenitis suppurativa is an inflammatory disease of the apocrine glands and follicular
epithelium, presenting most commonly as deep recurrent infections or chronic sinus tracts in
hair-bearing regions of the skin. It is most commonly seen in the axillary region in young
women, but the groin and perineum may also be affected.
After the diagnosis has been confirmed, a short course of treatment with local drainage and
suppressive antibiotics is appropriate. However, once the disease process is established, the only
appropriate therapy is complete excision of the scarred area.
The successful treatment of chronic hidradenitis with injection of botulinum toxin A has been
reported. Sweat glands have a sympathetic innervation. Botulinum toxin works by blocking the
release of acetylcholine from nerve endings, thus decreasing sweat production. However, this is
expensive and represents a temporary treatment of a chronic problem.
The mainstay of treatment for larger areas is complete excision of axillary skin followed by split-
thickness skin grafting. Negative pressure dressings are helpful to stabilize the graft. Although
numerous local fasciocutaneous pedicled and free flaps have been described, they increase the
risk of donor site morbidity and are generally not necessary. Healing by secondary intention has
also been described. However, it is preferable to undertake excision of one axilla at a time so the
patient is not disabled in the perioperative period.
References

1. Pollack WJ, Virnelli FR, Ryan RF. Axillary hidradenitis suppurativa: a simple and effective
surgical technique. Plast Reconstr Surg. 1972;49:22.
2. Slade DE, Powell BW, Mortimer PS. Hidradenitis suppurativa: pathogenesis and
management. Br J Plast Surg. 2003;56:451.
3. O'Reilly DJ, Pleat JM, Richards AM. Treatment of hidradenitis suppurativa with botulinum
toxin A. Plast Reconstr Surg. 2005;116:1575-1576.

This examination contains test materials that are owned and copyrighted by the American Society of
Plastic Surgeons and the Plastic Surgery Educational Foundation. Any reproduction of these materials or
any part of them, through any means, including but not limitied to, copying or printing electronic files,
reconstruction through memorization or dictation, and/or dissemination of these materials or any part of
them is strictly prohibited. Keep printed materials in a secure location when you are not reviewing them
and discard them in a secure manner, such as shredding, when you have completed the examination.

Page 69 of 287
American Society of Plastic Surgeons and the Plastic Surgery Educational Foundation
In-Service Examination

46. A 48-year-old woman comes to the office for consultation regarding reconstruction
of the right breast after mastectomy because of cancer. The patient is concerned
about maximizing the aesthetic result and minimizing any donor site deformity.
Physical examination shows a well-healed chest wall and a B-cup left breast with
Grade 3 ptosis. Soft-tissue reconstruction with a superior gluteal artery perforator
(SGAP) free flap is planned. Which of the following is a disadvantage of this
procedure?
A) Difficulty molding the gluteal fat
B) Gait dysfunction
C) Inability to fully hide the donor scar
D) Inability to provide a sensate flap
E) Lack of abundant soft tissue

The correct response is Option A.

In soft-tissue reconstruction with a superior gluteal artery perforator (SGAP) free flap, the firmer
consistency of gluteal fat causes difficulty in molding tissue, particularly in breast reconstruction.
Secondary procedures may be necessary to obtain the desired shape.

Gait dysfunction is extremely uncommon after a SGAP flap if care is taken to avoid injury to the
motor branch. The scar is generally well hidden by clothing and the donor site deformities are
generally minimal. Even in thin patients there is an abundance of adipose tissue in the gluteal
area. Although not commonly done, a sensate branch of the nervi clunium superiores can provide
sensation to the flap.

References

1. Blondeel PN, Van Landuyt K, Hamdi M, et al. Soft tissue reconstruction with the
superior gluteal artery perforator flap. Clin Plastic Surg. 2003;30:371-382.
2. DellaCroce FJ, Sullivan SK. Application and refinement of the superior gluteal artery
perforator free flap for bilateral simultaneous breast reconstruction. Plast Reconstr Surg.
2005;116:97-105.

This examination contains test materials that are owned and copyrighted by the American Society of
Plastic Surgeons and the Plastic Surgery Educational Foundation. Any reproduction of these materials or
any part of them, through any means, including but not limitied to, copying or printing electronic files,
reconstruction through memorization or dictation, and/or dissemination of these materials or any part of
them is strictly prohibited. Keep printed materials in a secure location when you are not reviewing them
and discard them in a secure manner, such as shredding, when you have completed the examination.

Page 70 of 287
American Society of Plastic Surgeons and the Plastic Surgery Educational Foundation
In-Service Examination

47. A 26-year-old woman of Asian descent who underwent rhinoplasty five years ago
has erosion of the silicone rubber (Silastic) prosthesis through the skin of the nasal
tip. Physical examination shows a depressed scar in this region. Secondary
rhinoplasty is planned. Use of which of the following grafts for this procedure is
most appropriate to minimize volume loss?
A) Dermis
B) Fat
C) Muscle
D) Cartilage
E) Bone

The correct response is Option D.

A cartilaginous graft would be most appropriate to correct the deformity described. The low
metabolic rate of cartilage leads to minimal volume loss. Fat graft survival depends on early
neovascularization and only approximately 50% of lipocytes in an autogenous graft survive.
Owing to the traumatic nature of the suction technique, fewer viable micrografts are transferred
with potential for survival so that surgically removed grafts (macrografts) have a greater
longevity. Animal studies have also demonstrated that adipocytes implanted in a vascularized
bed (muscle) survive better than those in dermis. Considerable resorption is seen in
nonvascularized bone grafts. Muscle alone is not routinely transferred as a graft. There are not
sufficient studies in the literature to support the routine use of dermis to correct this deformity.

References

1. Karacaoglu E, Kizilkaya E, Cermik H, et al. The role of recipient sites in fat-graft


survival: experimental study. Ann Plast Surg. 2005;55:63-68.
2. Brenner KA, McConnell MP, Evans GR, et al. Survival of diced cartilage grafts: an
experimental study. Plast Reconstr Surg. 2006;117:105-115.

This examination contains test materials that are owned and copyrighted by the American Society of
Plastic Surgeons and the Plastic Surgery Educational Foundation. Any reproduction of these materials or
any part of them, through any means, including but not limitied to, copying or printing electronic files,
reconstruction through memorization or dictation, and/or dissemination of these materials or any part of
them is strictly prohibited. Keep printed materials in a secure location when you are not reviewing them
and discard them in a secure manner, such as shredding, when you have completed the examination.

Page 71 of 287
American Society of Plastic Surgeons and the Plastic Surgery Educational Foundation
In-Service Examination

48. A previously healthy 55-year-old man is brought to the emergency department


three hours after sustaining partial-thickness scald burns to the arms, torso, and
legs, comprising 35% of the total body surface area. He has smoked one half pack
of cigarettes daily for the past 20 years. The patient is breathing comfortably on
room air and oxygen saturation is 99%. On physical examination, he is alert, calm,
and cooperative. The patient is admitted to the hospital for wound care. Three days
after the injury, respiratory distress develops and he is intubated. Which of the
following is the most likely mechanism of respiratory failure in this patient?
A) Aspiration pneumonia
B) Inflammatory interstitial edema
C) Inhalation injury
D) Pulmonary embolism
E) Severe chronic obstructive pulmonary disease

The correct response is Option B.

Burn injury results in local production of immunoregulatory mediators, which results in the
production of prostaglandins, thromboxane A2, and reactive oxygen metabolites. These
vasoactive inflammatory mediators result in edema and fluid extravasation from the vascular
system. Burns of more than 20% or 30% of the total body surface area result in significant
distant soft tissue edema and microvasacular injury. In the lungs, interstitial edema results in
decreased oxygenation and may progress to respiratory distress syndrome. Gram-negative
pneumonia is common among critically ill burn patients, especially intubated patients.

Aspiration is not a likely source in an alert patient. Smoking is likely to have contributed to this
patient's susceptibility to lung disease, although the primary, overwhelming factor involved is the
inflammatory response to a critical burn injury. His normal breathing and oxygen saturation on
admission rule out a severe underlying pulmonary process. Inhalation injury is common in fires
but not scald injury. Pulmonary embolism may result after prolonged immobilization but is much
less likely the source of respiratory failure and pneumonia in the acute burn patient.

References

1. Sheridan RL, Tompkins RG. Burns. In: Greenfield LJ, Mulholland MW, Oldham KT,
et al., eds. Surgery: Scientific Principles and Practice. 2nd ed. Philadelphia:
Lippincott-Raven; 1997:423-424.
2. Press B. Thermal, chemical and electrical injuries. In: Aston SJ, Beasley RW, Thorne
CH, eds. Grabb and Smith's Plastic Surgery. 5th ed. Philadelphia: Lippincott-Raven;
1997:161-184.

This examination contains test materials that are owned and copyrighted by the American Society of
Plastic Surgeons and the Plastic Surgery Educational Foundation. Any reproduction of these materials or
any part of them, through any means, including but not limitied to, copying or printing electronic files,
reconstruction through memorization or dictation, and/or dissemination of these materials or any part of
them is strictly prohibited. Keep printed materials in a secure location when you are not reviewing them
and discard them in a secure manner, such as shredding, when you have completed the examination.

Page 72 of 287
American Society of Plastic Surgeons and the Plastic Surgery Educational Foundation
In-Service Examination

49. A 65-year-old man who underwent three-vessel coronary artery bypass grafting
(CABG) five weeks ago comes to the office because he has a draining lesion near
the sternotomy incision. The CABG procedure included harvest of the patient's left
internal mammary artery. Temperature is 38.7°C (101.7°F). Physical examination
shows a 3-mm papule at the manubrium of the healed sternotomy incision. CT scan
of the chest shows a sinus tract leading to the internal sternal plate. In addition to
removal of sternal wires and debridement of the wound, which of the following is
the most appropriate management?
A) Continuous irrigation
B) Negative-pressure wound therapy
C) Omental flap
D) Pectoralis major muscle flaps
E) Rectus abdominis muscle flaps

The correct response is Option D.

The most effective treatment of the sternal wound infection described is initial debridement to
healthy bleeding tissue and bone, removal of all foreign bodies, and immediate closure with
bilateral pectoralis major muscle flaps. Post-sternotomy wounds are classified according to the
duration of time that has elapsed between the sternotomy and the clinical onset of the infection.
Type I infections occur within the first week after sternotomy and have no bony involvement.
Type II infections occur during the second to fourth weeks after sternotomy. Bony involvement
is frequent, but chostochondritis is rare. Type III infections occur months to years after
sternotomy and typically involve chronic draining sinus tracts. Osteomyelitis, chostochondritis,
and retained foreign bodies are all common.

The patient described had a Type III infection with a deep sinus tract as confirmed by CT scan.
Although delayed closure had previously been considered the treatment of choice for Type III
sternotomy wounds, more recent data suggest that a single-stage approach with bilateral
pectoralis major muscle flaps results in quicker recovery, improved cosmesis, and decreased
morbidity and mortality compared with staged approaches. Given the chronicity of the wound,
all foreign bodies, including retained suture material and wires, should be removed.

Pectoralis major turnover flaps provide additional bulk and might therefore be considered
preferable. However, the left internal mammary artery has been harvested in this patient,
compromising the blood supply to the left pectoralis major turnover flap.

This examination contains test materials that are owned and copyrighted by the American Society of
Plastic Surgeons and the Plastic Surgery Educational Foundation. Any reproduction of these materials or
any part of them, through any means, including but not limitied to, copying or printing electronic files,
reconstruction through memorization or dictation, and/or dissemination of these materials or any part of
them is strictly prohibited. Keep printed materials in a secure location when you are not reviewing them
and discard them in a secure manner, such as shredding, when you have completed the examination.

Page 73 of 287
American Society of Plastic Surgeons and the Plastic Surgery Educational Foundation
In-Service Examination

References

1. Ascherman JA, Patel SM, Malhotra SM, et al. Management of sternal wounds with
bilateral pectoralis major myocutaneous advancement flaps in 114 consecutively
treated patients: Refinements in techniques and outcomes analysis. Plast Reconstr
Surg. 2004;114:676-683.
2. Pairolero PC, Arnold PG. Management of infected median sternotomy wounds. Ann
Thorac Surg. 1986;42:1-2.
3. Pairolero PC, Arnold PG. Long term results of pectoralis major muscle transposition
for infected sternotomy wounds. Ann Surg. 1991;213:583.
4. Gur E, Stern D, Weiss J, et al. Clinical-radiological evaluation of poststernotomy
wound infection. Plast Reconstr Surg. 1998;101:348-355.

This examination contains test materials that are owned and copyrighted by the American Society of
Plastic Surgeons and the Plastic Surgery Educational Foundation. Any reproduction of these materials or
any part of them, through any means, including but not limitied to, copying or printing electronic files,
reconstruction through memorization or dictation, and/or dissemination of these materials or any part of
them is strictly prohibited. Keep printed materials in a secure location when you are not reviewing them
and discard them in a secure manner, such as shredding, when you have completed the examination.

Page 74 of 287
American Society of Plastic Surgeons and the Plastic Surgery Educational Foundation
In-Service Examination

(Please note that this pictorial appears in color in the online examination)

50. A 60-year-old woman comes to the office for evaluation of a pigmented skin lesion
on the left cheek (shown). The patient wants the lesion to be removed because it
often becomes irritated. She has a history of sun exposure and basal cell
carcinoma. Melanoma was diagnosed in the patient's father at 60 years of age.
Physical examination shows a slightly elevated, irregularly pigmented brown plaque
on the cheek and similar lesions on the chest. Biopsy of the lesion on the cheek
rules out malignancy. Which of the following is the most likely diagnosis?
A) Actinic keratosis
B) Compound nevus
C) Keratoacanthoma
D) Melasma
E) Seborrheic keratosis

The correct response is Option E.

Seborrheic keratosis is a common benign, pigmented lesion often seen in large numbers,
especially on the trunk, face, and arms of individuals older than 30 years. Seborrheic keratosis

This examination contains test materials that are owned and copyrighted by the American Society of
Plastic Surgeons and the Plastic Surgery Educational Foundation. Any reproduction of these materials or
any part of them, through any means, including but not limitied to, copying or printing electronic files,
reconstruction through memorization or dictation, and/or dissemination of these materials or any part of
them is strictly prohibited. Keep printed materials in a secure location when you are not reviewing them
and discard them in a secure manner, such as shredding, when you have completed the examination.

Page 75 of 287
American Society of Plastic Surgeons and the Plastic Surgery Educational Foundation
In-Service Examination

typically presents as a sharply circumscribed, waxy, papillomatous plaque with a friable,


hyperkeratotic surface, most often described as having a "stuck on" appearance. Pigmentation is
variable, from mild to deep black. Seborrheic keratosis is rarely symptomatic but may become
irritated and secondarily infected. Adequate treatment of seborrheic keratosis includes shave
excision, superficial electrodesiccation, and freezing with liquid nitrogen, because this type of
lesion does not undergo malignant transformation.

Actinic (solar) keratosis must be distinguished from seborrheic keratosis because it represents
dysplasia and is premalignant. Actinic keratoses, which are found primarily on exposed surfaces,
are discrete, flat, or elevated lesions with rough adherent hyperkeratosis. They are skin-colored,
yellow-brown or brown, possibly with a reddish tinge, and are often numerous in elderly
patients, making excision of all lesions impractical. Treatment with topical 5% 5-fluorouracil
(5-FU) is common. Nodular lesions should be excised.

Keratoacanthomas clinically and microscopically resemble squamous cell carcinomas.


Keratoacanthomas present as nodules (typically > 1 cm in diameter) with a crater-like center that
contains a keratin plug and are often found in hair-bearing sites of sun-exposed skin in elderly
patients. Although it is generally accepted that keratoacanthomas are benign neoplasms, because
of their potential for spontaneous regression over a four- to eight-week period after a four- to six-
week period of rapid growth, excision of these lesions is critical for proper diagnosis and
management.

Melasma presents as brown patches on the face, typically affecting both sides. The most
common sites of involvement are the cheeks, bridge of nose, forehead, and upper lip. Melasma
mostly occurs in women. A change in hormonal status may trigger melasma, and it is commonly
associated with pregnancy.

References

1. Fitzpatrick TB, Johnson RA, Polano MK, eds. Color Atlas and Synopsis of Clinical
Dermatology. 2nd ed. New York: McGraw-Hill; 1992.
2. Zarem HA, Lowe NJ. Benign growths and generalized skin disorders. In: Aston SJ,
Beasley RW, Thorne CH, eds. Grabb and Smith's Plastic Surgery. 5th ed.
Philadelphia: Lippincott-Raven; 1997:141-159.

This examination contains test materials that are owned and copyrighted by the American Society of
Plastic Surgeons and the Plastic Surgery Educational Foundation. Any reproduction of these materials or
any part of them, through any means, including but not limitied to, copying or printing electronic files,
reconstruction through memorization or dictation, and/or dissemination of these materials or any part of
them is strictly prohibited. Keep printed materials in a secure location when you are not reviewing them
and discard them in a secure manner, such as shredding, when you have completed the examination.

Page 76 of 287
American Society of Plastic Surgeons and the Plastic Surgery Educational Foundation
In-Service Examination

Section 2: Cosmetic
51. A 45-year-old woman who has had a 100-lb weight loss since undergoing gastric
banding two years ago comes to the office for consultation regarding loose skin on
her upper arms. Physical examination shows significant ptosis of the posteromedial
aspect of the upper arms and relaxation of the soft tissue. Fascia in which of the
following regions is also relaxed in this patient and is appropriate for use as an
anchor during brachioplasty to reduce widening and hypertrophy of the scars?
A) Axillary
B) Bicipital
C) Deltoid
D) Pectoral
E) Tricipital

The correct response is Option A.

An important etiologic mechanism of the aesthetic arm deformity is relaxation of a longitudinal


fascial system sling that extends from the clavicle to the soft tissues of the posteromedial aspect
of the arm via the clavipectoral and axillary fascia. Reanchoring the soft tissues of the
posteromedial aspect of the arm to the axillary fascia with nonabsorbable sutures addresses the
relaxation of the axillary fascia sling and forms the basis for the anchor brachioplasty. Similarly
to anchoring the soft tissues of the medial aspect of the thigh to the Colles fascia (perineum) in
medial thigh lifts, fascial anchoring in brachioplasties provides more predictable results while
reducing complications.

References

1. Lockwood T. Brachioplasty with superficial fascial system suspension. Plast


Reconstr Surg. 1995;96:912-920.
2. Teimourian B, Malekzadeh S. Rejuvenation of the upper arm. Plast Reconstr Surg.
1998;102:545-551.

This examination contains test materials that are owned and copyrighted by the American Society of
Plastic Surgeons and the Plastic Surgery Educational Foundation. Any reproduction of these materials or
any part of them, through any means, including but not limited to, copying or printing electronic files,
reconstruction through memorization or dictation, and/or dissemination of these materials or any part of
them is strictly prohibited. Keep printed materials in a secure location when you are not reviewing them
and discard them in a secure manner, such as shredding, when you have completed the examination.

Page 77 of 287
American Society of Plastic Surgeons and the Plastic Surgery Educational Foundation
In-Service Examination

52. A 32-year-old woman who underwent submuscular placement of smooth 240-ml


saline breast prostheses 10 years ago comes to the office for consultation
regarding replacement of the implants. She says she wants her breasts to be two
cup sizes larger. Currently, she wears a size 36C brassiere. Physical examination
shows good aesthetic outcome and no evidence of capsular contracture or rippling.
Which of the following is the most likely adverse effect of implant exchange in this
patient?
A) Inability to breast-feed
B) Increased capsular contracture
C) Increased risk of breast cancer
D) Increased risk of collagen vascular disease
E) Shrinkage of breast tissue

The correct response is Option E.

Patients seeking reoperation for dissatisfaction with breast size after initial implantation of breast
prostheses must be informed of the following long-term negative effects: thinning of tissue,
stretching of tissue, shrinkage of breast tissue, additional and more rapid sagging, palpable
implant edges and shell, visible implant edges, visible traction rippling, and possible additional
sensory loss. Shrinkage of breast tissue occurs with all prostheses; the larger the prosthesis, the
more shrinkage that occurs. Larger implants will not give this patient a more natural appearance.
The potential for lactation should not be impaired by breast prostheses, especially when the
prostheses are positioned in the subpectoral pocket.

References

1. Adams WP, Bengston BP, Glicksman CA, et al. Decisions and management
algorithms to address patient and Food and Drug Administration concerns regarding
breast augmentation and implants. Plast Reconstr Surg. 2004;114:1252-1262.
2. LaTrenta GS. Breast augmentation. In: Rees TD, La Trenta GS, eds. Aesthetic Plastic
Surgery. Philadelphia: WB Saunders; 1994:1003-1049.

This examination contains test materials that are owned and copyrighted by the American Society of
Plastic Surgeons and the Plastic Surgery Educational Foundation. Any reproduction of these materials or
any part of them, through any means, including but not limited to, copying or printing electronic files,
reconstruction through memorization or dictation, and/or dissemination of these materials or any part of
them is strictly prohibited. Keep printed materials in a secure location when you are not reviewing them
and discard them in a secure manner, such as shredding, when you have completed the examination.

Page 78 of 287
American Society of Plastic Surgeons and the Plastic Surgery Educational Foundation
In-Service Examination

53. A 48-year-old man comes to the office six days after undergoing bilateral
transcutaneous lower eyelid blepharoplasty because he has diplopia and pain in
the left orbit. Injury to the left inferior oblique muscle is suspected. Which of the
following findings will confirm the diagnosis?
A) Absent depression, extorsion, adduction of the left eye
B) Absent elevation, intorsion, adduction of the left eye
C) Absent extorsion, elevation, and abduction of the left eye
D) Absent intorsion, depression, and abduction of the left eye

The correct response is Option C.

The patient described has sustained injury to the inferior oblique muscle during a lower lid
blepharoplasty, which is characterized by absent extorsion, elevation, and abduction of the eye.

Elevation, intorsion, and abduction are controlled by the superior rectus muscle. Adduction is
controlled by the medial rectus muscle. Intorsion, depression, and abduction are caused by the
superior oblique muscle. Depression, extortion, and adduction are controlled by the inferior
rectus muscle.

References

1. Baylis H, Wilson M, Groth M. Complications of lower blepharoplasty. In: Putterman


A, ed. Cosmetic Oculoplastic Surgery. Philadelphia: WB Saunders; 1993:387.
2. Doxanas M, Anderson RL. Extraocular muscles. In: Clinical Orbital Anatomy.
Baltimore: Williams & Wilkins; 1984:126.

This examination contains test materials that are owned and copyrighted by the American Society of
Plastic Surgeons and the Plastic Surgery Educational Foundation. Any reproduction of these materials or
any part of them, through any means, including but not limited to, copying or printing electronic files,
reconstruction through memorization or dictation, and/or dissemination of these materials or any part of
them is strictly prohibited. Keep printed materials in a secure location when you are not reviewing them
and discard them in a secure manner, such as shredding, when you have completed the examination.

Page 79 of 287
American Society of Plastic Surgeons and the Plastic Surgery Educational Foundation
In-Service Examination

54. When referring to nasal anatomy, the angle of divergence is made between which
of the following structures?
A) Lateral crura of the lower lateral cartilages
B) Middle and lateral crura of the lower lateral cartilages
C) Middle and medial crura of the lower lateral cartilages
D) Middle crura of the lower lateral cartilages
E) Septum and upper lateral cartilage

The correct response is Option D.

The angle of divergence refers to the middle crura of the lower lateral cartilages. The angle of
divergence is the angle between the right middle crus and the left middle crus running from the
medial genu to the lateral genu, while looking at the nose from the anteroposterior view. The
angle from the middle and medial crura refers to the angle of rotation as the tip gently bends
cephalad from the columella to the tip-defining point. There is no specific name given to the
angle made by the lateral crura of the lower lateral cartilages. The septum and the upper lateral
cartilage form the angle of the internal valve and relate to issues of occlusion of the airway. The
middle and lateral crura form the lateral genu.

The ideal angle of divergence is approximately 30 to 60 degrees. A more obtuse angle produces
a long intercrural distance and a more boxy tip. A very acute angle of divergence makes a shorter
intercrural distance and a narrow lobule. Optimally, the angle of rotation is approximately 60
degrees. A more obtuse angle often results in a lower nostril-lobule ratio and a more square tip.
A shorter or absent middle crus will cause the tip to appear stubbed with inadequate projection.

References

1. Sheen JH, Sheen DP. Aesthetic Rhinoplasty. 2nd ed. St. Louis, Quality Medical
Publishing, 1987:47-43.
2. Byrd HS. Rhinoplasty. Selected Readings in Plastic Surgery. 2001;9:9-18.
3. Rohrich RJ, Adams WP. The boxy nasal tip: classification and management based on
alar cartilage suturing techniques. Plast Reconstr Surg. 2001;107:1849-1863.
4. Daniel RK. The nasal tip: anatomy aesthetics. Plast Reconstr Surg. 1992;89:216-224.

This examination contains test materials that are owned and copyrighted by the American Society of
Plastic Surgeons and the Plastic Surgery Educational Foundation. Any reproduction of these materials or
any part of them, through any means, including but not limited to, copying or printing electronic files,
reconstruction through memorization or dictation, and/or dissemination of these materials or any part of
them is strictly prohibited. Keep printed materials in a secure location when you are not reviewing them
and discard them in a secure manner, such as shredding, when you have completed the examination.

Page 80 of 287
American Society of Plastic Surgeons and the Plastic Surgery Educational Foundation
In-Service Examination

(Please note that this pictorial appears in color in the online examination)

55. A 17-year-old boy is brought to the office three days after he sustained trauma to
the ear during practice with his wrestling team. A photograph is shown. Which of
the following is the most appropriate management?
A) Incision and evacuation followed by application of a bolster dressing
B) Intravenous administration of an antibiotic for 48 hours followed by oral
administration of an antibiotic for five days
C) Needle aspiration
D) Observation for two weeks followed by drainage if fluid is present in the wound
E) Resection of the posterior cartilage and sculpting of the anterior cartilage

This examination contains test materials that are owned and copyrighted by the American Society of
Plastic Surgeons and the Plastic Surgery Educational Foundation. Any reproduction of these materials or
any part of them, through any means, including but not limited to, copying or printing electronic files,
reconstruction through memorization or dictation, and/or dissemination of these materials or any part of
them is strictly prohibited. Keep printed materials in a secure location when you are not reviewing them
and discard them in a secure manner, such as shredding, when you have completed the examination.

Page 81 of 287
American Society of Plastic Surgeons and the Plastic Surgery Educational Foundation
In-Service Examination

The correct response is Option A.

The history and photograph of the patient described indicate an othematoma caused by shearing
of the skin from the cartilage. Early diagnosis of this hematoma will prevent cauliflower ear
deformity. Complete evacuation via a small incision with application of a bolster dressing to
prevent reaccumulation of the fluid or blood is the most appropriate management.

The use of intravenous antibiotics to treat the hematoma is inappropriate, and there is no sign of
infection. Needle aspiration may help, but the hematoma is unlikely to be completely evacuated.
Allowing the hematoma to remain can result in calcification. Observation may lead to
cauliflower ear deformity. Resection of the posterior cartilage and sculpting of the anterior
cartilage is the treatment option for chronic calcification or cauliflower ear deformity, but not for
acute othematoma.

This examination contains test materials that are owned and copyrighted by the American Society of
Plastic Surgeons and the Plastic Surgery Educational Foundation. Any reproduction of these materials or
any part of them, through any means, including but not limited to, copying or printing electronic files,
reconstruction through memorization or dictation, and/or dissemination of these materials or any part of
them is strictly prohibited. Keep printed materials in a secure location when you are not reviewing them
and discard them in a secure manner, such as shredding, when you have completed the examination.

Page 82 of 287
American Society of Plastic Surgeons and the Plastic Surgery Educational Foundation
In-Service Examination

References

1. Brent B. Reconstruction of the ear. In: Aston SJ, Beasley RW, Thorne CHM, eds.
Grabb & Smith's Plastic Surgery. 5th ed. Philadelphia: Lippincott Raven;
1997:420-421.
2. Kelleher JC, Sullivan K, Baibak G, et al. The wrestler's ear. Plast Reconstr Surg.
1967;40:540.
3. Elsahy NI. Acquired ear defects. Clin Plast Surg. 2002;29:175-186.

This examination contains test materials that are owned and copyrighted by the American Society of
Plastic Surgeons and the Plastic Surgery Educational Foundation. Any reproduction of these materials or
any part of them, through any means, including but not limited to, copying or printing electronic files,
reconstruction through memorization or dictation, and/or dissemination of these materials or any part of
them is strictly prohibited. Keep printed materials in a secure location when you are not reviewing them
and discard them in a secure manner, such as shredding, when you have completed the examination.

Page 83 of 287
American Society of Plastic Surgeons and the Plastic Surgery Educational Foundation
In-Service Examination

56. For reduction mammaplasty, which of the following is the greatest advantage of a
vertical procedure over an inverted-T method?
A) Decreased risk of hematoma
B) Greater ability to breast-feed
C) Increased sensation of the nipple
D) Initial natural shape of the breast
E) Smaller scar

The correct response is Option E.


Vertical mammaplasty is a technique that uses adjustable markings, an upper pedicle for the
areola, and a central breast reduction with decreased undermining of the skin. Key features of the
vertical scar reduction mammaplasty include skin excision in only one direction, which reduces
scar burden, and central vertical glandular excision, which contributes to improved postoperative
shape by narrowing the breast while maximizing projection as a result of suturing the medial and
lateral pillars together. There is no wide periareolar skin excision; therefore, circumareolar scar
quality is not compromised by excess tension.
However, this method is more intuitive and inherently less precise than the inverted-T method.
Because of the central and posterior nature of glandular resection in vertical mammaplasty, it is
more difficult to determine the end point of resection.
A recent study comparing vertical mammary reduction with the Wise pattern technique showed
no statistical differences in the rate of hematomas and nipple numbness. Breast-feeding is
possible with either technique but may require supplementation with formula. The initial shape
in a vertical mammaplasty is poor; it has a flattened lower pole, and dog-ears are frequently
present. This improves after three to six months, although small revisions are sometimes
necessary.
References

1. Hidalgo DA, Elliot LF, Palumbo S, et al. Current trends in breast reduction. Plast
Reconstr Surg. 1999;104:806.
2. Lassus C. A 30-year experience with vertical mammaplasty. Plast Reconstr Surg.
1996;97:373.
3. Lejour M. Vertical mammaplasty. Plast Reconstr Surg. 1993;92:985-986.
4. Cruz-Korchin N, Korchin L. Vertical vs. Wise pattern breast reduction. Plast
Reconstr Surg. 2003;112:1573-1578.
5. Cruz-Korchin N, Korchin L. Breast feeding after vertical mammaplasty. Plast
Reconstr Surg. 2004;114:890-894.

This examination contains test materials that are owned and copyrighted by the American Society of
Plastic Surgeons and the Plastic Surgery Educational Foundation. Any reproduction of these materials or
any part of them, through any means, including but not limited to, copying or printing electronic files,
reconstruction through memorization or dictation, and/or dissemination of these materials or any part of
them is strictly prohibited. Keep printed materials in a secure location when you are not reviewing them
and discard them in a secure manner, such as shredding, when you have completed the examination.

Page 84 of 287
American Society of Plastic Surgeons and the Plastic Surgery Educational Foundation
In-Service Examination

57. A 23-year-old man comes to the office for follow-up examination four months after
undergoing rhinoplasty and horizontal sliding osseous genioplasty with miniplate
fixation. He says he has noticed increasing show of his lower teeth since the
surgery. Physical examination shows ptosis of the lower lip and a deep inferior
gingivobuccal sulcus. Which of the following is the most likely cause of these
findings?
A) Excessive traction by the digastric muscle
B) Inadequate fixation of the osteotomy segment
C) Inadequate repair of the mentalis muscle
D) Injury to the marginal mandibular nerve
E) Injury to the mental nerve

The correct response is Option C.


The patient described has a witch’s chin deformity as a complication of a lower buccal sulcus
incision for access during an osseous genioplasty. In making this incision, the mentalis muscle is
divided. Leaving a cuff of muscle superiorly is helpful because it provides a good tissue base for
suturing the muscle when closing the incision. This closure of the muscle helps to resuspend the
soft tissue of the chin and prevents ptosis of the lower lip and soft tissues.

Excessive traction by the digastric muscle and inadequate fixation of the osteotomy segment are
not likely causes for the findings in the patient described. Miniplate fixation should provide
adequate fixation to the osteomy segment. Furthermore, relapse or traction of the osteotomy
segment would not account for all of the findings in the patient described.

Injury to the marginal mandibular nerve is not likely to have caused the findings in the patient
described because it is not in the field of dissection for the procedure.

Injury to the mental nerve would cause numbness of the lower lip on the side of the injury and
would have no effect on lip position.

References

1. Guyuron B, Kadi JS. Problems following genioplasty: diagnosis and treatment. Clin
Plast Surg. 1997;24:507-514.
2. Wolford LM, Fields RT. Surgical planning. In: Booth PW, Schendel SA, Hausamen
J-E, eds. Maxillofacial Surgery. London: Churchill Livingstone; 1999:1205-1257.
3. Zide B. The mentalis muscle: an essential component of chin and lower lip position.
Plast Reconstr Surg. 2000;105:1213-1215.

This examination contains test materials that are owned and copyrighted by the American Society of
Plastic Surgeons and the Plastic Surgery Educational Foundation. Any reproduction of these materials or
any part of them, through any means, including but not limited to, copying or printing electronic files,
reconstruction through memorization or dictation, and/or dissemination of these materials or any part of
them is strictly prohibited. Keep printed materials in a secure location when you are not reviewing them
and discard them in a secure manner, such as shredding, when you have completed the examination.

Page 85 of 287
American Society of Plastic Surgeons and the Plastic Surgery Educational Foundation
In-Service Examination

58. A 40-year-old woman comes to the office for follow-up examination three weeks
after undergoing traditional brachioplasty for correction of skin laxity in the proximal
aspect of the upper arms. She has had persistent numbness along the medial
aspect of the elbow and forearm, extending to the level of the wrist. Which of the
following nerves was most likely injured during brachioplasty in this patient?
A) Intercostobrachial
B) Medial antebrachial cutaneous
C) Medial brachial cutaneous
D) Musculocutaneous
E) Posterior brachial cutaneous

The correct response is Option B.

Traditional methods of brachioplasty use a medial or posteromedial longitudinal incision to


resect excess skin and fat. The medial antebrachial cutaneous nerve branches, together with the
basilic vein, exit the deep fascia of the medial arm at the transition between the middle and distal
thirds of the arm. Care should be taken at the time of surgery to identify this nerve at this level
and maintain more fat on the deep fascia to prevent injury.

The intercostobrachial nerve and the medial brachial cutaneous nerve innervate the medial arm
skin. The musculocutaneous nerve terminates in the lateral antebrachial cutaneous nerve,
supplying sensation to the lateral forearm. The posterior brachial cutaneous nerve, a branch of
the radial nerve, supplies sensation to the posterior arm.

References

1. Kenkel JM, Aly AS, Capella JF, et al. Marking and operative techniques, body
contouring surgery after massive weight loss. Plast Reconstr Surg. 2006;117:45S-
73S.
2. Moore KL. The upper limb. Clinically Oriented Anatomy. Baltimore: Williams &
Wilkins; 1985:626-793.

This examination contains test materials that are owned and copyrighted by the American Society of
Plastic Surgeons and the Plastic Surgery Educational Foundation. Any reproduction of these materials or
any part of them, through any means, including but not limited to, copying or printing electronic files,
reconstruction through memorization or dictation, and/or dissemination of these materials or any part of
them is strictly prohibited. Keep printed materials in a secure location when you are not reviewing them
and discard them in a secure manner, such as shredding, when you have completed the examination.

Page 86 of 287
American Society of Plastic Surgeons and the Plastic Surgery Educational Foundation
In-Service Examination

59. A 55-year-old woman who underwent an upper and lower blepharoplasty procedure
via a subciliary approach five years ago comes to the office for consultation
regarding dry eyes. She says that the symptoms are worse in the left eye and that
she has to tape her eyes closed before sleeping to minimize dryness. Physical
examination shows prominent globes with 1 mm of scleral show on the left and no
scleral show on the right. Severe bilateral lid laxity is noted, along with 8 mm of
anterior distraction of the lower lids. Snap-back test is greater than one second.
Schirmer's test indicates adequate tear production. Results of thyroid testing are
within normal limits. Which of the following is the most appropriate management of
this patient's symptoms?
A) Full-thickness skin grafting to the lower eyelid
B) Lateral canthotomy and canthoplasty
C) Lateral tarsorrhaphy
D) Suspension via a sub-orbicularis oculi fat (SOOF) lift
E) Wedge resection lower lid tightening

The correct response is Option B.

The main symptom of dry eyes in the patient described is not likely caused by poor tear
production but by accelerated evaporative loss from poor tone of the lower eyelids. The patient's
excess laxity is best described by the 8 mm of distraction of the anterior lower lid and the poor
snap-back test results. The most appropriate management of this patient's symptoms is lateral
canthal support.

The minimal scleral show/ectropion indicates that full-thickness skin grafting is not likely to be
beneficial.

For 1–2 mm of distraction, lateral suspension via a sub-orbicularis oculi fat (SOOF) lift is
effective. For 3–6 mm of distraction, a lateral canthopexy lid tightening without shortening is
effective. For greater than 6 mm of distraction, a lateral canthal division and canthoplasty with
resection with 2–3 mm of full-thickness lid margin is effective.

With this patient's prominent eyes, the placement of the tendon should be more superior to avoid
clotheslining or tightening the lid underneath the limbus, such as would happen with wedge
resection.

This examination contains test materials that are owned and copyrighted by the American Society of
Plastic Surgeons and the Plastic Surgery Educational Foundation. Any reproduction of these materials or
any part of them, through any means, including but not limited to, copying or printing electronic files,
reconstruction through memorization or dictation, and/or dissemination of these materials or any part of
them is strictly prohibited. Keep printed materials in a secure location when you are not reviewing them
and discard them in a secure manner, such as shredding, when you have completed the examination.

Page 87 of 287
American Society of Plastic Surgeons and the Plastic Surgery Educational Foundation
In-Service Examination

References

1. Codner MA, Hanna MK. Upper and lower blepharoplasty. In: Nahai F, ed. The Art of
Aesthetic Surgery, Principles & Techniques. St. Louis: Quality Medical Publishing;
2005:704-705.
2. Flowers RS, DuVal C. Blepharoplasty and periorbital aesthetic surgery. In: Aston SJ,
Beasley RW, Thorne CHM, eds. Grabb & Smith's Plastic Surgery. 5th ed.
Philadelphia: Lippincott-Raven; 1997:628-629.
3. McCord CD. Avoidance and treatment of complications of aesthetic eyelid surgery.
In: Nahai F, ed. The Art of Aesthetic Surgery, Principles & Techniques. St. Louis:
Quality Medical Publishing; 2005:783-795.

This examination contains test materials that are owned and copyrighted by the American Society of
Plastic Surgeons and the Plastic Surgery Educational Foundation. Any reproduction of these materials or
any part of them, through any means, including but not limited to, copying or printing electronic files,
reconstruction through memorization or dictation, and/or dissemination of these materials or any part of
them is strictly prohibited. Keep printed materials in a secure location when you are not reviewing them
and discard them in a secure manner, such as shredding, when you have completed the examination.

Page 88 of 287
American Society of Plastic Surgeons and the Plastic Surgery Educational Foundation
In-Service Examination

60. A 25-year-old woman is scheduled to undergo rhinoplasty using lateral nasal


osteotomy using an external perforating technique. Which of the following arteries
is most susceptible to injury during this procedure?
A) Angular
B) Anterior ethmoidal
C) Dorsal nasal
D) Infraorbital
E) Lateral nasal

The correct response is Option A.

An external perforating osteotomy is performed at the nasofacial junction. When performing this
osteotomy, care must be taken to avoid the angular artery. This artery arises from the facial
artery and makes its way toward the nose. Once on the nose proper, the angular artery becomes
the lateral nasal artery. The lateral nasal artery provides blood supply to the nasal tip and is not
located in the region where the osteotomy is performed.

The external nasal branch of the anterior ethmoidal artery and the dorsal nasal artery supply
blood to the lateral nasal tissues and dorsum. These vessels are both located on the nose itself
and are above the level of the osteotomy. Finally, the infraorbital artery and its foramen are
located lateral to the osteotomy site.

References

1. Gryskiewicz JM, Gryskiewicz KM. Nasal osteotomies: a clinical comparison of the


perforating methods versus the continuous technique. Plast Reconstr Surg.
2004;113:1445-1458.
2. Rohrich RJ, Krueger JK, Adams WP. Importance of lateral nasal osteotomy: an
external perforated technique. In: Gunter JP, Rohrich RJ, Adams WP, ed. Dallas
Rhinoplasty: Nasal Surgery by the Masters. St. Louis: Quality Medical Publishing;
2002;1:615-631.
3. Oneal RM, Beil RJ Jr, Schlesinger J. Surgical anatomy of the nose. Clin Plast Surg.
1996;23:195-222.
4. Rohrich RJ, Janis JE, Adams WP, et al. An update on the lateral nasal osteotomy in
rhinoplasty: an anatomic endoscopic comparison of the external versus the internal
approach. Plast Reconstr Surg. 2003;111:2461-2463.

This examination contains test materials that are owned and copyrighted by the American Society of
Plastic Surgeons and the Plastic Surgery Educational Foundation. Any reproduction of these materials or
any part of them, through any means, including but not limited to, copying or printing electronic files,
reconstruction through memorization or dictation, and/or dissemination of these materials or any part of
them is strictly prohibited. Keep printed materials in a secure location when you are not reviewing them
and discard them in a secure manner, such as shredding, when you have completed the examination.

Page 89 of 287
American Society of Plastic Surgeons and the Plastic Surgery Educational Foundation
In-Service Examination

61. A 58-year-old nulligravid woman who is scheduled to undergo bilateral prophylactic


mastectomy comes to the office for consultation regarding immediate breast
reconstruction. She works full time as a fitness instructor. The patient currently
wears a size 34B brassiere and wants her brassiere size to be increased to a C
cup, but she wants to make sure that scarring is minimized. Height is 5 ft 5 in and
weight is 120 lb. Physical examination of the abdomen shows paucity of extra
tissue. Which of the following is the most appropriate breast reconstruction
procedure for this patient?
A) Bilateral autogenous reconstruction
B) Bilateral tissue expansion followed by implantation of prostheses
C) Delayed breast reconstruction after pathology is confirmed
D) Single-stage reconstruction with prostheses
E) TRAM flap followed by implantation of prostheses

The correct response is Option B.

In the thin patient described, autogenous tissue transfer often requires implantation of prostheses
to provide enough volume to create a breast mound. This can be done alone or in combination
with either the latissimus dorsi myocutaneous flap or the TRAM flap. A recent study suggests
that there may be less risk of recipient site morbidity in reconstruction with a TRAM flap with
prostheses placed or completely filled subsequently compared with immediate placement of
prostheses.

Bilateral latissimus dorsi or TRAM flaps may not be the first choice for a fitness instructor who
will most likely require prostheses. The recovery time and the associated donor site morbidity
may prevent her from returning to the gym as an instructor. Tissue expander placement and
placement of prostheses can often be done through the same incisions used for extirpation and as
such have the least amount of visible scar. No additional tissue is used to create the breast
mound. Immediate breast reconstruction can offer some psychological benefits for the woman
and may be associated with fewer surgeries overall. Prostheses placed without an expanded
pocket may not be as reliable in reconstruction of moderate-sized breasts.

References

1. Kronowitz SJ, Robb GL, Youssef A, et al. Optimizing autologous breast


reconstruction in thin patients. Plast Reconstr Surg. 2003;112:1768-1778.
2. Spear SL, Carter ME, Schwarz K. Prophylactic mastectomy: indications, options, and
reconstructive alternatives. Plast Reconstr Surg. 2005;115:891-909.

This examination contains test materials that are owned and copyrighted by the American Society of
Plastic Surgeons and the Plastic Surgery Educational Foundation. Any reproduction of these materials or
any part of them, through any means, including but not limited to, copying or printing electronic files,
reconstruction through memorization or dictation, and/or dissemination of these materials or any part of
them is strictly prohibited. Keep printed materials in a secure location when you are not reviewing them
and discard them in a secure manner, such as shredding, when you have completed the examination.

Page 90 of 287
American Society of Plastic Surgeons and the Plastic Surgery Educational Foundation
In-Service Examination

62. Which of the following structures of the eye acts as a suspensory system for the
globe?
A) Lockwood ligament
B) Medial canthal tendon
C) Orbital septum
D) Superior oblique tendon
E) Whitnall ligament

The correct response is Option A.

The Lockwood ligament, which attaches posterior to the lacrimal sac, acts as a suspensory
system for the globe. Together with the intermuscular septa and the Tenon capsule, it forms the
lower eyelid retractor system. Posteriorly, it arises from fibrous attachments to the inferior side
of the inferior rectus muscle and continues anteriorly as the capsulopalpebral fascia (lower eyelid
retractors). The medial aspect of the Lockwood ligament attaches to the posterior lacrimal crest,
and the lateral retinaculum attaches to the lateral orbital (Whitnall) tubercle.

The other structures do not function as part of the suspensory system for the globe. The medial
canthal tendon, the deep head of the preseptal orbicularis, and the deep head of the preseptal
orbicularis muscles are important in maintaining apposition of the eyelids to the globe. The
orbital septum and the superior oblique tendon do not suspend the globe. The Whitnall (superior
transverse) ligament attaches medially to the trochlea, laterally to the capsule of the lacrimal
gland, and to the lateral orbital wall.

References

1. Zide BM, Jelks GW. Medial canthus. In: Surgical Anatomy of the Orbit. New York:
Raven Press; 1985:41-46.
2. Doxanas MT. Surgical anatomy of the eyelids and orbits. In: Tse DT, ed. Color Atlas
of Ophthalmic Surgery. Oculoplastic Surgery. Philadelphia: JB Lippincott;
1992:1-16.

This examination contains test materials that are owned and copyrighted by the American Society of
Plastic Surgeons and the Plastic Surgery Educational Foundation. Any reproduction of these materials or
any part of them, through any means, including but not limited to, copying or printing electronic files,
reconstruction through memorization or dictation, and/or dissemination of these materials or any part of
them is strictly prohibited. Keep printed materials in a secure location when you are not reviewing them
and discard them in a secure manner, such as shredding, when you have completed the examination.

Page 91 of 287
American Society of Plastic Surgeons and the Plastic Surgery Educational Foundation
In-Service Examination

63. A 28-year-old woman comes to the clinic for consultation regarding augmentation of
the right breast to achieve symmetry with the left breast. She is happy with the size
of the left breast and does not want any alteration to it. Physical examination shows
asymmetry of breast volume and shape; the right breast is 40% smaller than the
left. The nipple-areola complex is much smaller on the ipsilateral hypoplastic right
side, and the anterior axillary fold is absent. Which of the following is the most likely
diagnosis?
A) Cystosarcoma phylloides
B) Poland syndrome
C) Scoliosis
D) Tuberous breast deformity
E) Normal asymmetry of the breasts

The correct response is Option B.

Poland syndrome is an anomaly associated with underdevelopment of the breast. It is believed to


be related to a vascular compromise of the subclavian vessel supplying the breast, ribs, pectoralis
major muscle, and ipsilateral upper extremity. The sternal head of the pectoralis muscle is most
often missing.

Options for reconstruction include autologous and prosthetic implants. If the deformity is minor
and good tissue cover exists, implant reconstruction is often adequate.

Tuberous breast deformity involves lack of breast tissue forming between the pectoralis muscle
and the skin. It is often defined by a high inframammary fold, large areola, and herniated breast
tissue through the enlarged areola. Anterior lateral thoracotomies performed before breast
development can be associated with underdeveloped breasts, but this does not explain all of the
findings in this patient. Previous breast surgery is also associated with asymmetries of volume
and shape but not with absence of the anterior axillary fold.

Cystosarcoma phylloides is similar to fibroadenoma. It usually presents as unilateral asymmetry


and can be benign. However, this would not explain all of the abnormalities in this patient.

References

1. Longaker MT, Glat PM, Colen LB, et al. Reconstruction of breast asymmetry in
Poland's chest-wall deformity using microvascular free flaps. Plast Reconstr Surg.
1997;99:429-436.

This examination contains test materials that are owned and copyrighted by the American Society of
Plastic Surgeons and the Plastic Surgery Educational Foundation. Any reproduction of these materials or
any part of them, through any means, including but not limited to, copying or printing electronic files,
reconstruction through memorization or dictation, and/or dissemination of these materials or any part of
them is strictly prohibited. Keep printed materials in a secure location when you are not reviewing them
and discard them in a secure manner, such as shredding, when you have completed the examination.

Page 92 of 287
American Society of Plastic Surgeons and the Plastic Surgery Educational Foundation
In-Service Examination

2. Grolleau J-L, Lanfrey E, Lavigne B, et al. Breast base anomalies: treatment strategy
for tuberous breasts, minor deformities, and asymmetry. Plast Reconstr Surg.
1999;104:2040-2048.
3. Bleiziffer S, Schreiber C, Burgkart R, et al. The influence of right anterolateral
thoracotomy in prepubescent female patients on late breast development and on the
incidence of scoliosis. J Thorac Cardiovasc Surg. 2004;127:1474-1480.
4. Bostwick J. Plastic and Reconstructive Surgery. 2nd ed. St. Louis: Quality Medical
Publishing; 2000:689.

This examination contains test materials that are owned and copyrighted by the American Society of
Plastic Surgeons and the Plastic Surgery Educational Foundation. Any reproduction of these materials or
any part of them, through any means, including but not limited to, copying or printing electronic files,
reconstruction through memorization or dictation, and/or dissemination of these materials or any part of
them is strictly prohibited. Keep printed materials in a secure location when you are not reviewing them
and discard them in a secure manner, such as shredding, when you have completed the examination.

Page 93 of 287
American Society of Plastic Surgeons and the Plastic Surgery Educational Foundation
In-Service Examination

64. A 60-year-old woman has a persistent asymmetric smile resulting from an injury to
the right marginal mandibular nerve during rhytidectomy with SMAS plication two
years ago. Physical examination shows elevation of the right oral commissure.
Which of the following interventions is the most appropriate initial step in restoring
symmetry of this patient's smile?
A) Anesthetic motor block of the left depressor labii inferioris muscle
B) Botulinum toxin (Botox) injection into the left depressor labii inferioris muscle
C) Repair of right marginal mandibular nerve
D) Resection of the left oral depressor muscles
E) Transection of the left marginal mandibular nerve

The correct response is Option A.

Performing an anesthetic motor block of the left depressor labii inferioris muscle is the most
appropriate initial step in restoring symmetry of the smile of the patient described because it will
demonstrate the potential outcome of resection of this muscle. If the patient is satisfied with
these temporary results, then longer-lasting (Botox) or permanent resection of the left depressor
labii inferioris muscle can be performed.

Marginal mandibular nerve palsy is a known complication of facelift procedures. After two
years, the neuromuscular junctions in the denervated muscle will not likely recover function.
Repairing the nerve will have little effect on the patient's smile.

Resuspension with a dynamic temporalis sling is adequate for restoration of oral competence and
establishment of movement in patients with global facial nerve palsy. It will not likely result in
the restoration of a symmetric smile.

Although resection of the left oral depressor muscles or transection of the left marginal
mandibular nerve may also improve the symmetry of the patient's smile, these procedures are
permanent and may not have the desired effect.

References

1. Godwin Y, Tomat L, Manktelow R. The use of local anesthetic motor block to


demonstrate the potential outcome of depressor labii inferioris resection in patients
with facial paralysis. Plast Reconstr Surg. 2005;116:957-961.
2. Breslow GD, Cabiling D, Kanchwala S, et al. Selective marginal mandibular
neurectomy for treatment of the marginal mandibular lip deformity in patients with
chronic unilateral facial palsies. Plast Reconstr Surg. 2005;116:1223-1232.

This examination contains test materials that are owned and copyrighted by the American Society of
Plastic Surgeons and the Plastic Surgery Educational Foundation. Any reproduction of these materials or
any part of them, through any means, including but not limited to, copying or printing electronic files,
reconstruction through memorization or dictation, and/or dissemination of these materials or any part of
them is strictly prohibited. Keep printed materials in a secure location when you are not reviewing them
and discard them in a secure manner, such as shredding, when you have completed the examination.

Page 94 of 287
American Society of Plastic Surgeons and the Plastic Surgery Educational Foundation
In-Service Examination

(Please note that this pictorial appears in color in the online examination)

65. A 16-year-old girl is brought to the office by her parents because she has had
worsening pain in the right breast (shown) for the past eight months. Ten years
ago, she sustained a full-thickness burn to the back, right chest, and abdomen
requiring excision and split-thickness skin grafting. Physical examination shows
contracture of the scar and lateral expansion of the breast. After release of the burn
scar contractures, a defect of 200 cm2 is created. Which of the following is most
appropriate for coverage of this defect?
A) Dermal regeneration template (Integra)
B) Latissimus dorsi musculocutaneous flap
C) Meshed split-thickness skin graft
D) Nonmeshed thin split-thickness skin graft
E) Z-plasty release of the constricted scar

The correct response is Option A.

The patient described is suffering from pain caused by a developing breast that is constricted
under a nonexpanding split-thickness skin graft placed at the time of the burn excision. The burn

This examination contains test materials that are owned and copyrighted by the American Society of
Plastic Surgeons and the Plastic Surgery Educational Foundation. Any reproduction of these materials or
any part of them, through any means, including but not limited to, copying or printing electronic files,
reconstruction through memorization or dictation, and/or dissemination of these materials or any part of
them is strictly prohibited. Keep printed materials in a secure location when you are not reviewing them
and discard them in a secure manner, such as shredding, when you have completed the examination.

Page 95 of 287
American Society of Plastic Surgeons and the Plastic Surgery Educational Foundation
In-Service Examination

scar contractures are first released to allow expansion of the underlying breast. The decision for
coverage material is then based on what will most likely permit further expansion of the breast.

Dermal regeneration template (Integra) has been shown to allow secondary expansion and is the
most appropriate choice for the scenario described. The wound is covered with an expandable
dermal layer, which is then covered again by a split-thickness skin graft, leaving a minimal
donor scar.

A Z-plasty would be inadequate to release this large scar.

Full-thickness skin grafting would allow greater breast expansion and less secondary burn scar
contracture, but a graft of the size needed would be difficult to obtain.

A latissimus dorsi flap would not be the best option for the patient described because she has a
history of burns on the back. Also, a latissimus dorsi flap would be bulky, and the patient
described does not require additional volume.

Nonmeshed thin split-thickness skin grafting would give a superior aesthetic result compared
with meshed split-thickness skin grafting; however, both would result in secondary burn scar
contracture, which would not allow further expansion of the breast.

References

1. MacLennan SE, Wells MD, Neale HW. Reconstruction of the burned breast. Clin Plast
Surg. 2000;27:113-119.
2. Palao R, Gomez P, Huguet P. Burned breast reconstructive surgery with Integra dermal
regeneration template. Br J Plast Surg. 2003;56:252-259.

This examination contains test materials that are owned and copyrighted by the American Society of
Plastic Surgeons and the Plastic Surgery Educational Foundation. Any reproduction of these materials or
any part of them, through any means, including but not limited to, copying or printing electronic files,
reconstruction through memorization or dictation, and/or dissemination of these materials or any part of
them is strictly prohibited. Keep printed materials in a secure location when you are not reviewing them
and discard them in a secure manner, such as shredding, when you have completed the examination.

Page 96 of 287
American Society of Plastic Surgeons and the Plastic Surgery Educational Foundation
In-Service Examination

66. A 41-year-old woman comes to the office for consultation regarding irregular skin
tone and uneven skin surface on the outer thighs one year after she underwent
liposuction of the hips and thighs. On physical examination, the irregularities are
radially oriented around a single small scar approximately 1 cm wide. No other
scars are visible. The most likely explanation for these findings is use of which of
the following during the liposuction procedure?
A) Deep large cannula suction with cross-tunneling
B) Fine cannula suction with cross-tunneling
C) Fine cannula suction without cross-tunneling
D) Superficial large cannula suction through a single port site
E) Superficial large cannula suction with cross-tunneling

The correct response is Option D.

One of the most common deformities of liposuction is surface irregularity, which can result from
several etiologies. A large cannula will create a large furrow that may be visualized if the
liposuction is not done evenly. Use of a single port may also lead to irregularities because the
suctioning is done from only one angle. Superficial suctioning is also more prone to visible
irregularities. The best way to avoid this problem is to use small cannulas in the deep fat, with
cross-tunneling from two sites such that the tunnels are at right angles to each other.

References

1. Gasparotti M, Lewis CM, Toledo LS. Superficial Liposculpture: Manual of


Technique. New York: Springer Verlag, 1993.
2. Illouz YG. Body contouring by lipolysis: a 5-year experience with over 3000 cases.
Plast Reconstr Surg. 1983;72:591-597.
3. Illouz YG. Refinements in the lipoplasty technique. Clin Plast Surg.
1989;16:217-233.
4. Illouz YG. History and current concepts of lipoplasty. Clin Plast Surg.
1996;23:721-730.

This examination contains test materials that are owned and copyrighted by the American Society of
Plastic Surgeons and the Plastic Surgery Educational Foundation. Any reproduction of these materials or
any part of them, through any means, including but not limited to, copying or printing electronic files,
reconstruction through memorization or dictation, and/or dissemination of these materials or any part of
them is strictly prohibited. Keep printed materials in a secure location when you are not reviewing them
and discard them in a secure manner, such as shredding, when you have completed the examination.

Page 97 of 287
American Society of Plastic Surgeons and the Plastic Surgery Educational Foundation
In-Service Examination

67. Which of the following is the estimated incidence of detection of occult breast
cancer during reduction mammaplasty?
A) 0.002%
B) 0.02%
C) 0.2%
D) 2%
E) 20%

The correct response is Option C.

The average risk of occult breast cancer in patients undergoing breast reduction has been
estimated to be 0.27% (0.11%, 0.75%, 0.71%, 0.05%, 0.06%, 0.16%, and 0.09% in seven journal
articles published between 1997 and 2006). Cancer is often found at the in situ or early stages.
Definitive therapy depends on size, pathology, location, and status of margins. Consultation with
an oncologist and further pathologic analysis are generally necessary.

References

1. Hage JJ, Karim RB. Risk of breast cancer among reduction mammaplasty patients
and the strategies used by plastic surgeons to detect such cancer. Plast Reconstr Surg.
2006;117:727-735.
2. Tang CL, Brown MH, Levine R, et al. Breast cancer found at the time of breast
reduction. Plast Reconstr Surg. 1999;103:1682-1686.

This examination contains test materials that are owned and copyrighted by the American Society of
Plastic Surgeons and the Plastic Surgery Educational Foundation. Any reproduction of these materials or
any part of them, through any means, including but not limited to, copying or printing electronic files,
reconstruction through memorization or dictation, and/or dissemination of these materials or any part of
them is strictly prohibited. Keep printed materials in a secure location when you are not reviewing them
and discard them in a secure manner, such as shredding, when you have completed the examination.

Page 98 of 287
American Society of Plastic Surgeons and the Plastic Surgery Educational Foundation
In-Service Examination

68. A 32-year-old woman comes to the office for consultation regarding wrinkling along
the sides of her nose. She received botulinum toxin (Botox) injections for forehead
lines, glabellar furrowing, and crow's feet one month ago. Physical examination
shows vertical rhytides on the nasal sidewalls. Contraction of which of the following
muscles is the most likely cause of these findings?
A) Corrugator
B) Frontalis
C) Nasalis
D) Orbicularis oculi
E) Procerus

The correct response is Option C.

Activity of the transverse portion of the nasalis muscle causes bunny lines.

The frontalis muscle and the corrugators are responsible for forehead lines and glabellar lines,
respectively. The lateral portions of the orbicularis oculi are responsible for crow's feet. The
procerus is responsible for horizontal nasal lines, but not wrinkles on the sides of the nose.

References

1. Carruthers J, Fagien S, Matarasso SL, Botox Consensus Group. Consensus


recommendations on the use of botulinum toxin Type A in facial aesthetics. Plast
Reconstr Surg. 2004;114(6 suppl):1S-22S.
2. Carruthers J, Carruthers A. Aesthetic botulinum A toxin in the mid and lower face
and neck. Dermatol Surg. 2003;29(5):468-476.

This examination contains test materials that are owned and copyrighted by the American Society of
Plastic Surgeons and the Plastic Surgery Educational Foundation. Any reproduction of these materials or
any part of them, through any means, including but not limited to, copying or printing electronic files,
reconstruction through memorization or dictation, and/or dissemination of these materials or any part of
them is strictly prohibited. Keep printed materials in a secure location when you are not reviewing them
and discard them in a secure manner, such as shredding, when you have completed the examination.

Page 99 of 287
American Society of Plastic Surgeons and the Plastic Surgery Educational Foundation
In-Service Examination

69. A 55-year-old woman is referred to the office by her primary physician because she
has had a painful, chronic area of scabbing on the right ear for the past three
months. Physical examination shows a 2 x 1-mm area of skin ulceration on the
superior helix of the ear. Surrounding inflammation and exposed cartilage are
noted. Which of the following is the most appropriate initial step in management?
A) Daily wound dressing with bacitracin ointment and avoidance of pressure on the ear
B) Excision with 5-mm margins and coverage with a retroauricular trap door flap
C) Excision with 5-mm margins and reconstruction with an Antia-Buch advancement
flap
D) Excisional biopsy and primary wound closure
E) Injection of triamcinolone and daily topical application of hydrocortisone cream

The correct response is Option D.

The patient described most likely has a benign condition referred to as chondrodermatitis
nodularis helicis, an inflammation of the cartilage of the ear that often leads to a painful open
area. This condition can mimic a skin cancer on physical examination. Unless the area is clearly
benign, biopsy is warranted to rule out skin cancer. Recurrence rate for this condition is high.
Excisional biopsy and primary wound closure will give a pathological confirmation and may
suffice as treatment.

Daily wound dressing with bacitracin ointment or injection of triamcinolone and daily topical
application of hydrocortisone cream are both inappropriate because neither would give the
appropriate tissue diagnosis. Excision and reconstruction or coverage with a flap are unwarranted
given no pathological diagnosis.

Chondrodermatitis nodularis helicis is a condition of unknown etiology predominantly found in


older men and often is associated with trauma from sleeping. It begins as an area of cartilage
inflammation and then ulcerates through the skin. The treatment is excision of the cartilage and
closure of the skin. Recurrence rates remain high. Preventive measures to decrease the
recurrence rate include avoidance of sleeping on the affected ear.

References

1. Elsahy N. Acquired ear defects. Clin Plast Surg. 2002;29:175-186.


2. Zaren HA, Lowe NJ. Benign growth and generalized skin disorders. In: Aston SJ,
Beasley RW, Thorne CHM, eds. Grabb and Smith’s Plastic Surgery. 5th ed.
Philadelphia: Lippincott-Raven; 1997:141-148.

This examination contains test materials that are owned and copyrighted by the American Society of
Plastic Surgeons and the Plastic Surgery Educational Foundation. Any reproduction of these materials or
any part of them, through any means, including but not limited to, copying or printing electronic files,
reconstruction through memorization or dictation, and/or dissemination of these materials or any part of
them is strictly prohibited. Keep printed materials in a secure location when you are not reviewing them
and discard them in a secure manner, such as shredding, when you have completed the examination.

Page 100 of 287


American Society of Plastic Surgeons and the Plastic Surgery Educational Foundation
In-Service Examination

70. A 27-year-old woman (weight, 60 kg) undergoes suction lipectomy of the abdomen,
flanks, and outer thighs in an outpatient setting. During the procedure, 3 L of
tumescent solution containing 0.1% lidocaine and 1:1,000,000 epinephrine in
normal saline are infused and 3 L of aspirate are removed using a 20-cm-long,
4-mm-wide cannula. The patient is discharged in good condition two hours after
completion of the procedure. She is found dead in her bed the next morning. Which
of the following is the most likely cause of death?
A) Epinephrine toxicity
B) Lidocaine toxicity
C) Perforation of the abdominal wall and colon
D) Perforation of the diaphragm and tension pneumothorax
E) Pulmonary embolism

The correct response is Option E.

Pulmonary embolization with fat or thrombus is the most common cause of death from suction
lipectomy. One survey of 917 aesthetic plastic surgeons showed the leading cause of death from
suction lipectomy to be thromboembolism (23%), followed by perforation of the
abdomen/viscera (15%), anesthesia/sedation/medication (10%), fat embolism (9%), and
unknown or confidential causes (29%).

References

1. Grazer FM, de Jong RH. Fatal outcomes from liposuction: census survey of cosmetic
surgeons. Plast Reconstr Surg. 2000;105:436-446.
2. Iverson RE, Lynch DJ, ASPS Committee on Patient Safety. Practice advisory on
liposuction. Plast Reconstr Surg. 2004;113:1478-1490.

This examination contains test materials that are owned and copyrighted by the American Society of
Plastic Surgeons and the Plastic Surgery Educational Foundation. Any reproduction of these materials or
any part of them, through any means, including but not limited to, copying or printing electronic files,
reconstruction through memorization or dictation, and/or dissemination of these materials or any part of
them is strictly prohibited. Keep printed materials in a secure location when you are not reviewing them
and discard them in a secure manner, such as shredding, when you have completed the examination.

Page 101 of 287


American Society of Plastic Surgeons and the Plastic Surgery Educational Foundation
In-Service Examination

71. A 28-year-old woman comes to the office one year after undergoing transaxillary
augmentation mammaplasty because she is dissatisfied with the shape of her
breasts (shown). Physical examination shows large prostheses positioned slightly
low on the chest and a transverse crease that is clearly visible in the lower pole of
each breast (double-bubble deformity). Which of the following is the most likely
cause of these findings?
A) Contractile forces of the muscles
B) Excessive lowering of the inframammary fold
C) Formation of capsular contracture
D) Poland syndrome
E) Ptosis of the overlying natural breast parenchyma

This examination contains test materials that are owned and copyrighted by the American Society of
Plastic Surgeons and the Plastic Surgery Educational Foundation. Any reproduction of these materials or
any part of them, through any means, including but not limited to, copying or printing electronic files,
reconstruction through memorization or dictation, and/or dissemination of these materials or any part of
them is strictly prohibited. Keep printed materials in a secure location when you are not reviewing them
and discard them in a secure manner, such as shredding, when you have completed the examination.

Page 102 of 287


American Society of Plastic Surgeons and the Plastic Surgery Educational Foundation
In-Service Examination

The correct response is Option B.

The most likely cause of this deformity is excessive or inadvertent lowering of the
inframammary fold. This deformity is difficult to correct without further surgery, so the surgeon
should be cognizant of this crucial fold area. If a patient desires a large prosthesis and/or if the
distance from the inframammary fold to the nipple is short, it may be desirable to lower the
inframammary fold. In these patients, the original inframammary fold must be completely
obliterated with serial cuts, often up to the dermis. If the inframammary fold is inadvertently
lowered, it is best to repair the fold with 2-0 sutures and reposition the prosthesis superiorly.

In the literature, both this defect and the defect of native breast tissue sagging over the lower pole
of a high prosthesis are called a "double bubble." Each deformity is caused by a different
problem and should be recognized and treated differently.

References

1. Massiha H. Augmentation in ptotic and densely glandular breasts: prevention,


treatment, and classification of double-bubble deformity. Ann Plast Surg.
2000;44:143-146.
2. Schusterman MA. Lowering the inframammary fold. Aesth Surg J.
2004;Sep/Oct:482-485.

This examination contains test materials that are owned and copyrighted by the American Society of
Plastic Surgeons and the Plastic Surgery Educational Foundation. Any reproduction of these materials or
any part of them, through any means, including but not limited to, copying or printing electronic files,
reconstruction through memorization or dictation, and/or dissemination of these materials or any part of
them is strictly prohibited. Keep printed materials in a secure location when you are not reviewing them
and discard them in a secure manner, such as shredding, when you have completed the examination.

Page 103 of 287


American Society of Plastic Surgeons and the Plastic Surgery Educational Foundation
In-Service Examination

72. In a worm's eye view of the Caucasian nose, which of the following is the most
common ratio of the lobular portion of the nose to the columella?
A) 3:1
B) 2:1
C) 1:1
D) 1:2
E) 1:3

The correct response is Option D.

The lobular portion of the nose from the worm's eye view should be in a 1:2 ratio with the
columella and nasal apertures. The lobular portion comprises one third and the collumellar
portion comprises two thirds of the total distance from the tip to the base. The nostrils should
have a teardrop configuration with the diameter of the base slightly larger than the diameter of
the apex. The long axis of each nostril points in a slight medial direction.

References

1. Rohrich RJ, Muzaffar AR. Primary rhinoplasty. In Achauer, BM, ed. Plastic Surgery:
Indications, Operations, and Outcomes. St. Louis: Mosby; 2000:263-247.
2. Rohrich RJ, Sheen JH. Secondary rhinoplasty. In: Grotting JC, Ed. Reoperative
Aesthetic and Reconstructive Plastic Surgery. St. Louis: Quality Medical Publishing;
1995:401-510.
3. Staffel JG. Basic Principles of Rhinoplasty. San Antonio: University of Texas Health
Science Center at San Antonio; 1996:13.

This examination contains test materials that are owned and copyrighted by the American Society of
Plastic Surgeons and the Plastic Surgery Educational Foundation. Any reproduction of these materials or
any part of them, through any means, including but not limited to, copying or printing electronic files,
reconstruction through memorization or dictation, and/or dissemination of these materials or any part of
them is strictly prohibited. Keep printed materials in a secure location when you are not reviewing them
and discard them in a secure manner, such as shredding, when you have completed the examination.

Page 104 of 287


American Society of Plastic Surgeons and the Plastic Surgery Educational Foundation
In-Service Examination

(Please note that this pictorial appears in color in the online examination)

73. A 32-year-old woman comes to the office for follow-up examination six months after
she underwent repair of a fracture of the left orbit. A preoperative photograph is
shown. Physical examination shows cicatricial ectropion of the lower eyelid. Which
of the following is the most appropriate management?
A) Complete tarsorrhaphy
B) Pentagonal wedge resection of the lower eyelid
C) Placement of a graft to the lower eyelid
D) Punctal occlusion of the lower eyelid
E) Resuspension of the lower eyelid retractors

The correct response is Option C.

Protection of the ocular surfaces is the primary goal in managing the cicatricial ectropion of the
lower eyelid in the patient described. The patient's lower eyelid lacks height. Scarring between
the capsulopalpebral fascia (lower eyelid retractors) and the orbital septum because of trauma or
following aggressive lower eyelid blepharoplasty can cause vertical contracture of the lower
eyelid, resulting in corneal exposure. Lower eyelid position can only be restored by placement of
an autogenous or alloplastic spacer graft (hard palate, donor sclera, auricular cartilage, tarsus,
and others), as in the postoperative image shown. In conjunction, eyelid-tightening procedures
such as lateral canthopexy and canthoplasty address horizontal skin laxity.

Complete tarsorrhaphy is not an option for a young and active patient. It is an acceptable
temporizing solution in cases of paralytic ectropion.

This examination contains test materials that are owned and copyrighted by the American Society of
Plastic Surgeons and the Plastic Surgery Educational Foundation. Any reproduction of these materials or
any part of them, through any means, including but not limited to, copying or printing electronic files,
reconstruction through memorization or dictation, and/or dissemination of these materials or any part of
them is strictly prohibited. Keep printed materials in a secure location when you are not reviewing them
and discard them in a secure manner, such as shredding, when you have completed the examination.

Page 105 of 287


American Society of Plastic Surgeons and the Plastic Surgery Educational Foundation
In-Service Examination

Pentagonal wedge resection of the lower eyelid is an adjunct procedure for cases of lower eyelid
laxity with horizontal skin excess.

Punctal occlusion of the lower eyelid would alleviate the dryness caused by the increased
exposure but would not restore normal anatomy of the lower eyelid in the patient described.

Repair of the lower eyelid retractors in conjunction with lateral canthal resuspension is used to
correct lower eyelid entropion. The procedure would not address the scarring present in the
patient described.

References

1. Patel MP, Shapiro MD, Spinelli HM. Combined hard palate spacer graft, midface
suspension, and lateral canthoplasty for lower eyelid retraction: a tripartite approach.
Plast Reconstr Surg. 2005;115:2105-2114.
2. Jelks GW, Glat PM, Jelks EB, et al. The inferior retinacular lateral canthoplasty: a
new technique. Plast Reconstr Surg. 1997;100:1262-1270.

This examination contains test materials that are owned and copyrighted by the American Society of
Plastic Surgeons and the Plastic Surgery Educational Foundation. Any reproduction of these materials or
any part of them, through any means, including but not limited to, copying or printing electronic files,
reconstruction through memorization or dictation, and/or dissemination of these materials or any part of
them is strictly prohibited. Keep printed materials in a secure location when you are not reviewing them
and discard them in a secure manner, such as shredding, when you have completed the examination.

Page 106 of 287


American Society of Plastic Surgeons and the Plastic Surgery Educational Foundation
In-Service Examination

74. A 16-year-old girl is referred to the office by her pediatrician for reduction
mammaplasty because of breast hypertrophy that has been worsening over the
past two years. She wears a size 36DD brassiere and has constant pain in the
shoulders and back from the weight of her breasts. Height is 5 ft 4 in, weight is
165 lb, and menarche was at 10 years of age. Physical examination shows juvenile
breast hypertrophy, shoulder grooving, intertrigal dermatitis, and striae. Abnormality
of which of the following is the most likely cause of this patient's condition?
A) End-organ responsiveness to estrogen
B) Estrogen level
C) Number of estrogen receptors
D) Progesterone level
E) Prolactin level

The correct response is Option A.

Abnormal end-organ responsiveness to estrogen is the predominant factor leading to breast


hypertrophy. It has been demonstrated that normal levels of estrogen, progesterone, and prolactin
exist in patients with juvenile breast hypertrophy. These patients also have a normal number of
estrogen receptors.

References

1. Pang S. Premature thelarche and premature adrenarche. Pediatr Ann. 1981;10:29.


2. Ilicki A, Prager Lewin R, Kauli R, et al. Premature thelarche—natural history and sex
hormone secretion in 68 girls. Acta Pediatr Scand. 1984;73:756-762.
3. Jabs AD, Frantz AG, Smith-Vaniz A, et al. Mammary hypertrophy is not associated
with increased estrogen receptors. Plast Reconstr Surg. 1990;86:64-66.

This examination contains test materials that are owned and copyrighted by the American Society of
Plastic Surgeons and the Plastic Surgery Educational Foundation. Any reproduction of these materials or
any part of them, through any means, including but not limited to, copying or printing electronic files,
reconstruction through memorization or dictation, and/or dissemination of these materials or any part of
them is strictly prohibited. Keep printed materials in a secure location when you are not reviewing them
and discard them in a secure manner, such as shredding, when you have completed the examination.

Page 107 of 287


American Society of Plastic Surgeons and the Plastic Surgery Educational Foundation
In-Service Examination

75. A 5-year-old boy is brought to the office by his parents for consultation regarding
correction of Stahl's ear deformity. Which of the following is the most appropriate
surgical management?
A) Partial detachment of the folded segment of helical cartilage from the scapha and
repositioning of the helix
B) Placement of sutures from the conchal bowl to the mastoid fascia
C) Rasping of the anterior surface of the antihelical cartilage to create the antihelix
D) Release of the superior auricle from the temporal skin with skin grafting to create a
retroauricular sulcus
E) Wedge excision of the third crus with helical advancement

The correct response is Option E.

Stahl's ear is an auricular deformity that primarily includes a third antihelical crus. One of the
many methods of repair involves wedge excision of the third crus with helical advancement. The
other techniques listed are used for other deformities of the ear.

The constricted ear is a deformity that involves a “constricted” helical rim with the superior
portion of the helix often folding over the scapha. This deformity has also been referred to as
“cup” or “lop” ear and occurs with varying degrees of severity. One method of repair includes
partially detaching the helix from the scapha and resuturing it to the scapha at the proper angle.

Prominent ears are characterized by a wide conchoscaphal angle and are often associated with
flattening of the antihelical fold. Various corrective methods have been described, including
rasping of the anterior surface of the antihelix, placement of retention sutures to recreate the
antihelical fold, conchal bowl resection, and suturing of the conchal bowl to the mastoid fascia.

Cryptotia is an adherence of the superior portion of the helix to the temporal skin with varying
degrees of severity. Surgical correction involves release of this abnormal connection and
frequently requires skin grafting, depending on the severity of the deformity.

References

1. Leber D. Ear reconstruction. In: Georgiade GS, Riefkohl RR, Levin LS, eds.
Georgiade Plastic, Maxillofacial and Reconstructive Surgery. 3rd ed. Baltimore:
Williams & Wilkins; 1997:497.
2. Tanzer R. The constricted ear. Plast Reconstr Surg. 1975;55:406.

This examination contains test materials that are owned and copyrighted by the American Society of
Plastic Surgeons and the Plastic Surgery Educational Foundation. Any reproduction of these materials or
any part of them, through any means, including but not limited to, copying or printing electronic files,
reconstruction through memorization or dictation, and/or dissemination of these materials or any part of
them is strictly prohibited. Keep printed materials in a secure location when you are not reviewing them
and discard them in a secure manner, such as shredding, when you have completed the examination.

Page 108 of 287


American Society of Plastic Surgeons and the Plastic Surgery Educational Foundation
In-Service Examination

3. Ono I, Gunji H, Tateshita T, et al. An operation for Stahl’s ear. Br J Plast Surg.
1996;49:564.
4. Mustarde J. The correction of prominent ears using mattress sutures. Br J Plast Surg.
1963;16:170.
5. Furnas D. The correction of prominent ears by concha-mastoid sutures. Plast
Reconstr Surg. 1968;42:189.

This examination contains test materials that are owned and copyrighted by the American Society of
Plastic Surgeons and the Plastic Surgery Educational Foundation. Any reproduction of these materials or
any part of them, through any means, including but not limited to, copying or printing electronic files,
reconstruction through memorization or dictation, and/or dissemination of these materials or any part of
them is strictly prohibited. Keep printed materials in a secure location when you are not reviewing them
and discard them in a secure manner, such as shredding, when you have completed the examination.

Page 109 of 287


American Society of Plastic Surgeons and the Plastic Surgery Educational Foundation
In-Service Examination

76. A 55-year-old woman undergoes modified radical mastectomy with immediate first-
stage reconstruction of the right breast with a tissue expander. Before the second
stage to exchange the tissue expander with a permanent prosthesis is initiated,
pathology results from analysis of tissue from the right breast indicate metastatic
carcinoma of four axillary lymph nodes. Radiation therapy is recommended. Which
of the following interventions will result in the best long-term appearance of the
reconstructed breast?
A) Complete the tissue expansion before radiation and exchange the tissue expander
with a prosthesis after radiation
B) Deflate the tissue expander before radiation and reinflate the tissue expander and
exchange with a prosthesis after radiation
C) Remove the tissue expander and reconstruct the breast with a TRAM flap before
radiation
D) Remove the tissue expander before radiation; after radiation, reinsert and expand a
tissue expander and then exchange with a prosthesis
E) Remove the tissue expander before radiation and reconstruct the breast with a
TRAM flap after radiation
The correct response is Option E.
Breast reconstruction after radiation therapy is best accomplished with autologous tissue such as
a TRAM flap. Any reconstruction method that uses prostheses or tissue expanders after radiation
therapy has a higher risk of complications and poor cosmetic outcome when compared with
reconstruction using autologous tissue. Radiation increases the risk of infection and exposure of
the prosthesis, leading to failure of breast reconstruction procedures using prostheses. Radiation
also increases the risk for capsular contracture, resulting in a poor aesthetic result.
Breasts that are reconstructed with a TRAM flap and subsequently radiated are prone to
distortion, shrinkage, and fibrosis, leading to a poor aesthetic outcome. Therefore, the best long-
term results are obtained when the breast is reconstructed with a TRAM flap after radiation
therapy has been completed.
References

1. Kroll SS, Baldwin B. A comparison of outcomes using three different methods of


breast reconstruction. Plast Reconstr Surg. 1992;90:455-462.
2. Krueger EA, Wilkins EG, Strawderman H, et al. Complications and patient
satisfaction following expander/implant breast reconstruction with and without
radiotherapy. Int J Radiat Oncol Biol Phys. 2001;49:713-721.

This examination contains test materials that are owned and copyrighted by the American Society of
Plastic Surgeons and the Plastic Surgery Educational Foundation. Any reproduction of these materials or
any part of them, through any means, including but not limited to, copying or printing electronic files,
reconstruction through memorization or dictation, and/or dissemination of these materials or any part of
them is strictly prohibited. Keep printed materials in a secure location when you are not reviewing them
and discard them in a secure manner, such as shredding, when you have completed the examination.

Page 110 of 287


American Society of Plastic Surgeons and the Plastic Surgery Educational Foundation
In-Service Examination

3. Tran NV, Chang DW, Gupta A, et al. Comparison of immediate and delayed free
TRAM flap breast reconstruction in patients receiving postmastectomy radiation
therapy. Plast Reconstr Surg. 2001;108:78-82.

This examination contains test materials that are owned and copyrighted by the American Society of
Plastic Surgeons and the Plastic Surgery Educational Foundation. Any reproduction of these materials or
any part of them, through any means, including but not limited to, copying or printing electronic files,
reconstruction through memorization or dictation, and/or dissemination of these materials or any part of
them is strictly prohibited. Keep printed materials in a secure location when you are not reviewing them
and discard them in a secure manner, such as shredding, when you have completed the examination.

Page 111 of 287


American Society of Plastic Surgeons and the Plastic Surgery Educational Foundation
In-Service Examination

77. A 50-year-old woman undergoes endoscopic brow lift for rejuvenation of the
forehead region. Which of the following interventions is necessary for proper
elevation of the lateral brow areas during this procedure?
A) Injection of botulinum toxin
B) Release of the orbital ligament
C) Resection of the corrugator supercilii muscles
D) Supraperiosteal dissection
E) Upper eyelid blepharoplasty

The correct response is Option B.

The orbital ligament is a band of connective tissue that fixes the superficial temporal fascia to the
bone of the lateral orbital rim. This tissue must be released during the course of the brow lift to
achieve adequate elevation of the lateral brow.

Injection of botulinum toxin and upper eyelid blepharoplasty can be used as adjunctive
procedures to improve the appearance of the forehead, brow, and periorbital regions, but these
procedures are not necessary for lateral brow elevation in a brow-lift procedure. Resection of the
corrugator supercilii muscles can be performed in the course of the brow-lift procedure to
improve the appearance of glabellar rhytides and furrows. Similarly, this procedure is not a
necessary component for lateral brow elevation.

The optimal plane of dissection in an endoscopic brow lift, supraperiosteal (subgaleal) versus
subperiosteal, is a controversial issue. One advantage of the subperiosteal plane of dissection is
the minimal bleeding encountered in this avascular plane. Also, sensory nerve damage may
better be avoided with subperiosteal dissection, which occurs below the plane of the supraorbital
nerve.

References

1. DeCordier BC, de la Torre JI, Al-Hakeem MS, et al. Endoscopic forehead lift: review
of technique, cases, and complications. Plast Reconstr Surg. 2002;110:1558-1568.
2. Knize DM. Galea aponeurotica and temporal fascias. In: Knize DM, ed. The
Forehead and Temporal Fossa Anatomy and Technique. Philadelphia: Lippincott
Williams & Wilkins; 2001:45-72.
3. Bernard RW. Endoscopic brow lift: a personal technique. In: Knize DM, ed. The
Forehead and Temporal Fossa Anatomy and Technique. Philadelphia: Lippincott
Williams & Wilkins; 2001:133-151.

This examination contains test materials that are owned and copyrighted by the American Society of
Plastic Surgeons and the Plastic Surgery Educational Foundation. Any reproduction of these materials or
any part of them, through any means, including but not limited to, copying or printing electronic files,
reconstruction through memorization or dictation, and/or dissemination of these materials or any part of
them is strictly prohibited. Keep printed materials in a secure location when you are not reviewing them
and discard them in a secure manner, such as shredding, when you have completed the examination.

Page 112 of 287


American Society of Plastic Surgeons and the Plastic Surgery Educational Foundation
In-Service Examination

(Please note that this pictorial appears in color in the online examination)

78. A 20-year-old woman comes to the office for follow-up examination two years after
she sustained burns over 80% of the total body surface area. She has difficulty
balancing her eyeglasses on her remnant ears. Physical examination shows a
bilateral burned pinna deformity with loss of the superior helix. A photograph of the
left ear is shown. Which of the following interventions is most appropriate at this
time?
A) Bilateral conchal transposition flap and skin grafting
B) Bilateral insertion of costochondral grafts under existing skin
C) Reconstruction of the ears with auricular osteointegrated prostheses
D) Reconstruction of the ears with Silastic prostheses and temporoparietal flap
coverage
E) Retroauricular release and grafting

This examination contains test materials that are owned and copyrighted by the American Society of
Plastic Surgeons and the Plastic Surgery Educational Foundation. Any reproduction of these materials or
any part of them, through any means, including but not limited to, copying or printing electronic files,
reconstruction through memorization or dictation, and/or dissemination of these materials or any part of
them is strictly prohibited. Keep printed materials in a secure location when you are not reviewing them
and discard them in a secure manner, such as shredding, when you have completed the examination.

Page 113 of 287


American Society of Plastic Surgeons and the Plastic Surgery Educational Foundation
In-Service Examination

The correct response is Option A.

The patient described has sustained a significant injury and multiple reconstructive procedures
are required. Although reconstruction of the ears may not be a high priority, it can be relatively
easily accomplished with a bilateral conchal transposition flap and skin grafting. A conchal
transposition flap has minimal donor site morbidity with a low complication rate and the results
can address the patient's difficulty regarding eyeglasses.

Bilateral reconstruction of the ears with costochondral grafts is not appropriate because it is
associated with significant donor site problems and is often a marginal result due to the severe
burned skin.

Reconstruction of the ears with auricular osteointegrated prostheses is not appropriate because it
has a high complication rate in burn patients.

Reconstruction with Silastic prostheses and temporoparietal flap coverage has a very high
exposure and infection rate and is rarely done. Reconstruction with Medpor has a lower
complication rate and is a potential option to consider for the patient described.

The chondrocutaneous composite flap based on the crux helix was described by Davis in 1974. It
has been advocated by many to assist in the reconstruction in the acquired deformities of the
superior helix. The flap is very reliable and has minimal donor site morbidity.

This examination contains test materials that are owned and copyrighted by the American Society of
Plastic Surgeons and the Plastic Surgery Educational Foundation. Any reproduction of these materials or
any part of them, through any means, including but not limited to, copying or printing electronic files,
reconstruction through memorization or dictation, and/or dissemination of these materials or any part of
them is strictly prohibited. Keep printed materials in a secure location when you are not reviewing them
and discard them in a secure manner, such as shredding, when you have completed the examination.

Page 114 of 287


American Society of Plastic Surgeons and the Plastic Surgery Educational Foundation
In-Service Examination

References

1. Davis J. Reconstruction of the upper third of the helix with a chondrocutaneous


composite flap based on the crus helix. In: Tanzer RC, Edgerton MT, eds. Symposium
on Reconstruction of the Auricle. St. Louis: Mosby; 1974:247.
2. Achauer BM. Reconstruction of burn deformities of the head and neck. In: Cohen M,
ed. Mastery of Plastic and Reconstructive Surgery. Boston: Little, Brown;
1994:416-428.
3. Donelan MB. Conchal transposition flap for postburn ear deformities. Plast Reconstr
Surg. 1989;83:641-654.

This examination contains test materials that are owned and copyrighted by the American Society of
Plastic Surgeons and the Plastic Surgery Educational Foundation. Any reproduction of these materials or
any part of them, through any means, including but not limited to, copying or printing electronic files,
reconstruction through memorization or dictation, and/or dissemination of these materials or any part of
them is strictly prohibited. Keep printed materials in a secure location when you are not reviewing them
and discard them in a secure manner, such as shredding, when you have completed the examination.

Page 115 of 287


American Society of Plastic Surgeons and the Plastic Surgery Educational Foundation
In-Service Examination

79. A 38-year-old woman comes to the office for consultation regarding reduction
mammaplasty. She currently wears a size 36DD brassiere and wants to be able to
wear a C cup brassiere postoperatively. Height is 5 ft 6 in and weight is 120 lb. On
physical examination, the distance from nipple to sternal notch is 35 cm. Selection
of a vertical mammaplasty technique in this patient is limited by which of the
following?
A) Desired postoperative cup size
B) Fat content of the breasts
C) Lateral positioning of the nipples
D) Length of the pedicle
E) Preoperative cup size

The correct response is Option D.

The contraindications to performing a vertical reduction mammaplasty are the length of the
pedicle and the amount and quality of the remaining skin. Blood supply to the nipple may be
compromised in a very long pedicle. It is impossible to give a numeric value to this length.
Lassus recommends a transposition of no more than 9 cm.

Vertical reduction mammaplasty more easily achieves consistently high-quality results when the
method is applied to a small or moderate reduction (< 800 g per side). Although reductions of
more than 1000 g are technically difficult to achieve, they are not impossible and have been
reported by some centers. With the adjunct of the L-shaped scar, larger amounts of skin can be
resected, limiting the need for postoperative revisions.

The lateral positioning of the nipples is not a contraindication to vertical reduction


mammaplasty. Although Lejour reported delayed wound healing in cases with high fat content of
the breast, this does not preclude the vertical method.

References

1. Hall-Findlay EJ. Vertical breast reduction using the superomedial pedicle. In: Spear
SL, ed. Surgery of the Breast: Principles and Art. 2nd ed. Philadelphia: Lippincott
Williams & Wilkins; 2006:1072-1092.
2. Lejour M. Vertical mammaplasty: early complications after 250 personal consecutive
cases. Plast Reconstr Surg. 1999;104:764-770.
3. Hidalgo DA. Vertical mammaplasty. Plast Reconstr Surg. 2005;115:1179-1197.

This examination contains test materials that are owned and copyrighted by the American Society of
Plastic Surgeons and the Plastic Surgery Educational Foundation. Any reproduction of these materials or
any part of them, through any means, including but not limited to, copying or printing electronic files,
reconstruction through memorization or dictation, and/or dissemination of these materials or any part of
them is strictly prohibited. Keep printed materials in a secure location when you are not reviewing them
and discard them in a secure manner, such as shredding, when you have completed the examination.

Page 116 of 287


American Society of Plastic Surgeons and the Plastic Surgery Educational Foundation
In-Service Examination

80. A 45-year-old woman comes to the office for consultation regarding severe skin
laxity of the lower abdomen and medial aspect of the thighs two years after she
underwent gastric bypass surgery. Weight loss during the first 12 months
postoperatively was 120 lb. The patient’s weight has been steady at 140 lb for the
past eight months. Which of the following incisional approaches is most appropriate
for contour of the thighs in this patient?
A) Anterior oblique
B) Crescentic groin
C) Medial longitudinal
D) Posterior oblique
E) Transverse

The correct response is Option C.

In a patient with severe medial thigh skin redundancy after massive weight loss, a medial
longitudinal approach is currently preferred because it removes a maximal amount of skin
redundancy along the entire length of the thigh while concealing the incision in the medial thigh
skin. A crescentic groin incision does attempt to conceal the scar in the groin but can distort the
vulvar structures when placed under tension, and is less effective when medial thigh laxity is
severe. An anterior oblique or posterior oblique incision is not preferred because of the
conspicuous location of the resulting scars. A transverse incision is inadequate to remove enough
skin and results in a poor cosmetic outcome.

References

1. Kenkel JM, Aly AS, Capella JF, et al. Marking and operative techniques, body
contouring surgery after massive weight loss. Plast Reconstr Surg.
2006;117:45SB73S.
2. Hurwitz DJ, Rubin PJ, Risin M, et al. Correcting the saddlebag deformity in the
massive weight loss patient. Plast Reconstr Surg. 2004;114:1313-1325.

This examination contains test materials that are owned and copyrighted by the American Society of
Plastic Surgeons and the Plastic Surgery Educational Foundation. Any reproduction of these materials or
any part of them, through any means, including but not limited to, copying or printing electronic files,
reconstruction through memorization or dictation, and/or dissemination of these materials or any part of
them is strictly prohibited. Keep printed materials in a secure location when you are not reviewing them
and discard them in a secure manner, such as shredding, when you have completed the examination.

Page 117 of 287


American Society of Plastic Surgeons and the Plastic Surgery Educational Foundation
In-Service Examination

81. A 48-year-old woman comes to the office for consultation regarding a breast lift
procedure. Physical examination shows location of the nipples 3 cm inferior to the
inframammary fold and below the lower contour of the breast and skin envelopes.
Which of the following is the most likely diagnosis of this patient's deformity?
A) No ptosis
B) Grade 1 ptosis
C) Grade 2 ptosis
D) Grade 3 ptosis
E) Pseudoptosis

The correct response is Option D.

Regnault defined the degree of ptosis by evaluating the relationship of the nipple to the
inframammary fold.

In Grade 1 or mild ptosis, the nipple is within 1 cm of the level of the inframammary fold and
above the lower contour of the breast and skin envelopes. In Grade 2 or moderate ptosis, the
nipple is 1 to 3 cm below the inframammary fold but above the lower contour of the breast and
skin envelopes. In Grade 3 or severe ptosis, the nipple is more than 3 cm below the
inframammary fold and below the lower contour of the breast and skin envelopes.

In cases of pseudoptosis, the nipple is above or at the level of the inframammary fold with the
majority of the breast tissue below, giving the impression of true ptosis.

References

1. Regnault P. Breast ptosis: definition and treatment. Clin Plast Surg. 1976;3:193-203.
2. Mathes SJ. In: Plastic Surgery: General Principles. 2nd ed. Philadelphia: WB
Saunders; 2006:87.

This examination contains test materials that are owned and copyrighted by the American Society of
Plastic Surgeons and the Plastic Surgery Educational Foundation. Any reproduction of these materials or
any part of them, through any means, including but not limited to, copying or printing electronic files,
reconstruction through memorization or dictation, and/or dissemination of these materials or any part of
them is strictly prohibited. Keep printed materials in a secure location when you are not reviewing them
and discard them in a secure manner, such as shredding, when you have completed the examination.

Page 118 of 287


American Society of Plastic Surgeons and the Plastic Surgery Educational Foundation
In-Service Examination

82. A 53-year-old woman comes to the office for consultation because she is
dissatisfied with the appearance of her eyebrows and forehead. Physical
examination shows hooding of the lateral orbital brow. The most appropriate
management is a brow lift procedure to move the peak of the eyebrow to which of
the following anatomic locations?
A) At the level of the supraorbital rim
B) Between the lateral limbus and lateral canthus
C) Between the midpupil and lateral limbus
D) Medial brow above the orbital rim
E) Medial to the medial canthus

The correct response is Option B.

With aging, the lateral brow often descends more noticeably than the medial brow. Brow lifts are
performed to restore the brow to a more pleasing shape and position. However, patients may fear
brow lift procedures because they do not want to have the brows overly elevated, resulting in a
constant surprised appearance. Several studies have examined eyebrow aesthetics. Although no
one brow position or shape is ideal, given our multicultural society and changes in fashion
aesthetics over time, there is a consensus in the literature regarding eyebrow aesthetics.

In women, the eyebrow should have a gentle arch shape with the peak residing between the
lateral limbus and lateral canthus. The medial brow should begin above or just medial to the
medial canthus and should be lower than the lateral peak. The peak of the brow arch should be
no more than 10 mm above the most caudal portion of the medial brow. The eyebrow should
arch above the supraorbital rim, with the medial portion at or below the supraorbital rim.

In men, eyebrows should be horizontal or flat with no arch shape. The male eyebrow is lower
than the female eyebrow and usually follows the orbital rim. The lateral portion of the brow is
typically fuller in men than in women.

References

1. Freund RM, Nolan WB III. Correlation between brow lift outcomes and aesthetic
ideals for eyebrow height and shape in females. Plast Reconstr Surg.
1996;97:1343-1348.
2. Gunter JP, Antrobus SD. Aesthetic analysis of the eyebrows. Plast Reconstr Surg.
1997;99:1808-1816.
3. Roth JM, Metzinger SE. Quantifying the arch position of the female eyebrow. Arch
Facial Plast Surg. 2003;5:235-239.

This examination contains test materials that are owned and copyrighted by the American Society of
Plastic Surgeons and the Plastic Surgery Educational Foundation. Any reproduction of these materials or
any part of them, through any means, including but not limited to, copying or printing electronic files,
reconstruction through memorization or dictation, and/or dissemination of these materials or any part of
them is strictly prohibited. Keep printed materials in a secure location when you are not reviewing them
and discard them in a secure manner, such as shredding, when you have completed the examination.

Page 119 of 287


American Society of Plastic Surgeons and the Plastic Surgery Educational Foundation
In-Service Examination

(Please note that this pictorial appears in color in the online examination)

83. A 16-year-old boy who underwent costochondral reconstruction of the right ear
20 days ago because of microtia has had pain, erythema, fluctuance, and swelling
of the ear as well as fever for the past 48 hours. A photograph is shown. In addition
to intravenous administration of an antibiotic, which of the following is the most
appropriate management?
A) Application of mafenide acetate (Sulfamylon)
B) Incision and drainage, washout, and placement of irrigating drains
C) Incision and removal of the cartilage construct and placement of irrigating drains
D) Needle aspiration and application of a compressive bolster dressing
E) Observation

The correct response is Option B.

The clinical scenario described is dire because of the likely loss of the costochondral ear

This examination contains test materials that are owned and copyrighted by the American Society of
Plastic Surgeons and the Plastic Surgery Educational Foundation. Any reproduction of these materials or
any part of them, through any means, including but not limited to, copying or printing electronic files,
reconstruction through memorization or dictation, and/or dissemination of these materials or any part of
them is strictly prohibited. Keep printed materials in a secure location when you are not reviewing them
and discard them in a secure manner, such as shredding, when you have completed the examination.

Page 120 of 287


American Society of Plastic Surgeons and the Plastic Surgery Educational Foundation
In-Service Examination

reconstruction. The cartilage may have gained enough vascularity to help avoid loss of the
construct. In addition to intravenous administration of an antibiotic, incision and removal of the
construct must be considered, but a trial of incision and drainage, washout, and placement of
irrigating drains is warranted in an attempt to avoid loss of this graft. If the fever and cellulitis do
not improve quickly, then removal of the construct is warranted as a secondary procedure.

Application of Sulfamylon, needle aspiration, and observation are not aggressive enough for this
severe infection.

References

1. Park C, Lew DH, Yoo WM. An analysis of 123 temporoparietal fascial flaps:
anatomic and clinical considerations in total auricular reconstruction. Plast Reconstr
Surg. 1999;104:1295-1306
2. Osorno G. Autogenous rib cartilage reconstruction of congenital ear defects: report of
110 cases with Brent's technique. Plast Reconstr Surg. 1999;104:1951-1962.

This examination contains test materials that are owned and copyrighted by the American Society of
Plastic Surgeons and the Plastic Surgery Educational Foundation. Any reproduction of these materials or
any part of them, through any means, including but not limited to, copying or printing electronic files,
reconstruction through memorization or dictation, and/or dissemination of these materials or any part of
them is strictly prohibited. Keep printed materials in a secure location when you are not reviewing them
and discard them in a secure manner, such as shredding, when you have completed the examination.

Page 121 of 287


American Society of Plastic Surgeons and the Plastic Surgery Educational Foundation
In-Service Examination

84. A 45-year-old woman with T3 N0 invasive ductal carcinoma in the inferior pole of
the left breast is scheduled to undergo segmental mastectomy and subsequent
radiation therapy. She currently wears a size 36DDD brassiere and is willing to
accept any cup size from C to DDD. Which of the following interventions will yield
the best cosmetic result in this patient?
A) Bilateral reduction mammaplasty
B) Implantation of a prosthesis in the left breast and mastopexy of the right breast
C) Latissimus dorsi musculocutaneous flap reconstruction of the left breast and
mastopexy of the right breast
D) TRAM reconstruction of the defect
E) No reconstruction
The correct response is Option A.
In a patient with large breasts, in whom a partial mastectomy is required, reduction
mammaplasty is a very good option. This procedure will potentially relieve symptoms of
macromastia, reduce the amount of breast tissue present in both breasts, and offer the best
aesthetic outcome.
Implantation of a prosthesis in a breast that will undergo radiation therapy increases the risk for
complications and would likely lead to a less symmetrical result.
The latissimus dorsi (or TRAM) flap could be used for immediate partial reconstruction but is
ideal for delayed reconstruction of partial mastectomy defects. Reduction mammaplasty does not
preclude future reconstruction options, but a latissimus flap reconstruction would.
With a T3 tumor, a significant portion of the lower pole of the breast is removed to obtain clear
margins. Even though the patient described has moderate to large breasts, there is a high
likelihood that she will develop a deformity of the left breast and asymmetry with the right breast
if no reconstruction is performed.
References

1. Kronowitz SJ, Feledy JA, Hunt KK, et al. Determining the optimal approach to breast
reconstruction after partial mastectomy. Plast Reconstr Surg. 2006;117:1-11.
2. Nahabedian MY. Discussion—Determining the optimal approach to breast
reconstruction after partial mastectomy. Plast Reconstr Surg. 2006;117:12-14.
3. Audretsch WP. Reconstruction of the partial mastectomy defect: classification and
method. In: Spear SL, ed. Surgery of the Breast. 2nd ed. Philadelphia: Lippincott
Williams & Wilkins; 2006:179-216.

This examination contains test materials that are owned and copyrighted by the American Society of
Plastic Surgeons and the Plastic Surgery Educational Foundation. Any reproduction of these materials or
any part of them, through any means, including but not limited to, copying or printing electronic files,
reconstruction through memorization or dictation, and/or dissemination of these materials or any part of
them is strictly prohibited. Keep printed materials in a secure location when you are not reviewing them
and discard them in a secure manner, such as shredding, when you have completed the examination.

Page 122 of 287


American Society of Plastic Surgeons and the Plastic Surgery Educational Foundation
In-Service Examination

85. A 42-year-old woman comes to the office for consultation regarding removal of
transverse wrinkles over the bridge of the nose. Administration of botulinum toxin
type A is planned. Targeting of which of the following muscles is most appropriate?
A) Corrugator supercilii
B) Frontalis
C) Orbicularis oculi
D) Procerus
E) Zygomaticus minor

The correct response is Option D.

The procerus muscle produces transverse wrinkles over the bridge of the nose. The corrugator
supercilii serves to draw the eyebrow inferiorly and medially, and as such produces the vertical
glabellar frown lines. The main antagonist of all of the eyebrow depressors is the frontalis
muscle. Contraction of the orbicularis oculi enhances the radially oriented folds at the lateral
canthus. The zygomaticus major muscle draws the angle of the mouth superiorly, laterally, and
posteriorly with actions of laughing, smiling, and chewing. The zygomaticus minor muscle
functions as one of the lip elevators and, with the zygomaticus major, contributes to the
nasolabial fold.

References

1. Fagien S. Botox for the treatment of dynamic and hyperkinetic facial lines and
furrows: adjunctive use in facial aesthetic surgery. Plast Reconstr Surg.
1999;103:701-713.
2. Carruthers J, Fagien S, Matarasso LS, and the Botox Consensus Group. Consensus
recommendations on the use of botulinum toxin type A in facial aesthetics. Plast
Reconstr Surg. 2004;114(Suppl. 6):1S-22S.
3. Fagien S. Extended use of botulinum toxin type A in facial aesthetic surgery. Aesth
Surg J. 1998;18:215-219.

This examination contains test materials that are owned and copyrighted by the American Society of
Plastic Surgeons and the Plastic Surgery Educational Foundation. Any reproduction of these materials or
any part of them, through any means, including but not limited to, copying or printing electronic files,
reconstruction through memorization or dictation, and/or dissemination of these materials or any part of
them is strictly prohibited. Keep printed materials in a secure location when you are not reviewing them
and discard them in a secure manner, such as shredding, when you have completed the examination.

Page 123 of 287


American Society of Plastic Surgeons and the Plastic Surgery Educational Foundation
In-Service Examination

86. A 32-year-old woman comes to the office because she has persistent paresthesia
of the lower lip three weeks after undergoing alloplastic chin augmentation via the
submental approach. Which of the following is the most appropriate management?
A) Gentle external massage
B) Oral administration of a corticosteroid
C) Surgical readjustment of the prosthesis
D) Surgical release of the mentalis muscle fibers
E) Observation

The correct response is Option C.

Paresthesia of the lip after insertion of an alloplastic chin prosthesis should be resolving or
decreasing by two to three weeks postoperatively. If not, the prosthesis should be removed and
trimmed superiorly at the level of the mental nerve foramen, or the implant wing should be
positioned lower to prevent continued impingement of the mental nerve. Leaving a prosthesis in
situ for eight weeks or longer with persistent numbness of the lip may result in some permanent
loss of sensation. It is best to adjust prostheses as early as possible—or by four weeks at the
latest—for satisfactory results.

References

1. Zide BM, Pfeifer TM, Longaker MT. Chin surgery: I. Augmentation—the allures and
the alerts. Plast Reconstr Surg. 1999;104:1843-1853.
2. Posnick JC, Al-Qattan MM, Stepner NM. Alteration in facial sensibility in
adolescents following sagittal split and chin osteotomies of the mandible. Plast
Reconstr Surg. 1996;97:920-927.

This examination contains test materials that are owned and copyrighted by the American Society of
Plastic Surgeons and the Plastic Surgery Educational Foundation. Any reproduction of these materials or
any part of them, through any means, including but not limited to, copying or printing electronic files,
reconstruction through memorization or dictation, and/or dissemination of these materials or any part of
them is strictly prohibited. Keep printed materials in a secure location when you are not reviewing them
and discard them in a secure manner, such as shredding, when you have completed the examination.

Page 124 of 287


American Society of Plastic Surgeons and the Plastic Surgery Educational Foundation
In-Service Examination

87. A 50-year-old man who underwent uncomplicated full abdominoplasty with suction
lipectomy two weeks ago comes to the emergency department because he has had
painless swelling of the right lower extremity for the past 24 hours. He currently
smokes one pack of cigarettes daily. Medical history includes no serious disease
conditions. Vital signs are stable and within normal limits. Physical examination
shows 7-cm enlargement of the circumference of the right leg with a positive
Homans sign. The chest is clear to auscultation, and the patient is not in acute
distress. Which of the following diagnostic studies is the most appropriate initial
step?
A) Doppler ultrasonography of the lower extremities
B) Lymphangiography
C) Spiral CT of the chest
D) Venography of the lower extremities
E) Ventilation-perfusion (VQ) scan

The correct response is Option A.

Among plastic surgery procedures, abdominoplasty has one of the highest rates of deep vein
thrombosis (DVT) and pulmonary embolism. It is essential to know the proper course of
management of this major cause of postoperative morbidity and mortality. Clinical workup may
vary with the severity of the symptoms and physical examination of the patient, and not all
clinical investigations may be available to the surgeon. However, a clear path of investigation
and treatment should be developed.

A Doppler ultrasonography is the most appropriate initial test for DVT because it is quick,
noninvasive, and readily available in most centers.

Venography of the lower extremities is also a diagnostic test, but it is invasive and has increased
risks.

Lymphangiography is a diagnostic test for lymphedema, not for DVT.

If pulmonary embolism is suspected, a spiral CT of the chest or a VQ scan of the lungs is


indicated.

This examination contains test materials that are owned and copyrighted by the American Society of
Plastic Surgeons and the Plastic Surgery Educational Foundation. Any reproduction of these materials or
any part of them, through any means, including but not limited to, copying or printing electronic files,
reconstruction through memorization or dictation, and/or dissemination of these materials or any part of
them is strictly prohibited. Keep printed materials in a secure location when you are not reviewing them
and discard them in a secure manner, such as shredding, when you have completed the examination.

Page 125 of 287


American Society of Plastic Surgeons and the Plastic Surgery Educational Foundation
In-Service Examination

References

1. Davison SP, Venturi ML, Attinger CE, et al. Prevention of venous thromboembolism
in the plastic surgery patient. Plast Reconstr Surg. 2004;114:43E-51E.
2. Most D, Kozlow J, Heller J, et al. Thromboembolism in plastic surgery. Plast
Reconstr Surg. 2005;115:20E-30E.
3. Rohrich RJ, Rios JL. Venous thromboembolism in cosmetic plastic surgery:
maximizing patient safety. Plast Reconstr Surg. 2003;112:871-872.

This examination contains test materials that are owned and copyrighted by the American Society of
Plastic Surgeons and the Plastic Surgery Educational Foundation. Any reproduction of these materials or
any part of them, through any means, including but not limited to, copying or printing electronic files,
reconstruction through memorization or dictation, and/or dissemination of these materials or any part of
them is strictly prohibited. Keep printed materials in a secure location when you are not reviewing them
and discard them in a secure manner, such as shredding, when you have completed the examination.

Page 126 of 287


American Society of Plastic Surgeons and the Plastic Surgery Educational Foundation
In-Service Examination

For each patient with blepharoptosis, select the most likely diagnosis (A–D).

A. Horner syndrome
B. Injury to the facial (VII) nerve
C. Myasthenia gravis
D. Senile ptosis

88. A 66-year-old man with visual obstruction of the left eye and elevation
of the supratarsal crease

89. A 60-year-old woman with general muscle fatigue and bilateral upper
visual field obstruction that worsens at the end of the day

The correct response for Item 88 is D and for Item 89 is C.

Horner syndrome is marked by ptosis, miosis, and anhidrosis. It occurs after loss of sympathetic
innervations of the superior cervical ganglion. The eyelid crease is usually not affected.

Injury to the facial (VII) nerve results in brow ptosis, not blepharoptosis.

Myasthenia gravis may cause unilateral or bilateral ptosis, which would be exaggerated by
fatigue. The condition typically affects young women or elderly men. Pharmacologic testing
establishes the diagnosis.

Senile ptosis is characterized by dehiscence of the levator aponeurosis, which is the most
common cause of ptosis in elderly individuals. Elevation of the supratarsal crease is seen.
Levator plication or advancement is required for correction.

References

1. Jelks GW, Jelks EB. Preoperative evaluation of the blepharoplasty patient: bypassing
the pitfalls. Clin Plastic Surg. 1993;20:213.
2. Martin JJ Jr, Tenzel RR, Acquired ptosis: dehiscences and disinsertions C are they
real or iatrogenic? Ophthal Plast Reconstr Surg. 1992;8:130-132.

This examination contains test materials that are owned and copyrighted by the American Society of
Plastic Surgeons and the Plastic Surgery Educational Foundation. Any reproduction of these materials or
any part of them, through any means, including but not limited to, copying or printing electronic files,
reconstruction through memorization or dictation, and/or dissemination of these materials or any part of
them is strictly prohibited. Keep printed materials in a secure location when you are not reviewing them
and discard them in a secure manner, such as shredding, when you have completed the examination

Page 127 of 287


American Society of Plastic Surgeons and the Plastic Surgery Educational Foundation
In-Service Examination

90. During a reduction mammaplasty procedure, preservation of sensation in the


nipple-areola complex is most dependent on which of the following?
A) Amount of tissue resected
B) Nipple-to-notch distance
C) Pedicle location
D) Preoperative breast volume
E) Skin incision used

The correct response is Option C.

The most important determinant in preserving sensation in the nipple-areola complex is the
anatomic location of glandular resection. Superior glandular pedicle techniques with tissue
resections at the base of the breast are associated with higher risk of injury to the nerve supply.
Lateral, inferior, and medial-based pedicles allow for better preservation of the nerve supply.

The amount of tissue resected has not been shown to have a statistically significant effect on
preservation of sensation in the nipple-areola complex.

Preoperative breast volume is a factor in preoperative nipple-areola sensation (ie, the sensitivity
of the nipple-areola decreases as the breast volume increases). However, preoperative breast
volume, type of skin incision used, or nipple-to-notch distance have not been found to be the
most important factor in preservation of nipple sensation.

References

1. Schlenz I, Rigel S, Schemper M, et al. Alteration of nipple and areola sensitivity by


reduction mammaplasty: a prospective comparison of five techniques. Plast Reconstr
Surg. 2005;115:743-751.
2. Nahabedian MY. Discussion—Alteration of nipple and areola sensitivity by reduction
mammaplasty: a prospective comparison of five techniques. Plast Reconstr Surg.
2005;115:752-754.

This examination contains test materials that are owned and copyrighted by the American Society of
Plastic Surgeons and the Plastic Surgery Educational Foundation. Any reproduction of these materials or
any part of them, through any means, including but not limited to, copying or printing electronic files,
reconstruction through memorization or dictation, and/or dissemination of these materials or any part of
them is strictly prohibited. Keep printed materials in a secure location when you are not reviewing them
and discard them in a secure manner, such as shredding, when you have completed the examination

Page 128 of 287


American Society of Plastic Surgeons and the Plastic Surgery Educational Foundation
In-Service Examination

91. A 50-year-old man is brought to the emergency department 90 minutes after he


sustained injuries in a motor vehicle collision. Physical examination shows a severe
abrasion injury to the right ear with partial degloving of the superior helical rim
extending into the concha. The skin over the upper third of the ear is missing at
variable depths. Which of the following is the most appropriate initial step in
management?
A) Conservative debridement and topical wound care
B) Debridement and coverage with a local flap
C) Debridement and skin grafting
D) Debridement, coverage with a temporoparietal fascia flap, and skin grafting
E) Resection and primary wound closure

The correct response is Option A.

The patient described has a partial-thickness injury involving the upper third of the ear. It is often
not clear initially if the perichondrium is viable in this situation. Although the patient may
require coverage of the exposed cartilage with a flap, initial management is most often
conservative. The most appropriate initial step is conservative debridement and topical wound
care including dressing changes until the devitalized tissues demarcate. This management allows
for a full assessment of the extent of the injury. It is important that the dressings prevent
desiccation and infection of the exposed perichondrium.

Debridement followed by immediate skin grafting is not appropriate because skin graft take is
often unreliable if the cartilage is exposed. Also, the extent of tissue injury caused by shearing
and blunt trauma is difficult to assess initially because of the soiled and edematous tissues of the
fresh trauma.

Debridement, coverage with a temporoparietal fascia flap, and skin grafting is effective in
providing a bed for skin grafting but is not the most appropriate initial step in management.

Resection and closure of a defect of this size can be performed, often with an Antia-Buch flap
after wedge resection with a pentagonal design. However, this procedure will reduce the size of
the ear and, therefore, is not appropriate.

References

1. Aguilar EF. Ear reconstruction. Clin Plast Surg. 2004;31:87-91.


2. El-Khatib HA, Al-Basti HB, Al-Ghould A, et al. Subtotal reconstruction of a burned
auricle. Burns. 2005;31:230-235.

This examination contains test materials that are owned and copyrighted by the American Society of
Plastic Surgeons and the Plastic Surgery Educational Foundation. Any reproduction of these materials or
any part of them, through any means, including but not limited to, copying or printing electronic files,
reconstruction through memorization or dictation, and/or dissemination of these materials or any part of
them is strictly prohibited. Keep printed materials in a secure location when you are not reviewing them
and discard them in a secure manner, such as shredding, when you have completed the examination

Page 129 of 287


American Society of Plastic Surgeons and the Plastic Surgery Educational Foundation
In-Service Examination

92. A 10-year-old girl is brought to the office by her parents for consultation regarding
congenital unilateral facial paralysis. She has had no previous treatment of this
condition. Physical examination shows no muscle activity on the affected side.
Which of the following is the most appropriate first step in management to restore
voluntary animation to the paralyzed side of the face?
A) Cross-facial nerve grafting
B) Gracilis neurovascular free tissue transfer
C) Masseter muscle transfer
D) Temporalis muscle transposition
E) Tensor fascia lata suspension

The correct response is Option A.

The best treatment for this child with unilateral facial paralysis involving the eye is a two-stage
surgical correction resulting in dynamic reanimation. In the first stage, cross-facial nerve grafting
is constructed from the unparalyzed side of the face to the paralyzed side. The sural nerve is
commonly used as a graft. The second stage consists of free tissue transfer of muscle, such as the
gracilis or pectoralis minor, to the paralyzed side of the face. Microneurovascular anastomoses
are completed, with the goal of functional reanimation of the paralyzed face.

Eyelid closure is a priority issue in treatment of facial paralysis. Lack of corneal protection can
result in keratitis, conjunctivitis, corneal ulceration, and visual impairment. Although use of
artificial tears and ointments is necessary, such treatment is neither permanently effective nor
corrective in any type of functional repair.

Transposition of the temporalis muscle is another option for dynamic reconstruction of the eye in
patients with facial paralysis. It has the advantage of immediate functional correction, compared
with the lag period required for muscle reinnervation by cross-facial nerve grafting. However,
one long-term study has shown overall better functional results, measured by eyelid closure and
movement, in patients undergoing correction with two-stage cross-facial nerve grafting and
muscle transfer compared with transposition of temporalis muscle. In addition,
microneurovascular tissue transfer is a better choice in children with facial paralysis, who may
be the best candidates to achieve functional adaptation after such a procedure. Furthermore, the
use of a muscle of mastication in children may have potential deleterious effects on skeletal
maturation.

Static correction of eyelid closure by means such as fascial or tendon sling creation,
tarsorrhaphy, and placement of a gold weight does not produce dynamic movement and,

This examination contains test materials that are owned and copyrighted by the American Society of
Plastic Surgeons and the Plastic Surgery Educational Foundation. Any reproduction of these materials or
any part of them, through any means, including but not limited to, copying or printing electronic files,
reconstruction through memorization or dictation, and/or dissemination of these materials or any part of
them is strictly prohibited. Keep printed materials in a secure location when you are not reviewing them
and discard them in a secure manner, such as shredding, when you have completed the examination

Page 130 of 287


American Society of Plastic Surgeons and the Plastic Surgery Educational Foundation
In-Service Examination

therefore, does not fully correct the obvious functional deficit apparent on the paralyzed side
with animation of the face. These options are more suitable for older patients with facial
paralysis, in whom static correction may be more desirable and who may not be optimal
candidates for more complex procedures such as free tissue transfer.

References

1. Frey M, Giovanoli P, Tzou CJ, et al. Dynamic reconstruction of eye closure by


muscle transposition or functional muscle transplantation in facial palsy. Plast
Reconstr Surg. 2004;114:865-875.
2. Harrison DH. The treatment of unilateral and bilateral facial palsy using free muscle
transfers. Clin Plast Surg. 2002;29:539-549.
3. Buncke HJ, Buncke GM, Kind GM, et al. Cross-facial and functional microvascular
muscle transplantation for longstanding facial paralysis. Clin Plast Surg.
2002;29:551-566.

This examination contains test materials that are owned and copyrighted by the American Society of
Plastic Surgeons and the Plastic Surgery Educational Foundation. Any reproduction of these materials or
any part of them, through any means, including but not limited to, copying or printing electronic files,
reconstruction through memorization or dictation, and/or dissemination of these materials or any part of
them is strictly prohibited. Keep printed materials in a secure location when you are not reviewing them
and discard them in a secure manner, such as shredding, when you have completed the examination

Page 131 of 287


American Society of Plastic Surgeons and the Plastic Surgery Educational Foundation
In-Service Examination

93. A 37-year-old woman is undergoing infiltration of wetting solution in preparation for


liposuction of the trochanteric region. As the infiltration progresses, supraventricular
tachycardia is noted on the electrocardiography monitor. Blood pressure remains
within normal limits. Which of the following is the most appropriate next step?
A) Administer procainamide 100 mg slowly over 5 minutes to a maximum dose of
1000 mg and perform 12-lead electrocardiography
B) Administer protamine 25 mg by intravenous infusion over 10 minutes and perform
12-lead electrocardiography
C) Continue the procedure as planned after obtaining chest radiographs
D) Discontinue the infiltration, check the composition of the infiltrate, and assess for
possible accidental intravascular injection
E) Discontinue the infiltration until the tachycardia resolves and immediately cardiovert

The correct response is Option D.


The infiltration of large amounts of lidocaine and epinephrine is not without risk. Medication
errors are missed because the surgeon fails to check that the operating room personnel have
properly constituted the wetting solution, with severe consequences. Wetting solution
preparation is often relegated to junior personnel who are unaware of the importance of correct
formulation. Intravascular injections can also contribute to lidocaine or epinephrine
complications and toxicity.
Supraventricular tachycardia is a normal heart response to adrenaline, but it nevertheless should
be assessed by the surgeon and anesthesiologist with a good electrocardiogram. The infiltration
should be discontinued. The vital signs of the patient described will determine further treatment.

Beta blockers, calcium channel blockers, procainamide, and other medications may be used if the
patient's rhythm does not normalize. Protamine is used to reverse heparin and is not used for
arrhythmia. Chest radiographs will not diagnose the etiology or the arrhythmia. Cardioversion is
not used if the patient's blood pressure is stable.

References

1. ACLS Providers Manual. Dallas: American Heart Association; 2002.


2. Kenkel JM, Brown SA, Love EJ, et al. Hemodynamics, electrolytes, and organ
histology of larger-volume liposuction in a porcine model. Plast Reconstr Surg.
2004;113:1391-1399.
3. Kenkel JM, Lipschitz AH, Luby M, et al. Pharmacokinetics and safety of lidocaine
and monoethylglycinexylidide in liposuction: a microdialysis study. Plast Reconstr
Surg. 2004;114:516-526.

This examination contains test materials that are owned and copyrighted by the American Society of
Plastic Surgeons and the Plastic Surgery Educational Foundation. Any reproduction of these materials or
any part of them, through any means, including but not limited to, copying or printing electronic files,
reconstruction through memorization or dictation, and/or dissemination of these materials or any part of
them is strictly prohibited. Keep printed materials in a secure location when you are not reviewing them
and discard them in a secure manner, such as shredding, when you have completed the examination

Page 132 of 287


American Society of Plastic Surgeons and the Plastic Surgery Educational Foundation
In-Service Examination

94. An active 72-year-old man with squamous cell carcinoma of the ear is scheduled to
undergo total resection. Medical history includes only well-controlled hypertension.
Which of the following is the primary limitation of autologous reconstruction in this
patient?
A) Cartilage framework
B) Medical comorbidities
C) Nutritional factors
D) Skin envelope
E) Temporal wasting

The correct response is Option A.

The most difficult problem in auricular reconstruction in elderly patients is in the fabrication of
the three-dimensional cartilage frame. This is because the harvested costal cartilages are calcified
and extremely brittle; therefore, modifications in the construction of the cartilage frame may be
required.

Limitations in elasticity and flexibility of the costal cartilage may not allow the fabrication of the
crus helicis–helical rim as one unit as in younger patients. However, it is possible to perform
auricular reconstruction in elderly patients with extreme caution in the construction of the
cartilage frame.

In younger patients, the major difficulty in auricular reconstruction is attainment of sufficient


skin surface area to cover the fabricated cartilage frame without resorting to skin grafts, but in an
elderly patient with a nontraumatic injury, there is ample skin. Medical comorbidities, if severe,
may limit surgery, but the ear and rib incisions are usually well tolerated. Temporal wasting may
make the use of a temporoparietal flap problematic, but this is generally not needed in an elective
case with normal postauricular skin. Nutritional factors are generally optimized before surgery.

References

1. Brent B. Reconstruction of the ear. In: Aston SJ, Beasley RW, Thorne CHM, eds.
Grabb and Smith's Plastic Surgery. 5th ed. Philadelphia: Lippincott-Raven;
1997:413-429.
2. Nagata S. Microtia: auricular reconstruction. In: Achauer BM, Eriksson E, Guyuron
B, et al., eds. Plastic Surgery: Indications, Operations and Outcomes. St. Louis:
Mosby; 2000:1023-1056.

This examination contains test materials that are owned and copyrighted by the American Society of
Plastic Surgeons and the Plastic Surgery Educational Foundation. Any reproduction of these materials or
any part of them, through any means, including but not limited to, copying or printing electronic files,
reconstruction through memorization or dictation, and/or dissemination of these materials or any part of
them is strictly prohibited. Keep printed materials in a secure location when you are not reviewing them
and discard them in a secure manner, such as shredding, when you have completed the examination

Page 133 of 287


American Society of Plastic Surgeons and the Plastic Surgery Educational Foundation
In-Service Examination

95. A 40-year-old woman is scheduled to undergo reconstruction of the right breast via
a free TRAM flap. She has smoked two packs of cigarettes daily for the past eight
years. This patient's smoking history increases her risk of which of the following
postoperative complications?
A) Hematoma
B) Mastectomy flap necrosis
C) Seroma
D) TRAM flap loss
E) Vessel thrombosis

The correct response is Option B.

One study found no significant increased risk of flap loss, vessel thrombosis, or fat necrosis in
free TRAM flaps performed on smokers. However, smokers did have an increased risk over their
nonsmoking counterparts in the area of mastectomy skin flap necrosis, abdominal hernia, and
abdominal flap necrosis.

Another study of 569 free TRAM flaps found that smokers had an increased rate of fat necrosis,
wound infection, abdominal flap necrosis, and mastectomy flap necrosis. There was no
correlation with free flap loss, thrombosis, hematoma, or seroma.

By using the deep inferior epigastric vessels, which are the primary blood supply to the TRAM
flap, risks to the flap itself are minimized even in smokers. But smoking continues to increase the
risk of donor site morbidity and native breast skin necrosis.

References

1. Chang DW, Reece GP, Wang B, et al. Effect of smoking on complications in patients
undergoing free TRAM flap breast reconstruction. Plast Reconstr Surg.
2000;105:2374-2380.
2. Selber JC, Kurichi JE, Vega SJ, et al. Risk factors and complications in free TRAM
flap breast reconstruction. Ann Plast Surg. 2006;56:492-497.

This examination contains test materials that are owned and copyrighted by the American Society of
Plastic Surgeons and the Plastic Surgery Educational Foundation. Any reproduction of these materials or
any part of them, through any means, including but not limited to, copying or printing electronic files,
reconstruction through memorization or dictation, and/or dissemination of these materials or any part of
them is strictly prohibited. Keep printed materials in a secure location when you are not reviewing them
and discard them in a secure manner, such as shredding, when you have completed the examination

Page 134 of 287


American Society of Plastic Surgeons and the Plastic Surgery Educational Foundation
In-Service Examination

96. A 26-year-old woman comes to the office after being referred by an ophthalmologist
because of scleral show of the left lower eyelid. Three weeks ago, she sustained a
laceration of the lower eyelid, which was sutured in the emergency department.
Which of the following is the most appropriate next step in management?
A) Tear supplementation and massage of the scar
B) Injection of a corticosteroid into the scar and silicone sheeting
C) Lysis of scar adhesions and septal cartilage interposition grafting
D) Excision of the scar and full-thickness skin grafting
E) Lateral canthopexy and cheek advancement flap

The correct response is Option A.

This patient has ectropion with eversion of the lower eyelid. In general, ectropion is classified as
involutional, cicatricial, or neurogenic. The patient described has a cicatricial ectropion. At three
weeks into the postoperative period, the wounds are still in the active phase of healing. Over the
next six to nine months, as the collagen with the scar remodels, the lid may return to a more
normal position. Therefore, during this time frame, the ectropion should be managed with scar
tissue massage and a regimen to prevent complications related to dry eyes, such as tear
supplementation and eye patching. Surgical intervention is warranted if there is no improvement
after this time frame or if conservative therapy is ineffective and there is a risk of visual
compromise.

Injection of a corticosteroid can reduce inflammation and soften the scar, allowing the eyelid to
return to a better position. However, use of corticosteroids in the lower eyelid is not advocated
because of the risks of tissue atrophy and skin discoloration.

Lysis of scar adhesions and the placement of an interpositional graft are indicated for cases of
midlamellar scar. Placing upward traction on the eyelid can identify scar contracture in this
location. If adequate skin is present, the lower eyelid should reach at least the level of the
midpupil. If this movement is restricted, midlamellar scarring is likely.

Lateral canthopexy is indicated for lower eyelid malposition associated with lower eyelid laxity.
Excision of the scar and skin grafting are indicated for ectropion secondary to cutaneous
deficiency.

Excision of the scar and cheek advancement with a lateral canthopexy would not be a primary
reconstruction procedure for this lesion.

This examination contains test materials that are owned and copyrighted by the American Society of
Plastic Surgeons and the Plastic Surgery Educational Foundation. Any reproduction of these materials or
any part of them, through any means, including but not limited to, copying or printing electronic files,
reconstruction through memorization or dictation, and/or dissemination of these materials or any part of
them is strictly prohibited. Keep printed materials in a secure location when you are not reviewing them
and discard them in a secure manner, such as shredding, when you have completed the examination

Page 135 of 287


American Society of Plastic Surgeons and the Plastic Surgery Educational Foundation
In-Service Examination

References

1. Carraway JH, Miller JR, Lewis BK. Reoperative blepharoplasty. In: Grotting JC, ed.
Reoperative Aesthetic and Reconstructive Plastic Surgery. St. Louis: Quality Medical
Publishing; 1995:323-361.
2. Jelks GW, Jelks EB. Repair of lower lid deformities. Clin Plast Surg.
1993;20:417-425.
3. Patipa M. The evaluation and management of lower eyelid retraction following
cosmetic surgery. Plast Reconstr Surg. 2000;106:438-459.

This examination contains test materials that are owned and copyrighted by the American Society of
Plastic Surgeons and the Plastic Surgery Educational Foundation. Any reproduction of these materials or
any part of them, through any means, including but not limited to, copying or printing electronic files,
reconstruction through memorization or dictation, and/or dissemination of these materials or any part of
them is strictly prohibited. Keep printed materials in a secure location when you are not reviewing them
and discard them in a secure manner, such as shredding, when you have completed the examination

Page 136 of 287


American Society of Plastic Surgeons and the Plastic Surgery Educational Foundation
In-Service Examination

97. During rhytidectomy in a 52-year-old man, the external jugular vein is entered.
During hemostasis, an adjacent nerve is noted to be cauterized. Which of the
following postoperative complications is most likely to occur in this patient?
A) Asymmetry of the lower lip
B) Difficulty swallowing
C) Gustatory sweating
D) Hoarseness
E) Paresthesia of the earlobe

The correct response is Option E.

The earlobe is innervated by the great auricular nerve, which follows closely with the external
jugular vein and runs on the same plane. The vein and the nerve run superficial to the platysma
and can often be injured while raising the tissue plane for rhytidectomy. Injury can occur when
the plane is adherent between the sternocleidomastoid muscle and the lateral border of the
platysma muscle.

Asymmetry of the lower lip would result from injury to the marginal mandibular nerve, which
lies deep to the platysma. The great occipital nerve may also be injured, which would cause
numbness of the mastoid area, but this nerve runs in a more posterior direction than the external
jugular vein and great auricular nerve. Difficulty swallowing may occur with injury to the ansa
cervicalis or to the pharyngeal musculature. Pharyngeal muscles are innervated by the branches
of trigeminal (V) nerve running deep within the carotid sheath. Hoarseness would result from
injury to the recurrent laryngeal nerve, which runs much deeper along the tracheoesophageal
groove, caudal to the thyroid.

References

1. McKinney P, Katrana DJ. Prevention of injury to the great auricular nerve during
rhytidectomy. Plast Reconstr Surg. 1980;66:675-679.
2. Aboudib JH Jr, De Castro CC. Anatomical variations analysis of the external jugular
vein, great auricular nerve, and posterosuperior border of the platysma muscle. Aesth
Plast Surg. 1997;21:75-78.
3. Shultz JD, Dodson TB, Meyer RA. Donor site morbidity of greater auricular nerve
graft harvesting. J Oral Maxillofacial Surg. 1992;50:803-805.
4. Sand T, Becser N. Neurophysiological and anatomical variability of the greater
auricular nerve. Acta Neurol Scand. 1998;98:333-339.

This examination contains test materials that are owned and copyrighted by the American Society of
Plastic Surgeons and the Plastic Surgery Educational Foundation. Any reproduction of these materials or
any part of them, through any means, including but not limited to, copying or printing electronic files,
reconstruction through memorization or dictation, and/or dissemination of these materials or any part of
them is strictly prohibited. Keep printed materials in a secure location when you are not reviewing them
and discard them in a secure manner, such as shredding, when you have completed the examination

Page 137 of 287


American Society of Plastic Surgeons and the Plastic Surgery Educational Foundation
In-Service Examination

98. A 38-year-old woman comes to the office three months after undergoing
augmentation mammaplasty to correct Grade 2 ptosis of the breasts because she
has become increasingly dissatisfied with the results. During the procedure, 350-ml
saline prostheses were placed submuscularly. Current physical examination shows
excess superior fullness and inferior positioning of the natural breast over the lower
pole of the prostheses. Which of the following is the most effective method of
correcting this result?
A) Closed capsulotomy
B) Exchange for larger implants
C) Mastopexy
D) Open capsulotomy
E) Radial scoring of the inframammary fold

The correct response is Option C.

Augmentation of the Grade 2 ptotic breast will result in a "double-bubble" appearance if the
nipple-areola complex is not also elevated with a skin excision. Therefore, Grade 2 ptosis, in
which the nipple is below the inframammary crease, requires a mastopexy. Attempting to correct
greater degrees of ptosis with implantation of a prosthesis alone requires the use of large volume
prostheses that may be aesthetically unpleasing. In addition, the poor quality skin envelope may
not be able to tolerate such expansion. Without the necessary skin tightening, the native ptotic
breast may hang over the less mobile and higher positioned underlying prosthesis. This
postoperative contour deformity (double bubble) can worsen as the patient ages. It is corrected
with a mastopexy.

References

1. Kirwan L. Augmentation of the ptotic breast: simultaneous periareolar


mastopexy/breast augmentation. Aesth Surg J. 1999;19:34-39.
2. Massiha H. Augmentation in ptotic and densely glandular breasts: prevention,
treatment, and classification of double-bubble deformity. Ann Plast Surg.
2000;44:143-146.

This examination contains test materials that are owned and copyrighted by the American Society of
Plastic Surgeons and the Plastic Surgery Educational Foundation. Any reproduction of these materials or
any part of them, through any means, including but not limited to, copying or printing electronic files,
reconstruction through memorization or dictation, and/or dissemination of these materials or any part of
them is strictly prohibited. Keep printed materials in a secure location when you are not reviewing them
and discard them in a secure manner, such as shredding, when you have completed the examination

Page 138 of 287


American Society of Plastic Surgeons and the Plastic Surgery Educational Foundation
In-Service Examination

99. A 24-year-old man who sustained total amputation of the left ear during an
explosion three months ago comes to the office for consultation regarding
reconstruction of the ear. Physical examination shows loss of the auricle and tragus
with scarring of the skin and scalp surrounding the external auditory canal. Which of
the following materials is most appropriate for the first stage of the reconstruction of
this patient's ear?
A) Full-thickness skin grafting to replace the scar and subsequent expansion
B) Radial forearm flap and Silastic framework
C) Temporalis muscle turnover flap and rib framework
D) Temporoparietal flap and rib graft framework
E) Tissue expansion of retroauricular skin and Medpor framework

The correct response is Option D.

The goal of reconstruction of the ear is to restore the cosmesis of the auricle and the function of
the superior helical rim in providing support for eyeglasses. Total reconstruction of the ear most
likely requires two to three stages. The first stage typically involves creating a cartilage
framework and placing it under vascularized tissue. In the scenario described, an ipsilateral
temporoparietal fascia pedicled flap or a contralateral free temporoparietal fascia flap (TPFF)
covered with a split-thickness skin graft (STSG) is most appropriate. STSGs are generally used
because they will shrink and form fit to the auricular cartilage over the TPFF or in the posterior
auricular skin crease.

Other options include a local tissue flap or a pre-expanded local tissue flap. A tissue expander
can be placed and may be appropriate for some patients. However, it is not likely to be
successful in the patient described because of the scarred skin in the region of expander
placement. A second surgery is usually necessary to recreate the posterior auricular crease and
the tragus or lobule. In young patients, rib cartilage is the most common first choice for
autogenous framework creation because it has less risk of exposure and is readily available. In
the patient described, Medpor would not be the most appropriate choice for the first stage of
reconstruction because it is associated with a high exposure rate when used with thin atrophic or
scarred soft tissue. The quality and quantity of skin redraping the auricular framework play a
significant role in the aesthetics of the final result. In older patients with postablative defects and
compromised surrounding tissue, prosthetic reconstruction can be offered as a viable, perhaps
simplistic, solution to staged auricular reconstruction. A younger patient is seldom compliant
with a prosthesis, and autologous reconstruction is a better option.

This examination contains test materials that are owned and copyrighted by the American Society of
Plastic Surgeons and the Plastic Surgery Educational Foundation. Any reproduction of these materials or
any part of them, through any means, including but not limited to, copying or printing electronic files,
reconstruction through memorization or dictation, and/or dissemination of these materials or any part of
them is strictly prohibited. Keep printed materials in a secure location when you are not reviewing them
and discard them in a secure manner, such as shredding, when you have completed the examination

Page 139 of 287


American Society of Plastic Surgeons and the Plastic Surgery Educational Foundation
In-Service Examination

References

1. Bhandari PS. Total ear reconstruction in postburn deformity. Clin Plast Surg.
2002;29:213-220.
2. Sevin K, Askar I, Saray A, et al. Exposure of high-density porous polyethylene
(Medpor) used for contour restoration and treatment. Br J Oral Maxillofac Surg.
2000;38:44-49.
3. Thorne C, Brecht LE, Bradly JP, et al. Auricular reconstruction: indications for
autogenous and prosthetic technique. Plast Reconstr Surg. 2001;107:1241-1251.

This examination contains test materials that are owned and copyrighted by the American Society of
Plastic Surgeons and the Plastic Surgery Educational Foundation. Any reproduction of these materials or
any part of them, through any means, including but not limited to, copying or printing electronic files,
reconstruction through memorization or dictation, and/or dissemination of these materials or any part of
them is strictly prohibited. Keep printed materials in a secure location when you are not reviewing them
and discard them in a secure manner, such as shredding, when you have completed the examination

Page 140 of 287


American Society of Plastic Surgeons and the Plastic Surgery Educational Foundation
In-Service Examination

100. One year after undergoing gastric bypass surgery, a 36-year-old woman comes to
the office for consultation regarding cosmetic improvement of the lower torso. She
has lost 120 lb since the procedure was performed, and her weight has been
stable for the past three months. She does not smoke cigarettes. Height is 5 ft 4 in
and weight is 175 lb. Physical examination shows circumferential laxity of the
truncal skin with lipodystrophy of the hips and rectus diastasis as well as an upper
midline incision. No hernias are noted. A lower body lift is scheduled. Which of the
following is the most likely complication of this procedure in this patient?
A) Infection
B) Postoperative hemorrhage
C) Pulmonary embolism
D) Seroma
E) Wound dehiscence

The correct response is Option D.

The most common complication of a lower body lift procedure is seroma. In a large retrospective
series, seromas occurred in 37.5% of all patients; wound dehiscence (12%), infection (8%), and
pulmonary embolism (6%) also occurred frequently after this type of surgery. Other
complications such as psychiatric difficulties, unscheduled hospitalizations, skin necrosis, and
postoperative hematomas may be common but are not reported as often. Complications seem to
be associated with higher preoperative body mass index (weight) and history of smoking.

References

1. Aly AS, Cram AE, Chao M, et al. Belt lipectomy for circumferential truncal excess:
the University of Iowa experience. Plast Reconstr Surg. 2003;111:398-413.
2. Van Huizum MA, Roche NA, Hofer SO. Circular belt lipectomy: a retrospective
follow-up study on perioperative complications and cosmetic outcome. Ann Plast
Surg. 2005;54:459-464.

This examination contains test materials that are owned and copyrighted by the American Society of
Plastic Surgeons and the Plastic Surgery Educational Foundation. Any reproduction of these materials or
any part of them, through any means, including but not limited to, copying or printing electronic files,
reconstruction through memorization or dictation, and/or dissemination of these materials or any part of
them is strictly prohibited. Keep printed materials in a secure location when you are not reviewing them
and discard them in a secure manner, such as shredding, when you have completed the examination

Page 141 of 287


American Society of Plastic Surgeons and the Plastic Surgery Educational Foundation
In-Service Examination

Section 3: Craniomaxillofacial

(Please note that this pictorial appears in color in the online examination)

101. A 14-year-old girl is brought to the office by her parents for consultation regarding
facial asymmetry. On the basis of the photograph shown, which of the following is
the most likely diagnosis?
A) Congenital cranial (VII) nerve palsy
B) Craniofacial microsomia
C) Goldenhar syndrome
D) Romberg disease
E) Unilateral coronal craniosynostosis

The correct response is Option D.

The patient shown has Romberg disease (progressive hemifacial atrophy). This is a unilateral
condition that affects girls more commonly than boys, and onset is in childhood with progression
from 2 to 10 years of age. It usually involves skin, subcutaneous tissue, muscle, and bone but

This examination contains test materials that are owned and copyrighted by the American Society of
Plastic Surgeons and the Plastic Surgery Educational Foundation. Any reproduction of these materials or
any part of them, through any means, including but not limited to, copying or printing electronic files,
reconstruction through memorization or dictation, and/or dissemination of these materials or any part of
them is strictly prohibited. Keep printed materials in a secure location when you are not reviewing them
and discard them in a secure manner, such as shredding, when you have completed the examination.

Page 142 of 287


American Society of Plastic Surgeons and the Plastic Surgery Educational Foundation
In-Service Examination

spares the cranial nerves and their function. Treatment is complex, with reconstruction of facial
bone structure and augmentation followed by cutaneous contour correction with free tissue
transfer. Parascapular flap is the flap of choice.

Unilateral coronal craniosynostosis can result in incomplete correction of vertical dystopia,


secondary to continued growth constriction at the affected side cranial base. Soft-tissue loss and
lower face asymmetry are not associated features. Hemifacial microsomia is a congenital,
nonprogressive abnormality of the first branchial arch derivatives (ie, mandible and auricle). The
mandibular ramus and condyle are variably hypoplastic, and overlying soft tissues (muscle,
subcutaneous fat) are often hypoplastic. Ipsilateral macrostomia (transverse facial cleft) can also
present in this syndrome. Goldenhar syndrome is a variant of craniofacial microsomia and is
distinguished by presence of concomitant ocular abnormalities, including epibulbar dermoid.

References

1. Siebert J, Soltanian H, Hazen A. Hemifacial atrophy. In: Plastic Surgery. Mathes SJ,
Hentz VR, ed. Philadelphia: Elsevier; 2005:555-569.
2. McCarthy J, Hopper R, Grayson B. Craniofacial microsomia. In: Plastic Surgery.
Mathes SJ, Hentz VR, ed. Philadelphia: Elsevier; 2005:113-134.

This examination contains test materials that are owned and copyrighted by the American Society of
Plastic Surgeons and the Plastic Surgery Educational Foundation. Any reproduction of these materials or
any part of them, through any means, including but not limited to, copying or printing electronic files,
reconstruction through memorization or dictation, and/or dissemination of these materials or any part of
them is strictly prohibited. Keep printed materials in a secure location when you are not reviewing them
and discard them in a secure manner, such as shredding, when you have completed the examination.

Page 143 of 287


American Society of Plastic Surgeons and the Plastic Surgery Educational Foundation
In-Service Examination

(Please note that this pictorial appears in color in the online examination)

102. A 7-year-old boy is brought to the office for consultation regarding the congenital
nerve palsy shown. The affected nerve is derived from which of the following
branchial arches?
A) First
B) Second
C) Third
D) Fourth
E) Fifth

The correct response is Option B.

The facial nerve (cranial nerve VII) is derived from the second branchial arch and consists
primarily of motor fibers that are distributed to the muscles of facial expression. Branchial arches
begin to develop in the fourth week as neural crest cells migrate into the head and neck region.
By the end of the fourth week, four pairs of branchial arches are visible externally. The fifth and

This examination contains test materials that are owned and copyrighted by the American Society of
Plastic Surgeons and the Plastic Surgery Educational Foundation. Any reproduction of these materials or
any part of them, through any means, including but not limited to, copying or printing electronic files,
reconstruction through memorization or dictation, and/or dissemination of these materials or any part of
them is strictly prohibited. Keep printed materials in a secure location when you are not reviewing them
and discard them in a secure manner, such as shredding, when you have completed the examination.

Page 144 of 287


American Society of Plastic Surgeons and the Plastic Surgery Educational Foundation
In-Service Examination

sixth arches are small and not visible on the surface of the embryo yet. The branchial arches are
separated by prominent clefts called branchial grooves.

References

1. Moore KL. The Developing Human. Philadelphia: WB Saunders; 1988:396-398.


2. Stricker M, VanDer Meulen J, Mazzola BR. Craniofacial Malformations. New York:
Churchill Livingstone; 1990:22-24.

This examination contains test materials that are owned and copyrighted by the American Society of
Plastic Surgeons and the Plastic Surgery Educational Foundation. Any reproduction of these materials or
any part of them, through any means, including but not limited to, copying or printing electronic files,
reconstruction through memorization or dictation, and/or dissemination of these materials or any part of
them is strictly prohibited. Keep printed materials in a secure location when you are not reviewing them
and discard them in a secure manner, such as shredding, when you have completed the examination.

Page 145 of 287


American Society of Plastic Surgeons and the Plastic Surgery Educational Foundation
In-Service Examination

103. A 24-year-old woman comes to the office because she has had popping of the left
temporomandibular joint and associated mild pain that have been worsening over
the past six months. The patient says she also has intermittent inability to fully
open her mouth, but she does not believe her symptoms restrict her daily routine.
Physical examination shows mild bilateral tenderness of the preauricular region
but no popping, clicking, or crepitus. Maximal incisal opening is 20 to 25 mm, and
lateral and protrusive excursions are reduced. MRI of the temporomandibular
joints shows mild anterior dislocation of the left meniscus with reduction in the
open mouth position. Which of the following is the most appropriate initial
management?
A) Administration of a nonsteroidal anti-inflammatory drug, occlusal splint therapy, and
restriction to soft diet
B) Comprehensive orthodontic therapy with bilateral sagittal ramus osteotomies of the
mandible
C) Initial intra-articular injection of a corticosteroid followed by serial injections if
symptoms do not resolve
D) Open arthrotomy with meniscectomy followed by a brief period of joint immobilization
and then physical therapy
E) Open arthrotomy with plication of the meniscus followed by a brief period of joint
immobilization and then physical therapy

The correct response is Option A.


With symptoms and clinical findings of mild pain and popping of the temporomandibular joint,
mild tenderness of the preauricular region, and transient joint locking, the patient described has
an early/intermediate stage internal derangement. MRI shows only mild changes in meniscal
position. Therefore, conservative therapy such as a soft diet, use of nonsteroidal anti-
inflammatory drugs, and splinting is indicated. Most patients with temporomandibular joint
disorders will achieve some relief of symptoms with nonsurgical therapy within a period of two
to three years. However, the pathologic process may continue, and follow-up is recommended.

References

1. Wilkes HW. Internal derangements of the temporomandibular joint. Arch


Otolaryngol Head Neck Surg. 1989;115:469-477.
2. Hall HD, Merrill RG, Sanders B. Guidelines for management of disorders of the
temporomandibular joint and related structures. J Tenn Dent Assoc. 2002;82:58-64.
3. Karlis V, Glickman R. Nonsurgical management of temporomandibular disorders. In:
Miloro M, eds. Oral and Maxillofacial Surgery. Hamilton, Ontario: BC Decker;
2004:949-961.

This examination contains test materials that are owned and copyrighted by the American Society of
Plastic Surgeons and the Plastic Surgery Educational Foundation. Any reproduction of these materials or
any part of them, through any means, including but not limited to, copying or printing electronic files,
reconstruction through memorization or dictation, and/or dissemination of these materials or any part of
them is strictly prohibited. Keep printed materials in a secure location when you are not reviewing them
and discard them in a secure manner, such as shredding, when you have completed the examination.

Page 146 of 287


American Society of Plastic Surgeons and the Plastic Surgery Educational Foundation
In-Service Examination

104. A 24-year-old man who underwent repair of isolated cleft lip and palate in infancy
comes to the office with his wife for consultation regarding the risk of their children
having cleft deformities because they are planning their first pregnancy. The
patient's family history includes isolated cleft lip and palate in his mother. The
patient's wife has no congenital deformities and has no known family history of
cleft palate. Which of the following percentages best represents the chance that
this couple will have a child with a cleft deformity?
A) 5%
B) 10%
C) 15%
D) 25%

The correct response is Option A.

Although there are cases in which clefting does transmit in an autosomal-dominant or autosomal-
recessive fashion, most isolated cleft lip/palate deformities follow a multifactorial model of
genetic transmission. Therefore, the risk that this patient will have a child with a cleft is only
slightly higher than the general population.

Families with Van der Woude syndrome, for example, are known to have an autosomal-
dominant clefting mutation. If this were the case, this patient would then have a 50% chance of
transmitting the cleft locus to a child. Similar Mendelian genetic models would apply to other
scenarios.

References

1. Moore KE. The Developing Human. Philadelphia: WB Saunders; 1988.


2. Cordero D, Schneider R, Helms J. Morphogenesis of the face. In: Lin KY, Ogle RC,
Jane JA, eds. Craniofacial Surgery: Science and Surgical Technique. Philadelphia:
W.B. Saunders; 2002:75-83.

This examination contains test materials that are owned and copyrighted by the American Society of
Plastic Surgeons and the Plastic Surgery Educational Foundation. Any reproduction of these materials or
any part of them, through any means, including but not limited to, copying or printing electronic files,
reconstruction through memorization or dictation, and/or dissemination of these materials or any part of
them is strictly prohibited. Keep printed materials in a secure location when you are not reviewing them
and discard them in a secure manner, such as shredding, when you have completed the examination.

Page 147 of 287


American Society of Plastic Surgeons and the Plastic Surgery Educational Foundation
In-Service Examination

105. A 76-year-old man undergoes a radial forearm flap procedure for reconstruction of
a defect of the floor of the mouth resulting from excision of carcinoma. During
dissection of vessels in the neck for microsurgical anastomosis, the surgeon has
difficulty visualizing an appropriate recipient artery because of high bifurcation of
the carotid artery. Which of the following muscles can be cut to provide better
exposure of the recipient artery?
A) Digastric
B) Geniohyoid
C) Omohyoid
D) Sternohyoid
E) Styloglossus

The correct response is Option A.

There are only two muscles that would be in the area of a high bifurcation of the carotid that may
need to be cut for better exposure: posterior belly of the digastric and the stylohyoid, which span
across the skull base to the hyoid. Because the stylohyoid is not mentioned, the digastric muscle
is the correct choice. The microsurgeon usually divides the digastric muscle at the level of the
fibrous loop for the intermediate digastric tendon. The other muscles are never dissected or
divided during this procedure.

References

1. Netter FH. Atlas of Human Anatomy. Summit, NJ: Ciba-Geigy; 1991:53.


2. Langman J. Medical Embryology. Baltimore: Williams & Wilkins, 1981:45.

This examination contains test materials that are owned and copyrighted by the American Society of
Plastic Surgeons and the Plastic Surgery Educational Foundation. Any reproduction of these materials or
any part of them, through any means, including but not limited to, copying or printing electronic files,
reconstruction through memorization or dictation, and/or dissemination of these materials or any part of
them is strictly prohibited. Keep printed materials in a secure location when you are not reviewing them
and discard them in a secure manner, such as shredding, when you have completed the examination.

Page 148 of 287


American Society of Plastic Surgeons and the Plastic Surgery Educational Foundation
In-Service Examination

106. A 37-year-old woman is brought to the emergency department two hours after she
sustained injuries to the face in a motor vehicle collision. Radiographs show an
orbital zygomatic fracture on the left side. On ophthalmologic examination, the left
pupil fails to constrict when direct light is shined in the eye; consensual pupillary
constriction is normal. Color perception is diminished in the left eye. Findings in
the right eye are normal. Which of the following is the most likely cause of these
findings?
A) Detachment of the retina
B) Extension of the fracture through the optic canal
C) Impingement of bone fragments on the optic (II) nerve
D) Shear force injury to the optic (II) nerve
E) Thrombosis of the retinal artery

The correct response is Option D.

Physical examination of the patient with an orbital zygomatic fracture includes an


ophthalmologic assessment. Along with evaluation for motility disturbances of the globe and
diplopia, a direct visual assessment is performed. The finding of traumatic optic neuropathy has
been reported in 2% to 5% of patients with severe facial trauma. In its most severe form,
traumatic optic neuropathy results in visual loss; however, findings may be more subtle, such as
diminished color perception. It is also possible to find an afferent pupillary defect in the affected
eye (if the pupil fails to constrict with direct light stimulation but constricts normally in a
consensual response when light is directed to the contralateral eye). Injury to the optic (II) nerve
can result from either mechanical or ischemic insults. The most likely mechanism of injury is
shear force to the optic (II) nerve. The dural sheath is firmly attached to the optic nerve at its
entrance into the optic foramen. Rapid deceleration injuries of the head can generate forces that
are concentrated at the optic foramen. An optic nerve injury can also result from fracture
fragments directly damaging the optic nerve or globe, thrombosis of the retinal artery, or retinal
detachment; however, these are less common than injury resulting from shear force.

References

1. Hollier L, Thorton J, Pazmino P, et al. The management of orbitozygomatic fractures.


Plast Reconstr Surg. 2003;111:2386.
2. Warner JE, Lessell S. Traumatic optic neuropathy. Int Ophthalmol Clin. 1995;35:57.

This examination contains test materials that are owned and copyrighted by the American Society of
Plastic Surgeons and the Plastic Surgery Educational Foundation. Any reproduction of these materials or
any part of them, through any means, including but not limited to, copying or printing electronic files,
reconstruction through memorization or dictation, and/or dissemination of these materials or any part of
them is strictly prohibited. Keep printed materials in a secure location when you are not reviewing them
and discard them in a secure manner, such as shredding, when you have completed the examination.

Page 149 of 287


American Society of Plastic Surgeons and the Plastic Surgery Educational Foundation
In-Service Examination

107. Which of the following muscles of facial expression is innervated on its anterior
surface?
A) Depressor anguli oris
B) Levator anguli oris
C) Levator labii superioris
D) Orbicularis oris
E) Zygomaticus major

The correct response is Option B.

The muscles of facial expression receive their innervation from cranial nerve VII (facial nerve).
The facial nerve innervates the muscles of facial expression along the posterior surface of the
muscle in most cases. The exceptions to this rule include the levator anguli oris, the buccinator,
and the mentalis muscle. At the modiolus, the fibers of the levator anguli oris coalesce with
fibers of the zygomaticus major, orbicularis oris, risorius, buccinator, and depressor anguli oris.
The levator anguli oris lies deep to the other muscles and receives its innervation along its
anterior surface.

References

1. Ewart CJ, Jaworski NB, Rekito AJ, et al. Levator anguli oris: a cadaver study
implicating its role in perioral rejuvenation. Ann Plast Surg. 2005;54:260-263.
2. Marinetti CJ. The lower muscular balance of the face used to lift labial commissures.
Plast Reconstr Surg. 1999;104:1153.

This examination contains test materials that are owned and copyrighted by the American Society of
Plastic Surgeons and the Plastic Surgery Educational Foundation. Any reproduction of these materials or
any part of them, through any means, including but not limited to, copying or printing electronic files,
reconstruction through memorization or dictation, and/or dissemination of these materials or any part of
them is strictly prohibited. Keep printed materials in a secure location when you are not reviewing them
and discard them in a secure manner, such as shredding, when you have completed the examination.

Page 150 of 287


American Society of Plastic Surgeons and the Plastic Surgery Educational Foundation
In-Service Examination

108. A 20-year-old woman comes to the office for consultation regarding malocclusion.
Cephalometric analysis shows SNA angle of 83 degrees (normal = 82 ± 3
degrees) and SNB angle of 85 degrees (normal = 80 ± 3 degrees). Which of the
following is the most likely cause of this patient's deformity?
A) Mandibular prognathism
B) Mandibular retrognathism
C) Maxillary retrusion
D) Retrogenia
E) Vertical maxillary excess

The correct response is Option A.

A normal SNA (sella-nasion-point A) angle is defined as 82 ± 3 degrees and measures the


position of point A (anterior maxilla) relative to the anterior cranial base (SN). A normal SNB
(sella-nasion-point B) angle is defined as 80 ± 3 degrees and measures the position of point B
(anterior mandible) relative to the cranial base (SN). The most common cause of a normal SNA
with an increased SNB is mandibular prognathism. Mandibular retrognathism would have a
normal SNA with a decreased SNB. Maxillary retrusion is associated with a decreased SNA and
a normal SNB. Vertical maxillary excess alone would not change the SNB but describes a patient
with long face syndrome.

References

1. Ferraro JW. Cephalometry and cephalometric analysis. In: Ferraro JW, ed.
Fundamentals of Maxillofacial Surgery. New York: Springer-Verlag; 1997:233-245.
2. Nanda RS, Merrill RM. Cephalometric assessment of sagittal relationship between
maxilla and mandible. Am J Orthodont Dentofac Orthoped. 1994;105:328-344.

This examination contains test materials that are owned and copyrighted by the American Society of
Plastic Surgeons and the Plastic Surgery Educational Foundation. Any reproduction of these materials or
any part of them, through any means, including but not limited to, copying or printing electronic files,
reconstruction through memorization or dictation, and/or dissemination of these materials or any part of
them is strictly prohibited. Keep printed materials in a secure location when you are not reviewing them
and discard them in a secure manner, such as shredding, when you have completed the examination.

Page 151 of 287


American Society of Plastic Surgeons and the Plastic Surgery Educational Foundation
In-Service Examination

109. Which of the following muscles is typically reoriented during repair of cleft palate?
A) Levator veli palatini
B) Muscularis uvulae
C) Palatoglossus
D) Palatopharyngeus
E) Superior pharyngeal constrictor

The correct response is Option A.

Within the normal soft palate, the levator veli palatine muscle forms a dynamic sling that
elevates the velum toward the posterior pharyngeal wall during the production of certain sounds.
In children born with cleft palate, there is a division of the musculature of the velum into
separate muscle bellies with abnormal insertions along the posterior edge of the hard palate. The
goals of cleft palate repair during infancy are twofold. The first goal is to establish a complete,
watertight closure of the secondary palate for separation of the oral and nasal cavities. The
second goal is to reorient and repair the levator musculature to allow for normal speech
formation. The muscularis uvulae is also repaired in cleft palate repair. However, this muscle is
typically not reoriented. Instead, it is simply approximated in the midline.

References

1. Costello BJ, Ruiz RL. Cleft lip and palate: comprehensive treatment planning and
primary repair. In: Miloro M, ed. Principles of Oral and Maxillofacial Surgery.
London: BC Decker; 2004:839-858.
2. VanderKolk CA. Cleft palate. In: Achauer BM, Eriksson E, Guyuron B, et al., eds.
Plastic Surgery: Indications, Operations, and Outcomes. Philadelphia: Mosby;
2000:799-808.

This examination contains test materials that are owned and copyrighted by the American Society of
Plastic Surgeons and the Plastic Surgery Educational Foundation. Any reproduction of these materials or
any part of them, through any means, including but not limited to, copying or printing electronic files,
reconstruction through memorization or dictation, and/or dissemination of these materials or any part of
them is strictly prohibited. Keep printed materials in a secure location when you are not reviewing them
and discard them in a secure manner, such as shredding, when you have completed the examination.

Page 152 of 287


American Society of Plastic Surgeons and the Plastic Surgery Educational Foundation
In-Service Examination

110. A 74-year-old man who is edentulous is scheduled to undergo resection of T4 N0


squamous cell carcinoma of the anterior aspect of the floor of the mouth. Bilateral
anterior segmental mandibulectomy from the middle of the body of the mandible is
planned. Which of the following is the most appropriate method of reconstruction?
A) Free fibular osteocutaneous flap
B) Nonvascularized iliac bone graft
C) Pectoralis myocutaneous flap with a reconstruction plate
D) Pectoralis myocutaneous flap with a rib graft
E) Platysmal myocutaneous flap with a reconstruction plate

The correct response is Option A.

Resection of the anterior mandible can result in poor oral competence. When reconstruction is
not attempted, the retained segments of the mandible tend to collapse toward the midline. This
lack of support results in the “Andy Gump” deformity with the associated oral incontinence and
cosmetic deformity. The approximate plate extrusion rate in patients with anterior defects who
have been reconstructed with plates and soft tissue flaps has been reported to be as high as 66%.
Free tissue transfer of a composite, osteocutaneous flap is thus advocated as the reconstructive
method of choice. Nonvascularized bone grafts are not indicated in patients who are expected to
receive postoperative radiation therapy and who have defects greater than 6 cm. With the advent
of free tissue transfer, the transfer of a vascularized rib on the pectoralis muscle is not a first
choice for reconstruction.

References

1. Schusterman MA, Reece GP, Kroll SS, et al. Use of the AO plate for immediate
mandibular reconstruction in cancer patients. Plast Reconstr Surg. 1991; 88:588-593.
2. Johnson JT, Myers EN, Deleyiannis FWB. Reconstruction of the Oral Cavity. 3rd ed.
Alexandria, VA: American Academy of Otolaryngology--Head and Neck Surgery
Foundation; 2003:41-47.

This examination contains test materials that are owned and copyrighted by the American Society of
Plastic Surgeons and the Plastic Surgery Educational Foundation. Any reproduction of these materials or
any part of them, through any means, including but not limited to, copying or printing electronic files,
reconstruction through memorization or dictation, and/or dissemination of these materials or any part of
them is strictly prohibited. Keep printed materials in a secure location when you are not reviewing them
and discard them in a secure manner, such as shredding, when you have completed the examination.

Page 153 of 287


American Society of Plastic Surgeons and the Plastic Surgery Educational Foundation
In-Service Examination

111. A 25-year-old man is brought to the emergency department 30 minutes after he fell
while riding a bicycle. He says he was thrown over the handlebars onto the road
surface and landed on his chin. Physical examination shows swelling along the
mandible and anterior open bite. Radiographs show a parasymphyseal fracture of
the mandible and bilateral fractures of the condylar neck. The left condylar head is
displaced medially out of the glenoid fossa. No other associated injuries are noted.
Which of the following is the most appropriate management of this fracture?
A) Mandibular-maxillary fixation for six weeks
B) Open reduction and internal fixation of the parasymphyseal fracture followed by
mandibular-maxillary fixation for six weeks
C) Open reduction and internal fixation of the parasymphyseal and left condylar neck
fractures followed by mandibular-maxillary fixation with elastics
D) Open reduction and internal fixation of the parasymphyseal and right condylar neck
fractures followed by mandibular-maxillary fixation with elastics
The correct response is Option C.
Rigid fixation of mandible fractures allows early mobilization and restoration of jaw function,
airway control, improved nutritional status, improved speech, better oral hygiene, patient
comfort, and an earlier return to the workplace. The technique of rigid internal fixation was
developed by AO/ASIF in Europe in the 1970s. The basic principles of the AO call for primary
bone healing under conditions of absolute stability. This is accomplished by interfragmentary
compression plates. An inferior border plate is used to counter compression forces, and a
superior border plate or arch bars can be used to counter traction or tension forces at the superior
border.

The subcondylar fracture poses a different and more complicated set of questions. Most
practitioners agree that most subcondylar fractures can be treated in a closed fashion. Closed
treatment implies control of the occlusion, aggressive physical therapy, and close follow-up.
Additionally, children should generally be treated with closed reduction. Absolute indications for
open treatment of subcondylar fractures are as follows: dislocation into the middle cranial fossa
or external auditory canal, lateral extracapsular displacement, inability to obtain adequate
occlusion, or open joint wound with foreign body or gross contamination.

References

1. Newman L. A clinical evaluation of the long-term outcome of patients treated for


bilateral fracture of the mandibular condyles. Br J Oral Maxillofac Surg.
1998;36:176-179.
2. Zide MF, Kent JN: Indications for open reduction of mandibular condyle fractures. J
Oral Maxillofac Surg. 1983;41:89-98.

This examination contains test materials that are owned and copyrighted by the American Society of
Plastic Surgeons and the Plastic Surgery Educational Foundation. Any reproduction of these materials or
any part of them, through any means, including but not limited to, copying or printing electronic files,
reconstruction through memorization or dictation, and/or dissemination of these materials or any part of
them is strictly prohibited. Keep printed materials in a secure location when you are not reviewing them
and discard them in a secure manner, such as shredding, when you have completed the examination.

Page 154 of 287


American Society of Plastic Surgeons and the Plastic Surgery Educational Foundation
In-Service Examination

112. An 18-year-old woman who underwent repair of cleft lip and palate during infancy
is scheduled to undergo maxillary advancement surgery. Which of the following
factors favors distraction osteogenesis over traditional orthognathic advancement?
A) Anticipated advancement of 5 mm
B) Anticipated advancement of 14 mm
C) Need for simultaneous alveolar cleft bone grafting
D) Need for simultaneous palatal expansion

The correct response is Option B.

Children with cleft lip often develop maxillary hypoplasia and Class III malocclusion.
Traditional orthognathic advancement is limited by the tension of the soft-tissue envelope.
Greater tension is associated with scarring, advancement distances of greater than 1 cm, and past
pharyngeal surgical procedures. Any of these issues will limit the advancement and increase the
relapse rate over the subsequent years.

Distraction osteogenesis allows the soft tissues to be expanded as the bone is advanced.

Palatal expansion requires horizontal movement. Distraction in multiple directions is difficult to


control.

References

1. Polley JW, Figueroa AA, Kidd M. Principles of distraction osteogenesis in


craniofacial surgery. In: Lin KY, Ogle RC, Jane JA, eds. Craniofacial Surgery:
Science and Surgical Technique. Philadelphia: W.B. Saunders; 2002:163-172.
2. Molina F. Distraction of the maxilla. In: McCarthy J, ed. Distraction of the
Craniofacial Skeleton. New York: Springer; 1999:204-218.

This examination contains test materials that are owned and copyrighted by the American Society of
Plastic Surgeons and the Plastic Surgery Educational Foundation. Any reproduction of these materials or
any part of them, through any means, including but not limited to, copying or printing electronic files,
reconstruction through memorization or dictation, and/or dissemination of these materials or any part of
them is strictly prohibited. Keep printed materials in a secure location when you are not reviewing them
and discard them in a secure manner, such as shredding, when you have completed the examination.

Page 155 of 287


American Society of Plastic Surgeons and the Plastic Surgery Educational Foundation
In-Service Examination

113. Which of the following facial anomalies is most common in patients with
craniofacial microsomia?
A) Facial nerve palsy
B) Mandibular hypoplasia
C) Maxillary hypoplasia
D) Microtia
E) Parotid gland hypoplasia

The correct response is Option B.

Craniofacial microsomia represents the second most common congenital anomaly affecting the
head and neck behind cleft of the lip and/or palate. Multiple craniofacial and other anomalies are
commonly seen as part of the constellation of findings. The most common anomalies are those
affecting the mandible (89% to 100%) and ear (66% to 99%). Less common associations that
must be sought involve the vertebrae and ribs (16% to 60%), ipsilateral facial nerve (10% to
45%), and genitourinary structures (4% to 15%), among others.

References

1. McCarthy JG, Grayson B. Craniofacial microsomia. In: McCarthy JG, Galiano RD,
Boutros SG, eds. Current Therapy in Plastic Surgery. Philadelphia: Saunders
Elsevier; 2006:506-516.
2. McCarthy JG, Hopper RA, Grayson BH. Craniofacial microsomia. In: Mathes S, ed.
Plastic Surgery. 2nd ed. Philadelphia: Saunders Elsevier; 2006.

This examination contains test materials that are owned and copyrighted by the American Society of
Plastic Surgeons and the Plastic Surgery Educational Foundation. Any reproduction of these materials or
any part of them, through any means, including but not limited to, copying or printing electronic files,
reconstruction through memorization or dictation, and/or dissemination of these materials or any part of
them is strictly prohibited. Keep printed materials in a secure location when you are not reviewing them
and discard them in a secure manner, such as shredding, when you have completed the examination.

Page 156 of 287


American Society of Plastic Surgeons and the Plastic Surgery Educational Foundation
In-Service Examination

114. Cleft palate occurs during which of the following weeks of fetal development?
A) 1–2
B) 3–4
C) 5–6
D) 7–8
E) 9–10

The correct response is Option D.

Errors occurring during the main embryonic period are responsible for major craniofacial
anomalies. Cleft palate, which results from a failure of the palatine shelves to fuse, occurs
primarily during weeks 7 to 8. Errors during weeks 1 to 2 result in death of the embryo. Clefting
of the lip occurs during weeks 5 to 6. Ear and eye abnormalities occur during weeks 10 to 12.
References

1. David LR. Applied embryology of the head and neck. In: Argenta L, ed. Basic
Science for Surgeons. Philadelphia: WB Saunders; 2004:173-188.
2. Moore KL, Persaud TVN. The Developing Human: Clinically Oriented Embryology.
6th ed. Philadelphia: WB Saunders; 1998.

This examination contains test materials that are owned and copyrighted by the American Society of
Plastic Surgeons and the Plastic Surgery Educational Foundation. Any reproduction of these materials or
any part of them, through any means, including but not limited to, copying or printing electronic files,
reconstruction through memorization or dictation, and/or dissemination of these materials or any part of
them is strictly prohibited. Keep printed materials in a secure location when you are not reviewing them
and discard them in a secure manner, such as shredding, when you have completed the examination.

Page 157 of 287


American Society of Plastic Surgeons and the Plastic Surgery Educational Foundation
In-Service Examination

115. A 38-year-old man sustains panfacial fractures in a motor vehicle collision. During
surgical reduction, which of the following structures is most appropriate to restore
the transverse dimension of the facial skeleton?
A) Condyle and posterior mandibular ramus
B) Mandibular arch
C) Nasomaxillary buttress
D) Pterygomaxillary buttress
E) Zygomaticomaxillary buttress

The correct response is Option B.

The buttresses of the face allow more precise reduction of facial fractures in both the transverse
and vertical dimensions. The vertical buttresses include the nasomaxillary, zygomaticomaxillary,
and pterygomaxillary buttresses. The condyle and posterior mandibular ramus make up yet
another buttress establishing posterior facial height. The horizontal buttresses, also known as
anterior-posterior buttresses, allow precise restoration of facial depth. They include the frontal,
zygomatic, maxillary, and mandibular buttresses. The mandibular buttress is composed of the
mandibular arch.

Of the options listed, the mandibular arch is the only structure that defines the horizontal buttress
of the face.

References

1. Louis PJ. Management of panfacial fractures. In: Miloro M, ed. Principles of Oral
and Maxillofacial Surgery. London: BC Decker; 2004:547-559.
2. Markowitz BL, Manson PN. Panfacial fractures: organization of treatment. Clin Plast
Surg. 1989;16:105-14.
3. Manson PN, Hoopes JE, Su CT. Structural pillars of the facial skeleton: an approach
to the management of Le Fort fractures. Plast Reconstr Surg. 1980;66:54-62.

This examination contains test materials that are owned and copyrighted by the American Society of
Plastic Surgeons and the Plastic Surgery Educational Foundation. Any reproduction of these materials or
any part of them, through any means, including but not limited to, copying or printing electronic files,
reconstruction through memorization or dictation, and/or dissemination of these materials or any part of
them is strictly prohibited. Keep printed materials in a secure location when you are not reviewing them
and discard them in a secure manner, such as shredding, when you have completed the examination.

Page 158 of 287


American Society of Plastic Surgeons and the Plastic Surgery Educational Foundation
In-Service Examination

116. An 18-year-old man who was born with a bilateral cleft lip and palate, which were
repaired in infancy, has completed orthodontic care and treatment. Maxillary
advancement is planned. Which of the following is the most likely dental
relationship in this patient?
A) Negative overbite
B) Positive overbite
C) Negative overjet
D) Positive overjet

The correct response is Option C.

Both overbite and overjet describe the relationship of the maxillary incisors to the mandibular
incisors. Cephalometric angles are frequently used to analyze the relationship of the teeth to the
maxilla and mandible.

Overbite is a vertical measure of the overlapping of the upper incisors to the lower incisors. It is
calculated by measuring in millimeters the vertical relationship of the perpendicular plane to the
occlusal plane. This value is positive when overlap is present and negative when there is no
overlap (as in the anterior open bite of Apert syndrome).

Overjet is a horizontal measure of the position of the upper and lower incisors in the
anteroposterior plane. Overjet is also measured in millimeters from the labial portion of the
incisal edge of the upper incisor to the labial edge of the lower incisor. This number is positive if
the maxillary teeth are forward of the mandibular teeth and negative if the lower teeth are
forward of the upper teeth.

For the patient with a Class III dental relationship, in which the maxilla is retrusive, overbite is
unaffected and the overjet is most likely negative.

References

1. Zoldos J. Pediatric orthognathic surgery. In: Bentz ML, ed. Pediatric Plastic Surgery.
Stamford, CT: Appleton and Lange; 1998:227-256.
2. Schendel S. Orthognathic surgery. In: VanderKolk CA, ed. Plastic Surgery:
Indications, Operations, and Outcomes. St. Louis: Mosby; 2000:871-896.

This examination contains test materials that are owned and copyrighted by the American Society of
Plastic Surgeons and the Plastic Surgery Educational Foundation. Any reproduction of these materials or
any part of them, through any means, including but not limited to, copying or printing electronic files,
reconstruction through memorization or dictation, and/or dissemination of these materials or any part of
them is strictly prohibited. Keep printed materials in a secure location when you are not reviewing them
and discard them in a secure manner, such as shredding, when you have completed the examination.

Page 159 of 287


American Society of Plastic Surgeons and the Plastic Surgery Educational Foundation
In-Service Examination

For the preoperatively marked rotation-advancement repair shown, select the


corresponding labeled flap (A–D).

117. Rotation

118. Advancement

The correct response for Item 117 is Option A and for Item 118 is Option B.

The A flap is marked on the medial lip element and is the rotation flap. This flap is designed as a
gentle curve from the height of Cupid's bow on the cleft side of the medial cleft element to the
junction of the philtrum and columella. This incision approaches the normal philtral collum as
much as needed to derotate and level Cupid's bow. If more rotation is needed to level Cupid's
bow, a back-cut is made, but the normal philtral collum is not violated. This incision is used in
philtral subunit reconstruction and leveling Cupid's bow.

The B flap is made on the non-cleft, lateral lip element and is the advancement flap. This flap is
the “wedge” that is placed in the defect created above the rotation flap as the medial lip element
is derotated to level Cupid's bow. This flap is marked by first finding the point on the white roll
that will correspond to the height of Cupid’s bow on the medial cleft element. This point is found
where the fullness of the lateral lip element begins to fade. A line is drawn from this point
cephalad and is the same length as the normal philtral collum as well as the marked edge of the
A (rotation) flap.

The C flap or columellar flap is made from the tissue medial to the A-flap incision. This small
flap is rotated into the columella, lengthening the cleft side of the columella. It is also sewn to the

This examination contains test materials that are owned and copyrighted by the American Society of
Plastic Surgeons and the Plastic Surgery Educational Foundation. Any reproduction of these materials or
any part of them, through any means, including but not limited to, copying or printing electronic files,
reconstruction through memorization or dictation, and/or dissemination of these materials or any part of
them is strictly prohibited. Keep printed materials in a secure location when you are not reviewing them
and discard them in a secure manner, such as shredding, when you have completed the examination.

Page 160 of 287


American Society of Plastic Surgeons and the Plastic Surgery Educational Foundation
In-Service Examination

D flap (alar base flap).

The D flap or alar base flap is cut along the alar-facial groove, and this incision corresponds to
the cephalad portion of the edge of the B flap (advancement flap). This alar groove incision is
made as small as possible and long enough to allow differential rotation of the D flap and the B
flap.

The L flap is the leading edge of the lateral lip that can be used to fill the nasal lining defect. The
M flap is the medially based flap from the rotation.

References

1. Losee JE, Selber JC, Arkoulakis N, et al. The cleft lateral lip element: do traditional
markings result in secondary deformities? Ann Plast Surg. 2003;50:594-600
2. Byrd HS. Unilateral cleft lip. In: Aston SJ, ed. Grabb and Smith's Plastic Surgery. 5th
edition. Philadelphia: Lippincott Raven; 1997:245.

This examination contains test materials that are owned and copyrighted by the American Society of
Plastic Surgeons and the Plastic Surgery Educational Foundation. Any reproduction of these materials or
any part of them, through any means, including but not limited to, copying or printing electronic files,
reconstruction through memorization or dictation, and/or dissemination of these materials or any part of
them is strictly prohibited. Keep printed materials in a secure location when you are not reviewing them
and discard them in a secure manner, such as shredding, when you have completed the examination.

Page 161 of 287


American Society of Plastic Surgeons and the Plastic Surgery Educational Foundation
In-Service Examination

119. A 9-year-old boy is brought to the office by his parents because he has not been
able to chew and swallow properly since he fell while riding a bicycle and landed
on his chin one week ago. Treatment in the emergency department immediately
after the injury included repair of a laceration to the chin. Which of the following
findings on physical examination indicates an undiagnosed bilateral fracture of the
condylar necks?
A) Class II malocclusion
B) Class III malocclusion
C) Positive overbite
D) Anterior open bite
E) Posterior open bite

The correct response is Option D.

The mechanism of injury in the patient described is likely to produce a bilateral condylar neck
fracture, which can be easily missed in the emergency department. As with the condylar neck
fractures, there is a loss of vertical ramus height. Consequently, the mandible itself rotates and,
with motion, the posterior molars will contact first. The anterior teeth will remain open—anterior
open bite. A posterior open bite occurs when the central incisors contact and prevent the molars
from occluding.

The Angle classification of dental occlusion was developed in 1890. It is based on the position of
the maxillary first molar in relationship to the first mandibular molar. In class I relation, the
mesiobuccal cusp of the maxillary first molar occludes in the mesiobuccal groove of the
mandibular first molar. In class II, the mandibular molar is displaced backward (distal) to the
maxillary first molar, while in class III relation, it is forward (mesial) to the maxillary molar.
This classification describes the anterior-posterior tooth relationship only.

Overbite describes the vertical relationship of the central mandibular and maxillary incisors. It is
a measure of the overlap between the incisal edges. When an anterior open bite is present, there
will also be a negative overbite.

References

1. Ash MM. Wheeler's Dental Anatomy: Physiology, and Occlusion. 7th ed.
Philadelphia: WB Saunders; 1993.
2. Schendel S. Orthognathic surgery. In: VanderKolk CA, ed. Plastic Surgery:
Indications, Operations, and Outcomes. St. Louis: Mosby; 2000:871-896.

This examination contains test materials that are owned and copyrighted by the American Society of
Plastic Surgeons and the Plastic Surgery Educational Foundation. Any reproduction of these materials or
any part of them, through any means, including but not limited to, copying or printing electronic files,
reconstruction through memorization or dictation, and/or dissemination of these materials or any part of
them is strictly prohibited. Keep printed materials in a secure location when you are not reviewing them
and discard them in a secure manner, such as shredding, when you have completed the examination.

Page 162 of 287


American Society of Plastic Surgeons and the Plastic Surgery Educational Foundation
In-Service Examination

120. A 5-year-old boy with submucus cleft palate has velopharyngeal incompetence.
Which of the following is the most likely underlying cause?
A) Incorrectly positioned levator palatine muscles
B) Incorrectly positioned palatoglossus muscles
C) Incorrectly positioned palatopharyngeal muscles
D) Incorrectly positioned tensor palatine muscles
E) Short soft palate

The correct response is Option A.

By definition, a submucus cleft palate involves the anomalous insertion of the levator palatine
muscles onto the posterior aspect of the hard palate. In normal anatomy, these muscles would be
oriented in a transverse fashion. When contracting, the palate then is forced to elevate in a
V-shaped fashion and shorten to some degree. Nevertheless, not all children will develop speech
difficulty or velopharyngeal incompetence.

The tensor palatine muscles and posterior tonsillar pillars (the palatopharyngeal muscles) are in
relatively normal location. Their function is impacted by the incorrectly positioned levator.

Children with a submucus cleft have normal palate length.

References

1. Elkadi H. Cleft palate repair: functional anatomy of the velopharynx. In: Salyer KE,
Bardach J, eds. Atlas of Craniofacial and Cleft Surgery. Philadelphia: Lippincott
Williams & Wilkins; 2000:679-782.
2. Witt P. Velopharyngeal insufficiency. In: VanderKolk CA, ed. Plastic Surgery:
Indications, Operations, and Outcomes. St. Louis: Mosby; 2000:819-834.

This examination contains test materials that are owned and copyrighted by the American Society of
Plastic Surgeons and the Plastic Surgery Educational Foundation. Any reproduction of these materials or
any part of them, through any means, including but not limited to, copying or printing electronic files,
reconstruction through memorization or dictation, and/or dissemination of these materials or any part of
them is strictly prohibited. Keep printed materials in a secure location when you are not reviewing them
and discard them in a secure manner, such as shredding, when you have completed the examination.

Page 163 of 287


American Society of Plastic Surgeons and the Plastic Surgery Educational Foundation
In-Service Examination

121. A 37-year-old man comes to the emergency department one hour after he was
struck in the left side of the face by a baseball bat. Radiographs show a fracture of
the left mandibular condyle, with anterior and medial displacement of the condyle.
The most likely cause of the displacement is a pulling force originating in which of
the following muscles?
A) Lateral pterygoid
B) Levator veli palatine
C) Masseter
D) Temporalis
E) Tensor veli palatini

The correct response is Option A.

The lateral pterygoid muscle inserts on two places: the neck of the mandibular condyle and
medial aspect of the articular disk. Its insertion on the condyle serves to pull the mandible
superiorly, anteriorly, and medially. The medial pterygoid inserts on the medial border of the
ramus and also pulls the mandible superiorly, anteriorly, and medially. The masseter inserts on
the lateral border of the mandible and coronoid and acts as an elevator. The temporalis inserts on
the coronoid process of the mandible rather than the condyle. The tensor veli palatini inserts in
the midline of the soft palate and has no role in mandibular excursion.

References

1. Gardner E, Gray DJ, O'Reilly R. Anatomy: A Regional Study of Human Structure. 3rd
ed. Philadelphia, WB Saunders, 1969; 679-680.
2. Punjabi AP and Herford AS. Mandibular fractures. In: Thaller SR and McDonald W
(ed): Facial Trauma. New York, Marcel Dekker, 2004: 381-414.

This examination contains test materials that are owned and copyrighted by the American Society of
Plastic Surgeons and the Plastic Surgery Educational Foundation. Any reproduction of these materials or
any part of them, through any means, including but not limited to, copying or printing electronic files,
reconstruction through memorization or dictation, and/or dissemination of these materials or any part of
them is strictly prohibited. Keep printed materials in a secure location when you are not reviewing them
and discard them in a secure manner, such as shredding, when you have completed the examination.

Page 164 of 287


American Society of Plastic Surgeons and the Plastic Surgery Educational Foundation
In-Service Examination

122. The Frankfort horizontal line is formed by connecting which of the following two
points on a standard cephalogram?
A) A point and B point
B) B point and menton
C) Nasion and porion
D) Orbitale and porion
E) Sella and orbitale

The correct response is Option D.

A line joining the porion and the orbitale forms the Frankfort horizontal line (see cephalogram
below). The porion is the superior aspect of the external auditory meatus. The orbitale is the most
inferior point around the orbital rim. The Frankfort horizontal line is used to determine other
reference measurements in the facial skeleton, such as the facial plane angle. The A point is the
most posterior point of the anterior surface of the maxilla. The B point is the most posterior point
on the anterior surface of the mandible. The menton is the most inferior point on the lower
border of the mandible. The nasion is the junction of the frontal bone and the nasal bones. The
sella is the midportion of the sella turcica. Lines and angles other than the Frankfort horizontal
line utilize these points to determine anomalous cranial skeletal structure.

This examination contains test materials that are owned and copyrighted by the American Society of
Plastic Surgeons and the Plastic Surgery Educational Foundation. Any reproduction of these materials or
any part of them, through any means, including but not limited to, copying or printing electronic files,
reconstruction through memorization or dictation, and/or dissemination of these materials or any part of
them is strictly prohibited. Keep printed materials in a secure location when you are not reviewing them
and discard them in a secure manner, such as shredding, when you have completed the examination.

Page 165 of 287


American Society of Plastic Surgeons and the Plastic Surgery Educational Foundation
In-Service Examination

References

1. Seward FS. Frankfort horizontal plane, facial angle and facial type. Aust Orthod J.
1968;1:106-108.
2. Rosen HM. Aesthetic Perspectives in Jaw Surgery. New York: Springer-Verlag;
1999:3-23.

This examination contains test materials that are owned and copyrighted by the American Society of
Plastic Surgeons and the Plastic Surgery Educational Foundation. Any reproduction of these materials or
any part of them, through any means, including but not limited to, copying or printing electronic files,
reconstruction through memorization or dictation, and/or dissemination of these materials or any part of
them is strictly prohibited. Keep printed materials in a secure location when you are not reviewing them
and discard them in a secure manner, such as shredding, when you have completed the examination.

Page 166 of 287


American Society of Plastic Surgeons and the Plastic Surgery Educational Foundation
In-Service Examination

123. A 25-year-old man is brought to the emergency department 30 minutes after


sustaining severe trauma to the head and face during a motor vehicle collision.
Examination shows clear-fluid rhinorrhea, indicating leakage of cerebrospinal fluid.
This patient has most likely sustained an injury to which of the following
structures?
A) Anterior ethmoidal air cells
B) Cribriform plate
C) Frontonasal duct
D) Orbital portion of frontal bone
E) Superior nasal concha

The correct response is Option B.

The cribriform plate (the horizontal component of the ethmoid bone) has many foramina that act
as passageways for olfactory (I) nerves and is in intimate contact with the meninges. Therefore,
injury to the cribriform plate may cause tearing of the meninges, allowing leakage of
cerebrospinal fluid via intact or fractured foramina. The other components of the ethmoid bone
are the vertical portion and the lateral masses.

Because of their location, the other structures listed are not likely to be involved with leakage of
cerebrospinal fluid. The lateral masses of the ethmoid bone extend from the periphery of the
cribriform plate, contain the anterior ethmoid air cells, and articulate with the orbital portion of
the frontal bone. The crista galli is a midline prominence that serves as a point of attachment for
intracranial soft tissue and that centers the cribriform plate. The frontonasal duct communicates
the frontal sinus of the nasal cavity. The superior nasal conchae are bony prominences located on
the medial side of each lateral mass and facing the nasal cavity.

References

1. Fleckenstein P, Tranum-Jensen J. Anatomy in Diagnostic Imaging. 2nd ed.


Philadelphia: WB Saunders; 2001:160-165.
2. Larsen WJ. The skull. In: Anatomy—Development, Function, Clinical Correlations.
Philadelphia: WB Saunders; 2002:539-571.
3. Kelley LL, Petersen CM. Sectional Anatomy for Imaging Professionals. St. Louis:
Mosby; 1997:7-8.

This examination contains test materials that are owned and copyrighted by the American Society of
Plastic Surgeons and the Plastic Surgery Educational Foundation. Any reproduction of these materials or
any part of them, through any means, including but not limited to, copying or printing electronic files,
reconstruction through memorization or dictation, and/or dissemination of these materials or any part of
them is strictly prohibited. Keep printed materials in a secure location when you are not reviewing them
and discard them in a secure manner, such as shredding, when you have completed the examination.

Page 167 of 287


American Society of Plastic Surgeons and the Plastic Surgery Educational Foundation
In-Service Examination

124. In the United States, occurrence of encephaloceles is most common in which of


the following anatomic regions?
A) Frontoethmoidal
B) Occipital
C) Parietal
D) Sphenoidal

The correct response is Option B.

In the United States, encephaloceles are most common in the occipital location. In Asia, they are
more common in the frontoethmoidal region. The prognosis with these malformations depends
upon the presence or absence of herniated brain tissue in the encephalocele.

References

1. Monteith SJ, Heppner PA, Law AJ, et al. Encephalocele-epidemiological variance in


New Zealand. J Clin Neurosci. 2005;12:557-558.
2. Petrick MG, Kwong PC. Anterior encephalocele with subcutaneous right facial
nodule. J Am Acad Dermatol. 2004;51(Suppl. 2):S77.

This examination contains test materials that are owned and copyrighted by the American Society of
Plastic Surgeons and the Plastic Surgery Educational Foundation. Any reproduction of these materials or
any part of them, through any means, including but not limited to, copying or printing electronic files,
reconstruction through memorization or dictation, and/or dissemination of these materials or any part of
them is strictly prohibited. Keep printed materials in a secure location when you are not reviewing them
and discard them in a secure manner, such as shredding, when you have completed the examination.

Page 168 of 287


American Society of Plastic Surgeons and the Plastic Surgery Educational Foundation
In-Service Examination

125. A 20-year-old man who sustained a naso-orbital-ethmoid fracture undergoes


transnasal wire canthopexy. Which of the following vectors, relative to the position
of the insertion of the medial canthal tendon, is most appropriate for placement of
the primary wire?
A) Anterior and inferior
B) Anterior and superior
C) Posterior and inferior
D) Posterior and superior

The correct response is Option D.

Transnasal key wire must be positioned posterior and superior to the insertion of the medial
canthal tendons, behind and above the lacrimal fossa (see diagram below). The fractures are
exposed via coronal approach and may require temporary removal of nasal bones for complete
access. The vector provided with this location of wire positioning allows optimal relationship
between the eyelid tissue and the globes. The canthal tendon is rarely completely detached from
the bone and usually has fibers to the anterior and posterior lacrimal crests. However, in cases of
tendon avulsion, the same principle applies, so the tendon must be transfixed posterior and
superior to the lacrimal fossa. The bony intercanthal distance should be between 16 and 23 mm.

Additional pull-out transnasal soft-tissue bolsters should be applied anteriorly to assist with
contouring of soft tissue against bone.

This examination contains test materials that are owned and copyrighted by the American Society of
Plastic Surgeons and the Plastic Surgery Educational Foundation. Any reproduction of these materials or
any part of them, through any means, including but not limited to, copying or printing electronic files,
reconstruction through memorization or dictation, and/or dissemination of these materials or any part of
them is strictly prohibited. Keep printed materials in a secure location when you are not reviewing them
and discard them in a secure manner, such as shredding, when you have completed the examination.

Page 169 of 287


American Society of Plastic Surgeons and the Plastic Surgery Educational Foundation
In-Service Examination

References

1. Clark N. Approach to upper maxillofacial fractures. In: Evans GRD, ed. Operative
Plastic Surgery. New York: McGraw-Hill; 2000:546-562.
2. Grant MP, Iliff NT, Manson PN. Naso-orbito-ethmoidal complex injuries. In: Evans
GRD, ed. Operative Plastic Surgery. New York: McGraw-Hill; 2000:575-584.

This examination contains test materials that are owned and copyrighted by the American Society of
Plastic Surgeons and the Plastic Surgery Educational Foundation. Any reproduction of these materials or
any part of them, through any means, including but not limited to, copying or printing electronic files,
reconstruction through memorization or dictation, and/or dissemination of these materials or any part of
them is strictly prohibited. Keep printed materials in a secure location when you are not reviewing them
and discard them in a secure manner, such as shredding, when you have completed the examination.

Page 170 of 287


American Society of Plastic Surgeons and the Plastic Surgery Educational Foundation
In-Service Examination

126. A 60-year-old man is diagnosed with melanoma of the forehead. In addition to


wide local excision, in which of the following tumor stages is sentinel lymph node
biopsy indicated?
A) Tis N0 M0
B) T2a N1 M0
C) T2b N0 M0
D) T3b N1 M0
E) T4a N0 M1

The correct response is Option C.

Sentinel lymph node biopsy (SLNB) is the standard of care for intermediate-thickness melanoma
of the trunk and extremities and is recommended when possible for intermediate melanoma of
the head and neck, such as those staged T1b N0 M0. SLNB is more complicated in the head and
neck because of greater variation in the lymphatic drainage systems in those areas and because of
the close proximity of the primary tumor to first-echelon lymph nodes. However, prospective
studies have reported no false negatives, and SLNB results are of great prognostic value and may
be used to guide adjuvant systemic protocols. Positive sentinel lymph nodes are identified in
15% to 21% of patients, and then complete nodal dissection yields additional positive nodes in
up to 20% of patients. The effect of elective lymph node dissection on patient survival rates has
not been determined. Nor has local or regional control in the clinically negative neck among
patients with stage I and II melanoma.

SLNB is indicated in intermediate melanoma with lymph nodes that are clinically negative.
Intermediate melanoma in the head and neck includes stages IB and II. In these stages, tumor
characteristics include T1b (Breslow thickness of less than 1 mm with ulceration, or Clark level
of IV or V), T2 (Breslow thickness of 1.01 to 2 mm), and T3 (Breslow thickness of 2.01 to 4
mm).

A T1a tumor is a thin, less aggressive melanoma with a Breslow thickness of less than 1 mm
without ulceration or a Clark level below III. Because it is not yet intermediate, this tumor is not
appropriate for SLNB. A patient with an N1 tumor has a clinically positive neck, which prohibits
SLNB. A T4 tumor is a deep melanoma, with a Breslow thickness of more than 4 mm. Because
it is no longer intermediate, it is not appropriate for SLNB.

This examination contains test materials that are owned and copyrighted by the American Society of
Plastic Surgeons and the Plastic Surgery Educational Foundation. Any reproduction of these materials or
any part of them, through any means, including but not limited to, copying or printing electronic files,
reconstruction through memorization or dictation, and/or dissemination of these materials or any part of
them is strictly prohibited. Keep printed materials in a secure location when you are not reviewing them
and discard them in a secure manner, such as shredding, when you have completed the examination.

Page 171 of 287


American Society of Plastic Surgeons and the Plastic Surgery Educational Foundation
In-Service Examination

References

1. Gillenwater AM, Byers RM. Melanoma of the head and neck. In: Harrison LB,
Sessions RB, Hong WK, eds. Head and Neck Cancer—A Multidisciplinary Approach.
Philadelphia: Lippincott Williams & Wilkins; 2004:598-619.
2. Eicher SA, Clayman GL, Myers JN, et al. A prospective study of intraoperative
lymphatic mapping for head and neck cutaneous melanoma. Arch Otolaryngol Head
Neck Surg. 2002;128:241-246.

This examination contains test materials that are owned and copyrighted by the American Society of
Plastic Surgeons and the Plastic Surgery Educational Foundation. Any reproduction of these materials or
any part of them, through any means, including but not limited to, copying or printing electronic files,
reconstruction through memorization or dictation, and/or dissemination of these materials or any part of
them is strictly prohibited. Keep printed materials in a secure location when you are not reviewing them
and discard them in a secure manner, such as shredding, when you have completed the examination.

Page 172 of 287


American Society of Plastic Surgeons and the Plastic Surgery Educational Foundation
In-Service Examination

(Please note that this pictorial appears in color in the online examination)

127. A 4-year-old boy is brought to the office by his parents for follow-up evaluation of
inability to fully open the mouth. Six weeks of physical therapy has resulted in no
change in the patient’s condition. On physical examination, maximal incisal
opening is 12 mm. A three-dimensional CT scan is shown. Which of the following
is the most likely diagnosis?
A) Bony ankylosis of the temporomandibular joints
B) Fibrous ankylosis of the temporomandibular joints
C) Hypertrophy of the coronoid processes
D) Möbius syndrome

The correct response is Option C.

Hyperplasia of the coronoid processes is a relatively rare condition that typically results in a slow
and progressive reduction in mandibular opening. The limited mouth opening is due to
mechanical impingement of the enlarged coronoid process against the posterior surface of the
zygoma. The differential diagnosis includes all causes of ankylosis and pseudoankylosis of the
temporomandibular joints. Radiographic imaging is necessary to identify hyperplasia of the
coronoid process and CT scan can most accurately demonstrate the pathology. Surgical
correction consisting of coronoidectomy via an intraoral approach with postoperative physical
therapy is the recommended method of treatment.

Bony ankylosis of the temporomandibular joints will be seen as osseous fusion of the condyle
with the glenoid fossa on CT. Fibrous ankylosis or pseudoankylosis of the temporomandibular
joints is due to fibrosis of the periarticular or paramandibular soft tissues and typically will not
show any bony pathology of the temporomandibular joints on CT. The coronoid processes would
be expected to be normal in both conditions. Möbius syndrome is characterized by a congenital

This examination contains test materials that are owned and copyrighted by the American Society of
Plastic Surgeons and the Plastic Surgery Educational Foundation. Any reproduction of these materials or
any part of them, through any means, including but not limited to, copying or printing electronic files,
reconstruction through memorization or dictation, and/or dissemination of these materials or any part of
them is strictly prohibited. Keep printed materials in a secure location when you are not reviewing them
and discard them in a secure manner, such as shredding, when you have completed the examination.

Page 173 of 287


American Society of Plastic Surgeons and the Plastic Surgery Educational Foundation
In-Service Examination

paralysis of cranial nerve VII, usually bilateral, with or without a combination of other cranial
nerve palsies. The coronoid process would be expected to be normal.

References

1. Gerbino G, Bianchi SD, Bernardi M, et al. Hyperplasia of the mandibular coronoid


process: long-term follow-up after coronoidectomy. J Craniomaxillofac Surg.
1997;25:169-173.
2. Tieghi R, Galie M, Piersanti L, et al. Bilateral hyperplasia of the coronoid processes:
clinical report. J Craniofac Surg. 2005;16(4):723-726.

This examination contains test materials that are owned and copyrighted by the American Society of
Plastic Surgeons and the Plastic Surgery Educational Foundation. Any reproduction of these materials or
any part of them, through any means, including but not limited to, copying or printing electronic files,
reconstruction through memorization or dictation, and/or dissemination of these materials or any part of
them is strictly prohibited. Keep printed materials in a secure location when you are not reviewing them
and discard them in a secure manner, such as shredding, when you have completed the examination.

Page 174 of 287


American Society of Plastic Surgeons and the Plastic Surgery Educational Foundation
In-Service Examination

128. Which of the following is the most common parotid mass in a 1-year-old child?
A) Acinic cell carcinoma
B) Hemangioma
C) Lymphoma
D) Pleomorphic adenoma
E) Warthin tumor

The correct response is Option B.

The most common parotid mass presenting in children is hemangioma. The history is usually one
of identification and rapid enlargement in the first few months of life, with gradual involution
over the next five to six years. Treatment is conservative observation of regression.

Acinic cell carcinoma and lymphoma are malignant tumors of the parotid gland. Acinic cell
carcinoma is a malignant tumor, often multifocal, with aggressive propensity for local invasion
of adjacent structures. Treatment is aggressive surgical resection. It is radiotherapy resistant.
Lymphoma arises as a local manifestation of systemic disease. Treatment is by removal of the
affected lymph node and systemic lymphoma treatment with chemotherapy.

Pleomorphic adenoma, a benign mixed tumor, is the most common tumor of salivary glands in
adults. This tumor is treated by resection and in rare cases (recurrence) can transform into
malignant mixed tumor.

Warthin tumor is a benign tumor and is the second most common tumor (in adults) of the
parotid. It is frequently bilateral (10%) and is most commonly found in men aged 50 to 60 years.
Treatment is removal of the tumor with only minimal margins needed.

References

1. Ariyan S, Narayan D, Ariyan C. Salivary gland tumors. In: Plastic Surgery. Mathes
SJ, Hentz VR, ed. Philadelphia: Elsevier; 2005:74-90.
2. Granick M, Solomon M. Salivary gland tumors. In: Aston SJ, Beasley RW, Thorne
CHM, eds. Grabb & Smith's Plastic Surgery. 5th ed. Philadelphia: Lippincott Raven;
1997:453-457.

This examination contains test materials that are owned and copyrighted by the American Society of
Plastic Surgeons and the Plastic Surgery Educational Foundation. Any reproduction of these materials or
any part of them, through any means, including but not limited to, copying or printing electronic files,
reconstruction through memorization or dictation, and/or dissemination of these materials or any part of
them is strictly prohibited. Keep printed materials in a secure location when you are not reviewing them
and discard them in a secure manner, such as shredding, when you have completed the examination.

Page 175 of 287


American Society of Plastic Surgeons and the Plastic Surgery Educational Foundation
In-Service Examination

129. In a 20-year-old man, examination of the occlusion shows that the mesial buccal
cusp of the maxillary first molar articulates between the first and second
mandibular molars. Cephalometric analysis shows SNA angle of 70 degrees
(normal 80 to 82 degrees) and SNB angle of 79 degrees (normal 79 to 80
degrees). Which of the following is the most likely diagnosis?
A) Long face syndrome
B) Maxillary deficiency
C) Maxillary excess
D) Retrognathia
E) True prognathism

The correct response is Option B.

In a Class III mesioclusion, the mesial buccal cusp of the maxillary first molar articulates
between the first and second mandibular molars. The mandible is mesially placed or prognathic
compared to the maxilla. This type of malocclusion may result from maxillary deficiency,
mandibular overgrowth, or a combination of both.

Long face syndrome results from vertical maxillary excess. Physical findings include excessive
gingival and upper incisor show at rest and during smiling, a long vertical face, and a
retrognathic mandible caused by backward autorotation or true retrognathism.

At rest, normal incisor show is approximately 2 to 3 mm. Decreased incisor show usually
indicates maxillary hypoplasia. Excessive gingival show indicates excess maxillary growth.

This patient has a normal SNB angle, indicating that he does not have true prognathism but that
it only appears excessive because the maxilla is retrusive. The SNA angle (normally 82 degrees)
relates the maxilla to the cranial base. The SNB angle (normally 79 degrees) relates the mandible
to the cranial base.

References

1. Posnick JC. Maxillary deficiencies. In: Fundamentals of Maxillofacial Surgery. New


York: Springer; 1997:287-292.
2. Rosen HM. Maxillary advancement for mandibular prognathism: indications and
rationale. Plast Reconstr Surg. 1991;87:823-832.

This examination contains test materials that are owned and copyrighted by the American Society of
Plastic Surgeons and the Plastic Surgery Educational Foundation. Any reproduction of these materials or
any part of them, through any means, including but not limited to, copying or printing electronic files,
reconstruction through memorization or dictation, and/or dissemination of these materials or any part of
them is strictly prohibited. Keep printed materials in a secure location when you are not reviewing them
and discard them in a secure manner, such as shredding, when you have completed the examination.

Page 176 of 287


American Society of Plastic Surgeons and the Plastic Surgery Educational Foundation
In-Service Examination

(Please note that this pictorial appears in color in the online examination)

130. The 27-year-old woman shown above comes to the office for evaluation because
she has had diplopia for the past 18 months. Physical examination shows orbital
dystopia, enophthalmos, and malar flattening on the right. On the basis of the
current findings, this patient's prior injury was most likely which of the following
fractures?
A) Le Fort II fracture
B) Le Fort III fracture
C) Orbital floor blow-out fracture
D) Orbital roof fracture
E) Orbitozygomatic complex fracture

This examination contains test materials that are owned and copyrighted by the American Society of
Plastic Surgeons and the Plastic Surgery Educational Foundation. Any reproduction of these materials or
any part of them, through any means, including but not limited to, copying or printing electronic files,
reconstruction through memorization or dictation, and/or dissemination of these materials or any part of
them is strictly prohibited. Keep printed materials in a secure location when you are not reviewing them
and discard them in a secure manner, such as shredding, when you have completed the examination.

Page 177 of 287


American Society of Plastic Surgeons and the Plastic Surgery Educational Foundation
In-Service Examination

The correct response is Option E.

The clinical findings of orbital dystopia, enophthalmos, and malar flattening on the patient's right
side are typical for orbitozygomatic fractures. With the downward pull of the muscles, the
zygoma rotates and the orbital volume is increased. This results in enophthalmos and possible
dystopia, depending on the degree of displacement. The rotation of the zygoma causes loss of
malar projection. This patient had only her orbital floor repaired with synthetic mesh. This repair
was inadequate for her type of fracture, and stresses the importance of correct diagnosis and
treatment in order to prevent secondary post-traumatic deformities.

Orbital floor or roof fractures would not result in malar flattening. Le Fort II and III fractures
would cause a malocclusion, which is not present in this patient.

References

1. Hollier LH, Thornton J, Pazmino P, et al. The management of orbitozygomatic


fractures. Plast Reconst Surg. 2003;111:2386-2393.
2. Longaker MT, Kawamoto HK. Evolving thoughts on correcting posttraumatic
enophthalmos. Plast Reconstr Surg. 1998;101:899-906.

This examination contains test materials that are owned and copyrighted by the American Society of
Plastic Surgeons and the Plastic Surgery Educational Foundation. Any reproduction of these materials or
any part of them, through any means, including but not limited to, copying or printing electronic files,
reconstruction through memorization or dictation, and/or dissemination of these materials or any part of
them is strictly prohibited. Keep printed materials in a secure location when you are not reviewing them
and discard them in a secure manner, such as shredding, when you have completed the examination.

Page 178 of 287


American Society of Plastic Surgeons and the Plastic Surgery Educational Foundation
In-Service Examination

131. A 4-month-old boy is brought to the emergency department by his parents for
evaluation after he had a seizure for the first time. Physical examination shows
low nasal bridge and short neck. On laboratory studies, serum calcium level is
6.0 mg/dL (normal = 9.0–10.5 mg/dL) and serum phosphate level is 7.0 mg/dL
(normal = 2.5–4.5 mg/dL). Which of the following is the most likely diagnosis?
A) Albright syndrome
B) Binder syndrome
C) Carpenter syndrome
D) Down syndrome
E) Ectodermal dysplasia

The correct response is Option A.

Albright syndrome is the only syndrome listed that affects calcium and phosphate metabolism. It
is caused by an autosomal-dominant mutation affecting receptor binding to adenylate cyclase.
The resultant craniofacial malformations include a low nasal bridge and a short neck. Binder
syndrome is manifested by maxillary hypoplasia and midface retrusion. Carpenter syndrome is a
rare autosomal-recessive condition that results in brachycephaly due to variable synostoses, in
addition to defects of the cardiovascular, musculoskeletal, and genital systems. Down syndrome
is caused by trisomy of the 21st chromosome. Craniofacial manifestations include
brachycephaly, a low nasal bridge, and inner epicanthal folds. Ectodermal dysplasia is an
X-linked recessive syndrome that results in hypoplastic skin and sparse dermal appendages.

References

1. Jones KL. Smith’s Recognizable Patterns of Human Malformation. 5th ed.


Philadelphia: W.B. Saunders; 1997:446-447
2. Braccini F, Bacciu A, Bruzzo M, et al. Craniofacial fibrous dysplasia associated with
primary hyperparathyroidism. Acta Biomed Ateneo Parmense. 1999;70:5-11.

This examination contains test materials that are owned and copyrighted by the American Society of
Plastic Surgeons and the Plastic Surgery Educational Foundation. Any reproduction of these materials or
any part of them, through any means, including but not limited to, copying or printing electronic files,
reconstruction through memorization or dictation, and/or dissemination of these materials or any part of
them is strictly prohibited. Keep printed materials in a secure location when you are not reviewing them
and discard them in a secure manner, such as shredding, when you have completed the examination.

Page 179 of 287


American Society of Plastic Surgeons and the Plastic Surgery Educational Foundation
In-Service Examination

(Please note that this pictorial appears in color in the online examination)

132. A 28-year-old man comes to the office six months after he sustained partial
amputation of the left ear due to a human bite (shown). For successful
reconstruction, which of the following anatomic areas of the ear must be
addressed?
A) Antihelix and triangular fossa
B) Antitragus and intertragic notch
C) Concha and scaphoid fossa
D) Crus of helix and tubercle
E) Helix and lobule

The correct response is Option E.

Even though the upper helix is still present, the middle helix below the auricular tubercle and the
lower helix in continuity with the lobule are absent. Precise understanding of the anatomy of the
ear is important in the planning of reconstruction regarding framework and soft-tissue
requirements.

The antihelix is intact and represents the border of the defect. Inferiorly, the antihelix ends in the
antitragus; superiorly, it divides into two crura. The triangular fossa separates the superior crus
from the inferior crus. The intertragic notch is located between the tragus and antitragus. The
concha constitutes the bowl-shaped area in the center of the ear from which the root or crus of
the helix arises. The scaphoid fossa is the groove between the upper helix and the superior crus

This examination contains test materials that are owned and copyrighted by the American Society of
Plastic Surgeons and the Plastic Surgery Educational Foundation. Any reproduction of these materials or
any part of them, through any means, including but not limited to, copying or printing electronic files,
reconstruction through memorization or dictation, and/or dissemination of these materials or any part of
them is strictly prohibited. Keep printed materials in a secure location when you are not reviewing them
and discard them in a secure manner, such as shredding, when you have completed the examination.

Page 180 of 287


American Society of Plastic Surgeons and the Plastic Surgery Educational Foundation
In-Service Examination

of the antihelix. The auricular tubercle is a thickening in the helix located between the upper and
middle thirds of the helix.

References

1. Berkovitz BKB, Moxham BJ. The ear. In: Berkovitz BKB, Moxham BJ, eds. Head
and Neck Anatomy: A Clinical Reference. London: Martin Dunitz; 2002:291-314.
2. Moore KL. The head. In: Moore KL, ed. Clinically Oriented Anatomy. 3rd ed.
Baltimore: Williams & Wilkins; 1992:637-780.
3. Williams PL, Warwick R, Dyson M, et al. The auditory and vestibular apparatus. In:
Williams PL, Warwick R, Dyson M, et al. Gray's Anatomy. 37th ed. Edinburgh:
Churchill Livingstone; 1989:1219-1228.

This examination contains test materials that are owned and copyrighted by the American Society of
Plastic Surgeons and the Plastic Surgery Educational Foundation. Any reproduction of these materials or
any part of them, through any means, including but not limited to, copying or printing electronic files,
reconstruction through memorization or dictation, and/or dissemination of these materials or any part of
them is strictly prohibited. Keep printed materials in a secure location when you are not reviewing them
and discard them in a secure manner, such as shredding, when you have completed the examination.

Page 181 of 287


American Society of Plastic Surgeons and the Plastic Surgery Educational Foundation
In-Service Examination

(Please note that this pictorial appears in color in the online examination)

133. An 8-month-old girl is brought to the office for evaluation of the skull asymmetry
shown. Physical examination shows posterior plagiocephaly with flattening of the
right occipital area and an ipsilateral mastoid bulge. CT scan is shown. Which of
the following is the most appropriate management for correction of this deformity?
A) No treatment
B) Modification of sleep position
C) Molding helmet therapy
D) Remodeling of the posterior vault
E) Bilateral fronto-orbital advancement and remodeling

This examination contains test materials that are owned and copyrighted by the American Society of
Plastic Surgeons and the Plastic Surgery Educational Foundation. Any reproduction of these materials or
any part of them, through any means, including but not limited to, copying or printing electronic files,
reconstruction through memorization or dictation, and/or dissemination of these materials or any part of
them is strictly prohibited. Keep printed materials in a secure location when you are not reviewing them
and discard them in a secure manner, such as shredding, when you have completed the examination.

Page 182 of 287


American Society of Plastic Surgeons and the Plastic Surgery Educational Foundation
In-Service Examination

The correct response is Option D.

This infant has premature obliteration of her right lambdoid suture (lambdoid synostosis),
resulting in posterior plagiocephaly. Lambdoid synostosis is treated with posterior vault
expansion and remodeling. Without treatment, increased intracranial pressure can occur.
Modification of the infant's sleep position and use of molding helmet therapy are appropriate
treatments for plagiocephaly caused by deformation forces, but not for plagiocephaly caused by
lambdoid synostosis. Bilateral fronto-orbital advancement and remodeling is not appropriate for
managing lambdoid synostosis because the synostosis and skull asymmetry are of the posterior
vault.

Management of posterior plagiocephaly requires differentiation of occipitoparietal flattening


caused by lambdoid synostosis from that caused by deformation forces because deformational
plagiocephaly does not require surgical correction. However, the two entities are often difficult
to distinguish. Lambdoid synostosis causes a more trapezium-shaped skull with occipitoparietal
flattening and an ipsilateral mastoid bulge. The ipsilateral ear is posteriorly positioned in relation
to the other ear. Deformational plagiocephaly causes a parallelogram-shaped skull and displaces
the ipsilateral ear anteriorly.

References

1. Huang MH, Mouradian WE, Cohen SR, Gruss JS. The differential diagnosis of
abnormal head shapes: separating craniosynostosis from positional deformities and
normal variants. Cleft Palate Craniofac J. 1998;35:204-211.
2. Mulliken JB, Vander Woude DL, Hansen M, LaBrie RA, Scott RM. Analysis of
posterior plagiocephaly: deformational versus synostotic. Plast Reconstr Surg.
1999;103(2):371-380.
3. Ehret FW, Whelan MF, Ellenbogen RG, Cunningham ML, Gruss JS. Differential
diagnosis of the trapezoid-shaped head. Cleft Palate Craniofac J. 2004;41:13-19.
4. Zoller JE, Mischkowski RA, Speder B. Preliminary results of standardized occipital
advancement in the treatment of lambdoid synostosis. J Craniomaxillofac Surg.
2002;30:343-348.

This examination contains test materials that are owned and copyrighted by the American Society of
Plastic Surgeons and the Plastic Surgery Educational Foundation. Any reproduction of these materials or
any part of them, through any means, including but not limited to, copying or printing electronic files,
reconstruction through memorization or dictation, and/or dissemination of these materials or any part of
them is strictly prohibited. Keep printed materials in a secure location when you are not reviewing them
and discard them in a secure manner, such as shredding, when you have completed the examination.

Page 183 of 287


American Society of Plastic Surgeons and the Plastic Surgery Educational Foundation
In-Service Examination

134. A 45-year-old man with microgenia is evaluated before undergoing augmentation


genioplasty using an alloplastic prosthesis. A lateral cephalogram shows
horizontal deficiency of 5 mm. A prosthesis of which of the following thicknesses is
most appropriate for correction of this patient's deformity?
A) 3 mm
B) 6 mm
C) 9 mm
D) 12 mm

The correct response is Option B.

The ratio of soft-tissue response to augmentation genioplasty is approximately 0.8 to 1.0.


Therefore, a 5-mm deficiency would best be corrected with a slightly thicker, 6-mm implant.

References

1. Ewing M, Ross RB. Soft tissue response to mandibular advancement and genioplasty.
Am J Orthod Dentofacial Orthop. 1992;101:550-555.
2. Ferraro JW. Cephalometry and cephalometric analysis. In: Ferarro JW, ed.
Fundamentals of Maxillofacial Surgery. New York: Springer-Verlag; 1997:233-245.

This examination contains test materials that are owned and copyrighted by the American Society of
Plastic Surgeons and the Plastic Surgery Educational Foundation. Any reproduction of these materials or
any part of them, through any means, including but not limited to, copying or printing electronic files,
reconstruction through memorization or dictation, and/or dissemination of these materials or any part of
them is strictly prohibited. Keep printed materials in a secure location when you are not reviewing them
and discard them in a secure manner, such as shredding, when you have completed the examination.

Page 184 of 287


American Society of Plastic Surgeons and the Plastic Surgery Educational Foundation
In-Service Examination

(Please note that this pictorial appears in color in the online examination)

135. A 7-year-old girl is brought to the office for consultation regarding the congenital
malformation shown. Which of the following anatomic structures is absent in this
patient?
A) Anterior nasal spine
B) Dentoalveolus
C) Hard palate
D) Maxilla
E) Nasal bones

The correct response is Option A.

The anterior nasal spine is absent in patients with Binder syndrome. In 1962, Von Binder
described a form of nasomaxillary hypoplasia that included nasomaxillary hypoplasia, a convex
lip, a vertically short nose, a flat frontonasal angle, an absent anterior nasal spine, limited nasal
mucosa, and hypoplastic frontal sinuses. It is believed to be caused by a disturbance of the
prosencephalic induction center at a critical phase in development.

This examination contains test materials that are owned and copyrighted by the American Society of
Plastic Surgeons and the Plastic Surgery Educational Foundation. Any reproduction of these materials or
any part of them, through any means, including but not limited to, copying or printing electronic files,
reconstruction through memorization or dictation, and/or dissemination of these materials or any part of
them is strictly prohibited. Keep printed materials in a secure location when you are not reviewing them
and discard them in a secure manner, such as shredding, when you have completed the examination.

Page 185 of 287


American Society of Plastic Surgeons and the Plastic Surgery Educational Foundation
In-Service Examination

References

1. Posnick JC. Craniofacial and Maxillofacial Surgery in Children and Young Adults.
Philadelphia: WB Saunders; 2000:446-468.
2. Stricker M, VanDer Meulen J, Mazzola BR. Craniofacial Malformations. New York:
Churchill Livingstone; 1990:464-465.

This examination contains test materials that are owned and copyrighted by the American Society of
Plastic Surgeons and the Plastic Surgery Educational Foundation. Any reproduction of these materials or
any part of them, through any means, including but not limited to, copying or printing electronic files,
reconstruction through memorization or dictation, and/or dissemination of these materials or any part of
them is strictly prohibited. Keep printed materials in a secure location when you are not reviewing them
and discard them in a secure manner, such as shredding, when you have completed the examination.

Page 186 of 287


American Society of Plastic Surgeons and the Plastic Surgery Educational Foundation
In-Service Examination

136. A 7-year-old boy is brought to the emergency department after sustaining trauma
to the face. He has nausea and vomiting but is alert and oriented to time and
place. Pulse rate is 48/min. Physical examination shows right periorbital
ecchymosis, diplopia, and limited vertical gaze. Visual acuity is within normal
limits. Which of the following is the most likely finding on CT scan of the head and
craniofacial skeleton?
A) Compression of the optic nerve
B) Displaced fracture of the zygoma
C) Large (greater than 3 cm) displaced fracture of the orbital floor
D) Minimally displaced fracture of the orbital floor

The correct response is Option D.

Based on the patient's age, signs, and symptoms, a CT scan is most likely to show a minimally
displaced fracture of the orbital floor, suggesting a trapdoor fracture of the orbital floor. This
type of fracture allows herniation of the orbital contents, which are then entrapped. It most
commonly occurs in children, possibly because of the relative elasticity of their bones, which
allows them to snap back into position after tissue herniates through the fracture. As in this
patient, a trapdoor fracture of the orbital floor can cause an oculocardiac reflex, which produces
bradycardia, nausea, and syncope.

A CT scan is not likely to show compression of the optic nerve because the patient does not have
a relative afferent pupillary defect. A CT scan also is unlikely to show a displaced fracture of the
zygoma because this type of fracture usually occurs in older patients. This study should not
disclose a large displaced fracture of the orbital floor because this type of fracture does not
typically cause oculocardiac reflex. Because the patient’s mental status is intact, a CT scan is not
likely to show a subarachnoid hemorrhage.

In a trapdoor fracture of the orbital floor, the muscles most likely to be entrapped are the inferior
rectus and inferior oblique muscles. If entrapment restricts ocular movement, immediate surgery
is recommended to prevent irreversible ischemia and fibrosis of the entrapped contents.

References

1. Grant JH, Patrinely JR, Weiss AH, et al. Trapdoor fracture of the orbit in a pediatric
population. Plast Reconstr Surg. 2002;109:482-489.
2. Sires BS, Stanley RB, Levine LM. Oculocardiac reflex caused by orbital floor
trapdoor fracture: an indication for urgent repair. Arch Ophthalmol.
1998;116:955-956.

This examination contains test materials that are owned and copyrighted by the American Society of
Plastic Surgeons and the Plastic Surgery Educational Foundation. Any reproduction of these materials or
any part of them, through any means, including but not limited to, copying or printing electronic files,
reconstruction through memorization or dictation, and/or dissemination of these materials or any part of
them is strictly prohibited. Keep printed materials in a secure location when you are not reviewing them
and discard them in a secure manner, such as shredding, when you have completed the examination.

Page 187 of 287


American Society of Plastic Surgeons and the Plastic Surgery Educational Foundation
In-Service Examination

137. Formation of the primary palate begins during which of the following weeks of
gestation?
A) First
B) Third
C) Fifth
D) Eighth
E) Eleventh

The correct response is Option C.

Formation of the primary palate begins during the fifth week of gestation and is completed by the
end of the sixth week of gestation. Anatomically, it is located anterior to the incisive foramen.
The premaxilla only contains the central and lateral incisors.

References

1. Sadove AM, van Aalst JA, Culp JA, et al. Cleft palate repair: art and issues. Clin
Plast Surg. 2004;31:231-241.
2. Witt PD, Marsh JL. Cleft palate deformities. In: Bentz ML, ed. Pediatric Plastic
Surgery. Stamford: Appleton & Lange; 1998:94.

This examination contains test materials that are owned and copyrighted by the American Society of
Plastic Surgeons and the Plastic Surgery Educational Foundation. Any reproduction of these materials or
any part of them, through any means, including but not limited to, copying or printing electronic files,
reconstruction through memorization or dictation, and/or dissemination of these materials or any part of
them is strictly prohibited. Keep printed materials in a secure location when you are not reviewing them
and discard them in a secure manner, such as shredding, when you have completed the examination.

Page 188 of 287


American Society of Plastic Surgeons and the Plastic Surgery Educational Foundation
In-Service Examination

138. A 25-year-old man comes to the office because he has numbness of the tip of the
nose six weeks after he sustained a naso-orbital-ethmoid fracture during a motor
vehicle collision. The most likely cause of the numbness is damage to which of the
following nerves?
A) Anterior ethmoidal
B) Infraorbital
C) Infratrochlear
D) Nasopalatine
E) Pterygopalatine

The correct response is Option A.

The anterior ethmoidal nerve innervates the nasal tip. The infratrochlear and infraorbital nerves
innervate the sidewalls and dorsum. The lateral branch of the pterygopalatine nerve supplies the
upper and middle turbinates, whereas the medial branch supplies the septum.

The nasopalatine nerve, which is the terminal branch of the pterygopalatine nerve, innervates the
maxillary incisor teeth, gingiva, and palate.

References

1. Netter, FH. Atlas of Human Anatomy. Summit, NJ: Ciba-Geigy; 1991:401.


2. Langman J. Medical Embryology. Baltimore: Williams & Wilkins, 1981.

This examination contains test materials that are owned and copyrighted by the American Society of
Plastic Surgeons and the Plastic Surgery Educational Foundation. Any reproduction of these materials or
any part of them, through any means, including but not limited to, copying or printing electronic files,
reconstruction through memorization or dictation, and/or dissemination of these materials or any part of
them is strictly prohibited. Keep printed materials in a secure location when you are not reviewing them
and discard them in a secure manner, such as shredding, when you have completed the examination.

Page 189 of 287


American Society of Plastic Surgeons and the Plastic Surgery Educational Foundation
In-Service Examination

139. A 64-year-old man comes to the office for evaluation because of a lesion in the
mouth and numbness of the chin. Examination shows a 3-cm ulcerative lesion on
the anterior floor of the mouth that is fixed to the periosteum of the lingual
mandible. A 4-cm mobile node is seen in the right submandibular triangle. Biopsy
of the lesion shows squamous cell carcinoma. Distant metastases are not
apparent. Which of the following is the most accurate TNM staging of this tumor?
A) T2 N2b M0
B) T3 N1 M0
C) T4 N1 M0
D) T4 N1 M1
E) T4 N2a M0

The correct response is Option E.

Tumors of the oral cavity are staged according to the TNM (Tumor, Node, Metastasis) system.
Chin numbness and fixation to the periosteum indicate invasion of the tumor into the mandible
and involvement of the inferior alveolar nerve. Therefore, the T stage of this tumor would be a
T4. A single 4-cm, mobile node would stage this tumor as an N2a. Absent distant metastases
stage this tumor as an M0.

Oral Cavity:
T1 Tumor <2 cm
T2 Tumor >2 but <4 cm
T3 Tumor >4 cm
Tumor invades adjacent structures such as cortical bone, tongue, skin, or soft tissues of
T4
the neck

N1 One ipsilateral node <3 cm


N2a One ipsilateral node >3 and <6 cm
N2b Multiple ipsilateral nodes <6 cm
N2c Bilateral contralateral nodes <6 cm
N3 Any nodes >6 cm

M0 No distal metastasis
M1 Distal metastasis

This examination contains test materials that are owned and copyrighted by the American Society of
Plastic Surgeons and the Plastic Surgery Educational Foundation. Any reproduction of these materials or
any part of them, through any means, including but not limited to, copying or printing electronic files,
reconstruction through memorization or dictation, and/or dissemination of these materials or any part of
them is strictly prohibited. Keep printed materials in a secure location when you are not reviewing them
and discard them in a secure manner, such as shredding, when you have completed the examination.

Page 190 of 287


American Society of Plastic Surgeons and the Plastic Surgery Educational Foundation
In-Service Examination

References

1. Alvi A, Myers EN, Johnson JT. Cancer of the oral cavity. In: Myers EN, Suen JY,
eds. Cancer of the Head and Neck. Philadelphia: WB Saunders; 1996:321-360.
2. American Joint Committee on Cancer. Lip and oral cavity. In: American Joint
Committee on Cancer: Manual for Staging of Cancer. 6th ed. Philadelphia:
Lippincott Williams & Wilkins; 2002:23-30.

This examination contains test materials that are owned and copyrighted by the American Society of
Plastic Surgeons and the Plastic Surgery Educational Foundation. Any reproduction of these materials or
any part of them, through any means, including but not limited to, copying or printing electronic files,
reconstruction through memorization or dictation, and/or dissemination of these materials or any part of
them is strictly prohibited. Keep printed materials in a secure location when you are not reviewing them
and discard them in a secure manner, such as shredding, when you have completed the examination.

Page 191 of 287


American Society of Plastic Surgeons and the Plastic Surgery Educational Foundation
In-Service Examination

(Please note that this pictorial appears in color in the online examination)

140. The 2-year-old boy shown has features typical of which of the following
syndromes?
A) Apert
B) Binder
C) Crouzon
D) Pfeiffer
E) Treacher Collins

The correct response is Option E.

The appearance of the patient shown demonstrates Treacher Collins syndrome, characterized by
bilaterally symmetrical abnormalities of structures within the first and second branchial arches.
Features include a convex facial profile with a retrusive lower jaw and chin, anti-mongoloid slant
of the palpebral fissures, lower lid colobomas, partial absence of eyelid cilia, absent or
malformed external ears, hypoplasia of the malar bones, and variable cleft palate.

Binder syndrome is characterized by nasomaxillary hypoplasia. Apert, Crouzon, and Pfeiffer

This examination contains test materials that are owned and copyrighted by the American Society of
Plastic Surgeons and the Plastic Surgery Educational Foundation. Any reproduction of these materials or
any part of them, through any means, including but not limited to, copying or printing electronic files,
reconstruction through memorization or dictation, and/or dissemination of these materials or any part of
them is strictly prohibited. Keep printed materials in a secure location when you are not reviewing them
and discard them in a secure manner, such as shredding, when you have completed the examination.

Page 192 of 287


American Society of Plastic Surgeons and the Plastic Surgery Educational Foundation
In-Service Examination

syndromes are all craniosynostotic syndromes that result in midfacial hypoplasia and a concave
facial profile of varying degrees.

References

1. Posnick JC. Craniofacial and Maxillofacial Surgery in Children and Young Adults.
Philadelphia: WB Saunders; 2000:391-418.
2. Freihofer HPM. Variations in the correction of Treacher-Collins syndrome. Plast
Reconstr Surg. 1997;99:647-657.

This examination contains test materials that are owned and copyrighted by the American Society of
Plastic Surgeons and the Plastic Surgery Educational Foundation. Any reproduction of these materials or
any part of them, through any means, including but not limited to, copying or printing electronic files,
reconstruction through memorization or dictation, and/or dissemination of these materials or any part of
them is strictly prohibited. Keep printed materials in a secure location when you are not reviewing them
and discard them in a secure manner, such as shredding, when you have completed the examination.

Page 193 of 287


American Society of Plastic Surgeons and the Plastic Surgery Educational Foundation
In-Service Examination

141. In a patient with fracture of the frontal sinus, laboratory analysis of which of the
following is most reliable to confirm cerebrospinal rhinorrhea?
A) Albumin
B) Beta-2 transferrin
C) Glucose
D) Glutamine
E) Lactic acid

The correct response is Option B.

Initially described for the detection of cerebrospinal fluid (CSF) in 1979, beta-2 transferrin is
currently accepted as the most reliable method of laboratory testing in determining the presence
of CSF in rhinorrhea and otorrhea. It has been reported to have a sensitivity near 100% and a
specificity near 95%. Beta-2 transferrin is a carbohydrate-free (desialylated) isoform of
transferrin that is almost exclusively found in CSF. It is also found in perilymph in the cochlea
and the aqueous and vitreous humor of the eye, but in lower concentrations. It is not present in
blood, nasal mucus, tears, or mucosal discharge. Its presence is identified using immunofixation
electrophoreses. Glucose testing of rhinorrhea or otorrhea was commonly used in the past, but
interpretation of the studies was confounded by contamination of glucose containing fluid (tears,
nasal mucus, blood in nasal mucus) or low CSF glucose levels (as seen in meningitis). Glucose
testing of CSF has been found to have an unacceptably high rate of false-positive results.

References

1. Warnecke A, Averbeck T, Wurster U, et al. Diagnostic relevance of β2-transferrin for


the detection of cerebrospinal fluid fistulas. Arch Otolaryngol Head Neck Surg.
2004;130:1178-1184.
2. Chan DTM, Poon WS, Ip CP, et al. How useful is glucose detection in diagnosing
cerebrospinal fluid leak? The rational use of CT and β2-transferrin assay in detection
of cerebrospinal fluid fistula. Asian J Surg. 2004;27:39-42.

This examination contains test materials that are owned and copyrighted by the American Society of
Plastic Surgeons and the Plastic Surgery Educational Foundation. Any reproduction of these materials or
any part of them, through any means, including but not limited to, copying or printing electronic files,
reconstruction through memorization or dictation, and/or dissemination of these materials or any part of
them is strictly prohibited. Keep printed materials in a secure location when you are not reviewing them
and discard them in a secure manner, such as shredding, when you have completed the examination.

Page 194 of 287


American Society of Plastic Surgeons and the Plastic Surgery Educational Foundation
In-Service Examination

For each structure of the neck, choose the fascial plane in which it is enveloped (A–C).

A. Pretracheal fascia
B. Prevertebral fascia
C. Superficial fascia

142. Trapezius muscle

143. Anterior scalene muscle

The correct response for Item 142 is Option C and for Item 143 is Option B.

The superficial fascia envelops the sternocleidomastoid, the trapezius, and the suprahyoid
muscles. The prevertebral fascia incorporates all of the scalene and paravertebral muscles. The
pretracheal fascia incorporates the thyroid and trachea.

References

1. Moore KL. Clinically oriented anatomy. 2nd ed. Baltimore: Williams & Wilkins;
1992:783-805.
2. Breisch EA. Cutaneous surgical anatomy of the head and neck. New York: Churchill
Livingstone; 1992:81-106.

This examination contains test materials that are owned and copyrighted by the American Society of
Plastic Surgeons and the Plastic Surgery Educational Foundation. Any reproduction of these materials or
any part of them, through any means, including but not limited to, copying or printing electronic files,
reconstruction through memorization or dictation, and/or dissemination of these materials or any part of
them is strictly prohibited. Keep printed materials in a secure location when you are not reviewing them
and discard them in a secure manner, such as shredding, when you have completed the examination.

Page 195 of 287


American Society of Plastic Surgeons and the Plastic Surgery Educational Foundation
In-Service Examination

144. A 6-month-old boy is referred by a pediatrician for evaluation of asymmetric shape


of the skull. Physical examination shows flattening of the right posterior skull,
prominence of the right side of the forehead and zygomatic complex, and forward
advancement of the right ear. Which of the following is NOT an option in the
management of this patient's condition?
A) Observation
B) Frequent repositioning exercises
C) Helmet-molding therapy
D) Three-dimensional CT scan of the head
E) Surgical remodeling of the cranial vault

The correct response is Option E.

Surgical management of plagiocephaly is not indicated at this young age.

Since the “back-to-sleep” campaign began recommending that young children be placed on their
backs to sleep in an effort to limit the incidence of sudden infant death syndrome (SIDS), many
children have been referred for evaluation of plagiocephaly. Physical examination continues to
be the diagnostic method for separating plagiocephaly as a result of positional molding (ie,
external pressures) from synostotic plagiocephaly.

Positional plagiocephaly results from external pressure on the skull—in this case, the right
posterior skull. As the brain grows, it pushes the ipsilateral structures forward, including the
zygomatic complex, forehead, and ear. From a vertex view, the skull takes on the appearance of
a parallelogram.

In contrast, when a growth suture is fused as in craniosynostosis, growth is restricted


perpendicular to the suture and accelerated parallel to that suture (Virchow’s Law). In cases of
suspected lambdoid synostosis, the growth of the contralateral forehead is accelerated and the
contralateral structures (such as the ear) are advanced. From the vertex, the skull becomes
trapezoid in appearance. Similarly from a posterior view, there is ipsilateral mastoid bossing and
the tilt.

A CT scan may be obtained for many reasons but is not necessary to establish the diagnosis in
this patient. Physical examination should be adequate to diagnose positional plagiocephaly.

Treatment of positional plagiocephaly initially involves behavioral changes and frequent


repositioning of the infant. Prone playtime activities are encouraged. Sometimes physical therapy

This examination contains test materials that are owned and copyrighted by the American Society of
Plastic Surgeons and the Plastic Surgery Educational Foundation. Any reproduction of these materials or
any part of them, through any means, including but not limited to, copying or printing electronic files,
reconstruction through memorization or dictation, and/or dissemination of these materials or any part of
them is strictly prohibited. Keep printed materials in a secure location when you are not reviewing them
and discard them in a secure manner, such as shredding, when you have completed the examination.

Page 196 of 287


American Society of Plastic Surgeons and the Plastic Surgery Educational Foundation
In-Service Examination

is required. Custom helmet molding therapy is recommended for severe skull asymmetry or for
patients with other developmental issues that would complicate behavioral modifications or
repositioning techniques. In this child, poor muscle tone and slight developmental delay lead us
to recommend helmet therapy. It is not unusual for the posterior fontanelle to be closed at this
time.

References

1. Ehret FW, Whelan MF, Ellenbogen RG, et al. Differential diagnosis of the
trapezoid-shaped head. Cleft Palate Craniofac J. 2004;41:13-19.
2. Huang M, Gruss JS, Clarren SK, et al. The differential diagnosis of posterior
plagiocephaly: true lambdoid synostosis versus positional molding. Plast Reconstr
Surg. 1996;98:765-774.
3. Teichgraeber JF, Seymour-Dempsey K, Baumgartner JE, et al. Molding helmet
therapy in the treatment of brachycephaly and plagiocephaly. J Craniofac Surg.
2004;15:118-123.

This examination contains test materials that are owned and copyrighted by the American Society of
Plastic Surgeons and the Plastic Surgery Educational Foundation. Any reproduction of these materials or
any part of them, through any means, including but not limited to, copying or printing electronic files,
reconstruction through memorization or dictation, and/or dissemination of these materials or any part of
them is strictly prohibited. Keep printed materials in a secure location when you are not reviewing them
and discard them in a secure manner, such as shredding, when you have completed the examination.

Page 197 of 287


American Society of Plastic Surgeons and the Plastic Surgery Educational Foundation
In-Service Examination

145. A fracture in which of the following locations is most likely to cause disruption of
mandibular growth?
A) Angle
B) Body
C) Condyle
D) Ramus
E) Symphysis

The correct response is Option C.

The area most commonly associated with growth disturbances in pediatric patients with
mandibular fractures is the condyle. The incidence of facial fractures in the pediatric patient
population is as high as 15% of all maxillofacial traumas; the most common location is the
mandible. The condyle in the pediatric patient, which is the growth center of the mandible, is the
most common site for fracture in this patient population as well. Conservative therapy is used in
nondisplaced fractures with no malocclusion. Displaced fractures are managed with closed
reduction and immobilization when possible, deferring open reduction to only the most severe
cases.

The angle, body, ramus, and symphysis are not growth centers. Fractures in these locations
would not cause disruption of mandibular growth.

References

1. Demianczuk A, Verchere C, Phillips J. The effect on facial growth of pediatric


mandibular fractures. J Craniofacial Surg. 1999;10:323-328.
2. Posnick J, Wells M, Pron G. Pediatric facial fractures: evolving patterns of treatment.
J Oral Maxillofac Surg. 1993;51:836-844.

This examination contains test materials that are owned and copyrighted by the American Society of
Plastic Surgeons and the Plastic Surgery Educational Foundation. Any reproduction of these materials or
any part of them, through any means, including but not limited to, copying or printing electronic files,
reconstruction through memorization or dictation, and/or dissemination of these materials or any part of
them is strictly prohibited. Keep printed materials in a secure location when you are not reviewing them
and discard them in a secure manner, such as shredding, when you have completed the examination.

Page 198 of 287


American Society of Plastic Surgeons and the Plastic Surgery Educational Foundation
In-Service Examination

146. In distraction osteogenesis of the mandible, which of the following is a


disadvantage of an external device compared with an internal device?
A) Ability to alter the vector of distraction
B) Ability to manipulate the bony regenerate
C) Greater expansion length
D) Shorter moment arm at the device–bone interface

The correct response is Option D.

Distraction osteogenesis of the mandible may be performed using external devices or internal
devices. External devices are secured to the bony segments via percutaneous pins. Internal
devices are placed subperiosteally, typically via an intraoral incision, and are anchored to the
bone with screws passing through foot plates.

Advantages of external devices include the potential for three-dimensional distraction, the ability
to “mold” the bony regenerate, adjust dental relationships prior to consolidation, a greater length
of distraction, and greater ease of placement.

Disadvantages include creation of more prominent facial scars and an increased distance from
the body of the distractor to the bone surface. The latter leads to a longer “moment arm” at the
device–bone interface and an increased possibility of pin loosening. Internal devices avoid the
creation of a significant facial scar, and are much more inconspicuous and cumbersome. They do
not allow for alteration of the distraction vector after the device is placed and generally the
length of distraction is less.

References

1. Yu JC, Fearon J, Havlik RJ, et al. Distraction osteogenesis of the craniofacial


skeleton. Plast Reconstr Surg. 2004;114:1-20.
2. Hollier LH, McCarthy JG. Distraction osteogenesis. In: Goldwyn RM, Cohen MN,
eds. The Unfavorable Result in Plastic Surgery: Avoidance and Treatment.
Philadelphia: Lippincott Williams & Wilkins; 2001:210-217.

This examination contains test materials that are owned and copyrighted by the American Society of
Plastic Surgeons and the Plastic Surgery Educational Foundation. Any reproduction of these materials or
any part of them, through any means, including but not limited to, copying or printing electronic files,
reconstruction through memorization or dictation, and/or dissemination of these materials or any part of
them is strictly prohibited. Keep printed materials in a secure location when you are not reviewing them
and discard them in a secure manner, such as shredding, when you have completed the examination.

Page 199 of 287


American Society of Plastic Surgeons and the Plastic Surgery Educational Foundation
In-Service Examination

147. If the 13-year-old boy whose CT scan is shown has an epibulbar dermoid, the
most appropriate classification of his condition is which of the following
syndromes?
A) Goldenhar
B) Stickler
C) Treacher Collins
D) Van der Woude
E) Velocardiofacial

The correct response is Option A.

The CT scan demonstrates a patient with the type IIb mandibular deformity of hemifacial or
craniofacial microsomia. Patients with hemifacial microsomia with associated epibulbar
dermoids and spinal or scapular deformities are classified as having Goldenhar syndrome.

Van der Woude syndrome is an autosomal-dominant cleft lip/palate syndrome. The defining
feature in addition to cleft lip and/or palate is “lip pits.” These lip pits are often found on the
lower lip and are accessory salivary glands.

Velocardiofacial syndrome (previously known as DiGeorge syndrome, conotruncal anomaly,

This examination contains test materials that are owned and copyrighted by the American Society of
Plastic Surgeons and the Plastic Surgery Educational Foundation. Any reproduction of these materials or
any part of them, through any means, including but not limited to, copying or printing electronic files,
reconstruction through memorization or dictation, and/or dissemination of these materials or any part of
them is strictly prohibited. Keep printed materials in a secure location when you are not reviewing them
and discard them in a secure manner, such as shredding, when you have completed the examination.

Page 200 of 287


American Society of Plastic Surgeons and the Plastic Surgery Educational Foundation
In-Service Examination

Sphrintzen syndrome, Catch-22 syndrome, and 22 minus syndrome) is a syndrome diagnosed by


the addition or deletion of the 22q.11 gene. Common findings include congenital heart
anomalies, clefting and velopharyngeal insufficiency, and facies that include a broad nasal root
and narrow alar bases, elongated face with low tone, low-set ears with folded helixes, malar
flattening, and retrognathia. It is estimated that up to 8% to 10% of patients with isolated cleft
palate carry the 22q.11 deletion/addition.

Treacher Collins syndrome or mandibulofacial dysostosis has three pathognomonic features:


lower eyelid colobomas, zygomatic hypoplasia with hypoplastic or absent zygomas and
inferolateral orbital rim clefting, and bilateral mandibular hypoplasia. Bilateral ear anomalies
ranging from simple deformities to complete microtia are often additional findings.

Stickler syndrome is associated with Pierre Robin sequence (micrognathia, glossoptosis,


respiratory distress, and possibly cleft palate) and includes possible skeletal anomalies as well as
optical conditions such as myopia.

References

1. Molina F. Hemifacial microsomia and Goldenhar syndrome. In: Lin KY, ed.
Craniofacial Surgery Science and Surgical Technique. Philadelphia: WB Saunders;
2002:272.
2. Jones KL. Oculo-auriculo-vertebral spectrum. In: Smith's Recognizable Patterns of
Human Malformation. 5th ed. Philadelphia: WB Saunders; 1997:642.

This examination contains test materials that are owned and copyrighted by the American Society of
Plastic Surgeons and the Plastic Surgery Educational Foundation. Any reproduction of these materials or
any part of them, through any means, including but not limited to, copying or printing electronic files,
reconstruction through memorization or dictation, and/or dissemination of these materials or any part of
them is strictly prohibited. Keep printed materials in a secure location when you are not reviewing them
and discard them in a secure manner, such as shredding, when you have completed the examination.

Page 201 of 287


American Society of Plastic Surgeons and the Plastic Surgery Educational Foundation
In-Service Examination

148. In the Tessier system, which of the following classifications represents the most
common facial cleft?
A) Tessier No. 0
B) Tessier No. 3
C) Tessier No. 7
D) Tessier No. 9
E) Tessier No. 14

The correct response is Option C.

The most common facial cleft is a Tessier No. 7. This is a cleft that begins at the lateral oral
commissure and extends laterally. From a soft-tissue standpoint, it creates macrostomia. A
Tessier No. 0 cleft involves the midline of the upper lip and nose. The Tessier No. 14 cleft is the
cranial extension of this. A Tessier No. 3 cleft involves the lateral nasal ala and the medial
canthus of the eye. A Tessier No. 9 cleft is actually the least common cleft. It extends from the
superolateral orbit into the temporal region.

This examination contains test materials that are owned and copyrighted by the American Society of
Plastic Surgeons and the Plastic Surgery Educational Foundation. Any reproduction of these materials or
any part of them, through any means, including but not limited to, copying or printing electronic files,
reconstruction through memorization or dictation, and/or dissemination of these materials or any part of
them is strictly prohibited. Keep printed materials in a secure location when you are not reviewing them
and discard them in a secure manner, such as shredding, when you have completed the examination.

Page 202 of 287


American Society of Plastic Surgeons and the Plastic Surgery Educational Foundation
In-Service Examination

References

1. Kawamoto HK. Rare craniofacial clefts. In: McCarthy JG, ed. Plastic Surgery.
Philadelphia: WB Saunders; 1990:2952-2955.
2. Kawamoto HK. The kaleidoscopic world of rare craniofacial clefts: order out of
chaos (Tessier classification). Clin Plast Surg. 1976;3:529-572.

This examination contains test materials that are owned and copyrighted by the American Society of
Plastic Surgeons and the Plastic Surgery Educational Foundation. Any reproduction of these materials or
any part of them, through any means, including but not limited to, copying or printing electronic files,
reconstruction through memorization or dictation, and/or dissemination of these materials or any part of
them is strictly prohibited. Keep printed materials in a secure location when you are not reviewing them
and discard them in a secure manner, such as shredding, when you have completed the examination.

Page 203 of 287


American Society of Plastic Surgeons and the Plastic Surgery Educational Foundation
In-Service Examination

149. Cranialization is indicated for patients with which of the following conditions of the
frontal sinus?
A) Fracture of the anterior sinus wall
B) Fracture of the nasofrontal duct
C) Fracture of the posterior sinus wall
D) Fracture of the sinus floor into orbital roof
E) Post-traumatic frontal sinus mucocele

The correct response is Option C.

Although cranialization is indicated for fractures of the posterior sinus wall, not all frontal sinus
fractures involving the posterior wall require cranialization. Exceptions would be noted in cases
of undisplaced fracture in which the dura is deemed intact by clinical observation and
confirmatory testing. Even in patients with leakage of cerebrospinal fluid (CSF), many surgeons
will allow up to 10 days for the CSF leak to resolve on its own before resorting to cranialization,
as long as all fractures are undisplaced and the nasofrontal duct is patent. In addition, cases of
posterior wall fracture with obstructed nasofrontal ducts can be addressed with sinus obliteration
and not cranialization, as long as there is no brain injury and little or no comminution of the
posterior wall. If there are other potential sources of a CSF leak, sorting out the cause can be
done preoperatively with a metrizamide CT scan. The cranialization procedure consists of
obliterating the nasofrontal duct and removing the sinus mucosa and posterior wall of the sinus,
effectively incorporating the sinus space into the cranial cavity. This prevents future meningitis
and brain abscess by sterilizing this connection.

Management of an anterior table fracture with an intact posterior table and an intact frontonasal
duct should be addressed by plating the anterior table to restore forehead contour.

If the nasofrontal duct is obstructed, then obliteration of the frontal sinus is indicated.

When the sinus floor is fractured medial to the supraorbital foramen, there is a good chance that
the nasofrontal duct is injured, in which case sinus obliteration is indicated. If it is not injured,
then plating is indicated to restore the orbital roof if fracture lines are displaced.

Post-traumatic mucoceles of the frontal sinus are managed by sinus obliteration. The exception
is, however, that cranialization could be indicated in a few instances where the mucocele has
become extremely expanded and destructive on the posterior table.

This examination contains test materials that are owned and copyrighted by the American Society of
Plastic Surgeons and the Plastic Surgery Educational Foundation. Any reproduction of these materials or
any part of them, through any means, including but not limited to, copying or printing electronic files,
reconstruction through memorization or dictation, and/or dissemination of these materials or any part of
them is strictly prohibited. Keep printed materials in a secure location when you are not reviewing them
and discard them in a secure manner, such as shredding, when you have completed the examination.

Page 204 of 287


American Society of Plastic Surgeons and the Plastic Surgery Educational Foundation
In-Service Examination

References

1. Gonty AA. Management of frontal sinus fractures and associated injuries. In: Fonseca
RJ, ed. Oral and Maxillofacial Surgery. Philadelphia: WB Saunders; 2000:301-326.
2. Lettieri S. Facial trauma. In: Vander Kolk CA, ed. Plastic Surgery: Indications,
Operations, and Outcomes. St. Louis: Mosby; 2000:923-940.

This examination contains test materials that are owned and copyrighted by the American Society of
Plastic Surgeons and the Plastic Surgery Educational Foundation. Any reproduction of these materials or
any part of them, through any means, including but not limited to, copying or printing electronic files,
reconstruction through memorization or dictation, and/or dissemination of these materials or any part of
them is strictly prohibited. Keep printed materials in a secure location when you are not reviewing them
and discard them in a secure manner, such as shredding, when you have completed the examination.

Page 205 of 287


American Society of Plastic Surgeons and the Plastic Surgery Educational Foundation
In-Service Examination

150. Which of the following muscles of the mouth is innervated by the mandibular
branch of the facial (VII) nerve?
A) Buccinator
B) Depressor anguli oris
C) Levator anguli oris
D) Orbicularis oris
E) Risorius

The correct response is Option B.

The other mimetic muscles innervated by the mandibular branch of the facial nerve include the
depressor labii inferioris and the mentalis muscle. These muscles control movement of the lower
lip to either pull it down and laterally or to protrude it. Paralysis of the nerve results in elevation
of the angle of the mouth and of the lower lip on the affected side. This nerve is at risk for injury
during rhytidectomy or neck lift procedures, parotidectomy, and neck dissection.

The buccinator, levator anguli oris, orbicularis oris, and risorius are all innervated by the buccal
branch of the facial nerve. Anatomic variation may be encountered with the risorius muscle,
which is rarely innervated by the mandibular branch.

References

1. Clemente CD. Neck and head. In: Clemente CD, ed. Anatomy: A Regional Atlas of
the Human Body. 4th ed. Baltimore: Williams & Wilkins; 1997:435-576.
2. Williams PL, Warwick R, Dyson M, et al. The cranial nerves. In: Williams PL,
Warwick R, Dyson M, et al. Gray's Anatomy. 37th ed. Edinburgh: Churchill
Livingstone; 1989:1094-1120.

This examination contains test materials that are owned and copyrighted by the American Society of
Plastic Surgeons and the Plastic Surgery Educational Foundation. Any reproduction of these materials or
any part of them, through any means, including but not limited to, copying or printing electronic files,
reconstruction through memorization or dictation, and/or dissemination of these materials or any part of
them is strictly prohibited. Keep printed materials in a secure location when you are not reviewing them
and discard them in a secure manner, such as shredding, when you have completed the examination.

Page 206 of 287


American Society of Plastic Surgeons and the Plastic Surgery Educational Foundation
In-Service Examination

Section 4: Hand

151. Hematologic evaluation is indicated before surgical intervention for which of the
following conditions?
A) Atypical cleft hand
B) Constriction band syndrome
C) Radial hypoplasia
D) Small finger polydactyly
E) Thumb duplication

The correct response is Option C.

Radial aplasia and hypoplasia are associated with several syndromes including VACTERL
(vertebral, anal, cardiac, tracheoesophageal, renal/radial, limb), Holt-Oram, TAR
(thrombocytopenia-absent radius), and Fanconi anemia. Both TAR syndrome and Fanconi
anemia are contraindications for early surgical intervention until the underlying hematologic
abnormalities have stabilized or been corrected.

The other congenital disorders listed tend to occur sporadically or familially and are not
associated with specific hematologic abnormalities.

References

1. Kozin SH. Upper-extremity congenital anomalies. J Bone Joint Surg Am.


2003;85:1564-1576.
2. Gluckman E. Allogeneic bone marrow transplantation in Fanconi anemia. Bone
Marrow Transplant. 1996;18:S33-S35.

This examination contains test materials that are owned and copyrighted by the American Society of
Plastic Surgeons and the Plastic Surgery Educational Foundation. Any reproduction of these materials or
any part of them, through any means, including but not limited to, copying or printing electronic files,
reconstruction through memorization or dictation, and/or dissemination of these materials or any part of
them is strictly prohibited. Keep printed materials in a secure location when you are not reviewing them
and discard them in a secure manner, such as shredding, when you have completed the examination.

Page 207 of 287


American Society of Plastic Surgeons and the Plastic Surgery Educational Foundation
In-Service Examination

(Please note that this pictorial appears in color in the online examination)

152. A 57-year-old man is referred to the office by his primary care physician for
management of contracture of the little finger (shown) that has been present for
more than five years. Flexion of the digit has been increasing during the past year.
Surgical management is planned via a transverse palmar approach with
longitudinal incisions based over the contracted cord and later converted to the
necessary V-Y advancement Z-plasties. Which of the following is the most likely
long-term outcome in this patient?
A) Anesthesia of the digit
B) Carpal tunnel syndrome
C) Complex regional pain syndrome type I
D) Cord recurrence
E) Vascular insufficiency

The correct response is Option D.

Complications of Dupuytren disease have been reported to be 17% overall. Nerve injuries can be
a devastating complication after surgery for Dupuytren disease. The digital nerves are displaced
medially by the spiral cord, making them more prone to injury. Fortunately, nerve transection is
relatively rare, reported in only 1.5% of patients with Dupuytren disease. Complex regional pain
syndrome (reflex sympathetic dystrophy) may occur after Dupuytren disease, but this is an
uncommon complication (approximately 4% to 8%). It can be confused with flare reaction,
which can intense vasodilation; however, there is no associated pain. Infection in patients with
Dupuytren disease is not significantly greater than other similar surgeries. Arterial injuries occur

This examination contains test materials that are owned and copyrighted by the American Society of
Plastic Surgeons and the Plastic Surgery Educational Foundation. Any reproduction of these materials or
any part of them, through any means, including but not limited to, copying or printing electronic files,
reconstruction through memorization or dictation, and/or dissemination of these materials or any part of
them is strictly prohibited. Keep printed materials in a secure location when you are not reviewing them
and discard them in a secure manner, such as shredding, when you have completed the examination.

Page 208 of 287


American Society of Plastic Surgeons and the Plastic Surgery Educational Foundation
In-Service Examination

in less than 1% of Dupuytren cases.

Recurrence after Dupuytren is significant; a recent study placed recurrence at 18% to 24%. Other
studies, with 10-year follow-up, have noted that no patients were free of disease. Patients should
have realistic expectations on the possible complications and recurrence risks prior to surgery.

References

1. Bulstrode NW, Jemec B, Smith PJ. The complications of Dupuytren’s contracture


surgery. J Hand Surg. 2005;30A:1021-1025.
2. Citron ND, Nunez V. Recurrence after surgery for Dupuytren’s disease: a randomized
trial of two skin incisions. J Hand Surg. 2005;30B:563-566.

This examination contains test materials that are owned and copyrighted by the American Society of
Plastic Surgeons and the Plastic Surgery Educational Foundation. Any reproduction of these materials or
any part of them, through any means, including but not limited to, copying or printing electronic files,
reconstruction through memorization or dictation, and/or dissemination of these materials or any part of
them is strictly prohibited. Keep printed materials in a secure location when you are not reviewing them
and discard them in a secure manner, such as shredding, when you have completed the examination.

Page 209 of 287


American Society of Plastic Surgeons and the Plastic Surgery Educational Foundation
In-Service Examination

153. A 17-year-old boy is referred to the office by his primary care physician for
consultation regarding lack of active flexion of the distal interphalangeal joint of the
ring finger of the dominant right hand. Six weeks ago, he sustained an injury to the
hand while practicing with his high school football team. Immediately after the
injury, the team's trainer initiated alternating application of hot and cold packs to
the hand and gave the patient a finger splint, which he wore intermittently for five
weeks. Current physical examination shows a tender mass at the distal
interphalangeal joint of the ring finger. A radiograph is shown. Which of the
following is the most appropriate next step?
A) Fusion of the distal interphalangeal joint
B) Fusion of the proximal interphalangeal joint
C) Reduction and fixation
D) Tendon repair with single-stage tendon grafting
E) Two-stage tendon reconstruction with implantation of a silicone rod

The correct response is Option C.

The patient described has an avulsion of the flexor digitorum profundus (FDP) tendon from the
distal phalanx, otherwise known as a jersey finger. This injury most commonly involves the ring
finger. Leddy has classified these injuries into three types to direct treatment. In type 1 injuries,
the proximal FDP retracts to the palm. These injuries must be repaired within two weeks to avoid

This examination contains test materials that are owned and copyrighted by the American Society of
Plastic Surgeons and the Plastic Surgery Educational Foundation. Any reproduction of these materials or
any part of them, through any means, including but not limited to, copying or printing electronic files,
reconstruction through memorization or dictation, and/or dissemination of these materials or any part of
them is strictly prohibited. Keep printed materials in a secure location when you are not reviewing them
and discard them in a secure manner, such as shredding, when you have completed the examination.

Page 210 of 287


American Society of Plastic Surgeons and the Plastic Surgery Educational Foundation
In-Service Examination

a tendon graft. In Type 2 injuries, the tendon retracts to the level of the proximal interphalangeal
(PIP), where it is usually maintained by a distal phalanx bone fragment by the A3 pulley. Type 2
injuries can usually be repaired without the need for a graft if treated within three months.
However, over time, type 2 injuries can convert to type 1 injuries with proximal migration of the
tendon to the palm. In Type 3 injuries, the bone fragment prevents tendon retraction proximal to
the A4 pulley. Type 3 injuries can usually be repaired without a tendon graft, at any time, even
after three months. Both type 2 and 3 injuries can be confirmed by radiography, which will
demonstrate the small bone piece in the finger. Critical to the outcome of profundus avulsion
injuries is early identification of the injury.

References

1. Leddy JP. Avulsions of the flexor digitorum profundus. Hand Clin. 1985;1:77.
2. Strickland JW. Flexor tendons: acute injuries. In: Green's Operative Hand Surgery.
Philadelphia: Churchill Livingstone; 1999:432.

This examination contains test materials that are owned and copyrighted by the American Society of
Plastic Surgeons and the Plastic Surgery Educational Foundation. Any reproduction of these materials or
any part of them, through any means, including but not limited to, copying or printing electronic files,
reconstruction through memorization or dictation, and/or dissemination of these materials or any part of
them is strictly prohibited. Keep printed materials in a secure location when you are not reviewing them
and discard them in a secure manner, such as shredding, when you have completed the examination.

Page 211 of 287


American Society of Plastic Surgeons and the Plastic Surgery Educational Foundation
In-Service Examination

154. A 23-year-old man comes to the emergency department immediately after he


sustained an injury to the tip and nail bed of the index finger of the right hand.
Physical examination shows avulsion of the nail bed and nail plate. Ninety percent
of the sterile matrix is missing and cannot be located on the underside of the nail
plate. Which of the following is the most appropriate intervention for repair of the
missing nail bed?
A) Application of regenerative dermal matrix (Alloderm)
B) Grafting of the sterile matrix from the great toe
C) Grafting of the sterile matrix from the injured nail bed
D) Healing by secondary intention
E) Skin grafting from the hypothenar area

The correct response is Option B.

If the area of loss of sterile matrix is significant, grafting with sterile matrix provides the best
outcome. More than half of the sterile matrix is missing from the injured nail bed of the patient
described, which is not enough to provide for coverage of the defect. Therefore, use of sterile
matrix from the great toe as the donor site is the most appropriate intervention in the scenario
described.

Regenerative dermal matrix is not sterile matrix and application of it would not reconstruct either
the germinal or sterile matrix.

Allowing the nail bed injury of the patient described to heal by secondary intention would result
in nail nonadherence and risk bony infection.

The sterile matrix should not be reconstructed with a standard skin graft because this would also
result in nail nonadherence and risk frequent infection.

References

1. Zook EG, Brown RE. The perionychium hand. In: Green’s Operative Hand Surgery.
Philadelphia: Churchill Livingstone; 1999:432.
2. Zook EG. Reconstruction of the functional and aesthetic nail. Hand Clin.
2002;18:577-594.

This examination contains test materials that are owned and copyrighted by the American Society of
Plastic Surgeons and the Plastic Surgery Educational Foundation. Any reproduction of these materials or
any part of them, through any means, including but not limited to, copying or printing electronic files,
reconstruction through memorization or dictation, and/or dissemination of these materials or any part of
them is strictly prohibited. Keep printed materials in a secure location when you are not reviewing them
and discard them in a secure manner, such as shredding, when you have completed the examination.

Page 212 of 287


American Society of Plastic Surgeons and the Plastic Surgery Educational Foundation
In-Service Examination

155. A 35-year-old woman who underwent above-the-knee amputation of the left leg for
soft-tissue sarcoma 20 years ago comes to the office because she has had
constant stabbing pain in the left thigh for the past year. Medical history includes
weight loss of 50 lb through diet and exercise over the past two years. Physical
examination shows a 3 × 2-cm mass in the posterior aspect of the left thigh, just
proximal to the amputation scar. Extreme tenderness is noted on palpation of the
mass. Which of the following diagnostic studies is most appropriate?
A) Electromyography
B) Local anesthetic nerve block
C) Magnetic resonance angiography
D) Positron emission tomography
E) Three-phase bone scanning

The correct response is Option B.

The history and physical examination described is most consistent with a neuroma in a lower
limb above-the-knee amputation. After a nerve is cut, nerve cell bodies start to adapt, sending out
branches to the distal nerve in an attempt to reach the distal end organs. An irregular mass of
tissue, clinically known as a neuroma, forms at the end of the proximal stump. If the neuroma is
superficial, painful symptoms can result. The Tinel sign, which is provoked pain on percussion
of the nerve, is useful for identifying a neuroma. Diagnosis of the neuroma (shown) can be
achieved by performing a diagnostic nerve block. The nerve proximal to the neuroma is blocked
with an anesthetic solution. If the pain resolves, a definitive diagnosis can often be made.

Electromyography is of little help in diagnosing neuroma, which is usually a clinical diagnosis.


There is no clinical indication for magnetic resonance angiography or positron emission
tomography. This scenario does not point to arterial disease or recurrent tumor. A three-phase
bone scan is sometimes used but does not have adequate predictive value.

This examination contains test materials that are owned and copyrighted by the American Society of
Plastic Surgeons and the Plastic Surgery Educational Foundation. Any reproduction of these materials or
any part of them, through any means, including but not limited to, copying or printing electronic files,
reconstruction through memorization or dictation, and/or dissemination of these materials or any part of
them is strictly prohibited. Keep printed materials in a secure location when you are not reviewing them
and discard them in a secure manner, such as shredding, when you have completed the examination.

Page 213 of 287


American Society of Plastic Surgeons and the Plastic Surgery Educational Foundation
In-Service Examination

References

1. Zimmermann M. Pathobiology of neuropathic pain. Eur J Pharmacol.


2001;429:23-37.
2. Ernberg LA, Adler RS, Lane J. Ultrasound in the detection and treatment of a painful
stump neuroma. Skeletal Radiol. 2003;32:306-309.

This examination contains test materials that are owned and copyrighted by the American Society of
Plastic Surgeons and the Plastic Surgery Educational Foundation. Any reproduction of these materials or
any part of them, through any means, including but not limited to, copying or printing electronic files,
reconstruction through memorization or dictation, and/or dissemination of these materials or any part of
them is strictly prohibited. Keep printed materials in a secure location when you are not reviewing them
and discard them in a secure manner, such as shredding, when you have completed the examination.

Page 214 of 287


American Society of Plastic Surgeons and the Plastic Surgery Educational Foundation
In-Service Examination

156. A 25-year-old firefighter is transferred to the burn unit for management of burns to
the hands 10 days after he sustained burns to 85% of the total body surface area
(TBSA). His condition is stable, and vital signs are within normal limits. He has
undergone multiple excision and grafting procedures. Current physical
examination shows large areas of exposed tendon over the dorsum of both hands
after excision of the burned skin. Partial-thickness burns of adjacent tissue are
noted. Because of the extent of this patient’s TBSA burns, regional and free tissue
flaps are precluded. The most appropriate intervention for wound coverage is
application of which of the following?
A) Allograft skin graft
B) Dermal regeneration template (Integra)
C) Expansion of adjacent tissue
D) Porcine xenografts
E) Temporary biosynthetic skin substitute (Biobrane)

The correct response is Option B.

The patient described requires wound coverage with dermal regeneration template (Integra).
Exposure of the extensor tendons precludes the use of skin grafts and no local flaps are available
in this patient’s burned hands because of adjacent scarring. Allograft skin would have similar
problems of adherence to the exposed tendons and would require overly epidermal autografts.
Regional and free flaps are also unavailable.

The use of tissue expansion to increase the size of full-thickness grafts and to improve the
availability of local flaps while decreasing donor site morbidity has been suggested. The patient
described is not a candidate for this procedure because of the local partial-thickness burns. Even
if the local tissue was uninjured and could be expanded, the presence of exposed tendons does
not allow for the protracted time associated with expander reconstruction. Expanders in the upper
extremity have also been associated with a high rate of complications such as infection and
extrusion.

Temporary biosynthetic skin substitute (Biobrane) consists of a collagen gel dermal analog and a
Silastic silicone rubber epidermal analog. It is indistinguishable from porcine xenograft in terms
of pliability, wound adherence, and formation of granulation tissue. Both Biobrane and porcine
xenograft may be used for coverage of partial-thickness burns. Both allow wound re-
epithelialization, at which point the Biobrane or allograft can be peeled off.

Bilaminate skin substitutes such as Integra (collagen-glycosaminoglycan dermis/Silastic

This examination contains test materials that are owned and copyrighted by the American Society of
Plastic Surgeons and the Plastic Surgery Educational Foundation. Any reproduction of these materials or
any part of them, through any means, including but not limited to, copying or printing electronic files,
reconstruction through memorization or dictation, and/or dissemination of these materials or any part of
them is strictly prohibited. Keep printed materials in a secure location when you are not reviewing them
and discard them in a secure manner, such as shredding, when you have completed the examination.

Page 215 of 287


American Society of Plastic Surgeons and the Plastic Surgery Educational Foundation
In-Service Examination

epidermis) and Alloderm regenerative tissue matrix (autologous-allogeneic dermis) plus a split-
thickness skin graft emulate the desirable properties of dermis and epidermis, particularly with
respect to pore size. The outer membrane requires a small pore size to act as a barrier to
microorganisms while allowing water vapor permeability. The inner layer requires a larger pore
size to permit ingrowth of granulation tissue from the wound bed. Use of Integra and Alloderm
for full-thickness wound coverage has been associated with less scarring than with conventional
skin grafts. This decreased scarring may in turn minimize secondary contracture and improve
outcome, particularly in the hand. Both require thin epidermal grafts for coverage, though only
Alloderm is able to support immediate skin grafting.

References

1. McCauley RL, Asuku ME. Upper extremity burn reconstruction. In: Mathes SJ, ed.
Plastic Surgery. Philadelphia: WB Saunders; 2006:605-645.
2. Dantzer E, Braye FM. Reconstructive surgery using an artificial dermis (Integra):
results with 39 grafts. Br J Plast Surg. 2001;54:659-664.
3. Trong-Duo C, Shao-Ling C, Tz-Wen L, et al. Reconstruction of the upper extremities
with artificial skin. Plast Reconstr Surg. 2001;108:378-384.

This examination contains test materials that are owned and copyrighted by the American Society of
Plastic Surgeons and the Plastic Surgery Educational Foundation. Any reproduction of these materials or
any part of them, through any means, including but not limited to, copying or printing electronic files,
reconstruction through memorization or dictation, and/or dissemination of these materials or any part of
them is strictly prohibited. Keep printed materials in a secure location when you are not reviewing them
and discard them in a secure manner, such as shredding, when you have completed the examination.

Page 216 of 287


American Society of Plastic Surgeons and the Plastic Surgery Educational Foundation
In-Service Examination

157. A 35-year-old man comes to the emergency department 30 minutes after he


sustained a knife wound to the right hand during an altercation. Medical history
includes end-stage renal disease, for which the patient undergoes dialysis through
an arteriovenous fistula in the right upper arm. Physical examination of the hand
shows Zone II lacerations of the volar aspect of the index, long, ring, and small
fingers. Both tendons and digital nerves are lacerated. The estimated duration of
surgical repair is three hours. Which of the following is the most appropriate use of
a tourniquet during this procedure?
A) Apply the tourniquet to the right distal forearm, and use reperfusion breaks every
90 minutes
B) Apply the tourniquet to the right upper arm for the first two hours only
C) Cool the right extremity for 30 minutes, then apply the tourniquet to the upper arm for
as long as needed
D) Do not use a tourniquet because of the possibility of causing thrombosis of the
arteriovenous shunt
E) Urgently schedule creation of an arteriovenous shunt in the left arm by a vascular
surgeon and perform the procedure on the right hand with use of an upper arm
tourniquet with reperfusion breaks every 60 minutes

The correct response is Option A.

The tourniquet provides the hand surgeon with a bloodless field, which is essential to perform
the exacting technique required in hand surgery. Any time a tourniquet is applied proximal to an
arteriovenous shunt for dialysis, there is a chance that the shunt may clot while the tourniquet is
inflated. In a review of 14 dialysis patients on whom upper arm tourniquets were used, 2 of 14
patients experienced shunt thrombosis and required fistula repair. The hand surgeon can
minimize this possibility by applying the tourniquet distal to the shunt rather than on the upper
arm. A forearm tourniquet can be used safely and effectively. The recommended inflation
pressure is 75 to 100 mmHg above systolic pressure. Higher pressures will increase the incidence
of complications such as nerve compression injuries. There is no universally accepted safe
tourniquet time; however, two hours is the generally accepted time limit. When tourniquet times
are longer than two hours, most surgeons will deflate the tourniquet for reperfusion breaks. The
reperfusion time is related to the duration of inflation (see the table below, adapted from
reference 2). The reperfusion times in the table are based on the time required for venous pH to
return to normal after deflation of the tourniquet.

This examination contains test materials that are owned and copyrighted by the American Society of
Plastic Surgeons and the Plastic Surgery Educational Foundation. Any reproduction of these materials or
any part of them, through any means, including but not limited to, copying or printing electronic files,
reconstruction through memorization or dictation, and/or dissemination of these materials or any part of
them is strictly prohibited. Keep printed materials in a secure location when you are not reviewing them
and discard them in a secure manner, such as shredding, when you have completed the examination.

Page 217 of 287


American Society of Plastic Surgeons and the Plastic Surgery Educational Foundation
In-Service Examination

Duration of Tourniquet Inflation Recommended Reperfusion Times Before Reinflation


(min) (min)
30 3–5
60 5–10
90 10–15
120 15–20

Applying the tourniquet to the right upper arm for the first two hours only is not appropriate for
the procedure described because the tourniquet is applied proximal to the shunt, and then the
operation is completed without the tourniquet, which sacrifices the bloodless field.

Cooling the right extremity and then applying the tourniquet to the upper arm is not appropriate
because the tourniquet is applied proximal to the shunt. This method also brings up the concept
of decreasing ischemic injury with hypothermia. Although this technique theoretically can
lengthen acceptable tourniquet times, it is cumbersome and more complex than reperfusion
breaks. Not using a tourniquet is not appropriate for the procedure described because it does not
provide the necessary bloodless field. Scheduling creation of a second shunt is unnecessary with
a forearm tourniquet.

References

1. Maury AC, Roy WS. A prospective, randomized, controlled trial of forearm versus
upper arm tourniquet tolerance. J Hand Surg [Br]. 2002;27:359-360.
2. Wilgis EFS. Observation on the effects of tourniquet ischemia. J Bone Joint Surg Am.
1971;53:1343-1346.
3. Green DP. General principles. In: Green DP, Hotchkiss RN, Pederson WC, et al, eds.
Operative Hand Surgery. 5th ed. Philadelphia: Elsevier; 2005:11-17.
4. Semer NB, Goldberg NH, Cuono CB. Upper extremity entrapment neuropathy and
tourniquet use with inpatients undergoing hemodialysis. J Hand Surg [Am].
1989;14:897-900.

This examination contains test materials that are owned and copyrighted by the American Society of
Plastic Surgeons and the Plastic Surgery Educational Foundation. Any reproduction of these materials or
any part of them, through any means, including but not limited to, copying or printing electronic files,
reconstruction through memorization or dictation, and/or dissemination of these materials or any part of
them is strictly prohibited. Keep printed materials in a secure location when you are not reviewing them
and discard them in a secure manner, such as shredding, when you have completed the examination.

Page 218 of 287


American Society of Plastic Surgeons and the Plastic Surgery Educational Foundation
In-Service Examination

158. A 62-year-old woman with symptomatic carpometacarpal arthritis of the thumb is


scheduled to undergo resection of the trapezium for relief of pain and swelling.
Which of the following additional steps in management of this patient's condition is
critical to the successful outcome of the surgical procedure?
A) Fluoroscopic assessment of the trapezial resection
B) Postoperative immobilization of the thumb in a spica splint
C) Reconstruction of the suspending ligament
D) Resection of the trapezial space
E) Temporary fixation of the metacarpal base with Kirschner wires

The correct response is Option B.

Osteoarthritis of the carpometacarpal joint of the thumb is frequently seen by hand surgeons and
often requires surgical reconstruction. Classical teaching by Eaton and Littler supports the
technique of ligament reconstruction and tendon interposition arthroplasty. Reported results
using this technique are very good, with more than 80% relief of symptoms. Several authors
have recently reported that trapezium excision and cast immobilization may be just as effective
in relieving symptoms as the more extensive placement of a soft-tissue spacer and ligament
reconstruction. Although the other management options are effective, they are not as essential to
successful outcome as the simple act of cast or splint immobilization for four to six weeks in the
postoperative period.

References

1. Manske PR. Commentary: excision of the trapezium. J Hand Surg [Am].


2004;29:1078-1079.
2. Davis TR, Brady O, Dias JJ. Excision of the trapezium for osteoarthritis of the
trapeziometacarpal joint: a study of the benefit of ligament reconstruction or tendon
interposition. J Hand Surg [Am]. 2004;29:1069-1077.
3. Gervis WH. Excision of the trapezium for osteoarthritis of the trapezio-metacarpal
joint. J Bone Joint Surg. 1949;31B:537-539.
4. Eaton RG, Littler JW. Ligament reconstruction for the painful carpometacarpal joint.
J Bone Joint Surg. 1973;55A:1655-1666.
5. Froimson AI. Tendon arthroplasty of the trapeziometacarpal joint. Clin Orthop.
1970;70:191-199.
6. Dell PC, Muniz RB. Interposition arthroplasty of the trapeziometacarpal joint for
osteoarthritis. Clin Orthop. 1987;220:27-34.

This examination contains test materials that are owned and copyrighted by the American Society of
Plastic Surgeons and the Plastic Surgery Educational Foundation. Any reproduction of these materials or
any part of them, through any means, including but not limited to, copying or printing electronic files,
reconstruction through memorization or dictation, and/or dissemination of these materials or any part of
them is strictly prohibited. Keep printed materials in a secure location when you are not reviewing them
and discard them in a secure manner, such as shredding, when you have completed the examination.

Page 219 of 287


American Society of Plastic Surgeons and the Plastic Surgery Educational Foundation
In-Service Examination

159. A 28-year-old man is brought to the emergency department 30 minutes after he


sustained avulsion injuries to the nondominant left hand when it became caught in
a motor vehicle fanbelt. Physical examination shows amputation of the index
finger at the level of the proximal interphalangeal joint as well as a 2 x 1-cm area
of soft-tissue loss. Replantation of the amputated digit is performed, and the
resulting 2 x 1-cm soft-tissue avulsion volar defect is covered with an arterialized
venous flow-through flap with overlying skin interposed as a vein graft in the
arterial repair. Which of the following is the most likely early complication of this
flap procedure?
A) Arterial thrombosis
B) Congestion of the flap
C) Failure of the replantation
D) Hematoma from vessel leak
E) Loss of flap due to infection

The correct response is Option B.

Venous flow-through flaps (VFTFs) are unusual but are gaining acceptance for certain kinds of
hand and finger wounds. The ideal site for coverage with a VFTF is a long and narrow defect
needing thin soft tissue. VFTFs typically become congested in the first week and then decongest
over the following two weeks as they revascularize from the wound bed. VFTFs cannot reliably
transfer composite tissue such as bone and tendon or cover a wide defect such as an entire palm.
Because VFTFs do not bring in vascularization to the wound bed as well as classic flaps, they are
not indicated in radiated or potentially infected wound beds. A small defect such as the 2-cm
defect needing coverage during the replantation of the finger in the scenario described is the
ideal candidate for this flap.

When compared with simple vein grafts, VFTFs are not associated with increased rates of
arterial thrombosis, failure of replantation, or hematoma. Although VFTFs are more susceptible
to infection than typical flaps, congestion of the flap with superior epidermolysis is a much more
likely complication.

References

1. Brooks D. Invited Discussion: arterialized venous free flaps. J Reconstr Microsurg.


2002;18:575-577.

This examination contains test materials that are owned and copyrighted by the American Society of
Plastic Surgeons and the Plastic Surgery Educational Foundation. Any reproduction of these materials or
any part of them, through any means, including but not limited to, copying or printing electronic files,
reconstruction through memorization or dictation, and/or dissemination of these materials or any part of
them is strictly prohibited. Keep printed materials in a secure location when you are not reviewing them
and discard them in a secure manner, such as shredding, when you have completed the examination.

Page 220 of 287


American Society of Plastic Surgeons and the Plastic Surgery Educational Foundation
In-Service Examination

2. De Lorenzi F, van der Hulst RR, den Dunnen WF, et al. Arterialized venous free flaps
for soft-tissue reconstruction of digits: a 40-case series. J Reconstr Microsurg.
2002;18:569-574.
3. Titley OG, Chester DL, Park AJ. A-a type, arterialized, venous, flow-through, free
flap for simultaneous digital revascularization and soft tissue reconstruction-revisited.
Ann Plast Surg. 2004;53:185-191.
4. Rose EH. Small flap coverage of hand and digit defects. Clin Plast Surg.
1989;16:427-442.

This examination contains test materials that are owned and copyrighted by the American Society of
Plastic Surgeons and the Plastic Surgery Educational Foundation. Any reproduction of these materials or
any part of them, through any means, including but not limited to, copying or printing electronic files,
reconstruction through memorization or dictation, and/or dissemination of these materials or any part of
them is strictly prohibited. Keep printed materials in a secure location when you are not reviewing them
and discard them in a secure manner, such as shredding, when you have completed the examination.

Page 221 of 287


American Society of Plastic Surgeons and the Plastic Surgery Educational Foundation
In-Service Examination

160. A 55-year-old man with well-controlled type 1 diabetes mellitus and a history of
kidney transplantation comes to the clinic because he has had increasing pain,
swelling, and redness of the right index finger over the past four days. Five days
ago, he sustained an injury to the finger when a thorn became lodged under the
skin while he was gardening. Medical records show that hemoglobin A1c was 12%
three weeks ago. Temperature is 37.9°C (100.2°F). On laboratory studies, white
blood cell count is 16,000/mm3 and serum glucose level is 495 mg/dL.
Examination of the right hand shows severe flexor tenosynovitis of the index
finger. Which of the following factors in this patient increases his risk of
amputation of the digit?
A) Hemoglobin A1c greater than 10%
B) History of kidney transplantation
C) Insulin dependence
D) Serum glucose level greater than 450 mg/dL
E) White blood cell count greater than 14,000/mm3

The correct response is Option B.

History of renal failure or kidney transplantation is associated with the highest risk of amputation
in diabetic patients with hand infections. Amputation rates in this population range from 75% to
100%.

Multiple studies have shown that diabetes mellitus negatively impacts the prognosis of hand
infections. Hand infections in patients with diabetes take longer to resolve and are more likely to
require multiple debridements than infections in patients without diabetes. In addition, initiation
of treatment is more likely to be delayed in patients with diabetes. In 50% of cases, the initial
surgical procedure does not control the infection adequately. Amputation rates for hand
infections in patients with diabetes range from 7% to 63%. In up to 28% of patients undergoing
amputation, a repeat procedure at a more proximal level is required.

Insulin dependence, history of poorly controlled diabetes (as indicated by elevated hemoglobin
A1c), white blood cell count greater than 14,000/mm3, and serum glucose level greater than 450
mg/dL have not been shown to significantly increase the risk of amputation in diabetic patients
with hand infections.

This examination contains test materials that are owned and copyrighted by the American Society of
Plastic Surgeons and the Plastic Surgery Educational Foundation. Any reproduction of these materials or
any part of them, through any means, including but not limited to, copying or printing electronic files,
reconstruction through memorization or dictation, and/or dissemination of these materials or any part of
them is strictly prohibited. Keep printed materials in a secure location when you are not reviewing them
and discard them in a secure manner, such as shredding, when you have completed the examination.

Page 222 of 287


American Society of Plastic Surgeons and the Plastic Surgery Educational Foundation
In-Service Examination

References

1. Connor RW, Kimbrough RC, Dabezies EJ. Hand infections in patients with diabetes
mellitus. Orthopedics. 2001;24:1057.
2. Gonzalez MH, Bochar S, Novotny J, et al. Upper extremity infections in patients with
diabetes mellitus. J Hand Surg [Am]. 1999;24A:682.

This examination contains test materials that are owned and copyrighted by the American Society of
Plastic Surgeons and the Plastic Surgery Educational Foundation. Any reproduction of these materials or
any part of them, through any means, including but not limited to, copying or printing electronic files,
reconstruction through memorization or dictation, and/or dissemination of these materials or any part of
them is strictly prohibited. Keep printed materials in a secure location when you are not reviewing them
and discard them in a secure manner, such as shredding, when you have completed the examination.

Page 223 of 287


American Society of Plastic Surgeons and the Plastic Surgery Educational Foundation
In-Service Examination

(Please note that this pictorial appears in color in the online examination)

161. A 73-year-old man is referred to the office by his primary care physician for
evaluation of discoloration of the nail of the left thumb (shown), which has been
present for the past seven years. The patient says the appearance of the nail has
not changed recently. Biopsy of the nail matrix shows malignant melanoma of
indeterminate depth. Which of the following surgical procedures is the most
appropriate management?
A) Elective lymph node dissection and amputation at the metacarpophalangeal joint
B) Elective lymph node dissection and nail ablation
C) Sentinel node biopsy and amputation at the level of the interphalangeal joint
D) Sentinel node biopsy and excision of skin with 1-cm margins
E) Sentinel node biopsy and ray amputation of the carpometacarpal joint

The correct response is Option C.

The annual incidence of melanoma has increased significantly from a lifetime risk of 1:1500 in
1900 to the current risk of 1:35. Nail apparatus melanomas, and specifically subungual
melanomas, pose a difficult problem because they are often diagnosed late. Late diagnosis
correlates with thicker melanomas and greater risk of metastasis. Approximately 20% to 25% of
subungual melanomas may be amelanotic. Sentinel node biopsy has become the standard of care,
and since its advent, elective lymph node dissection is no longer recommended.

Because of the proximity of the nail matrix to the periosteum and bone, adequate resection
margins are not achieved with standard wide excision of 1 to 2 cm, and recommendation for

This examination contains test materials that are owned and copyrighted by the American Society of
Plastic Surgeons and the Plastic Surgery Educational Foundation. Any reproduction of these materials or
any part of them, through any means, including but not limited to, copying or printing electronic files,
reconstruction through memorization or dictation, and/or dissemination of these materials or any part of
them is strictly prohibited. Keep printed materials in a secure location when you are not reviewing them
and discard them in a secure manner, such as shredding, when you have completed the examination.

Page 224 of 287


American Society of Plastic Surgeons and the Plastic Surgery Educational Foundation
In-Service Examination

excision is at the distal interphalangeal joint of the finger or interphalangeal joint of the thumb.
Melanoma can track along the neurovascular bundles. Amputation at a more proximal level is
not needed and does not improve prognosis. More proximal amputation also results in a critical
loss of function, particularly concerning the thumb.

The presence of pigment in the paronychial area (Hutchinson sign) is shown in the photograph.
An area of pigmentation in the eponychium is almost pathognomonic for subungual malignant
melanoma.

References

1. O’Leary JA, Berend KR, Johnson JL, et al. Subungual melanoma. Clin Orthop Rel
Res. 2000;378:206-212.
2. Hove LM, Akslen LA. Clinicopathological characteristics of melanomas of the hand.
J Hand Surg. 1999;24B:460-464.
3. Finley RK, Driscoll DL, Blumenson LE, et al. Subungual melanoma: an eighteen-
year review. Surgery. 1994;116:96-100.
4. Dawber RP, Colver GB. The spectrum of malignant melanoma of the nail apparatus.
Semin Dermatol. 1991;10:82-87.
5. Saida T. Heterogeneity of the site of origin of malignant melanoma in ungual areas:
"subungual" malignant melanoma may be a misnomer. Br J Dermatol. 1992;126:529.
6. Saida T, Ohshima Y. Clinical and histopathologic characteristics of early lesions of
subungual malignant melanoma. Cancer. 1989;63:556-560.
7. Pack GT, Oropeza R. Subungual melanoma. Surg Gynecol Obstet. 1967;124:571-582.
8. Heaton KM, El-Naggar A, Ensign LG, et al. Surgical management and prognostic
factors in patients with subungual melanoma. Ann Surg. 1994;219:197-204.
9. Patterson RH, Helwig EB. Subungual malignant melanoma: a clinical-pathological
study. Cancer. 1980;46:2074-2087.
10. Kopf AW, Bart RS, Rodriguez-Sains RS. Malignant melanoma: a review. J Dermatol
Surg Oncol. 1977;3:41-125.
11. Banfield CC, Redburn JC, Dawber RP. The incidence and prognosis of nail apparatus
melanoma: a retrospective study of 105 patients in four English regions. Br J
Dermatol. 1998;139:276-279.

This examination contains test materials that are owned and copyrighted by the American Society of
Plastic Surgeons and the Plastic Surgery Educational Foundation. Any reproduction of these materials or
any part of them, through any means, including but not limited to, copying or printing electronic files,
reconstruction through memorization or dictation, and/or dissemination of these materials or any part of
them is strictly prohibited. Keep printed materials in a secure location when you are not reviewing them
and discard them in a secure manner, such as shredding, when you have completed the examination.

Page 225 of 287


American Society of Plastic Surgeons and the Plastic Surgery Educational Foundation
In-Service Examination

162. A 28-year-old professional baseball player comes to the emergency department


one hour after he sustained injuries to the index finger of the dominant right hand
during a game. A radiograph is shown. Which of the following is the most
appropriate management?
A) Buddy taping the index and long fingers for three weeks
B) Closed reduction, splinting, and repeat radiography at four weeks
C) Extension block splinting with early protected motion
D) Open reduction with rigid screw fixation with early protected motion
E) Skeletal dynamic traction splinting for three weeks

This examination contains test materials that are owned and copyrighted by the American Society of
Plastic Surgeons and the Plastic Surgery Educational Foundation. Any reproduction of these materials or
any part of them, through any means, including but not limited to, copying or printing electronic files,
reconstruction through memorization or dictation, and/or dissemination of these materials or any part of
them is strictly prohibited. Keep printed materials in a secure location when you are not reviewing them
and discard them in a secure manner, such as shredding, when you have completed the examination.

Page 226 of 287


American Society of Plastic Surgeons and the Plastic Surgery Educational Foundation
In-Service Examination

The correct response is Option D.

Condylar fractures of the proximal phalanx are inherently unstable. The patient described has a
displaced condylar fracture. The standard of care is open reduction and internal fixation with
either screws or Kirschner wire. Screw fixation allows for early active range of motion.
Percutaneous pinning or fixation with cannulated screws under C-arm control is a reasonable
approach; however, it can be more difficult to obtain reduction using these techniques.

Nondisplaced condylar fractures can be treated with splinting alone. However, displacement is
likely and radiographs must be taken frequently to monitor for displacement. Similarly, closed
reduction can be performed, but frequent follow-up radiographs are needed.

Buddy taping is not appropriate because the intrinsic instability of condylar fractures of the
proximal phalanx is ignored. A displacement of 2 mm also requires open reduction and internal
fixation. Skeletal dynamic traction splinting is appropriate for severely comminuted proximal
interphalangeal (PIP) joint fractures such as those seen with pylon-type injuries.

References

1. Stern PJ. Fractures of the metacarpals and phalanges. In: Green DP, Hotchkiss RN,
Pederson WC, et al, eds. Operative Hand Surgery. 5th ed. Philadelphia: Elsevier;
2005:302-330.
2. Weiss APC, Hastings H II. Distal unicondylar fractures of the proximal phalanx. J
Hand Surg [Am]. 1993;18:594-599.

This examination contains test materials that are owned and copyrighted by the American Society of
Plastic Surgeons and the Plastic Surgery Educational Foundation. Any reproduction of these materials or
any part of them, through any means, including but not limited to, copying or printing electronic files,
reconstruction through memorization or dictation, and/or dissemination of these materials or any part of
them is strictly prohibited. Keep printed materials in a secure location when you are not reviewing them
and discard them in a secure manner, such as shredding, when you have completed the examination.

Page 227 of 287


American Society of Plastic Surgeons and the Plastic Surgery Educational Foundation
In-Service Examination

163. A 64-year-old woman with a four-year history of symptomatic Dupuytren


contracture of the ring and small fingers of the nondominant left hand undergoes
palmar and digital fasciectomy with local infiltration of 1% Xylocaine and
1:100,000 epinephrine. Four hours after completion of the procedure, circulation to
the ring finger is inadequate. The most appropriate management at this time is
local injection of which of the following?
A) Hyaluronidase
B) 1% Lidocaine
C) Nitroglycerin solution
D) Papaverine
E) Phentolamine
The correct response is Option E.
Recent trials have demonstrated that it is unlikely that Xylocaine with 1:100,000 epinephrine can
cause irreversible finger ischemia. More likely causes of the finger ischemia in the patient
described are underlying vascular disease, iatrogenic injury, or splinting in extension placing too
much stretch on the foreshortened vessels. However, if it is believed that arterial inflow may be
compromised by the infiltration of epinephrine, then the treatment of choice is phentolamine
rescue by local infiltration at the site of vasoconstriction. Phentolamine in experimental studies
“consistently and reliably reverses adrenaline vasoconstriction in the finger in an average of 1
hour and 25 minutes.”

Phentolamine is an alpha-adrenergic blocking agent and, as such, is useful in cases of


epinephrine-induced vasospasm. It is of no benefit in cases of arterial ischemia from organic
stenosis, such as in atherosclerotic disease.

References

1. Lalonde D, Bell M, Benoit P, et al. A multicenter prospective study of 3,110


consecutive cases of elective epinephrine use in the fingers and hand: the Dalhousie
Project clinical phase. J Hand Surg [Am]. 2005;30:1061-1067.
2. Nodwell T, Lalonde D. How long does it take phentolamine to reverse
adrenaline-induced vasoconstriction in the finger and hand? A prospective
randomized blinded study the Dalhousie project experimental phase. Can J Plast
Surg. 2003;11:187-190.
3. Arneklo-Nobin B, Edvinsson L, Eklof B, et al. Analysis of vasospasm in hand arteries
by in vitro pharmacology, hand angiography and finger plethysmography. Gen
Pharmacol. 1983;14:65-67.

This examination contains test materials that are owned and copyrighted by the American Society of
Plastic Surgeons and the Plastic Surgery Educational Foundation. Any reproduction of these materials or
any part of them, through any means, including but not limited to, copying or printing electronic files,
reconstruction through memorization or dictation, and/or dissemination of these materials or any part of
them is strictly prohibited. Keep printed materials in a secure location when you are not reviewing them
and discard them in a secure manner, such as shredding, when you have completed the examination.

Page 228 of 287


American Society of Plastic Surgeons and the Plastic Surgery Educational Foundation
In-Service Examination

(Please note that this pictorial appears in color in the online examination)

164. A 20-year-old man comes to the office after he sustained transection of the ulnar
nerve of the left arm during a rollover motor vehicle accident. Physical examination
shows a 5-cm transverse laceration in the proximal ulnar aspect of his left forearm. A
photograph of the laceration after suture removal is shown. Microscope-assisted
epineural repair of the transection is performed. As function is restored over time,
which of the following movements will most likely be last to return?
A) Abduction of the small finger
B) Adduction of the thumb
C) Flexion of the metacarpophalangeal joint of the small finger
D) Flexion of the proximal interphalangeal joint of the small finger
E) Ulnar-sided flexion of the wrist

The correct response is Option B.

The ulnar nerve is the terminal portion of the medial cord of the brachial plexus, after the medial
head of the median nerve has separated from it, with fibers from C8–T1. It initially lies medial to
the axillary artery and then to the brachial artery as it travels distally in the upper arm. It pierces
the intermuscular septum and then follows the medial head of the triceps muscle to the groove
between the olecranon process and the medial epicondyle. It gives off no branches in the arm.
After the ulnar nerve passes through the cubital tunnel, it gives off articular branches and
branches to the flexor carpi ulnaris (FCU) and the medial half of the flexor digitorum profundus
(FDP). It travels between the two heads of the FCU and continues into the forearm between this
muscle and the FDP.

This examination contains test materials that are owned and copyrighted by the American Society of
Plastic Surgeons and the Plastic Surgery Educational Foundation. Any reproduction of these materials or
any part of them, through any means, including but not limited to, copying or printing electronic files,
reconstruction through memorization or dictation, and/or dissemination of these materials or any part of
them is strictly prohibited. Keep printed materials in a secure location when you are not reviewing them
and discard them in a secure manner, such as shredding, when you have completed the examination.

Page 229 of 287


American Society of Plastic Surgeons and the Plastic Surgery Educational Foundation
In-Service Examination

In the distal half of the forearm, it is joined on its lateral side by the ulnar artery. Proximal to the
wrist, the nerve gives off the dorsal sensory branch, providing sensation to the dorsal wrist and
ulnar aspect of the hand. The ulnar nerve continues into the hand via Guyon’s canal, where it
splits into a superficial sensory branch and a deep motor branch. The superficial branch supplies
the palmaris brevis and the skin of the hypothenar eminence and digital nerves to the small and
ulnar side of the ring finger. The motor branch passes between the abductor digiti minimi (ADM)
and the flexor digiti minimi (FDM), with the deep branch of the ulnar artery, perforates the
opponens digiti minimi and follows the deep volar arch across the interossei, and finally
innervates the adductor pollicis (AD) and the deep head of the flexor pollicis brevis.

Based on its path, ulnar-sided flexion of the wrist (FCU) would be expected early, followed by
abduction of the small finger (ADM) and flexion of the metacarpophalangeal joint (FDM).
Interosseus muscle function would manifest as ability to abduct and adduct the fingers, followed
last by adduction of the thumb (AD). Flexion of the proximal interphalangeal joints of the fingers
is a function of the flexor digitorum superficialis, which is innervated by the median nerve.

References

1. Netter FH. Atlas of Human Anatomy. 3rd ed. Teterboro, NJ: ICON Learning Systems;
2003:plates 416 and 459.
2. Chiu DTW. Nerve repairs in the upper limbs. In: Aston SJ, Beasley RW, Thorne
CHM, eds. Grabb and Smith's Plastic Surgery. 5th ed. Philadelphia: Lippincott-
Raven; 1997:889-893.

This examination contains test materials that are owned and copyrighted by the American Society of
Plastic Surgeons and the Plastic Surgery Educational Foundation. Any reproduction of these materials or
any part of them, through any means, including but not limited to, copying or printing electronic files,
reconstruction through memorization or dictation, and/or dissemination of these materials or any part of
them is strictly prohibited. Keep printed materials in a secure location when you are not reviewing them
and discard them in a secure manner, such as shredding, when you have completed the examination.

Page 230 of 287


American Society of Plastic Surgeons and the Plastic Surgery Educational Foundation
In-Service Examination

165. A 57-year-old man has a painless mass on the right wrist that has been enlarging
gradually over the past two years. Physical examination shows a 4-cm mass at
the wrist level deep to the flexor carpi ulnaris. The mass is smooth and firm, is
nonadherent to surrounding structures, and is not bony. Tinel sign is present at
the site of the mass, but no ulnar nerve sensory or motor deficits are noted. Plain-
film radiographs show no abnormalities. MRI shows a homogeneous mass within
the ulnar nerve. Which of the following is the most likely diagnosis?
A) Aneurysm of the ulnar artery
B) Enchondroma
C) Epidermal inclusion cyst
D) Giant cell tumor of the tendon sheath
E) Schwannoma

The correct response is Option E.

The most likely pathology of this mass is a schwannoma of the ulnar nerve. These benign nerve
tumors are typically painless proximal to the wrist. Schwannomas of the digits tend to be painful.
A Tinel sign can often be demonstrated. Nerve function typically is not disturbed. Because of the
size and location, MRI is effective in characterizing and localizing the mass. With magnification,
marginal excision of schwannomas is easily performed because they are almost self-extruding
from the nerve. Compared with neurofibromas, schwannomas are noninfiltrative. The recurrence
rate is approximately 4%. The risk of nerve deficit is higher for excision after recurrence.

An aneurysm of the ulnar artery presents as a pulsatile mass. Vasospastic or thromboembolic


findings may be present. The ulnar nerve may be compressed by the aneurysm. Surgery will
preclude thromboembolic events. If the digital brachial index is <0.7, arterial reconstruction is
required.

An epidermal inclusion cyst results from implantation of epithelial cells into the underlying soft
tissue or bone after an injury. Involvement of the thumb or index finger is most common. The
cyst can abscess and require drainage. Otherwise, the cyst can be marginally excised from
surrounding soft tissue or curettaged from bone. Interestingly, imaging of bony involvement can
demonstrate cortical erosion or bone destruction that is more typical for malignancy.

A giant cell tumor of the tendon sheath is a rubbery mass that is more common along the flexor
surfaces of the digits. The slowly enlarging tumor can compress or splay adjacent digital nerves
and arteries. Recurrence is minimized with a complete marginal excision and bipolar
cauterization of the tumor bed. The recurrence rate is reported to be 5% to 50%.

This examination contains test materials that are owned and copyrighted by the American Society of
Plastic Surgeons and the Plastic Surgery Educational Foundation. Any reproduction of these materials or
any part of them, through any means, including but not limited to, copying or printing electronic files,
reconstruction through memorization or dictation, and/or dissemination of these materials or any part of
them is strictly prohibited. Keep printed materials in a secure location when you are not reviewing them
and discard them in a secure manner, such as shredding, when you have completed the examination.

Page 231 of 287


American Society of Plastic Surgeons and the Plastic Surgery Educational Foundation
In-Service Examination

References

1. Ho PK, Weiland AJ, McClinton MA, et al. Aneurysms of the upper extremity. J
Hand Surg [Am]. 1987;12:39-46.
2. O’Connor MI, Bancroft LW. Benign and malignant cartilage tumors of the hand.
Hand Clin. 2004;20:317-323, vi.
3. Ingari JV, Faillace JJ. Benign tumors of fibrous tissue and adipose tissue in the hand.
Hand Clin. 2004;20:243-248, v.
4. Forthman CL, Blazar PE. Nerve tumors of the hand and upper extremity. Hand Clin.
2004;20:233-242, v.
5. Rockwell GM, Thoma A, Salama S. Schwannoma of the hand and wrist. Plast
Reconstr Surg. 2003;111:1227-1232.

This examination contains test materials that are owned and copyrighted by the American Society of
Plastic Surgeons and the Plastic Surgery Educational Foundation. Any reproduction of these materials or
any part of them, through any means, including but not limited to, copying or printing electronic files,
reconstruction through memorization or dictation, and/or dissemination of these materials or any part of
them is strictly prohibited. Keep printed materials in a secure location when you are not reviewing them
and discard them in a secure manner, such as shredding, when you have completed the examination.

Page 232 of 287


American Society of Plastic Surgeons and the Plastic Surgery Educational Foundation
In-Service Examination

166. A 35-year-old man comes to the emergency department immediately after he


sustained an injury to the long finger of the dominant right hand while using a table
saw. Physical examination shows a segmental injury to the extensor tendon.
Sensation and vascular supply are intact along both sides of the finger.
Radiographs show a severely comminuted proximal interphalangeal (PIP) joint
with loss of bone. Arthrodesis of the PIP joint is performed using a tension band
technique. Soft-tissue coverage is adequate. Which of the following is the most
likely complication of this procedure?
A) Decreased two-point discrimination
B) Lumbrical plus deformity
C) Malunion
D) Nonunion
E) Quadriga effect

The correct response is Option E.

A proximal interphalangeal (PIP) fusion will result in a limitation in long finger profundus
excursion. Any action that prevents the full excursion of the profundus tendon of one finger will
affect the other fingers and produce some component of quadriga. “Quadriga” refers to the
global limitation in profundus excursion resulting from injury or limitations in motion occurring
at a single finger profundus tendon. Amputations are a common cause of quadriga. After finger
amputations, spontaneous adhesions of the resected profundus tendon may occur in the finger
stump or palm. Because of the normal interconnections of the profundus tendons, such adhesions
can block the excursion of the profundus tendons to intact fingers, resulting in the quadriga
syndrome, or profundus tendon blockage. This causes a decrease in the power and range of
movement of the terminal joints of the uninjured fingers when they are fully flexed. PIP fusion is
often well tolerated in the index finger, because the relatively independent profundus function of
the index finger does not impose a significant quadriga effect on the other fingers during power
grasp. PIP fusion of the long finger, however, has been shown to decrease the excursion of all
profundus tendons, reducing grip strength. PIP fusion restricts profundus excursion to a greater
extent than distal interphalangeal (DIP) or metacarpophalangeal (MP) fusion.

A change in two-point discrimination will occur if there has been a significant injury to one or
both of the digital nerves.

The lumbrical plus deformity produces paradoxical extension of the PIP joint when the finger is
flexed at the MP and is caused by retraction of the profundus tendon and tightening of the
lumbricals following amputation.

This examination contains test materials that are owned and copyrighted by the American Society of
Plastic Surgeons and the Plastic Surgery Educational Foundation. Any reproduction of these materials or
any part of them, through any means, including but not limited to, copying or printing electronic files,
reconstruction through memorization or dictation, and/or dissemination of these materials or any part of
them is strictly prohibited. Keep printed materials in a secure location when you are not reviewing them
and discard them in a secure manner, such as shredding, when you have completed the examination.

Page 233 of 287


American Society of Plastic Surgeons and the Plastic Surgery Educational Foundation
In-Service Examination

Malunion and nonunion are complications that may occur following any fusion procedure but are
unlikely if proper surgical technique is followed.

References

1. Moran SL, Berger RA. Biomechanics and hand trauma: what you need. Hand Clin.
2003;19:17-31.
2. Vedder NB, Hanel DP. The mangled upper extremity. In: Green DP, Hotchkiss RN,
Pederson WC, et al., eds. Green's Operative Hand Surgery. Philadelphia: Elsevier;
2005:1587-1628.

This examination contains test materials that are owned and copyrighted by the American Society of
Plastic Surgeons and the Plastic Surgery Educational Foundation. Any reproduction of these materials or
any part of them, through any means, including but not limited to, copying or printing electronic files,
reconstruction through memorization or dictation, and/or dissemination of these materials or any part of
them is strictly prohibited. Keep printed materials in a secure location when you are not reviewing them
and discard them in a secure manner, such as shredding, when you have completed the examination.

Page 234 of 287


American Society of Plastic Surgeons and the Plastic Surgery Educational Foundation
In-Service Examination

167. A 40-year-old woman who is a pitcher on a softball team has had swelling and
discomfort of the ring finger of the dominant right hand since she sustained an
injury during a game five days ago. She is concerned because she is not able to
grip a softball and has several games scheduled over the next few months.
Physical examination is limited because of edema and pain. The lateral
radiograph is shown. Which of the following interventions is the most appropriate
initial management of this patient's finger?
A) Buddy taping of the ring and long fingers
B) Extension block splinting of the proximal interphalangeal joint
C) Forearm-based intrinsic plus splinting
D) Hyperextension splinting of the distal interphalangeal joint
E) Injection of a corticosteroid into the proximal interphalangeal joint

The correct response is Option B.

The patient described has a fracture dislocation of the proximal interphalangeal (PIP) joint,
which is a frequent sports injury. The most common pattern involves the volar lip of the middle
phalanx. With the stability of the PIP joint disrupted, the finger subluxes or dislocates dorsally.
Early identification of the injury pattern and severity is critical to successful outcome. After
evaluation and radiography, the injury should be splinted in a protected position. A dorsal
semiflexed splint should protect the PIP joint in a stable, reduced position. The dorsal splint,
which blocks extension, protects the injured joint from further complications.

An injection of a corticosteroid will alleviate pain and swelling but is not indicated for a

This examination contains test materials that are owned and copyrighted by the American Society of
Plastic Surgeons and the Plastic Surgery Educational Foundation. Any reproduction of these materials or
any part of them, through any means, including but not limited to, copying or printing electronic files,
reconstruction through memorization or dictation, and/or dissemination of these materials or any part of
them is strictly prohibited. Keep printed materials in a secure location when you are not reviewing them
and discard them in a secure manner, such as shredding, when you have completed the examination.

Page 235 of 287


American Society of Plastic Surgeons and the Plastic Surgery Educational Foundation
In-Service Examination

fracture-dislocation injury. Buddy taping is useful for PIP joint collateral ligament injury.
Hyperextension casting of the distal interphalangeal joint is indicated for a mallet injury or
terminal tendon disruption. An intrinsic plus splint is useful for many soft tissue and bony
injuries. However, the injury in the patient described requires specific positioning of the PIP
joint.

References

1. Weiss A-PC. Cerclage fixation for fracture dislocation of the proximal


interphalangeal joint. Clin Orthop. 1996;327:21-28.
2. Williams RMM, Kiefhaber TR, Sommerkamp TG, et al. Treatment of unstable dorsal
proximal interphalangeal fracture/dislocations using a hemi-hamate autograft. J Hand
Surg [Am]. 2003;28A:856-865.
3. Seno N, Hashizume H, Inoue H, et al. Fractures of the base of the middle phalanx of
the finger: classification, management and long-term results. J Bone Joint Surg Br.
1997;79:758-763.
4. O’Rourke SK, Gaur S, Barton NJ. Long-term outcome of articular fractures of the
phalanges: an eleven year follow up. J Hand Surg [Br]. 1989;14:183-193.

This examination contains test materials that are owned and copyrighted by the American Society of
Plastic Surgeons and the Plastic Surgery Educational Foundation. Any reproduction of these materials or
any part of them, through any means, including but not limited to, copying or printing electronic files,
reconstruction through memorization or dictation, and/or dissemination of these materials or any part of
them is strictly prohibited. Keep printed materials in a secure location when you are not reviewing them
and discard them in a secure manner, such as shredding, when you have completed the examination.

Page 236 of 287


American Society of Plastic Surgeons and the Plastic Surgery Educational Foundation
In-Service Examination

168. A 48-year-old woman is referred to the office by her primary care physician for
evaluation of a painless ulcer on the tip of the index finger of the right hand that
has been present for the past three months. Medical history includes scleroderma
with thin, tight, fibrotic skin on the face and perioral region. On physical
examination, dry eschar and visible bone are noted at the tip of the index finger.
The skin of the other digits is thin and shiny. No flexion contractures of the
interphalangeal joints are noted. Radiographs of the index finger show mild
resorption of the tuft. Which of the following is the most appropriate management
at this time?
A) Amputation of the distal phalanx and direct closure of the stump at the level of the
distal interphalangeal joint
B) Conservative debridement of the soft tissue of the tip of the finger and resection of
exposed bone
C) Ray amputation of the digit and primary closure
D) Resection of the distal phalanx to the level of the distal interphalangeal joint and soft-
tissue healing by secondary intention
E) Urgent digital sympathectomy and debridement of eschar

The correct response is Option B.

Scleroderma is an autoimmune disease that includes a disorder of the connective tissue and small
blood vessels. This disease can affect the skin, hands, gastrointestinal tract, heart, lungs, and
kidneys. The sclerosis or fibrosis is commonly seen in the skin of the face and perioral area. This
sclerosis of the face gives the patient a mask-type appearance characteristic of scleroderma. The
CREST symptom complex has been used to describe the most common findings of systemic
sclerosis: calcinosis, Raynaud phenomenon, esophageal dysfunction, sclerodactyly, and
telangiectasia. Ulceration at the fingertips is a common finding secondary to poor circulation.
Skin breakdown can also occur at the proximal interphalangeal (PIP) and metacarpophalangeal
(MP) joints caused by bony joint deformities as well as poor circulation.

Tip ulcerations are common because of the poor circulation associated with scleroderma. Many
of these ulcerations will successfully heal over time without surgery. An initial conservative
approach should therefore be taken. This approach includes conservative debridement, topical
antibiotics (eg, silver sulfadiazine), and limited resection of exposed bone. If active infection is
present, antibiotics should be administered.

If ulceration and pain fail to respond to conservative treatment, more aggressive intervention is
required, including amputation and possible digital sympathectomy. Unfortunately, many

This examination contains test materials that are owned and copyrighted by the American Society of
Plastic Surgeons and the Plastic Surgery Educational Foundation. Any reproduction of these materials or
any part of them, through any means, including but not limited to, copying or printing electronic files,
reconstruction through memorization or dictation, and/or dissemination of these materials or any part of
them is strictly prohibited. Keep printed materials in a secure location when you are not reviewing them
and discard them in a secure manner, such as shredding, when you have completed the examination.

Page 237 of 287


American Society of Plastic Surgeons and the Plastic Surgery Educational Foundation
In-Service Examination

patients who undergo digital sympathectomy continue to have pain and suffer recurrent
ulceration.

Calcinosis is the deposition of calcium within the dermis or subcutaneously. These deposits can
cause pain, spontaneously cause skin breakdown, and drain calcific material. Symptomatic
lesions require excision or partial removal to alleviate symptoms.

Amputation, digital sympathectomy, and resection of the distal phalanx with secondary healing
are not appropriate because conservative therapy should be attempted first.

References

1. Ruch DS, Holden M, Smith BP, et al. Periarterial sympathectomy in scleroderma


patients: intermediate-term follow-up. J Hand Surg [Am]. 2002;27:258-264.
2. Bogoch ER, Gross DK. Surgery of the hand in patients with systemic sclerosis:
outcomes and considerations. J Rheumatol. 2005;32:642-648.
3. Gahhos F, Ariyan S, Frazier WH, et al. Management of sclerodermal finger ulcers. J
Hand Surg [Am]. 1984;9:320-327.
4. Feldon P, Terrono AL, Nalebuff EA, et al. Rheumatoid arthritis and other connective
tissue diseases. In: Green DP, Hotchkiss RN, Pederson WC, et al, eds. Operative
Hand Surgery. 5th ed. Philadelphia: Elsevier; 2005:2057-2059.
5. McCall TE, Petersen DP, Wong LB. The use of digital artery sympathectomy as a
salvage procedure for severe ischemia of Raynaud's disease and phenomenon. J Hand
Surg [Am]. 1999;34:173- 177.

This examination contains test materials that are owned and copyrighted by the American Society of
Plastic Surgeons and the Plastic Surgery Educational Foundation. Any reproduction of these materials or
any part of them, through any means, including but not limited to, copying or printing electronic files,
reconstruction through memorization or dictation, and/or dissemination of these materials or any part of
them is strictly prohibited. Keep printed materials in a secure location when you are not reviewing them
and discard them in a secure manner, such as shredding, when you have completed the examination.

Page 238 of 287


American Society of Plastic Surgeons and the Plastic Surgery Educational Foundation
In-Service Examination

169. A 35-year-old highway construction worker is brought to the emergency


department after he sustained injuries to the right leg when he was struck by a
passing vehicle. The patient’s condition is stable and vital signs are within normal
limits. Physical examination shows a Gustilo Class IIIB open fracture of the tibia.
No other serious injuries are noted. Irrigation and debridement performed in the
operating room result in a large soft-tissue defect of the upper third of the anterior
leg and an underlying tibial defect of 11 cm. Which of the following interventions is
most appropriate for wound closure and healing in this patient?
A) Application of hydroxyapatite crystals under a vascularized muscle flap
B) Cross-leg flap over an ipsilateral fibula transfer flap
C) Free muscle flap and a traditional tricortical iliac bone graft
D) Rotation pedicle muscle flap over a contralateral free fibula graft
E) Vacuum-assisted closure (VAC) therapy and bone lengthening with the Ilizarov
apparatus

The correct response is Option D.

Classifications of open fractures of the tibia allow for grouping based on severity. These
classifications can then be used to determine the most appropriate treatment and its prognosis.
Gustilo Class IIIB is an open fracture with extensive soft-tissue damage and exposed bone and
requires soft-tissue coverage. The goal of treatment is wound closure and bone healing to allow
ambulation. The first step of treatment is early and adequate debridement in concert with bone
stabilization. This treatment is followed by vascularized soft-tissue coverage and bone
reconstruction.

In bone defects greater than 6–8 cm, a vascularized fibular graft is the treatment of choice. A free
fibula graft can be taken with a skin island; however, if the defect is large, a second flap is
required.

In the scenario described, a rotation pedicle muscle flap over a contralateral free fibula graft is
most appropriate for wound closure and healing because it provides for vascularized soft-tissue
coverage. A gastrocnemius flap is a reasonable choice in the upper third of the tibia.

Application of hydroxyapatite crystals under a vascularized muscle flap is inappropriate because


this defect is too large for cancellous bone grafting or hydroxyapatite.

A cross-leg flap is only indicated if the other methods listed are not available. An ipsilateral
fibula pedicled bone graft is possible; however, with both tibia and fibula fractures, transfer of
the fibula can aggravate instability.

This examination contains test materials that are owned and copyrighted by the American Society of
Plastic Surgeons and the Plastic Surgery Educational Foundation. Any reproduction of these materials or
any part of them, through any means, including but not limited to, copying or printing electronic files,
reconstruction through memorization or dictation, and/or dissemination of these materials or any part of
them is strictly prohibited. Keep printed materials in a secure location when you are not reviewing them
and discard them in a secure manner, such as shredding, when you have completed the examination.

Page 239 of 287


American Society of Plastic Surgeons and the Plastic Surgery Educational Foundation
In-Service Examination

Free muscle flap and a traditional tricortical iliac bone graft provides for soft-tissue coverage;
however, the defect size of 11 cm is too large for a traditional iliac bone graft. A vascularized
iliac crest free flap will have significant curvature as well as inadequate length.

A vacuum-assisted closure (VAC) system applies negative pressure to the wound to decrease the
edema and reduce wound size. This process is slow and does not provide bone coverage in a
high-energy injury. The Ilizarov bone-lengthening technique is a long and arduous process of
distracting the debrided bone ends. It is less successful in injuries with significant soft-tissue
injury and scarring.

References

1. Francel TJ, Vander Kolk GA, Hoopes JE, et al. Microvascular soft-tissue
transplantation for reconstruction of acute open tibial fractures: timing of coverage
and long-term functional results. Plast Reconstr Surg. 1992;89:478.
2. Lee KS, Han SB, Baek JR. Free vascularized osteocutaneous fibular graft to the tibia
in 51 consecutive cases. J Reconstr Microsurg. 2004;20:277.
3. Heller L, Phillips K, Levin LS. Pedicled osteocutaneous fibula flap for reconstruction
in the lower extremity. Plast Reconstr Surg. 2002;109:2037.

This examination contains test materials that are owned and copyrighted by the American Society of
Plastic Surgeons and the Plastic Surgery Educational Foundation. Any reproduction of these materials or
any part of them, through any means, including but not limited to, copying or printing electronic files,
reconstruction through memorization or dictation, and/or dissemination of these materials or any part of
them is strictly prohibited. Keep printed materials in a secure location when you are not reviewing them
and discard them in a secure manner, such as shredding, when you have completed the examination.

Page 240 of 287


American Society of Plastic Surgeons and the Plastic Surgery Educational Foundation
In-Service Examination

170. A 37-year-old man comes to the emergency department because he has pain and
swelling of the right hand as well as numbness of the fingertips two hours after
undergoing CT scan of the abdomen because of abdominal pain. Physical
examination shows lack of full active extension of the fingers and exquisite
tenderness on passive adduction and abduction of the fingers. Pulses are intact.
High-pressure infiltration of intravenous radiographic contrast medium into the
hand is suspected. Which of the following is the most appropriate management?
A) Administration of nonsteroidal anti-inflammatory drugs and follow-up evaluation in
the office
B) Elevation of the arm, application of ice, and re-evaluation in six hours
C) Emergency fasciotomy
D) Initiation of heparin therapy for anticoagulation
E) Subcutaneous injection of calcium gluconate into the arm

The correct response is Option C.

The condition described is clearly a developing compartment syndrome that should be


emergently decompressed in the operating room. The diagnosis should not be delayed until
pulses are absent because absence of pulses is a late finding. Re-evaluation in six hours is not
appropriate for the patient described because, by that time, irreversible muscle necrosis may
occur, resulting in potential long-term disability. Similarly, having the patient follow up in the
office is not appropriate. Initiation of heparin therapy for anticoagulation may be indicated in an
ischemic event but is not appropriate in this scenario given the history and findings of the patient
described. Subcutaneous injection of calcium gluconate is the treatment of choice for
hydrofluoric acid burns, not for intravenous contrast infiltration.

In the case of a “nonalert” patient, compartment pressures may be measured using an


intracompartmental pressure monitor. Such devices are commercially available or may be
fabricated. Interstitial compartment pressures of > 30 mmHg are consistent with compartment
syndrome.

References

1. Stein DA, Lee S, Raskin KB. Compartment syndrome of the hand caused by
computed tomography contrast infiltration. Orthopedics. 2003;26:333-334.
2. Wedler V, Guggenheim M, Moron M, et al. Extensive hydrofluoric acid injuries: a
serious problem. J Trauma. 2005;58:852-857.

This examination contains test materials that are owned and copyrighted by the American Society of
Plastic Surgeons and the Plastic Surgery Educational Foundation. Any reproduction of these materials or
any part of them, through any means, including but not limited to, copying or printing electronic files,
reconstruction through memorization or dictation, and/or dissemination of these materials or any part of
them is strictly prohibited. Keep printed materials in a secure location when you are not reviewing them
and discard them in a secure manner, such as shredding, when you have completed the examination.

Page 241 of 287


American Society of Plastic Surgeons and the Plastic Surgery Educational Foundation
In-Service Examination

171. A 21-year-old man comes to the office because he has had absence of sensation
of the right thumb since he underwent repair of the digital nerve on the opposition
side of the digit one year ago. On physical examination, Tinel sign is absent at the
surgical scar. After surgical exploration and debridement of the thumb, a 3-cm
nerve gap is noted. Which of the following is the most appropriate method of
digital reconstruction?
A) Mobilization of the free nerve ends and primary repair
B) Polyglycolic acid tube conduit
C) Posterior interosseous nerve grafts
D) Radial central nerve grafts
E) Vein conduit

The correct response is Option C.

Repair or reconstruction of a digital nerve requires a tensionless technique. With tension, repair
of the nerve will cause scarring and form a neuroma. If the gap is too large, a graft or conduit is
required for a tensionless reconstruction. The terminal posterior interosseous nerve graft is ideal
because it provides a nerve with similar caliber for the repair. Harvesting of this nerve avoids a
donor deficit because at the level of the distal forearm wrist it only provides for wrist sensation.

Large nerve defects should not be reconstructed with vein or polyglycolic acid tube conduits.
The radial sensory nerve is too large in diameter. Additionally, the sensory loss would be more
problematic and would risk a painful neuroma.

References

1. Maser BM, Vedder N. Nerve repair and nerve grafting. In: Russell RC, ed. Plastic
Surgery. St. Louis: Mosby; 2000:1775.
2. Zeineh L, Wilhelmi BJ, Zook EG. Managing acute nerve injuries in extremities. Oper
Tech Plastic Reconstr Surg. 2003;9:111-116.

This examination contains test materials that are owned and copyrighted by the American Society of
Plastic Surgeons and the Plastic Surgery Educational Foundation. Any reproduction of these materials or
any part of them, through any means, including but not limited to, copying or printing electronic files,
reconstruction through memorization or dictation, and/or dissemination of these materials or any part of
them is strictly prohibited. Keep printed materials in a secure location when you are not reviewing them
and discard them in a secure manner, such as shredding, when you have completed the examination.

Page 242 of 287


American Society of Plastic Surgeons and the Plastic Surgery Educational Foundation
In-Service Examination

172. A 72-year-old man is referred to the hand clinic because he has had paronychia of
the left thumb for the past three months. Treatment by the patient’s primary care
physician, including warm soaks and antifungal therapy, resulted in no change in
his condition. Physical examination of the finger shows a 0.3 x 0.6-cm
erythematous lesion extending from the lunula to the eponychial fold and involving
the nail bed. No palpable lymph nodes are noted. Radiography shows no bony
involvement. Biopsy of the lesion shows moderately differentiated squamous cell
carcinoma. Which of the following is the most appropriate management?
A) Amputation of the entire distal phalanx
B) Complete ablation of the nail matrix
C) Excision of the lesion with 2-mm margins
D) Excision of the lesion with 15-mm margins
E) Ray amputation

The correct response is Option A.

The condition of the patient described requires amputation of the entire distal phalanx.

Chronic paronychia is usually caused by a candidal infection. Failure of medical treatment with
antifungals requires culture, radiographic evaluation to exclude bony involvement, and biopsy to
exclude malignancy, particularly squamous cell carcinoma.

Squamous cell carcinoma is the most common primary malignancy of the hand. Lesions not
involving the nail bed require excision with 1-cm margins; 2-mm margins are inadequate for
squamous cell carcinoma. Tumors involving the nail bed or bone necessitate amputation of the
entire distal phalanx.

Ray amputation of the thumb results in excessive functional loss of the extremity and does not
improve long-term survival.

References

1. Fink JA, Akelman E. Nonmelanotic malignant skin tumors of the hand. Hand Clin.
1995;11:255-264.
2. Krull EA. Epidermoid carcinoma of subungual and periungual tissues. In: Krull EA,
Zook EG, Baran R, et al, eds. Nail Surgery: A Text and Atlas. Philadelphia:
Lippincott Williams & Wilkins; 2001:297-304.
3. Fleegler EJ. Benign and malignant soft tissue tumors of the upper limb. In: Mathes
SJ, ed. Plastic Surgery. Philadelphia: WB Saunders; 2006:949-977.

This examination contains test materials that are owned and copyrighted by the American Society of
Plastic Surgeons and the Plastic Surgery Educational Foundation. Any reproduction of these materials or
any part of them, through any means, including but not limited to, copying or printing electronic files,
reconstruction through memorization or dictation, and/or dissemination of these materials or any part of
them is strictly prohibited. Keep printed materials in a secure location when you are not reviewing them
and discard them in a secure manner, such as shredding, when you have completed the examination.

Page 243 of 287


American Society of Plastic Surgeons and the Plastic Surgery Educational Foundation
In-Service Examination

173. A 29-year-old woman is referred to the office because she has flu-like symptoms
as well as pain and stiffness in the joints of the hands. The pain keeps her awake
at night and the stiffness occurs in the morning and lasts approximately two hours.
The patient was evaluated for these symptoms in the emergency department two
weeks ago. Analysis of aspirate from the affected joints was negative for white
blood cells and organisms. A one-week course of cephalexin was initiated at that
time. Current temperature is 38.1°C (100.6°F). Examination of the hands shows
inflammation, swelling, and tenderness of the proximal interphalangeal joints of
the index and long fingers. Which of the following is the most likely diagnosis?
A) Gout
B) Osteoarthritis
C) Pseudogout
D) Rheumatoid arthritis
E) Septic arthritis

The correct response is Option D.

The patient described has the hallmarks of rheumatoid arthritis, the diagnosis of which is made
on clinical grounds. The criteria for diagnosis were published in 1988; these criteria are the gold
standard for diagnosis. Patients with the following symptoms for six or more weeks are
suspected to have rheumatoid arthritis: morning stiffness; soft tissue swelling; swelling of the
proximal interphalangeal, metacarpophalangeal, and/or wrist joints; and symmetric arthritis.
They may also have subcutaneous nodules, a positive rheumatoid factor, and positive findings on
plain radiographs.

Although the other diagnoses (gout, osteoarthritis, pseudogout, and septic arthritis) should not be
ruled out, the clinical findings described make rheumatoid arthritis the most likely diagnosis. A
complete blood count, erythrocyte sedimentation rate, serum rheumatoid factor, and antinuclear
antibodies, although not adequate for making a conclusive diagnosis, will make diagnosis more
conclusive when combined with plain radiographs and clinical findings.

Gout would be indicated by an elevated serum uric acid level or uric acid crystals in the joint
aspirate. Calcium pyrophosphate crystals would be found in the joint aspirate in pseudogout.
Leukocytes and pathogens would be noted in the aspirate in septic arthritis.

This examination contains test materials that are owned and copyrighted by the American Society of
Plastic Surgeons and the Plastic Surgery Educational Foundation. Any reproduction of these materials or
any part of them, through any means, including but not limited to, copying or printing electronic files,
reconstruction through memorization or dictation, and/or dissemination of these materials or any part of
them is strictly prohibited. Keep printed materials in a secure location when you are not reviewing them
and discard them in a secure manner, such as shredding, when you have completed the examination.

Page 244 of 287


American Society of Plastic Surgeons and the Plastic Surgery Educational Foundation
In-Service Examination

References

1. Brown FE, Collins ED, Harmatz AS. Rheumatoid arthritis of the hand and wrist. In:
Achauer BM, Eriksson E, Guyuron B, et al., eds. Plastic Surgery. Philadelphia:
Mosby; 2000:2249-2274.
2. Arnett F, Edworthy S, Bloch D, et al. The American Rheumatism Association 1987
revised criteria for classification of rheumatoid arthritis. Arthritis Rheum.
1988;31:315-324.

This examination contains test materials that are owned and copyrighted by the American Society of
Plastic Surgeons and the Plastic Surgery Educational Foundation. Any reproduction of these materials or
any part of them, through any means, including but not limited to, copying or printing electronic files,
reconstruction through memorization or dictation, and/or dissemination of these materials or any part of
them is strictly prohibited. Keep printed materials in a secure location when you are not reviewing them
and discard them in a secure manner, such as shredding, when you have completed the examination.

Page 245 of 287


American Society of Plastic Surgeons and the Plastic Surgery Educational Foundation
In-Service Examination

(Please note that this pictorial appears in color in the online examination)

174. A 42-year-old man comes to the office for initial consultation regarding an injury to
the small finger of the right hand for which he was treated at an outside hospital
one week ago. No medical records or radiographs related to the injury are
available. The device on the patient's finger (shown) was most likely placed for
correction of which of the following conditions?
A) Central slip avulsion
B) Fracture dislocation of the proximal interphalangeal joint
C) Fracture of the middle phalangeal shaft
D) Laceration of the flexor tendon
E) Mallet finger deformity

The correct response is Option B.

Operative treatment of a fracture dislocation of the proximal interphalangeal (PIP) joint is


generally indicated when more than 30% to 40% of the volar articular surface is involved. With
this fragment volume, the PIP joint is unstable and displaced. Treatment options include open
reduction and internal fixation, extension block pinning, or use of a dynamic traction device.
Several manufactured devices are on the market; however, the simple device pictured has been
shown to be very effective and can be made using easily available Kirschner wires. The
technique relies on the fact that distraction of the finger will reliably reduce the fracture and
restore the joint anatomy. It also allows the patient to move the joint during fracture healing,

This examination contains test materials that are owned and copyrighted by the American Society of
Plastic Surgeons and the Plastic Surgery Educational Foundation. Any reproduction of these materials or
any part of them, through any means, including but not limited to, copying or printing electronic files,
reconstruction through memorization or dictation, and/or dissemination of these materials or any part of
them is strictly prohibited. Keep printed materials in a secure location when you are not reviewing them
and discard them in a secure manner, such as shredding, when you have completed the examination.

Page 246 of 287


American Society of Plastic Surgeons and the Plastic Surgery Educational Foundation
In-Service Examination

decreasing the incidence of PIP stiffness.

A simple central slip avulsion can be treated with extension splinting. The device shown would
not stabilize a fracture of the phalangeal shaft. An unstable shaft fracture could be managed by
pinning or plate fixation. Repair of the flexor tendon would not be helped by a dynamic
intradigital traction device. A mallet injury would be treated with hyperextension splinting.

References

1. Badia A, Riano F, Ravikoff J, et al. Dynamic intradigital external fixation for


proximal interphalangeal joint fracture dislocations. J Hand Surg [Am].
2005;30A:154-160.
2. Glickel SZ, Barron OA, Eaton RG. Dislocations and ligament injuries in the digits.
In: Green DP, Hotchkiss RN, Pederson C, eds. Green’s Operative Hand Surgery. 4th
ed. New York: Churchill Livingstone; 1999:772-808.

This examination contains test materials that are owned and copyrighted by the American Society of
Plastic Surgeons and the Plastic Surgery Educational Foundation. Any reproduction of these materials or
any part of them, through any means, including but not limited to, copying or printing electronic files,
reconstruction through memorization or dictation, and/or dissemination of these materials or any part of
them is strictly prohibited. Keep printed materials in a secure location when you are not reviewing them
and discard them in a secure manner, such as shredding, when you have completed the examination.

Page 247 of 287


American Society of Plastic Surgeons and the Plastic Surgery Educational Foundation
In-Service Examination

175. A 43-year-old man who works as an electrical lineman is brought to the


emergency department one hour after he sustained a high-voltage electrical injury.
Physical examination shows a 1.5 × 2-cm area of eschar on the palmar aspect of
the right hand and an area of charred tissue on the right foot that is presumed to
be the exit wound. Pulses in all extremities are within normal limits. Evaluation of
the hand shows absence of sensation and muscle weakness. Which of the
following diagnostic studies is most appropriate to evaluate the extent of muscle
damage in the patient's hand?
A) Angiography
B) CT
C) Electromyography
D) MRI
E) Technetium-99m scanning

The correct response is Option D.

High-voltage electrical injuries (greater than 1000 volts) to the extremities present a difficult
diagnostic dilemma. Often, the injury is significantly more extensive than indicated by the
cutaneous examination. Muscle necrosis may be present under normal skin. Electrical injuries
often involve deep structures and subfascial edema may occur, leading to further muscle
necrosis. Various studies have been used to evaluate the burned extremity when there is no need
for urgent escharotomy. MRI is the most appropriate choice because it is the most sensitive of
the tests listed. MR images do not demonstrate tissue edema.

In patients who have sustained high-voltage electrical injuries, observation is appropriate. In


electrical burns, sensation often cannot be used as an adequate judge because paresthesia is
common because of the low resistance of the nerve to electric current. Myoglobinuria is
associated with significant muscle damage and may predict the need for fasciotomies.

References

1. Cancio LC, Jimenez-Reyna JF, Barillo DJ, et al. One hundred ninety-five cases of
high-voltage electric injury. J Burn Care Rehab. 2005;26:331-340.
2. Danielson JR, Capelli-Schellpfeffer M, Lee RC. Upper extremity electrical injury. In:
Tubiana R, ed. Hand Clinics. Philadelphia: WB Saunders; 2000:225-234.

This examination contains test materials that are owned and copyrighted by the American Society of
Plastic Surgeons and the Plastic Surgery Educational Foundation. Any reproduction of these materials or
any part of them, through any means, including but not limited to, copying or printing electronic files,
reconstruction through memorization or dictation, and/or dissemination of these materials or any part of
them is strictly prohibited. Keep printed materials in a secure location when you are not reviewing them
and discard them in a secure manner, such as shredding, when you have completed the examination.

Page 248 of 287


American Society of Plastic Surgeons and the Plastic Surgery Educational Foundation
In-Service Examination

176. A 62-year-old man who is a practicing orthodontist comes to the office because he
has had pain in the thumb of the dominant left hand at the level of the
trapeziometacarpal (TM) joint for the past year. He has no symptoms in the right
hand. On physical examination, grind test of the left thumb is positive.
Radiographs of both thumbs are shown. Which of the following interventions
involving the left thumb is most appropriate at this time?
A) Injection of a corticosteroid followed by splinting of the TM joint
B) Metacarpal extension osteotomy of the TM joint
C) Partial excision of the trapezium and tissue interposition arthroplasty
D) Resection of the trapezium followed by silicone implant arthroplasty
E) Splinting of the TM joint and application of a bone stimulator

The correct response is Option A.

The trapeziometacarpal (TM) joint is frequently affected with osteoarthritis. Arthritis of the
trapezium can affect five articulations: TM, trapezium-second metacarpal, trapezium-trapezoid,

This examination contains test materials that are owned and copyrighted by the American Society of
Plastic Surgeons and the Plastic Surgery Educational Foundation. Any reproduction of these materials or
any part of them, through any means, including but not limited to, copying or printing electronic files,
reconstruction through memorization or dictation, and/or dissemination of these materials or any part of
them is strictly prohibited. Keep printed materials in a secure location when you are not reviewing them
and discard them in a secure manner, such as shredding, when you have completed the examination.

Page 249 of 287


American Society of Plastic Surgeons and the Plastic Surgery Educational Foundation
In-Service Examination

scaphotrapezial, and scaphotrapezoidal. When all of these surfaces are involved, the condition is
called pantrapezial arthritis. This complex of trapezial articulations is referred to as the basal
joint of the thumb.

The key to the scenario described is twofold. The first important fact is that radiographic staging
may not relate to symptoms. The second important tenet is that conservative nonoperative
therapy such as splinting and injection of corticosteroids should be tried first. If conservative
therapy is unsuccessful, then arthroplasty is indicated. The patient described has pantrapezial
arthritis in the right basilar joint, but he has no symptoms. Therefore, no intervention is needed.
The left basilar joint (the patient's dominant hand) is symptomatic, and an initial trial of
conservative therapy is indicated. One study found that 70% of patients undergoing conservative
therapy were able to avoid surgery in the long term.

The grind test is performed by holding the thumb metacarpal and applying axial compression
while rotating the metacarpal in a circular motion. Pain and crepitance indicate a positive result.

Early stages of basal arthritis can be treated with ligament reconstruction. The goal of this
operation is reconstruction of the volar beak ligament of the thumb metacarpal and reinforcement
of the dorsal ligaments. Metacarpal extension osteotomy is an alternative operation for early
basal arthritis. The goal of this operation is to unload the palmar joint surfaces. More advanced
stages of basal arthritis require either partial or complete resection of the trapezium. Silicone
implant arthroplasty has fallen out of favor because of long-term complications such as implant
wear, implant fracture, and subluxation. The most common treatment for advanced pantrapezial
arthritis is excision of the trapezium with ligament reconstruction and metacarpal suspension.
Hematoma and distraction arthroplasty is excision of the trapezium without tendon interposition.
In this technique, Kirschner wires are placed for six weeks to maintain the trapezial space. Bone
stimulation does not play a role in the treatment of basilar joint arthritis.

References

1. Tomaino MM, King J, Leit M. Thumb basal joint arthritis. In: Green DP, Hotchkiss
RN, Pederson WC, et al, eds. Operative Hand Surgery. 5th ed. Philadelphia: Elsevier;
2005:461-485.
2. Freedman DM, Eaton RG, Glickel SZ. Long-term results of volar ligament
reconstruction for symptomatic basal joint laxity. J Hand Surg [Am].
2000;25:297-304.
3. Tomaino MM. Treatment of Eaton stage I trapeziometacarpal disease with metacarpal
extension osteotomy. J Hand Surg [Am]. 2000;25:1100-1106.
4. Tomaino MM, Pellegrini VD Jr, Burton RI. Arthroplasty of the basal joint of the
thumb: Long-term follow-up after ligament reconstruction with tendon interposition.
J Bone Joint Surg Am. 1995;77:346-355.

This examination contains test materials that are owned and copyrighted by the American Society of
Plastic Surgeons and the Plastic Surgery Educational Foundation. Any reproduction of these materials or
any part of them, through any means, including but not limited to, copying or printing electronic files,
reconstruction through memorization or dictation, and/or dissemination of these materials or any part of
them is strictly prohibited. Keep printed materials in a secure location when you are not reviewing them
and discard them in a secure manner, such as shredding, when you have completed the examination.

Page 250 of 287


American Society of Plastic Surgeons and the Plastic Surgery Educational Foundation
In-Service Examination

(Please note that this pictorial appears in color in the online examination)

177. A 3-month-old infant is brought to the office for consultation regarding deformity of
the right hand (shown). The infant is otherwise healthy and has no other
abnormalities. On physical examination, no abnormalities other than those of the
right hand are noted. Radiographs show no bony syndactyly. The most
appropriate management of this patient's deformity is release of the syndactyly
and which of the following?
A) Amputation of the digits
B) First dorsal interosseus flap
C) Full-thickness skin grafting
D) Radial artery flap
E) Split-thickness skin grafting

The correct response is Option C.

This examination contains test materials that are owned and copyrighted by the American Society of
Plastic Surgeons and the Plastic Surgery Educational Foundation. Any reproduction of these materials or
any part of them, through any means, including but not limited to, copying or printing electronic files,
reconstruction through memorization or dictation, and/or dissemination of these materials or any part of
them is strictly prohibited. Keep printed materials in a secure location when you are not reviewing them
and discard them in a secure manner, such as shredding, when you have completed the examination.

Page 251 of 287


American Society of Plastic Surgeons and the Plastic Surgery Educational Foundation
In-Service Examination

Syndactyly is the second most common congenital hand abnormality after polydactyly, occurring
in 1 in 2000 live births. Syndactyly can be described as complete, when the interdigital web
extends to the full length of the digit, or incomplete. Although there have been reports of the use
of skin flaps or primary closure after radical subcutaneous tissue debulking, skin grafts are
almost always necessary. Full-thickness skin grafting is the appropriate management of
syndactyly in a child. Effective grafts for this procedure include the lateral groin crease, plantar
foot, antecubital fossa, inner arm, or wrist flexion crease.

Amputation of the digits is almost never required for a simple syndactyly.

Pedicle flaps such as a dorsal interosseus flap or radial artery flap are not required because tissue
can be closed with adjacent flaps and skin grafts alone.

A split-thickness skin graft is a poor choice in a young child because it is difficult to obtain and
has a significant amount of contracture.

References

1. Dao KD, Shin AY, Billings A, et al. Surgical treatment of congenital syndactyly of
the hand. J Am Acad Orthop Surg. 2004;12:39-48.
2. Sawabe K, Suzuki Y, Suzuki S. Temporal skin grafts following straight incision for
syndactyly correction. Ann Plast Surg. 2005;55:139-142.
3. Cetik O, Ozsar BK, Eksioglu F, et al. Contrary intermittent skin release of complete
syndactyly without skin grafts in adults. Ann Plast Surg. 2005;55:359-362.

This examination contains test materials that are owned and copyrighted by the American Society of
Plastic Surgeons and the Plastic Surgery Educational Foundation. Any reproduction of these materials or
any part of them, through any means, including but not limited to, copying or printing electronic files,
reconstruction through memorization or dictation, and/or dissemination of these materials or any part of
them is strictly prohibited. Keep printed materials in a secure location when you are not reviewing them
and discard them in a secure manner, such as shredding, when you have completed the examination.

Page 252 of 287


American Society of Plastic Surgeons and the Plastic Surgery Educational Foundation
In-Service Examination

178. A 30-year-old man comes to the emergency department 30 minutes after he


sustained traumatic avulsion amputation of the thumb at the level of the
interphalangeal joint. The amputated digit was wrapped in a wet towel immediately
after the injury and has been kept on ice since that time. Physical examination
shows complete avulsions of the extensor pollicis longus and flexor pollicis longus
tendons from their muscle bellies. Which of the following is the most appropriate
management of this patient’s injury?
A) Replantation and tendon repair
B) Replantation of the amputated digit and immediate fusion of the interphalangeal joint
C) Revision amputation and delayed pollicization
D) Revision amputation and subsequent transfer of the great toe
E) Revision amputation, immediate shortening of the bone, and closure of the skin

The correct response is Option B.

Because the extensor pollicis longus (EPL) and flexor pollicis longus (FPL) tendons are avulsed,
repair or reconstruction of their function is difficult. When these tendons are reinserted, there is a
risk of infection. The main contribution of the thumb to overall hand function is through its
length and ability to oppose the fingers to grab and hold objects. The critical length of the thumb
is the level of the interphalangeal (IP) joint. It is optimal to preserve the length of the thumb to at
least the IP level. Thumb length is more important than motion. Considerable motion can be
preserved through the metacarpophalangeal and carpometacarpal joints of the thumb. Therefore,
thumb function is not significantly altered with fusion of the IP joint. Furthermore, fusion of the
IP joint can allow for less tension across the microneurovascular repairs. This has been shown to
improve survival rates for thumb replantations as well as produce reliable recovery of two-point
discrimination through nerve repairs that are not under tension. Furthermore, shortening of the
thumb allows for avoidance of the potential need for grafts to repair the artery or nerves. If the IP
joint of the thumb is fused, there is no need to repair the FPL tendon. Revision amputation with
closure, pollicization, or toe transfer would not yield the functional outcome of a successful
replantation. Specifically, transfer of the great toe is not needed if thumb length is at the IP level.

References

1. Chow JA, Bilos ZJ, Chunprapaph B. Thirty thumb replantations. Plast Reconstr Surg.
1979;64:626-630.
2. Wilhelmi BJ, Lee WPA, Pagensteert GI, et al. Replantation in the mutilated hand.
Hand Clin. 2003;19:89-120.

This examination contains test materials that are owned and copyrighted by the American Society of
Plastic Surgeons and the Plastic Surgery Educational Foundation. Any reproduction of these materials or
any part of them, through any means, including but not limited to, copying or printing electronic files,
reconstruction through memorization or dictation, and/or dissemination of these materials or any part of
them is strictly prohibited. Keep printed materials in a secure location when you are not reviewing them
and discard them in a secure manner, such as shredding, when you have completed the examination.

Page 253 of 287


American Society of Plastic Surgeons and the Plastic Surgery Educational Foundation
In-Service Examination

179. A 44-year-old man comes to the emergency department immediately after he


sustained an injury to the tip of the left thumb while working with a table saw.
Physical examination shows a 1.5 × 1.5-cm wound involving the volar tip of the
thumb with bone visible within the base of the wound. Which of the following is the
most appropriate management?
A) Cross-finger flap
B) Island Moberg flap
C) Secondary healing
D) Skin grafting
E) Thenar flap

The correct response is Option B.

The Moberg flap is the most effective intervention for thumb tip defects of 1.5 cm or smaller.
Some of the tactics that have been described to facilitate distal advancement of the flap include
flexion of the thumb interphalangeal crease, extension of the lateral incisions proximal to the
metaphalangeal crease, and islandization of the flap by releasing the skin at the base of the flap
and skin grafting the donor wound.

The cross-finger flap can be used for the thumb but is best suited for fingers of younger patients
because of the risk of flexion contractures when used in older patients. Secondary healing is
inappropriate in the case described because of the large size of the wound. Skin grafting is not
appropriate for the patient described because bone is exposed. The thenar flap is harvested from
the thumb and, therefore, is not used to reconstruct the thumb.

References

1. Louis DS, Jebson PJL, Graham TJ. Amputations. In: Green’s Operative Hand
Surgery. Philadelphia: Churchill Livingstone; 1999:55.
2. Lille S, Mowlavi A, Russell RC. Management of fingertip injuries. In: Russell RC,
ed. Plastic Surgery. St. Louis: Mosby; 2000:1779.

This examination contains test materials that are owned and copyrighted by the American Society of
Plastic Surgeons and the Plastic Surgery Educational Foundation. Any reproduction of these materials or
any part of them, through any means, including but not limited to, copying or printing electronic files,
reconstruction through memorization or dictation, and/or dissemination of these materials or any part of
them is strictly prohibited. Keep printed materials in a secure location when you are not reviewing them
and discard them in a secure manner, such as shredding, when you have completed the examination.

Page 254 of 287


American Society of Plastic Surgeons and the Plastic Surgery Educational Foundation
In-Service Examination

180. A 67-year-old woman comes to the office because she is unable to extend the
distal phalanx of the right thumb eight weeks after sustaining a fracture of the right
distal radius. At the time of the injury, she was seen in the emergency department
and the fracture was treated with closed reduction and casting for eight weeks.
Which of the following is the most appropriate next step in management?
A) Direct repair of the extensor pollicis longus tendon
B) Fusion of the interphalangeal joint of the thumb
C) Splinting of the thumb in extension for six weeks
D) Tenolysis of the extensor pollicis longus tendon
E) Transfer of extensor indicis proprius to extensor pollicis longus tendon

The correct response is Option E.

Minimally or nondisplaced fractures of the distal radius may be associated with significant
complications, including nonunion, malunion, stiffness, and tendon ruptures. The extensor
pollicis longus (EPL) and the extensor digitorum communis tendons are the most frequently
ruptured tendons after distal radius fracture. Rupture of the EPL tendon is more common in
minimally or nondisplaced fractures of the distal radius. The management of EPL ruptures is
surgical exploration and transfer of the extensor indicis proprius tendon to the distal stump of the
EPL tendon. This tendon transfer is a simple procedure that retains the extensor function of the
index finger.

Generally, EPL rupture occurs late. A zone of relative ischemia within the tendon weakens the
EPL until rupture occurs. With rupture of the EPL tendon, the proximal muscle and tendon
retracts. Direct repair is not possible or would place excessive tension on the repair. Grafting of
the gap lays the tendon graft in a scarred bed and motors the muscle-tendon unit with a
contracted muscle. Fusion of the interphalangeal joint of the thumb would not be optimal
treatment because a simple tendon transfer exists; however, it may serve to be a viable option if
there are multiple tendon ruptures and no sufficient transfers exist. Splinting will not restore
tendon function. Tenolysis is indicated for tendon adhesions but has no role in the management
of a tendon rupture. Interphalangeal joint arthrodesis is a more reasonable option with a similarly
ruptured flexor pollicis longus where the flexor sheath is scarred.

References

1. DeSmet L, Sioen W, Spaepen. Changes in key pinch strength after excision of the
trapezium and total joint arthroplasty. J Hand Surg [Br]. 2004;29:40-41.
2. Bonatz E, Kramer TD, Masear VR. Rupture of the extensor pollicis longus tendon.
Am J Orthop. 1996;25:118-122.

This examination contains test materials that are owned and copyrighted by the American Society of
Plastic Surgeons and the Plastic Surgery Educational Foundation. Any reproduction of these materials or
any part of them, through any means, including but not limited to, copying or printing electronic files,
reconstruction through memorization or dictation, and/or dissemination of these materials or any part of
them is strictly prohibited. Keep printed materials in a secure location when you are not reviewing them
and discard them in a secure manner, such as shredding, when you have completed the examination.

Page 255 of 287


American Society of Plastic Surgeons and the Plastic Surgery Educational Foundation
In-Service Examination

181. A 21-year-old woman who rows regularly with her university’s crew rowing club
comes to the office because she has had pain on movement of the right thumb as
well as pain and swelling of the distal forearm for the past month. She has not
sustained trauma to the hand or arm. Physical examination shows tenderness and
crepitus 4 cm proximal to the wrist and over the distal and radial aspects of the
forearm. Slight discomfort is noted on ulnar deviation of the wrist with the thumb
clasped in the palm. No grinding or crepitance of the thumb carpometacarpal joint
is noted. Which of the following is the most appropriate initial step in management
of this patient’s condition?
A) Injection of a corticosteroid into the first dorsal compartment
B) Injection of a corticosteroid into the third dorsal compartment
C) Splinting of the carpometacarpal joint of the thumb
D) Splinting of the wrist in extension
E) Observation

The correct response is Option D.

The patient described has findings consistent with intersection syndrome, a tenosynovitis of the
second dorsal compartment (common radial wrist extensors) associated with pain and swelling of
the muscle bellies of the abductor pollicis longus (APL) and extensor pollicis brevis (EPB), 4 cm
proximal to the wrist joint (ie, where the muscle bellies of APL and EPB cross extensor carpi
radialis longus and extensor carpi radialis brevis). As in other stenotic conditions of the hand and
wrist, it is associated with repetitive motions of the wrist and is frequently seen in athletes,
especially rowers and weightlifters. Initial nonoperative treatment consists of rest, nonsteroidal
anti-inflammatory drugs (NSAIDs), splinting of the wrist in 15 degrees of extension, and, in
some patients, injection of a corticosteroid into the second dorsal compartment. For those with
persistent pain, surgical release of the second dorsal compartment beginning at the wrist and
extending proximal to the area of swelling may be necessary.

De Quervain disease, a tenosynovitis of the first dorsal compartment, presents with wrist pain on
the radial side aggravated by movement of the thumb, most often in women aged 40–60 years.
Local tenderness and swelling 1 to 2 cm proximal to the radial styloid and knifelike pain on
Finklestein test (clasped thumb in palm while wrist is ulnarly deviated) is diagnostic. In mild
cases, rest, NSAIDs, and splinting of the wrist in gentle extension and the thumb widely
abducted may be successful. Injection of a corticosteroid into the first dorsal compartment may
be successful in 50%–80% of patients following one or two injections. If nonsurgical modalities
are unsuccessful, then surgical release of the first dorsal compartment is successful, if variations
of this compartment are appreciated and addressed.

This examination contains test materials that are owned and copyrighted by the American Society of
Plastic Surgeons and the Plastic Surgery Educational Foundation. Any reproduction of these materials or
any part of them, through any means, including but not limited to, copying or printing electronic files,
reconstruction through memorization or dictation, and/or dissemination of these materials or any part of
them is strictly prohibited. Keep printed materials in a secure location when you are not reviewing them
and discard them in a secure manner, such as shredding, when you have completed the examination.

Page 256 of 287


American Society of Plastic Surgeons and the Plastic Surgery Educational Foundation
In-Service Examination

Tenosynovitis of the extensor pollicis longus (third dorsal compartment) is rare but requires early
diagnosis and urgent operative treatment to prevent tendon rupture, a complication rarely seen
with de Quervain disease, trigger finger, or trigger thumb. It presents with pain, swelling,
tenderness, and often crepitus at Lister’s tubercle. Surgical treatment consists of third dorsal
compartment release and tendon transposition radial to Lister’s tubercle. Injection of a
corticosteroid is rarely indicated.

Degenerative arthritis of the thumb most frequently affects the carpometacarpal joint. Diagnosis
can be made by a positive grind test (ie, crepitations elicited with axial loading combined with
the rotation of the thumb metacarpal), and confirmed with radiographs that show destruction of
the articular surface and joint space. Early-stage disease may be treated with rest, splinting,
NSAIDs, and thenar strengthening. If this fails to relieve pain, then ligament reconstruction may
be performed, based on the extent of joint surface destruction. As the patient progresses to
end-stage degeneration of the carpometacarpal joint, carpometacarpal arthrodesis, trapezium
hemiprosthesis, ligament reconstruction-tendon interposition arthroplasty, or hemi- or complete
trapeziectomy may be appropriate management.

References

1. Wolfe SW. Tenosynovitis. In: Green DP, Hotchkiss RN, Pederson WC, eds. Green’s
Operative Hand Surgery. Vol. 1. 4th ed. New York: Churchill Livingstone;
1999:2022-2044.
2. Pool JU, Pellegrini VD Jr. Arthritis of the thumb basal joint complex. J Hand Ther.
2000;13:91-107.

This examination contains test materials that are owned and copyrighted by the American Society of
Plastic Surgeons and the Plastic Surgery Educational Foundation. Any reproduction of these materials or
any part of them, through any means, including but not limited to, copying or printing electronic files,
reconstruction through memorization or dictation, and/or dissemination of these materials or any part of
them is strictly prohibited. Keep printed materials in a secure location when you are not reviewing them
and discard them in a secure manner, such as shredding, when you have completed the examination.

Page 257 of 287


American Society of Plastic Surgeons and the Plastic Surgery Educational Foundation
In-Service Examination

182. A 25-year-old man comes to the clinic for follow-up examination six months after
undergoing open reduction and internal fixation of a grade IIIB fracture of the distal
third of the tibia and soft-tissue coverage with a neuroadipofascial flap. On
physical examination, turbid fluid is draining from the flap. The surrounding skin is
erythematous and edematous. Radiographs show evidence of osteomyelitis.
Debridement of the involved tibia is performed, resulting in a 15-cm bony defect,
and the leg is placed in an external fixator. Which of the following interventions is
most appropriate for reconstruction of bone in this patient?
A) Free fibula flap
B) Iliac crest bone flap
C) Iliac crest bone grafting
D) Ilizarov bone lengthening
E) Open cancellous bone grafting

The correct response is Option A.

The most appropriate choice for management of this large bony defect is a free fibula flap. This
flap can provide up to 26 cm of vascularized bone in an adult. Other options for free vascularized
bone include the scapula and iliac crest, both of which can provide up to 12 cm in length. Not
only are these flaps of insufficient length to adequately treat this patient, but the scapula lacks the
tubular width and depth of the fibula and the iliac crest has excessive curvature. For bone defects
smaller than 6–8 cm, bone grafts are sufficient when placed in a well-vascularized soft-tissue
pocket. Larger bony defects require vascularized bone.

The Ilizarov bone lengthening technique is not optimal in a patient who requires such a long
bone segment, although longer defects have been documented in the literature. With this
technique, a corticotomy is made through healthy bone distant from the defect. The intercalary
segment is pulled through the limb with wires until the defect is closed, with new bone formation
in the widening distraction gap. Treatment time of 1.9 months in fixation for each centimeter of
defect reconstructed requires a highly motivated and compliant patient.

Open cancellous bone grafting to open granulating skeletal defects, also known as the Papineau
technique, is limited to 6-cm defects. Larger deficiencies require multiple donor sites, adding to
donor site morbidity and discomfort.

References

1. Anthony JP, Mathes SJ. Update on chronic osteomyelitis. Clin Plast Surg.
1991;18:515-523.

This examination contains test materials that are owned and copyrighted by the American Society of
Plastic Surgeons and the Plastic Surgery Educational Foundation. Any reproduction of these materials or
any part of them, through any means, including but not limited to, copying or printing electronic files,
reconstruction through memorization or dictation, and/or dissemination of these materials or any part of
them is strictly prohibited. Keep printed materials in a secure location when you are not reviewing them
and discard them in a secure manner, such as shredding, when you have completed the examination.

Page 258 of 287


American Society of Plastic Surgeons and the Plastic Surgery Educational Foundation
In-Service Examination

2. Heller L, Levin LS. Lower extremity microsurgical reconstruction. Plast Reconstr


Surg. 2001;108:1029-1041.
3. Serafin D. The superficial circumflex iliac artery-iliac crest flap. In: Atlas of
Microsurgical Composite Tissue Transplantation. Philadelphia: WB Saunders;
1996:69-74.
4. Serafin D. The fibula flap. In: Atlas of Microsurgical Composite Tissue
Transplantation. Philadelphia: WB Saunders; 1996:547-573.
5. Serafin D. The scapular osteocutaneous flap. In: Atlas of Microsurgical Composite
Tissue Transplantation. Philadelphia: WB Saunders; 1996:347-358.
6. Green SA. Skeletal defects: a comparison of bone grafting and bone transport for
segmental skeletal defects. Clin Orthop. 1994;301:111-117.
7. Delimar D, Klobucar H, Jelic M, et al. Treatment of defect pseudoarthroses with bone
segment transport. Acta Chir Orthop Traumatol Cech. 2001;68:109-111.

This examination contains test materials that are owned and copyrighted by the American Society of
Plastic Surgeons and the Plastic Surgery Educational Foundation. Any reproduction of these materials or
any part of them, through any means, including but not limited to, copying or printing electronic files,
reconstruction through memorization or dictation, and/or dissemination of these materials or any part of
them is strictly prohibited. Keep printed materials in a secure location when you are not reviewing them
and discard them in a secure manner, such as shredding, when you have completed the examination.

Page 259 of 287


American Society of Plastic Surgeons and the Plastic Surgery Educational Foundation
In-Service Examination

183. The most appropriate intervention to achieve a bloodless field during hand surgery
is application of a tourniquet to the upper arm at which of the following pressures?
A) Equal to diastolic blood pressure
B) Equal to systolic blood pressure
C) 30 mmHg greater than diastolic blood pressure
D) 50 mmHg greater than mean arterial blood pressure
E) 100 mmHg greater than systolic blood pressure

The correct response is Option E.

For surgical procedures involving the hand, the tourniquet pressure should be based on the
patient’s preoperative systolic blood pressure.

Tourniquets have been used in extremity surgery for hundreds of years. In 1873, Johann
Friedrich August von Esmarch first introduced a bandage to exsanguinate the extremity before
applying a tourniquet. Tourniquets are used to achieve a bloodless field during surgery, allowing
greater visualization of the structures. The safe duration of tourniquet time ranges from 45
minutes to four hours; a duration of two hours is most widely accepted by hand surgeons.
Tourniquet pressure is related to both the pressure generated by the machine and the width of the
cuff. The cuff should be properly fitted to the arm. A rough guideline is use of a cuff width equal
to the diameter of the upper arm. Tourniquet pressure has been widely debated, with standard
pressures ranging from 250 to 300 mmHg.

References

1. Boiko M, Roffman M. Evaluation of a novel tourniquet device for bloodless surgery


of the hand. J Hand Surg. 2004;29B:185-187.
2. Maury AC, Roy WS. A prospective, randomized controlled trial of forearm versus
upper arm tourniquet tolerance. J Hand Surg. 2002;27B:359-360.
3. Blond L, Madsen JL. Exsanguination of the upper limb in healthy young volunteers. J
Bone Joint Surg. 2002;84:489-491.

This examination contains test materials that are owned and copyrighted by the American Society of
Plastic Surgeons and the Plastic Surgery Educational Foundation. Any reproduction of these materials or
any part of them, through any means, including but not limited to, copying or printing electronic files,
reconstruction through memorization or dictation, and/or dissemination of these materials or any part of
them is strictly prohibited. Keep printed materials in a secure location when you are not reviewing them
and discard them in a secure manner, such as shredding, when you have completed the examination.

Page 260 of 287


American Society of Plastic Surgeons and the Plastic Surgery Educational Foundation
In-Service Examination

184. A 26-year-old man comes to the office because he has worsening pain and
erythema in the thumb and small finger of the dominant right hand two days after
he punctured the thumb with a wood splinter. The most appropriate initial step is
exploration of the thumb, small finger, and which of the following?
A) Dorsum of the hand
B) Midpalmar space
C) Parona space
D) Posterior adductor space
E) Ring finger

The correct response is Option C.

The radial and ulnar bursas are connected through the Parona space, which lies between the
pronator quadratus fascia and the flexor digitorum profundus tendon sheath. The flexor tendon
sheath of the small finger often connects with the ulna bursa, which extends proximal to the
transverse carpal ligament. This connection can give rise to a “horseshoe abscess.” This abscess
results when an infection starting in the thumb or small finger progresses proximal through the
wrist and then into the opposite flexor tendon sheath through the Parona space. The Parona space
can be explored by performing an extended carpal tunnel release.

Another potential space in the hand is the thenar space, which is bordered ulnarly by the vertical
septum between the flexor sheath and metacarpal of the long finger, dorsally by the fascia of the
adductor pollicis, and radially by the thenar muscle fascia. It should be noted that this space does
not include the thenar muscles.

The midpalmar space is bordered radially by the vertical septum between the flexor sheath and
metacarpal of the long finger, dorsally by the fascia over the interossei of the third and fourth
web spaces, ulnarly by the fascia over the hypothenar muscles, and volarly by the flexor tendons.

The posterior adductor space is defined as the space dorsal to the adductor pollicis and volar to
the first dorsal interosseous.

The dorsum of the hand contains the posterior interosseous space, which is dorsal to the first
dorsal interosseous.

The ring finger would be explored if the patient had displayed signs of tenosynovitis of this digit.

This examination contains test materials that are owned and copyrighted by the American Society of
Plastic Surgeons and the Plastic Surgery Educational Foundation. Any reproduction of these materials or
any part of them, through any means, including but not limited to, copying or printing electronic files,
reconstruction through memorization or dictation, and/or dissemination of these materials or any part of
them is strictly prohibited. Keep printed materials in a secure location when you are not reviewing them
and discard them in a secure manner, such as shredding, when you have completed the examination.

Page 261 of 287


American Society of Plastic Surgeons and the Plastic Surgery Educational Foundation
In-Service Examination

References

1. Cohen MJ, Kaplan L. Histology and ultrastructure of the human flexor tendon sheath.
J Hand Surg [Am]. 1987;12:25-29.
2. Neviaser RJ. Tenosynovitis. Hand Clin. 1989;5:525-531.
3. Scheldrup EW. Tendon sheath patterns in the hand: an anatomical study based on 367
hand dissections. Surg Gynecol Obstet. 1951;93:16-22.

This examination contains test materials that are owned and copyrighted by the American Society of
Plastic Surgeons and the Plastic Surgery Educational Foundation. Any reproduction of these materials or
any part of them, through any means, including but not limited to, copying or printing electronic files,
reconstruction through memorization or dictation, and/or dissemination of these materials or any part of
them is strictly prohibited. Keep printed materials in a secure location when you are not reviewing them
and discard them in a secure manner, such as shredding, when you have completed the examination.

Page 262 of 287


American Society of Plastic Surgeons and the Plastic Surgery Educational Foundation
In-Service Examination

185. A 2-year-old boy is referred to the office by a pediatrician for surgical correction of
a deformity of the right hand. Physical examination shows Wassel Type 4
duplication of the thumb. In surgical management of this patient's condition, which
of the following is the most critical point?
A) Amputation of the ulnar duplication
B) Maintenance of the ulnar collateral ligament at the metacarpophalangeal joint
C) Reattachment of the abductor pollicis brevis
D) Repair of the flexor pollicis longus tendon
E) Repair of the nail bed

The correct response is Option B.

There are seven classes of thumb duplication as described by Wassel. Type 4 is the most
common. Type 1 involves a duplication of the distal phalanx in which the two are connected at
the base. Type 2 duplications occur when the two distal phalanges are separate. Type 3
duplications are of the proximal phalanx and are connected at the base. Type 4 duplications are
of the distal and proximal phalanxes when there are four distal and proximal phalanges. Type 5
are duplications of the distal and proximal phalanxes with a duplication of the metacarpal still
fused at the base. Type 6 duplications result in six separate bones, two distal phalanges, two
proximal phalanges, and two metacarpals that are separate.

In the correction of Type 4 duplications, it is most critical to preserve the ulnar collateral
ligament to stabilize pinch. Typically, the radial thumb is less functional and less developed. The
radial collateral ligament (RCL) is reconstructed with a ligament-periosteal flap. The abductor
pollicis brevis is included in this RCL reconstruction. Most often, the flexor pollicis longus in the
ulnar element is intact and does not require repair or reconstruction. As seen in the photograph of
the thumb duplication in the patient described, the nail complex does not require intervention.

This examination contains test materials that are owned and copyrighted by the American Society of
Plastic Surgeons and the Plastic Surgery Educational Foundation. Any reproduction of these materials or
any part of them, through any means, including but not limited to, copying or printing electronic files,
reconstruction through memorization or dictation, and/or dissemination of these materials or any part of
them is strictly prohibited. Keep printed materials in a secure location when you are not reviewing them
and discard them in a secure manner, such as shredding, when you have completed the examination.

Page 263 of 287


American Society of Plastic Surgeons and the Plastic Surgery Educational Foundation
In-Service Examination

References

1. Ezaki M, Kay SPJ, Light TR, et al. Congenital hand. In: Green's Operative Hand
Surgery. Philadelphia: Churchill Livingstone; 1999:432.
2. Laub DR, Ladd AL, Hentz VR. Congenital anomalies of the hand. In: Russell RC, ed.
Plastic Surgery. St. Louis: Mosby; 2000:1743.

This examination contains test materials that are owned and copyrighted by the American Society of
Plastic Surgeons and the Plastic Surgery Educational Foundation. Any reproduction of these materials or
any part of them, through any means, including but not limited to, copying or printing electronic files,
reconstruction through memorization or dictation, and/or dissemination of these materials or any part of
them is strictly prohibited. Keep printed materials in a secure location when you are not reviewing them
and discard them in a secure manner, such as shredding, when you have completed the examination.

Page 264 of 287


American Society of Plastic Surgeons and the Plastic Surgery Educational Foundation
In-Service Examination

186. A 48-year-old man comes to the office for consultation regarding a draining sinus
tract in the pretibial area of the left leg that developed approximately one week
ago. He has not had fever, chills, or recent trauma to the leg. Medical history
includes an open fracture of the tibia six years ago treated with an external fixator.
Which of the following diagnostic studies is the most appropriate initial step?
A) Arteriography
B) CT
C) MRI
D) Plain radiography
E) Three-phase bone scan

The correct response is Option D.

After examination of the patient, plain-film radiography is the most appropriate next step in the
case of the patient described.

Chronic osteomyelitis may present months to years after the initiating trauma. Contaminated
fractures may heal but continue to harbor sequestrum and devitalized bone. Once the diagnosis
of osteomyelitis is suspected, an accurate culture is obtained through a bone biopsy and all
necrotic bone must be removed. Bone is best obtained for cultures and pathology during open
debridement. Cultures of the sinus tract and biopsies of the skin are inadequate to characterize
bone pathogens. Cultures must be obtained prior to initiation of treatment with antibiotics.
Culture-specific antibiotics are administered for six weeks after the final debridement.

Arteriography may be considered for later reconstructive options for the patient described.

Radiographs, CT, MRI, or bone scan will not identify the pathogen.

Dressing changes fail to address the underlying etiology of this patient's problem.

References

1. Stevenson TR. Chronic osteomyelitis in 1809–1819. In: Cohen M, Goldwyn RM, eds.
Mastery of Plastic and Reconstructive Surgery. Vol. 3. Boston: Little, Brown;
1994:1809-1819.
2. Patzakis MJ, Zalavras CG. Chronic posttraumatic osteomyelitis and infected
nonunion of the tibia: current management concepts. J Am Acad Orthop Surg.
2005:13:417-427.

This examination contains test materials that are owned and copyrighted by the American Society of
Plastic Surgeons and the Plastic Surgery Educational Foundation. Any reproduction of these materials or
any part of them, through any means, including but not limited to, copying or printing electronic files,
reconstruction through memorization or dictation, and/or dissemination of these materials or any part of
them is strictly prohibited. Keep printed materials in a secure location when you are not reviewing them
and discard them in a secure manner, such as shredding, when you have completed the examination.

Page 265 of 287


American Society of Plastic Surgeons and the Plastic Surgery Educational Foundation
In-Service Examination

187. A 58-year-old woman who is a pianist comes to the office because she has had
increasing clumsiness of the right hand for the past four months. She says the
awkward movement of her hand makes it difficult for her to play the piano.
Physical examination shows marked atrophy of the first dorsal interosseous
muscle. Two-point discrimination is 3 to 4 mm in all fingers. The most likely cause
of these findings is nerve compression at which of the following sites?
A) Arcade of Struthers
B) Carpal tunnel
C) Guyon canal
D) Ligament of Struthers
E) Osborne ligament

The correct response is Option C.

Isolated intrinsic motor weakness is most often a result of compression of the motor branch of
the ulnar nerve with the ulnar tunnel, also referred to as the Guyon canal. The ulnar tunnel is
divided into three zones. Within zone 2 of the ulnar tunnel, the deep motor branch passes around
the hook of the hamate and between the pisohamate ligament and the fibrous arch of the flexor
digiti minimi. The motor branch goes on to innervate the abductor digiti minimi, the flexor digiti
minimi, the opponens digiti minimi, the small and ring lumbricals, the palmar and dorsal
interosseous muscles, the adductor pollicis, and the deep head of the flexor pollicis brevis
muscle. The terminal portion of the nerve innervates the first dorsal interosseous muscle. Causes
for compression include ganglion, hamate hook fracture, lipoma, and hypothenar hammer
syndrome.

Although isolated motor nerve compression can occur, the ulnar nerve is most commonly
compressed at the level of the elbow, which is referred to as cubital tunnel syndrome. Cubital
tunnel syndrome classically presents with sensory changes within the small and ring finger
followed by weakness in the intrinsic muscles and, in severe cases, weakness of the profundus
tendon to the small and ring finger. Surgical release in cases of cubital tunnel syndrome involves
release of the Osborne ligament as well as the arcade of Struthers.

The carpal tunnel is the most common site of median nerve compression and is not the most
common site of compression in this scenario. The ligament of Struthers may be involved in
median nerve compression at the level of the elbow.

This examination contains test materials that are owned and copyrighted by the American Society of
Plastic Surgeons and the Plastic Surgery Educational Foundation. Any reproduction of these materials or
any part of them, through any means, including but not limited to, copying or printing electronic files,
reconstruction through memorization or dictation, and/or dissemination of these materials or any part of
them is strictly prohibited. Keep printed materials in a secure location when you are not reviewing them
and discard them in a secure manner, such as shredding, when you have completed the examination.

Page 266 of 287


American Society of Plastic Surgeons and the Plastic Surgery Educational Foundation
In-Service Examination

References

1. Sweet S, Weiss LE. Ulnar nerve compression at the wrist. In: Berger RA, Weiss
APC, eds. Hand Surgery. Philadelphia: Lippincott Williams & Wilkins;
2004:909-915.
2. Gelberman RG. Ulnar tunnel syndrome. In: Gelberman RG, ed. Operative Nerve
Repair and Reconstruction. Philadelphia: Lippincott Williams & Wilkins;
1991:1131-1143.
3. McGowan AJ. The results of transposition of the ulnar nerve for traumatic ulnar
neuritis. J Bone Joint Surg Br. 1950;32:293-301.

This examination contains test materials that are owned and copyrighted by the American Society of
Plastic Surgeons and the Plastic Surgery Educational Foundation. Any reproduction of these materials or
any part of them, through any means, including but not limited to, copying or printing electronic files,
reconstruction through memorization or dictation, and/or dissemination of these materials or any part of
them is strictly prohibited. Keep printed materials in a secure location when you are not reviewing them
and discard them in a secure manner, such as shredding, when you have completed the examination.

Page 267 of 287


American Society of Plastic Surgeons and the Plastic Surgery Educational Foundation
In-Service Examination

188. A 22-year-old man has undergone multiple debridements of a wound of the right
hand since he sustained a gunshot wound six days ago. Physical examination
shows a significant soft-tissue defect and a 4-cm segment of bone loss from the
third metacarpal. A radial forearm osteofasciocutaneous flap procedure for
simultaneous reconstruction of the soft-tissue and skeletal defects is planned.
Which of the following portions of the radius is the most appropriate site for
harvesting the cortical bone graft?
A) Radial aspect, between the brachioradialis and pronator teres insertions
B) Radial aspect, between the brachioradialis insertion and flexor pollicis longus origin
C) Radial aspect, between the pronator teres insertion and flexor pollicis longus origin
D) Ulnar aspect, between the brachioradialis and pronator teres insertions
E) Ulnar aspect, between the pronator teres insertion and flexor pollicis longus origin

The correct response is Option A.

When raising a radial forearm osteofasciocutaneous flap, cortical bone should be harvested from
the radial aspect of the radius between the brachioradialis and pronator teres muscle insertions. A
cuff of the flexor pollicis longus to the radius is maintained to preserve the periosteal vessels. A
small cuff of periosteum is maintained on both sides of the radial vessels. All of the other
optional donor sites are anatomically not appropriate. A segment up to 10 cm in length and up to
40% of the cross-sectional area of the radius may be harvested. The superficial branch of the
radial nerve, running adjacent to the brachioradialis, is at risk and is sometimes sacrificed.

References

1. Vedder NB, Muzaffar AR. Soft-tissue coverage of the hand. In: Hand Surgery Update
3. Am Soc Surg Hand. 2003;34:479-492.
2. Mathes SJ, Nahai F. Radial forearm flap. In: Reconstructive Surgery. Principles,
Anatomy & Technique. New York: Churchill Livingstone; 1997:775-802.

This examination contains test materials that are owned and copyrighted by the American Society of
Plastic Surgeons and the Plastic Surgery Educational Foundation. Any reproduction of these materials or
any part of them, through any means, including but not limited to, copying or printing electronic files,
reconstruction through memorization or dictation, and/or dissemination of these materials or any part of
them is strictly prohibited. Keep printed materials in a secure location when you are not reviewing them
and discard them in a secure manner, such as shredding, when you have completed the examination.

Page 268 of 287


American Society of Plastic Surgeons and the Plastic Surgery Educational Foundation
In-Service Examination

189. A 65-year-old man who plays golf three times weekly has severe Dupuytren
contracture of the small finger of the dominant right hand. The dense cord extends
along the ulnar aspect of the hand and digit. The contracture of the
metacarpophalangeal joint is 60 degrees, and the contracture of the proximal
interphalangeal joint is 95 degrees. Which of the following is the most likely origin
of the ulnar cord?
A) Abductor digiti minimi
B) Abductor pollicis brevis
C) Antebrachial fascia
D) Cleland ligament
E) Volar carpal ligament

The correct response is Option A.

In the small finger, the ulnar cord typically originates from the musculotendinous junction of the
abductor digiti minimi. From this location, the pretendinous band, spiral band, lateral digital
sheath, and Grayson ligament can become involved, which can result in significant contractures
at the metacarpophalangeal and proximal interphalangeal joints.

The other structures listed are not typically involved in Dupuytren contracture. The abductor
pollicis brevis is the most radial thenar muscle and does not affect the ulnar cord. The
antebrachial fascia is the superficial forearm fascia and is not related to Dupuytren contracture.
The contractile cords involve Grayson ligaments and not Cleland ligaments. The Cleland
ligament is dorsal to the neurovascular bundle. The volar carpal ligament covers the Guyon
canal.

References

1. McFarlane RM. Patterns of the diseased fascia in the fingers in Dupuytren’s


contracture: displacement of the neurovascular bundle. Plast Reconstr Surg.
1974;54:31-44.
2. Hueston JT. Dupuytren’s contracture. In: Jupiter JB, ed. Flynn’s Hand Surgery. 4th
ed. Baltimore: Williams & Wilkins; 1991:864-889.

This examination contains test materials that are owned and copyrighted by the American Society of
Plastic Surgeons and the Plastic Surgery Educational Foundation. Any reproduction of these materials or
any part of them, through any means, including but not limited to, copying or printing electronic files,
reconstruction through memorization or dictation, and/or dissemination of these materials or any part of
them is strictly prohibited. Keep printed materials in a secure location when you are not reviewing them
and discard them in a secure manner, such as shredding, when you have completed the examination.

Page 269 of 287


American Society of Plastic Surgeons and the Plastic Surgery Educational Foundation
In-Service Examination

190. A 13-year-old boy is brought to the office by his parents for removal of a cast that
was applied to the right arm in the emergency department four weeks ago. The
long arm cast was applied with the wrist in flexion after closed reduction of a
fracture of the radius. On physical examination after removal of the cast, the
patient is unable to extend the long finger with the wrist in extension or in neutral
position. Full active range of motion of the long finger is noted with the wrist in
flexion. Which of the following is the most likely cause of these findings?
A) Entrapment of the flexor tendon
B) Injury to the median nerve
C) Ischemic necrosis of the flexor bellies
D) Posterior interosseous nerve palsy
E) Quadriga effect

The correct response is Option A.

Flexor tendon entrapment or adhesion in a fracture of the forearm is an unusual complication. It


has usually been described in children and adolescents. All patients exhibit the tenodesis effect—
that is, lack of extension of the involved digit with a neutral or extended wrist and full range of
motion with the wrist flexed.

A patient who had sustained an injury to the median nerve would present with numbness of the
fingers innervated by the median nerve and finger extension would not be affected. Ischemic
necrosis of the muscle bellies is unlikely in the patient described because active motion is normal
with the wrist flexed and there are no other symptoms to suggest a compartment syndrome.
Posterior interosseous nerve palsy usually affects all the fingers rather than just one. Quadriga
effect typically occurs after excessive shortening of a common flexor tendon or repairing a flexor
tendon end to a terminal extensor tendon.

References

1. Akita S, Kawai H. Entrapment of the flexor digitorum superficialis in the radius


fracture site. J Hand Surg [Am]. 2005;30:308-311.
2. Watson PA, Blair W. Entrapment of the index flexor digitorum profundus tendon
after fracture of both forearm bones in a child. Iowa Orthop J. 1999;19:127-128.

This examination contains test materials that are owned and copyrighted by the American Society of
Plastic Surgeons and the Plastic Surgery Educational Foundation. Any reproduction of these materials or
any part of them, through any means, including but not limited to, copying or printing electronic files,
reconstruction through memorization or dictation, and/or dissemination of these materials or any part of
them is strictly prohibited. Keep printed materials in a secure location when you are not reviewing them
and discard them in a secure manner, such as shredding, when you have completed the examination.

Page 270 of 287


American Society of Plastic Surgeons and the Plastic Surgery Educational Foundation
In-Service Examination

191. An otherwise healthy 55-year-old woman who works as a secretary comes to the
office because she has had increasing pain in the left wrist as well as loss of
mobility and impaired function for the past year. Physical examination shows
tenderness over the dorsoradial wrist, anatomic snuff box, and scaphoid
tuberosity. A radiograph is shown. Which of the following is the most appropriate
management?
A) Dexamethasone injection into the radiocarpal joint and splint
B) Radioscaphoid arthrodesis
C) Scapho-trapezio-trapezoid arthrodesis
D) Scaphoidectomy and four-corner fusion
E) Total wrist arthrodesis

The correct response is Option D.

The patient described has scapholunate advanced collapse (SLAC wrist). Components of a
SLAC wrist include radioscaphoid, capitolunate, and scaphocapitate arthritis. The capitate
wedges itself between the scaphoid and lunate as it slides off the ulnar articular surface of the
scaphoid. Typically, the articulation of the lunate with the distal radius is preserved, although
there are advanced degenerative changes elsewhere in the wrist.

In the radiograph shown, the radiolunate and the scapho-trapezio-trapezoid (STT) joints appear
preserved. The STT joint is unlikely to be the source of pain considering the clinical history and
the radiographic appearance. Therefore, STT arthrodesis is incorrect. Radioscaphoid arthrodesis
is unlikely to relieve all symptoms in this patient because of the presence of multiple other
arthritic joints. Injection of dexamethasone into the wrist may provide some relief; however, this
is not a long-term solution. Total wrist arthrodesis is excellent for control of pain caused by

This examination contains test materials that are owned and copyrighted by the American Society of
Plastic Surgeons and the Plastic Surgery Educational Foundation. Any reproduction of these materials or
any part of them, through any means, including but not limited to, copying or printing electronic files,
reconstruction through memorization or dictation, and/or dissemination of these materials or any part of
them is strictly prohibited. Keep printed materials in a secure location when you are not reviewing them
and discard them in a secure manner, such as shredding, when you have completed the examination.

Page 271 of 287


American Society of Plastic Surgeons and the Plastic Surgery Educational Foundation
In-Service Examination

advanced osteoarthritis; however, this pain control comes at the expense of loss of mobility.
Because of the preservation of the radiolunate joint, a limited wrist arthrodesis (scaphoidectomy
and four-corner fusion) is a superior alternative. This is because the arthritic joints can be fused
or eliminated while preserving wrist motion. Therefore, total wrist arthrodesis is not the best
option.

References

1. Imbriglia JE. Four corner arthrodesis. In: Blair WF, ed. Techniques in Hand Surgery.
Baltimore: Williams & Wilkins; 1996:865.
2. Watson HK, Weinzweig J. Intercarpal arthrodesis. In: Green DP, ed. Green’s
Operative Hand Surgery. New York: Churchill Livingstone; 1999:108.
3. Hastings H. Wrist (radiocarpal) arthrodesis. In: Green DP, ed. Green’s Operative
Hand Surgery. New York: Churchill Livingstone, 1999:108.

This examination contains test materials that are owned and copyrighted by the American Society of
Plastic Surgeons and the Plastic Surgery Educational Foundation. Any reproduction of these materials or
any part of them, through any means, including but not limited to, copying or printing electronic files,
reconstruction through memorization or dictation, and/or dissemination of these materials or any part of
them is strictly prohibited. Keep printed materials in a secure location when you are not reviewing them
and discard them in a secure manner, such as shredding, when you have completed the examination.

Page 272 of 287


American Society of Plastic Surgeons and the Plastic Surgery Educational Foundation
In-Service Examination

192. A 50-year-old woman who is obese undergoes carpal tunnel release using a
lidocaine Bier block for anesthesia. During the procedure, the patient becomes
restless and complains of a metallic taste in her mouth and ringing in her ears. As
the tourniquet is rechecked, the patient begins having seizures. Which of the
following interventions is the most appropriate next step?
A) Administer intravenous fluids
B) Administer intravenous lidocaine
C) Administer intravenous thiopental
D) Establish an airway
E) Hyperventilate with oxygen

The correct response is Option D.


The patient described has experienced a cuff leak leading to local anesthetic toxicity. At lower
doses, most toxic reactions involve the central nervous system in a biphasic fashion. Initially
there is an excitatory phase that manifests as muscle twitching in the face and distal extremities,
followed by tremors and progressing to generalized tonic-clonic seizures. With further increases
in anesthetic, a depression phase follows with drowsiness, unconsciousness, and respiratory
arrest.
Treatment of patients who demonstrate systemic toxicity with convulsions first requires the
establishment of a secure airway, followed by hyperventilation with oxygen. This often
terminates the seizures by reducing arterial CO2, preventing the cycle of acidosis, increased local
anesthetic uptake by the central nervous system, and lowered seizure threshold. If seizures
continue, 50 to 100 mg of intravenous sodium thiopental may also terminate the convulsions.
Cardiovascular toxicity usually occurs at higher anesthetic blood levels. If there is hypotension,
however, intravenous fluids, elevation of the legs, and vasopressor support should correct it. If
bupivacaine was used for the block, bupivacaine-induced arrhythmias are not extremely
responsive to intravenous lidocaine. Because of its recognized cardiotoxicity and reports of
several deaths following its use, bupivacaine is no longer recommended for use in Bier blocks.
Obesity, particularly in patients with funnel-shaped arms, may complicate the establishment of
good tourniquet occlusion in Bier block anesthesia. Thus, selecting another method of anesthesia
in obese patients may be prudent to avoid potential local anesthetic toxicity associated with
tourniquet leak.
References

1. Vaughan TM, Burt J. Local anesthetics. Sel Readings Plast Surg. 1999;9:9-11.
2. Ramamurthy S, Hickey R. Anesthesia. In: Green DP, ed. Green's Operative Hand
Surgery. 4th ed. New York: Churchill Livingstone; 1999:22-47.

This examination contains test materials that are owned and copyrighted by the American Society of
Plastic Surgeons and the Plastic Surgery Educational Foundation. Any reproduction of these materials or
any part of them, through any means, including but not limited to, copying or printing electronic files,
reconstruction through memorization or dictation, and/or dissemination of these materials or any part of
them is strictly prohibited. Keep printed materials in a secure location when you are not reviewing them
and discard them in a secure manner, such as shredding, when you have completed the examination.

Page 273 of 287


American Society of Plastic Surgeons and the Plastic Surgery Educational Foundation
In-Service Examination

193. A 37-year-old woman has had intermittent pain in the tip of the long finger of the
dominant right hand during the past three years. She says the pain is excruciating
and occurs randomly. She has extreme sensitivity to cold in the finger. Physical
examination shows extreme tenderness of the central matrix of the long finger.
Loupe magnification of the nail complex and finger pad shows no abnormalities.
Which of the following is the most likely diagnosis?
A) Buerger disease
B) Epidermal inclusion cyst
C) Glomus tumor
D) Hemangiopericytoma
E) Kaposi sarcoma

The correct response is Option C.

Glomus tumors occur most frequently in the fingertip. In the digit, most glomus tumors occur
subungually. The digital pulp is a less common location. Symptoms of glomus tumors typically
include pain, sensitivity to cold, and tenderness on palpation. On close evaluation, a blue-purple
mass can sometimes be observed. The Love sign is extreme pain on direct, focal pressure. The
Hildreth sign is ablation of pain with proximal tourniquet inflation. MRI can localize the glomus
tumor.

The surgical approach is either direct transungual or lateral subperiosteal. Complete excision
results in rapid resolution of symptoms. Recurrence varies from 6.6% to 33%. Incidence of nail
deformity with the transungual approach is 3.3% to 10%.

Buerger disease or thromboangiitis obliterans (TAO) is an inflammatory, occlusive, and


nonatherosclerotic vascular disease. The angiitis most commonly affects the small and
medium-sized arteries, veins, and nerves. Tobacco use and development and progression of TAO
are clearly linked. The precise etiology of TAO is unknown. The arteriographic findings include
normal proximal arteries, absent atherosclerosis or emboli, and focal and multifocal distal
segmental occlusions of small and medium-sized vessels, interspersed with normal-appearing
segments. Irregular stenosis with corkscrew appearance is the classic arteriographic finding.
Cessation of tobacco use is the critical first step in successful treatment.

An epidermal inclusion cyst develops after an injury traps epithelial cells in the underlying soft
tissue or bone. These cysts slowly enlarge and typically are not painful. However, an epidermal
inclusion cyst can abscess and require drainage.

This examination contains test materials that are owned and copyrighted by the American Society of
Plastic Surgeons and the Plastic Surgery Educational Foundation. Any reproduction of these materials or
any part of them, through any means, including but not limited to, copying or printing electronic files,
reconstruction through memorization or dictation, and/or dissemination of these materials or any part of
them is strictly prohibited. Keep printed materials in a secure location when you are not reviewing them
and discard them in a secure manner, such as shredding, when you have completed the examination.

Page 274 of 287


American Society of Plastic Surgeons and the Plastic Surgery Educational Foundation
In-Service Examination

Hemangiopericytoma is a rare tumor that can involve soft tissue or bone. Approximately 30% to
50% of cases develop in the limbs. It derives from vascular Zimmermann pericytes. These differ
from glomus tumor and hemangioma. Biopsy is critical to diagnosis, and treatment involves
functional wide local excision. The tumor can recur locally, and malignant forms can
metastasize, primarily to the lung and skeleton.

There are several clinical types of Kaposi sarcoma (KS). Classic KS runs an indolent course over
10 to 15 years. Most cases are seen in elderly men of Italian or Eastern European Jewish
ancestry. However, HIV-associated KS has a fulminant, disseminated, and most often fatal
course. Other clinical forms of KS occur in recipients of renal allotransplantation, patients
undergoing immunosuppressive therapy, prepubescent children, or young black African men.
Recently, a gamma herpes virus—human herpes virus type 8 (HHV-8)—was identified in KS
tissue from patients with classic, African, transplantation-related, and AIDS-associated KS.

References

1. Mills JL, Taylor LM Jr, Porter JM. Buerger’s disease in the modern era. Am J Surg.
1987;154:123-129.
2. Price EW, Lopez R, Jaffe KA, et al. Ulnar mass in a 51-year-old woman. Clin Orthop
Relat Res. 2004;(424):272-279.
3. Vasisht B, Watson HK, Joseph E, et al. Digital glomus tumors: a 29-year experience
with a lateral subperiosteal approach. Plast Reconstr Surg. 2004;114:1486-1489.
4. Pantanowitz L, Dezube BJ. Advances in the pathobiology and treatment of Kaposi
sarcoma. Curr Opin Oncol. 2004;16:443-449.

This examination contains test materials that are owned and copyrighted by the American Society of
Plastic Surgeons and the Plastic Surgery Educational Foundation. Any reproduction of these materials or
any part of them, through any means, including but not limited to, copying or printing electronic files,
reconstruction through memorization or dictation, and/or dissemination of these materials or any part of
them is strictly prohibited. Keep printed materials in a secure location when you are not reviewing them
and discard them in a secure manner, such as shredding, when you have completed the examination.

Page 275 of 287


American Society of Plastic Surgeons and the Plastic Surgery Educational Foundation
In-Service Examination

194. A 25-year-old police officer is brought to the emergency department 30 minutes


after he sustained amputation injuries to the dominant right hand. Physical
examination shows guillotine-type amputation of the thumb at the level of the
metacarpophalangeal (MP) joint and index finger at the proximal interphalangeal
(PIP) joint. The amputated parts were wrapped in moist gauze and placed in a
plastic bag immediately after the injury. Transfer of the patient to a replantation
facility will not be possible for at least 18 to 24 hours because a hurricane in the
region has disabled all ground and air emergency transport. The emergency
department physician has experience suturing minor lacerations of the hand but
has no microscopic surgical experience. He contacts the replantation facility by
telephone for consultation with a plastic surgeon. Which of the following is the
most appropriate instruction regarding management of the patient's condition until
transport to a replantation facility is possible?
A) Debride and close the wounds
B) Dermabrade the epidermis of the amputated parts and insert them under the skin of
the abdomen through two separate incisions
C) Reattach the amputated parts with sutures as a composite graft and apply splints to
the involved digits
D) Replant the amputated parts with step-by-step guidance of the plastic surgeon via
telephone
E) Wrap the amputated parts in saline-soaked gauze and place them in a plastic bag on
a bed of ice

The correct response is Option E.

The time interval between amputation and replantation can change a replantable situation to an
unreplantable one. There are no strict guidelines for ischemia times. There have been case
reports of a successful hand replantation performed after 54 hours of cold ischemia and a
successful digit replantation after 42 hours of warm ischemia. Acceptable ischemia time is
dependent on the amount of muscle in the amputated part: the more muscle, the shorter the
acceptable ischemia time. General guidelines are that if warm ischemia time is greater than 6
hours for amputations proximal to the carpus or 12 hours for the digits, replantation is usually
not recommended. Cold ischemia can double these acceptable time limitations. The patient
discussed is a young police officer with an amputation of the thumb and index finger. The thumb
is the only opposable digit and, as such, is critical for hand function. Thumb amputation is a
strong indication for replantation. The dangerous weather in the scenario described precludes
quick transfer; however, the cold ischemia time would be approximately 24 hours in a digit with
no muscle. This time frame is within the limits for a successful outcome. Therefore, the most
appropriate management in the scenario described is to wrap the amputated parts in saline-

This examination contains test materials that are owned and copyrighted by the American Society of
Plastic Surgeons and the Plastic Surgery Educational Foundation. Any reproduction of these materials or
any part of them, through any means, including but not limited to, copying or printing electronic files,
reconstruction through memorization or dictation, and/or dissemination of these materials or any part of
them is strictly prohibited. Keep printed materials in a secure location when you are not reviewing them
and discard them in a secure manner, such as shredding, when you have completed the examination.

Page 276 of 287


American Society of Plastic Surgeons and the Plastic Surgery Educational Foundation
In-Service Examination

soaked gauze and place them on ice.

Debriding and closing the wounds is not appropriate because this would treat the cold ischemia
time as a contraindication for replantation and commit this patient to late reconstruction.

Reattaching the amputated parts with sutures as a composite graft and applying splints to the
involved digits is not appropriate because of the size of the amputated digit. Small composite
grafts can survive; however, this applies to young children with distal tip amputations.
Furthermore, suturing the parts back on will create a condition of warm ischemia instead of cold
ischemia. In situations of near-complete amputations, the attached digits can be placed in an ice
saline slurry. Dermabrading the epidermis of the amputated parts and inserting them under the
skin of the abdomen through two separate incisions will not work for similar reasons.

Replanting the amputated parts with step-by-step guidance of the plastic surgeon via telephone is
inappropriate because of the poor outcome associated with an inexperienced operator.

References

1. Baek S-M, Kim S-S. Successful digital replantation after 42 hours of warm ischemia.
J Reconstr Microsurg. 1992;8:455.
2. VanderWilde RS, Wood MB, Zu ZG. Hand replantation after 54 hours of cold
ischemia: a case report. J Hand Surg [Am]. 1992;17:217-220.
3. Pederson W. Replantation. Plast Reconstr Surg. 2001;107:823-842.
4. Goldner RD, Urbaniak JR. Replantation. In: Green DP, Hotchkiss RN, Pederson WC,
et al, eds. Operative Hand Surgery. 5th ed. Philadelphia: Elsevier; 2005:1569-1586.

This examination contains test materials that are owned and copyrighted by the American Society of
Plastic Surgeons and the Plastic Surgery Educational Foundation. Any reproduction of these materials or
any part of them, through any means, including but not limited to, copying or printing electronic files,
reconstruction through memorization or dictation, and/or dissemination of these materials or any part of
them is strictly prohibited. Keep printed materials in a secure location when you are not reviewing them
and discard them in a secure manner, such as shredding, when you have completed the examination.

Page 277 of 287


American Society of Plastic Surgeons and the Plastic Surgery Educational Foundation
In-Service Examination

195. A 27-year-old man who sustained complete transection of the median nerve at the
level of the midforearm 18 months ago is scheduled to undergo opponensplasty
for improvement of hand function. Transfer of which of the following tendons is
most appropriate to restore the oppositional force of the thumb?
A) Abductor digiti minimi
B) Extensor carpi radialis brevis
C) Extensor indicis proprius
D) Flexor digitorum superficialis of the long finger
E) Palmaris longus

The correct response is Option D.

Opponensplasty is the term used to describe the transfer of a muscle tendon unit to restore the
oppositional force of the thumb that is often absent after an injury to the median nerve. The three
major principles of tendon transfer are as follows: (1) the transferred muscle should be used to
provide a single function only; (2) the transferred tendon must have an excursion sufficient to
produce the desired movement; (3) the transferred muscle should be as strong as, if not stronger
than, the muscle that is being reconstructed. Muscle power or strength is assessed by examining
the cross-sectional area of the muscle, which directly correlates with power.

In a study by Cooney, cross-sectional area and analysis of moment arm were used to determine
the best donor muscle for oppositional transfer. The flexor digitorum superficialis (FDS) of the
long finger and the extensor carpi ulnaris (ECU) muscles closely approximated thenar muscle
strength and potential excursion. Abduction from the palm was greatest after transfer of the FDS
from the long and ring fingers and after ECU and extensor carpi radialis longus (ECRL)
transfers. Pulley location was found to influence the motion and strength of transfers in both the
flexion and abduction planes. Cooney stresses the importance of directing the force of the
transfer toward the pisiform. Transfers distal to the pisiform, such as those using the extensor
digitorum quinti or the abductor digitorum quinti, produce more flexion than abduction.
Transfers proximal to the pisiform, such as the FDS using the flexor carpi ulnaris loop as a
pulley, produce more abduction and less metacarpal flexion.

References

1. Brand PW. Tendon transfers for median and ulnar nerve paralysis. Orthop Clin North
Am. 1970;1:447-454.
2. Cooney WP, Linscheid RL, An KN. Opposition of the thumb: an anatomic and
biomechanical study of tendon transfers. J Hand Surg [Am]. 1984;9:777-786.
3. Cooney WP. Tendon transfer for median nerve palsy. Hand Clin. 1988;4:155-165.

This examination contains test materials that are owned and copyrighted by the American Society of
Plastic Surgeons and the Plastic Surgery Educational Foundation. Any reproduction of these materials or
any part of them, through any means, including but not limited to, copying or printing electronic files,
reconstruction through memorization or dictation, and/or dissemination of these materials or any part of
them is strictly prohibited. Keep printed materials in a secure location when you are not reviewing them
and discard them in a secure manner, such as shredding, when you have completed the examination.

Page 278 of 287


American Society of Plastic Surgeons and the Plastic Surgery Educational Foundation
In-Service Examination

196. A 36-year-old man is brought to the emergency department after he sustained an


open, comminuted fracture of the proximal third of the tibia in a motorcycle
collision. After the patient's condition has been stabilized, he is transferred to the
operating room. Popliteal artery disruption is noted and warm ischemia time is six
hours. Surgical exploration of the wound shows avulsion of the posterior tibial
nerve. Which of the following is the most appropriate next step in management?
A) Implantation of the end of the posterior tibial nerve into the muscles of the posterior
compartment
B) Primary amputation of the leg
C) Vascular repair and coverage with a distally based anterolateral thigh flap
D) Vascular repair and coverage with a medial gastrocnemius flap
E) Vascular repair and repair of the posterior tibial nerve

The correct response is Option B.

Surgeons are always hesitant to amputate primarily, but in certain instances, it is the best option.
Generally, two absolute criteria for primary amputation after open tibial fractures with vascular
injury exist: anatomically complete disruption of the posterior tibial nerve in an adult, and crush
injury with warm ischemia time of more than six hours.

Relative indications for amputation include serious associated polytrauma, severe ipsilateral foot
trauma, and anticipated protracted course to obtain coverage and tibial reconstruction.

Several classification systems exist to determine salvage versus amputation, including MESI
(Mangled Extremity Syndrome Index), PSI (Predictive Salvage Index), MESS (Mangled
Extremity Severity Score), LSI (Limb Salvage Index), and NISSSA (acronym for nerve injury,
ischemia, soft-tissue contamination, skeletal injury, shock, and age).

References

1. Dirschl DR, Dahners LE. The mangled extremity: when should it be amputated? J Am
Acad Orthop Surg. 1996;4:182-190.
2. Lange RH, Bach AW, Hansen ST Jr, et al. Open tibial fractures with associated
vascular injuries: prognosis for limb salvage. J Trauma. 1985;25:203-208.
3. Gravvanis AI, Iconomou TG, Panayotou PN, et al. Medial gastrocnemius muscle flap
versus distally based anterolateral thigh flap: conservative or modern approach to the
exposed knee joint? Plast Reconstr Surg. 2005;116:932-934.
4. Pan SC, Yu JC, Shieh SJ, et al. Distally based anterolateral thigh flap: an anatomic
and clinical study. Plast Reconstr Surg. 2004;114:1768-1775.

This examination contains test materials that are owned and copyrighted by the American Society of
Plastic Surgeons and the Plastic Surgery Educational Foundation. Any reproduction of these materials or
any part of them, through any means, including but not limited to, copying or printing electronic files,
reconstruction through memorization or dictation, and/or dissemination of these materials or any part of
them is strictly prohibited. Keep printed materials in a secure location when you are not reviewing them
and discard them in a secure manner, such as shredding, when you have completed the examination.

Page 279 of 287


American Society of Plastic Surgeons and the Plastic Surgery Educational Foundation
In-Service Examination

197. A 10-year-old boy is scheduled to undergo surgical treatment of spastic cerebral


palsy. His function is limited by an internal rotation of the arm, flexion of the elbow,
pronation of the forearm, flexion of the wrist, and flexed in palm thumb. Which of
the following factors is the most significant predictor of the surgical outcome in this
patient?
A) Intelligence quotient
B) Level of voluntary control
C) Stereognosis
D) Two-point discrimination
E) Type of cerebral palsy

The correct response is Option B.

Cerebral palsy is a nonprogressive disorder of the central nervous system in children that
primarily affects the motor system. Clinical evaluation review passive range of motion,
sensibility, motor function, and functional classification. The most significant predictor of
surgical outcome is preoperative level of voluntary control. Intelligence, stereognosis, two-point
discrimination, and type of cerebral palsy do not have as much of an impact of postoperative
functional outcome.

References

1. Johnstone BR, Richardson PW, Coombs CJ, et al. Functional and cosmetic outcome
of surgery for cerebral palsy in the upper limb. Hand Clin. 2003;19:679-686
2. Dahlin LB, Komoto-Tufveson Y. Salgeback S. Surgery of the spastic hand in cerebral
palsy: improvement in stereognosis and hand function after surgery. J Hand Surg
[BR]. 1998;23:334-339.
3. Waters PM, Van Heest A. Spastic hemiplegia of the upper extremity in children.
Hand Clin. 1998;14:119-134.
4. Van Heest AE, House JH, Cariello C. Upper extremity surgical treatment of cerebral
palsy. J Hand Surg [AM]. 1999;24:323-330.

This examination contains test materials that are owned and copyrighted by the American Society of
Plastic Surgeons and the Plastic Surgery Educational Foundation. Any reproduction of these materials or
any part of them, through any means, including but not limited to, copying or printing electronic files,
reconstruction through memorization or dictation, and/or dissemination of these materials or any part of
them is strictly prohibited. Keep printed materials in a secure location when you are not reviewing them
and discard them in a secure manner, such as shredding, when you have completed the examination.

Page 280 of 287


American Society of Plastic Surgeons and the Plastic Surgery Educational Foundation
In-Service Examination

For each clinical scenario, select the most appropriate pharmacologic agent (A–E).

A. Acyclovir
B. First-generation cephalosporin
C. Prednisone
D. Rifampin isoniazid
E. Third-generation cephalosporin

198. A 45-year-old man has a two-day history of pain and swelling of the proximal
interphalangeal joint of the right index finger. Physical examination shows
pain on passive motion of the affected joint and erythema and tenderness
extending into the right hand. Gram's stain of fluid aspirated from the joint
shows gram-positive cocci.

199. A 19-year-old man has a seven-day history of swelling and pain on the volar
surface of the left index finger. He has no history of trauma to the finger.
Gram’s stain of fluid aspirated from the joint shows gram-negative diplococci.

The correct response for Item 198 is Option B and for Item 199 is Option E.

Septic arthritis may result from extension of an adjacent subcutaneous abscess or by intra-
articular contamination caused by a laceration or puncture wound. The joint is a poorly
vascularized potential space, favoring colonization. Early diagnosis and drainage are crucial to
treatment, as a joint infection can progress rapidly to destruction of articular cartilage.

The two most common organisms that cause hand infections are Staphylococcus aureus and
B-hemolytic streptococci. Minor staphylococcal and streptococcal infections are treated with
first-generation cephalosporins. More significant infections of the interphalangeal joint should be
performed through a midaxial incision.

Neisseria gonorrhoeae usually manifests as a primary venereal infection. However, it can


disseminate and sometimes present as a secondary hand infection, which is often confused with a
purulent tenosynovitis or arthritis. It is important to distinguish gonococcal from pyogenic
infection because, unlike a pyogenic infection, a gonococcal infection does not usually destroy
tendon or articular cartilage. Therefore, incision, drainage, and debridement are unnecessary and
should be avoided. Disseminated gonococcal infection is the most common cause of acute
infectious arthritis in sexually active adults. A history or evidence of trauma are lacking. Fluid
aspiration with Gram staining for gram-negative diplococci allows definitive diagnosis.
Hospitalization and intravenous administration of a third-generation cephalosporin is
recommended.

This examination contains test materials that are owned and copyrighted by the American Society of
Plastic Surgeons and the Plastic Surgery Educational Foundation. Any reproduction of these materials or
any part of them, through any means, including but not limited to, copying or printing electronic files,
reconstruction through memorization or dictation, and/or dissemination of these materials or any part of
them is strictly prohibited. Keep printed materials in a secure location when you are not reviewing them
and discard them in a secure manner, such as shredding, when you have completed the examination.

Page 281 of 287


American Society of Plastic Surgeons and the Plastic Surgery Educational Foundation
In-Service Examination

Manifestations of Sweet syndrome can masquerade as acute hand infections. Sweet syndrome,
originally described as an acute febrile neutrophilic dermatosis, belongs to a class of skin lesions
that histologically have intense epidermal and/or dermal inflammatory infiltrate of neutrophils
without evidence of infection or vasculitis. The lesions can erupt at sites of minor trauma. The
clinical picture is consistent with infection initially. The unresponsiveness of these lesions to
antimicrobial therapy and the lack of associated cellulitis is a clue to the diagnosis. The treatment
of choice involves a tapering dose of corticosteroids.

Herpetic whitlows are often confused with paronychia or felon and are treated mistakenly as
such. Initial signs include intense pain and erythema of the fingertip, followed by edema and
tenderness. A Tzanck smear of vesicular fluid may show multinucleated giant cells. Primary
herpes simplex infections typically resolve without treatment within three weeks. Incision and
drainage of herpetic whitlow is contraindicated because surgical treatment converts a closed
wound to open and may result in a secondary bacterial infection or viral superinfection. To date,
there have been no controlled studies that assess the efficacy of acyclovir for the treatment of
herpetic whitlow, but case reports suggest that it both suppresses and decreases the length and
severity of recurrent infections when taken orally.

References

1. Spann M, Talmor M, Nolan WB. Hand infections: basic principles and management.
Surg Infect. 2004;5:210-222.
2. O’Halloran LE, Bennett LL, Grothaus PC, et al. Sweet’s syndrome presenting as
acute hand infection. Plast Reconstr Surg. 2005;116:497.

This examination contains test materials that are owned and copyrighted by the American Society of
Plastic Surgeons and the Plastic Surgery Educational Foundation. Any reproduction of these materials or
any part of them, through any means, including but not limited to, copying or printing electronic files,
reconstruction through memorization or dictation, and/or dissemination of these materials or any part of
them is strictly prohibited. Keep printed materials in a secure location when you are not reviewing them
and discard them in a secure manner, such as shredding, when you have completed the examination.

Page 282 of 287


American Society of Plastic Surgeons and the Plastic Surgery Educational Foundation
In-Service Examination

200. A 43-year-old man with rapidly progressive scleroderma comes to the office for
follow-up examination because of skin atrophy, sclerosis, and telangiectases
resulting from his disease condition. On physical examination, multiple
contractures and skin lesions are noted. The largest lesion is located on the
dorsum of the metacarpophalangeal joint of the index finger of the right hand, with
exposure of the dorsal joint capsule and dorsal bone surfaces. Which of the
following is the most appropriate surgical procedure for correction of this
deformity?
A) Coverage with a local flap
B) Cross-finger flap
C) Full-thickness skin grafting
D) Reconstruction with bilaminate neodermis
E) Split-thickness skin grafting

The correct response is Option D.

In the scenario described, because one of the fingers is not contracted and still functional,
salvage is important for rudimentary activities. This view, in surgery, shows the open
metacarpophalangeal joint and degenerated bone at the base of the phalanx. These open
structures required suitable coverage. Histology showed stenotic fibrotic arteries typical of

This examination contains test materials that are owned and copyrighted by the American Society of
Plastic Surgeons and the Plastic Surgery Educational Foundation. Any reproduction of these materials or
any part of them, through any means, including but not limited to, copying or printing electronic files,
reconstruction through memorization or dictation, and/or dissemination of these materials or any part of
them is strictly prohibited. Keep printed materials in a secure location when you are not reviewing them
and discard them in a secure manner, such as shredding, when you have completed the examination.

Page 283 of 287


American Society of Plastic Surgeons and the Plastic Surgery Educational Foundation
In-Service Examination

immunopathic angiopathy, and impaired circulation made any surgery risky. Conventional
options for closure, including topical care and skin grafts, are likely to fail. Sclerotic skin renders
local flaps a technical impossibility, and amputation of the index finger is a more destructive
option. The Integra healed the open bones and joint and preserved a functioning finger, shown
below at six months. The material is compliant enough to allow full flexion. Threats to the finger
are gone, and daily function is possible.

References

1. Gottleib ME, Furman J. Successful management and surgical closure of chronic and
pathological wounds using Integra. J Burns. [serial online] 2004;3:4. Available from:
URL: http://www.journalofburns.com.
2. Druecke D, Lamme EN, Hermann S, et al. Modulation of scar tissue formation using
different dermal regeneration templates in the treatment of experimental
full-thickness wounds. Wound Repair Regen. 2004;12:518-527.

This examination contains test materials that are owned and copyrighted by the American Society of
Plastic Surgeons and the Plastic Surgery Educational Foundation. Any reproduction of these materials or
any part of them, through any means, including but not limited to, copying or printing electronic files,
reconstruction through memorization or dictation, and/or dissemination of these materials or any part of
them is strictly prohibited. Keep printed materials in a secure location when you are not reviewing them
and discard them in a secure manner, such as shredding, when you have completed the examination.

Page 284 of 287


American Society of Plastic Surgeons and the Plastic Surgery Educational Foundation
In-Service Examination

Thank you for your participation.

This examination contains test materials that are owned and copyrighted by the American Society of
Plastic Surgeons and the Plastic Surgery Educational Foundation. Any reproduction of these materials or
any part of them, through any means, including but not limited to, copying or printing electronic files,
reconstruction through memorization or dictation, and/or dissemination of these materials or any part of
them is strictly prohibited. Keep printed materials in a secure location when you are not reviewing them
and discard them in a secure manner, such as shredding, when you have completed the examination.

Page 285 of 287


American Society of Plastic Surgeons and the Plastic Surgery Educational Foundation
In-Service Examination

This examination contains test materials that are owned and copyrighted by the American Society of
Plastic Surgeons and the Plastic Surgery Educational Foundation. Any reproduction of these materials or
any part of them, through any means, including but not limited to, copying or printing electronic files,
reconstruction through memorization or dictation, and/or dissemination of these materials or any part of
them is strictly prohibited. Keep printed materials in a secure location when you are not reviewing them
and discard them in a secure manner, such as shredding, when you have completed the examination.

Page 286 of 287


American Society of Plastic Surgeons and the Plastic Surgery Educational Foundation
In-Service Examination

This examination contains test materials that are owned and copyrighted by the American Society of
Plastic Surgeons and the Plastic Surgery Educational Foundation. Any reproduction of these materials or
any part of them, through any means, including but not limited to, copying or printing electronic files,
reconstruction through memorization or dictation, and/or dissemination of these materials or any part of
them is strictly prohibited. Keep printed materials in a secure location when you are not reviewing them
and discard them in a secure manner, such as shredding, when you have completed the examination.

Page 287 of 287

You might also like